Témata: Laicke otazky

JuDu - 25/11/2006 - 13:39

Dobry den,

Mam velmi laicku otazku: je dopredu urceny smer letu satelitov okolo zemegule ? Ak sa nevyuzije pomocna sila otacanie Zeme, je mozne 'poslat' satelit na obeznu drahu bez rozdielu smeru (na vychod alebo na zapad) ? Kolko percent robi ta pomoc od Zeme ?
Dalsie otazky su okolo bodov L1 a L2. Viem, ze v tychto bodoch su vyrovnane gravitacne sily Zem-Mesiac a Zem-Slnko. V akej je to ale vyske (L1)? Je to naozaj bod vztiahnuty na polohu Zeme ? Lebo v tom pripade je to elipsa - ak sa to sleduje mimo Zeme z dalsej planety. Je podla mna logicke, ze sa to stale nachadza medzi Zemou a Mesiacom.
Kde sa nachadza bod L2 ?

Dakujem za pochopenie ...


ales - 25/11/2006 - 22:28

> je dopredu urceny smer letu satelitov okolo zemegule ? Ak sa nevyuzije pomocna sila otacanie Zeme, je mozne 'poslat' satelit na obeznu drahu bez rozdielu smeru (na vychod alebo na zapad) ? Kolko percent robi ta pomoc od Zeme ?

Družice lze v principu vypustit na libovolnou dráhu kolem Země, tedy i směrem "na západ" (v takovém případě se mluví o tzv. retrográdních drahách se sklonem nad 90° [rovníková dráha směrem "na východ" má sklon 0°, polární dráha má sklon 90° a rovníková dráha směrem "na západ" by měla sklon 180°]).

Oběžná rychlost pro jakoukoliv nízkou dráhu je vždy cca 7900 m/s (ať je sklon dráhy jakýkoliv). Pokud dráha vede směrem "na východ" lze od této potřebné oběžné rychlosti odečíst rychlost rotace Země v místě kosmodromu, odkud raketa startovala. Na rovníku je to cca 460 m/s (Sea Launch, Kourou), na Cape Canaveral (28°s.š.) je to cca 400 m/s a např. na Bajkonuru (47°s.š.) je to cca 315 m/s.

Při startu východním směrem z Cape Canaveral tedy teoreticky (se zanedbáním všech ztrát) stačí družici urychlit o cca 7500 m/s pro dosažení oběžné dráhy, zatímco pro start západním směrem by bylo třeba družici urychlit o cca 8300 m/s. Rozdíl v potřebné rychlosti je tedy cca +-5%, což ovšem v nosnosti rakety může činit až desítky procent (protože se zde uplatňuje logaritmický průběh Ciolkovského rovnice).


> Dalsie otazky su okolo bodov L1 a L2. Viem, ze v tychto bodoch su vyrovnane gravitacne sily Zem-Mesiac a Zem-Slnko. V akej je to ale vyske (L1)?

Librační body jsou vztaženy vždy ke dvěma hlavním tělesům a vždy jich existuje pět. To znamená, že soustava Země-Měsíc má librační body L1 až L5 (viz. např. http://www.freemars.org/l5/aboutl5.html nebo http://www.ottisoft.com/samplact/Lagrange%20point%20L1.htm ), ale také soustava Slunce-Země má své (jiné) body L1 až L5 (viz. např. http://math.ucr.edu/home/baez/lagrange.html nebo http://www.esa.int/esaSC/SEMM17XJD1E_index_0.html ). Podobné je to u dalších planet a jejich měsíců.

Librační bod L1 soustavy Země-Měsíc je na spojnici obou těles ve vzdálenosti cca 325000 km od Země a cca 60000 km od Měsíce. Vzhledem k tomu, že Měsíc kolem Země obíhá po elipse, tak samozřejmě i bod L1 se vůči Zemi pohybuje po elipse.

Librační bod L2 soustavy Země-Měsíc je "za Měsícem" (z pohledu od Země) ve vzdálenosti něco málo přes 60000 km. Bod L2 soustavy Slunce-Země je "za Zemí" (při pohledu od Slunce) ve vzdálenosti cca 1,5 mil. km.

(snad je moje odpověď alespoň trochu srozumitelná)


JuDu - 26/11/2006 - 12:57

Vazeny pan Holub,

Dakujem za vycerpavajucu odpoved, naozaj to pomohlo.

Dovidenia

Juraj


JuDu - 5/12/2006 - 10:15

Ahojte vsetci,

Novy den, nova otazka ...
Teraz vazne: ak sa satelit nachadza vo vyske viac ako 36000 km nad rovnikom, tak ma obeznu dobu okolo Zeme prave jeden den, takze akoby bol stale na jednom mieste (z pohladu zo Zeme).
Teraz moja zaludna otazka: ak spojime vsetky mozne body, tak to vytvori jeden prstenec okolo Zeme (v rovine rovnika). Netvytvoria mozne polohy jeden valec, rotacnu parabolu alebo ine rotacne teleso ? Je to mozne len nad rovnikom ?

Dakujem za pochopenie ...

S pozdravom

Juraj


Jirka - 5/12/2006 - 10:40

Geostacionarni draha je jen jedna a ma tvar prstence, mozna lepe receno kuznice se stredem v gravitacnim stredu Zeme. Jakakoliv jina obezna draha kolem Zeme se zanedbanim vlivu ostatnich teles a nehomogenit v gravitacnim poli Zeme ma vzdy tvar elipsy (ve specialnim pripade take kruznice). Tech moznych drah je tolik, ze vpodstate kompletne vyplnuji cely prostor.
Obeznych drah s periodou 24 hodin taky existuje nepreberne mnozstvi, ale jen na geostacionarni draze se satelit jevi jako nehybne visici nad Zemi.


ales - 5/12/2006 - 11:52

> ... Netvytvoria mozne polohy jeden valec, rotacnu parabolu alebo ine rotacne teleso ? Je to mozne len nad rovnikom ?

Nejsem si jist, jestli to Jirka popsal dostatečně jasně, takže pro jistotu:
- pravá geostacionární dráha musí ležet přesně nad rovníkem, musí být kruhová a ve výši cca 36000 km tak, aby měla periodu shodnou s periodou rotace Země
- taková dráha je u Země jen jedna a je to kružnice (žádný válec ani nic jiného)
- po této jediné dráze se pohybují všechny geostacionární družice (v různých místech té kružnice)
- (vše výše uvedené samozřejmě platí jen při zanedbání všech možných rušivých vlivů a nepřesností)


TH - 5/12/2006 - 12:15

citace:
Ahojte vsetci,

Novy den, nova otazka ...
Teraz vazne: ak sa satelit nachadza vo vyske viac ako 36000 km nad rovnikom, tak ma obeznu dobu okolo Zeme prave jeden den, takze akoby bol stale na jednom mieste (z pohladu zo Zeme).
Teraz moja zaludna otazka: ak spojime vsetky mozne body, tak to vytvori jeden prstenec okolo Zeme (v rovine rovnika). Netvytvoria mozne polohy jeden valec, rotacnu parabolu alebo ine rotacne teleso ? Je to mozne len nad rovnikom ?

Dakujem za pochopenie ...

S pozdravom

Juraj


Z toho ako je ta otazka formulovana, mam pocit, ze si predstavujes, ze druzica by mohla Zem obiehat po kruhovej drahe , ktorej rovina by bola rovnobezna s rovinou rovnika, ale bolo by to na inej zemepisnej sirke. Tak by druzica mohla byt "zavesena" napriklad priamo nad Bratislavou. Ak to naozaj myslis tak, tak to nie je mozne. Rovina drahy druzice musi vzdy krizovat rovnik. Specialnym pripadom je samozrejme, ked rovina drahy druzice a rovina rovnika su totozne. To je pripad geostacionarnych druzic.


JuDu - 5/12/2006 - 15:54

Dakujem kazdemu za odpoved, o jednu nejasnost mam menej vo svojej hlave.
Este raz dakujem a tesim sa nabuduce.
Juraj


JuDu - 9/1/2007 - 12:37

Dobry den,

Teraz som pozeral na katalog kozmickych telies a co som bol schopny najst, tak najstarsie teleso (ludskou rukou vyrobene) je vo vesmire 1958-002B, je to na drahe 653 km/3841 km. Je tam uvedene, ze predpokladana doba zivota je 300 rokov.
Ak draha ISS je len o niekolko kilometrov nizsia (320 - 350 km), tak preco je tam taky vyrazny brzdiaci ucinok zemskej atmosfery, ze to musi byt stale korigovany ?
Teleso 1958-002B bolo skontrolovane v rokoch 2006, 2003 a 2001. A medzi tym ? Ako vedia, ze presne identifikuju to teleso ?

Dakujem (aj za pochopenie)


Lukavský - 9/1/2007 - 18:40

Dráhy vesmírných těles (planet i družic) se řídí známými zákony (Keplerovy) a lze je spočítat dopředu i dozadu na několik let bez větších chyb. U pasivních družic to není tak přesné, protože se zde projevuje vliv nepředvídatelné změny hustoty atmosféry vlivem sluneční činnosti, ale i tak je dráha známa na dlouhou dobu a je nepravděpodobné, že by se nějaká družice sama od sebe ztratila (pokud nezmění svoji dráhu například explozí nebo srážkou s něčím jiným). Proto stačí jen občas se ujistit, že je družice v předpovídané poloze a jen čas od času provést opravu výpočtu polohy.
Hlavní vliv na životnost družice má výška její dráhy a poměr hmotnost/velikost. Na stejné dráze vydrží mnohem déle těleso s velkou hmotností proti tělesu stejné velikosti, ale lehkému. Čím je družice větší, tím má větší plochu kterou se tře o zbytky atmosféry a je tím více bržděna => má kratší životnost. Družice Vanguard 1 (1058-002B) má sice hmotnost jen asi 1,5 kg, zle její rozměry jsou miniaturní – je to koule o průměru asi 15 cm, takže ji lze s ISS jen těžko srovnávat. Navíc dráha ISS je kulová ve výšce cca 350 km a Vanguard je na protáhlé dráze, kde je i minimální výška dvojnásobná, ale v této výšce se pohybuje jen krátkou dobu. Většinu oběhu tráví ve větších výškách s řidší atmosférou (čím vyšší dráha tím pomaleji po ní letí – 2. Keplerův zákon) a proto je také její životnost i bez korekcí řádově mnohem větší než ISS.
V SPACE 40 http://www.lib.cas.cz/www/space.40/1958/I002B.HTM je pěkně vidět, že se její dráha za posledních skoro 50 let moc nezměnila nebo se můžeme podívat na Explorer 1 (1958-001A), jak apogeum dráhy tohoto dvoumetrového válce o průměru 16 cm a váze 5 kg klesalo až družice zanikla. Počáteční parametry dráhy byly srovnatelné s Vanguard 1, jen perigeum bylo ve výši ISS.[Upraveno 25.1.2007 poslal avitek]


JuDu - 10/1/2007 - 07:55

Dobry den pan Lukavsky,
Dakujem za presnu odpoved.
Juraj


JuDu - 25/1/2007 - 07:07

Dobry den prajem,

Teraz som precital cast z PDF materialu o ISS, a tam (okrem inych veci) bolo spomenute, ze na sovietskej vesmirnej stanici ALMAZ bol instalovany 'vesmirny' kanon. Mohol by niekto o tomto projekte nieco povedat ? Co akcia-reakcia ? Pocas strelby ako korigovali drahu aby sa stanica neroztocila ?

Dakujem


avitek - 25/1/2007 - 09:18

citace:

Teleso 1958-002B bolo skontrolovane v rokoch 2006, 2003 a 2001. A medzi tym ? Ako vedia, ze presne identifikuju to teleso ?



Na stránkách SPACE-40 je uveden jen velice omezený výběr parametrů dráh, protože si uvědomte, že za posledních cca. 15 roků bylo pro všechny katalogizované objekty (těch létajících k dnešnímu dni je 10154) zveřejněno celkem 60237920 dráhových elementů, tedy průměrně na jeden jediný objekt 6000! To by byly ve SPACE-40 neuvěřitelně dlouhé stránky, kdybych tam dával všechno. V podstatě výběr je dělán automaticky programem, a to tehdy, dojde-li k významné změně parametrů (ať již manévrem, nebo přirozeným způsobem); jinak, nedojde-li k velké změně, tak v posledních 5 rocích dávám nové parametry dráhy do SPACE jednou za 3 roky (proto např. u Vanguardu ty roky 2003 a 2006).

Podívám-li se na objekt 1958-002B Vanguard 1, tak od začátku letošního roku bylo zveřejněno celkem 26 nových elementů dráhy; znamená to, že byly aktualizovány každý den! A za tu dobu se dráha nezměnila měřitelným způsobem.

Tam, kdy se dráhy mění rychle, např. družice nebo jiné objekty krátce před zánikem v atmsféře, bývá denně několik nových drah, prakticky na každém oběhu nejméně jedna, tekdy asi 15 až 20 drah denně.

[Upraveno 25.1.2007 poslal avitek]


avitek - 25/1/2007 - 10:28

citace:
... bolo spomenute, ze na sovietskej vesmirnej stanici ALMAZ bol instalovany 'vesmirny' kanon. Mohol by niekto o tomto projekte nieco povedat ? Co akcia-reakcia ? Pocas strelby ako korigovali drahu aby sa stanica neroztocila ?


I když oficiální místa tuto skutečnost nikdy nepotvrdila, byl nejméně na prvních dvou Almazech vypuštěných do vesmíru (pod názvy Saljut 2 a Saljut 3) instalovan rychlopalný kanon. Jednalo se o adaptaci leteckého kanonu Nudelman-Richtěr NR-23 pod názvem Ščit-2 (česky Štít).

Byl montován na vnějším povrchu stanice rovnoběžně s její podélnou osou. Při střelbě byla orientace stanice automaticky kontrolována systémem silových gyroskopů (tzv. gyrodyny). Vzhledem k otřesům způsobeným střelbou, bylo zakázáno zkoušet kanon v době přítomnosti posádky na stanici. Měl se použít jen v případě nezbytné sebeobrany stanice.

Parametry kanonu:
typ: jednohlavňový
hmotnost bez střeliva: 39 kg
ráže: 23 mm
hmotnost střely: 200 g
úsťová rychlost: 690 m/s
teoretická kadence (rychlost střelby): 800 ran/min
praktická kadence: asi 700 ran/min

Měl sloužit jako obrana proti blízké inspekci stanice americkými raketoplány, eventuálně proti automatickým tahačům, umožňujícím navedení stanice do atmosféry a tím její zničení.

Na Saljutu 5 podle tvrzení jednoho z členů záložní posádky (kosmonaut Lisun) nebyl. Naproti tomu Popovič, který velel Saljutu 3, jeho exeistenci potvrdil, s dodatkem, že naštěstí jej nemusel použít.

Podle neoficiálních zpráv byl na jedné stanici (z dříve uvedeného Saljut 3) byl úspěšně vyzkoušen krátce před zánikem stanice, která byla záměrně navedena do atmosféry Země 1975-01-24.

Nepříliš podrobný snímek kanonu je na

http://www.russianspaceweb.com/almaz_ops2.html

V zelené krabici na následujícím obrázku je uložen kanon:

http://www.friends-partners.org/partners/mwade/graphics/a/almazgun.jpg

Dost rozmazaný snímek kanonu:

http://www.astronautix.com/graphics/a/almazgu2.jpg

Dále se tvrdí, že další Almaz (Salut 5) měl být vyzbrojen 2 neřízenými raketovými střelami.


Jirka - 25/1/2007 - 15:43

Ja si myslim ze to musi byt fama.
Jak by ten kanon zamerovali?
Mozna ze o tom nekdo mohl skutecne chvili uvazovat, ale myslim ze prakticky to muselo byt nerealizovatelne. Mozna ze tu zpravu vypustili Rusove zamerne na zmateni nepritele.


Abraxis - 25/1/2007 - 17:16

citace:
Ja si myslim ze to musi byt fama.
Jak by ten kanon zamerovali?
Mozna ze o tom nekdo mohl skutecne chvili uvazovat, ale myslim ze prakticky to muselo byt nerealizovatelne. Mozna ze tu zpravu vypustili Rusove zamerne na zmateni nepritele.


No, muze to pochopitelne byt fama, ale pri zvazovanem urceni (tedy obrana pred "prepadnutim stanice raketoplanem") by to naprosto stacilo - mireni proste otocenim cele stanice a presnost nemusi byt nic moc - staci "efektivni dostrel" na 10 metru...


Jirka - 25/1/2007 - 17:28

To by bylo ovsem dobre jen v tom pripade, ze by si nekdo chtel s takto vybavenou stanici dat randezvous.
Ani pak neni vubec jiste, jestli by zavcas dokazali identifikovat nepratelsky raketoplan touzici po spojeni s takovou stanici. Mozna by jen stacilo provadet uhybne manevry, nebo berany duc.
Kazdopadne by takovy kanon zrejme nebyl nic platny proti klasickym ASATum a urcite tehdy nemeli automaticky system schopny detekovat zavcas nejakou hrozbu, natoz zamerit kanon na vzdalenost vetsi nez par desitek metru.
Pokud to melo byt delano manualne, pak by potrebovali hned nekolik kosmonautu aby dreli nepretrzite hlidky.
Mozna ze tehdy premysleli jinak.


Hawk - 25/1/2007 - 18:17

Třeba s ním chtěli ničit klíčové satelity USA na LEO. V té době ještě ASATy nebyly.


avitek - 25/1/2007 - 21:30

citace:
Jak by ten kanon zamerovali?


Naprosto stejně, jako se zaměřuje kanon stihačky při vzdušných soubojích. Namíříte celé latedlo, v tomto případě celou stanici. Byl montován fixně.

Existenci potvrdili kosmonauti po glasnosti. V době, kdy létely první Almazy se o tom nevědělo.


avitek - 25/1/2007 - 21:42

citace:
Třeba s ním chtěli ničit klíčové satelity USA na LEO. V té době ještě ASATy nebyly.



Není pravda. Letové zkoušky ASATů v SSSR začaly už v roce 1963 (Poljot 1), ostrá zbraň (Kosmos 249) byla vyzkoušena v roce 1968.

První stanice Almaz (Saljut 2) startovala v roce 1973 (ta havarovala na dráze ještě dřív, než se k ní vydala první posádka) další (Saljut 3) letěl v roce 1974. Tedy o 5 resp. 6 let později než začaly ostré zkoušky sovětských ASATů.

Navíc používat dvacetitunovou stanici na "honění" nějakých družic je nesmysl. Za tím účelem začal vývoj Sojuzu VI (vojennyj istrebitěl), ale ten se nedostal dál než na rýsovací stoly. Zjistilo se, že i to je ve srovnání s bezpilotními malými ASATy neefektivní, neoparativní a neekonomické. Vyhrály automaty (jako i jinde, např. fotografická rozvědka z oběžné dráhy - tu měly taky dělat Almazy).



[Upraveno 26.1.2007 poslal avitek]


Jirka - 26/1/2007 - 10:50

citace:
citace:
Jak by ten kanon zamerovali?


Naprosto stejně, jako se zaměřuje kanon stihačky při vzdušných soubojích. Namíříte celé latedlo, v tomto případě celou stanici. Byl montován fixně.



Posadka by to mozna pomoci dalekohledu mohla skutecne udelat. Jenomze to by byla obrana jen proti objektum ktere by se stanici chteli dokovat. Pochybuju ze je mozne sestrelit rucnim zamerovanim objekt pohybujici se rychlosti treba jen nekolika stovek m/s. Navic by trosky zniceneho objektu stejne mohly zasahnout stanici.
Taky by museli objekt zavcas detekovat. To znamena, ze by museli dostat hlasku ze zeme, ze se k min blizi cizi objekt, pak by ho museli lokalizovat, preorientovat celou stanici a zacit palbu.
Me to prijde jako hrozne nerealisticky scenar.
Logictejsi mi prijde ze to byl test utocne zbran. Mohli se pokusit zlikvidovat nahodou kolem proletajici satelity. Kdyby meli celou flotilu takovych stanic s automatickym zamerovanim, tak by to snad slo. V pripade valky by par nepratelskych satelitu mohli sundat.
Mozna by mohli sundat nejaky satelit i v dobe miru, bylo by tezke jim neco dokazat.

Existenci potvrdili kosmonauti po glasnosti. V době, kdy létely první Almazy se o tom nevědělo.


avitek - 26/1/2007 - 13:48

citace:
Posadka by to mozna pomoci dalekohledu mohla skutecne udelat. Jenomze to by byla obrana jen proti objektum ktere by se stanici chteli dokovat.


Pokud jste četl jeden z mých příspěvků pořádně, účel skutečně byl (alespoň deklarovaný), obrana pro blízké inspekci, případně dokování tahače, nebo útok protidružicí.

Pro zaměřování měli periskopický dalekohled.


Radek - 27/1/2007 - 18:01

Koukám tu na Vaši diskusi a pokusím se doplnit.
V dokumentu o životě V.N.Čelomeje je část věnována zbrani Vámi probírané. Vývoj Almazu patříl do kompetence OKB-52 v době po roce 1966 CKB Mašinostrojenija, což je Čelomejovo OKB.

V krátkém výstřižku (celý tam dát nemohu) jsem krom filmových záběrů zmíněného kanonu (nevím do jaké míry jsou autentické) dal popis, který vypráví Vladimir Poljačenko, hlavní konstruktér NPO Mašinostrojenija. Na konci hovoří člověk, kterého asi všichni poznáte i bez titulku.

Dokument pod jménem „Almaz“ stáhnete na ftp://81.19.45.134/

Přihlašovací jméno „redboy“ heslo není třeba. Vel. cca. 13M


JuDu - 13/2/2007 - 14:25

Ahojte,

Mozem teraz naozaj 'jemne laicku' otazku ?
Ak sila posobiaca na nejake teleso smeruje mimo tazika, tak vyvolava krutiaci moment. Zatial je vsetko OK, ale ako je to s upravou drahy pre ISS ? Robi to klasicky Sojuz (Progress) tym sposobom ze zapali svoj hlavny pohon ? Ako je tam vyriesena otazka smerovania ? Podla mojho nazoru tam musia 'pustit' urcite percento cez vsetky trysky aby bolo smerovanie dodrzane. Tieto trysky su ale planovane na pohon kozm. lode a nie stanice, tak ako je tam s vydrzou ?

Dakujem ...


avitek - 13/2/2007 - 14:39

citace:
Ak sila posobiaca na nejake teleso smeruje mimo tazika, tak vyvolava krutiaci moment. Zatial je vsetko OK, ale ako je to s upravou drahy pre ISS ? Robi to klasicky Sojuz (Progress) tym sposobom ze zapali svoj hlavny pohon ? ...


Pro korekce dráhy se nepoužívají hlavní motory SKD, ale pouze motory DPO (dvigatěli přičalivanija i orientacii) a to pouze a jen lodí Progress. Sojuzy se z bezpečnostních důvodů ke korekcím dráhy komplexu nepoužívají vůbec.

Motory DPO jsou skutečně značně slabé (viz popis některého z posledních Progressů na SPACE-40, např. http://www.lib.cas.cz/www/space.40/2006/I045A.HTM ), proto při korekcích pracují po dobu desítek minut. Dají se však kombinovat tak, aby se případně částečně kompenzovaly krouticí momenty, způsobené nesymetrií tahu vzhledem k těžišti. Aby to nembylo moc problematické, používají se zejména lodě, připojené k zadnímu stykovacímu uzlu modulu SM. Při jednom z posledních takových manévrů dne 2006-11-29 došlo dokonce k problémům, protože odchylka orientace překročila naprogramované meze v palubním řídicím počítači systému SUDN. Bylo to způsobeno mj. tím, že po připojení konstrukce ITS P3/P4 je stanice značně nesymetrická.


Ervé - 15/2/2007 - 07:34

Takže SKD se nepoužívá, protože nesymetrie stanice neumožňuje vyrovnávat jeho tah jen motorky DPO. Taky to znamená, že palivem se plýtvá, protože část tahu se navzájem ruší kvůli nesymetrii a navíc se používají motorky s nízkým Isp. Chtělo by to iontový motor, který by využíval přebytku el. energie v době spánku. Vasimr už se k testování na ISS nepřipravuje ?


Ladze - 22/2/2007 - 12:14

Dotaz

Jak na ISS nebo i jinde mimo přitažlivost zvážím hmotnost předmětu když nemám "tu tíhu"? Jak vlastně odlétající lodě zjistí kolik naložily mat. (váhu předmětů). Předpokládám, že se musí zadat do palubního počítače kolik váží pro výpočty tahů.

Díky


MIZ - 22/2/2007 - 12:27

citace:
Dotaz

Jak na ISS nebo i jinde mimo přitažlivost zvážím hmotnost předmětu když nemám "tu tíhu"? Jak vlastně odlétající lodě zjistí kolik naložily mat. (váhu předmětů). Předpokládám, že se musí zadat do palubního počítače kolik váží pro výpočty tahů.

Díky

Jen lehce upřesním: Hmotnost tam musí být napsaná "od výrobce". Vážením nečeho zjistím tíhu té věci, což je síla, jakou je přitahováno k zemi. Na oběžné dráze v beztíži má vše tíhu nula. Hmotnost je ale samořejmě konstantní a hraje roli např. pro velikost síly potřebné ke změně dráhy apod.


Ladze - 22/2/2007 - 12:39


Jen lehce upřesním: Hmotnost tam musí být napsaná "od výrobce". Vážením nečeho zjistím tíhu té věci, což je síla, jakou je přitahováno k zemi. Na oběžné dráze v beztíži má vše tíhu nula. Hmotnost je ale samořejmě konstantní a hraje roli např. pro velikost síly potřebné ke změně dráhy apod.






Tohle ja vím taky, ale zajímají mě ty věci které to na sobě nemají. To bude asi část experimentů, možná odpadky(ne vše odveze progres nebo ano?)


MIZ - 22/2/2007 - 12:44

Ještě mě napadlo, že by místo váhy mohla fungovat nějaká malá centrifuga. Ale tu tam asi nemají...

IMHO to dělají odhadem.


arccos - 22/2/2007 - 12:55

Vážit v beztížném stavu se dá pomocí vah založených na principu setrvačnosti. Konkrétní příklad ale nemam.


tomas.pribyl - 22/2/2007 - 13:27

citace:
Vážit v beztížném stavu se dá pomocí vah založených na principu setrvačnosti. Konkrétní příklad ale nemam.


Tomto zpusobem se da zvazit (a vazi) cela ISS. Zasobovaci lod Progress provede nejaky motoricky manevr: vime, jaky tah maji jeji motory, jaky maji vektor tahu... Kdyz zname presnou obeznou drahu stanice pred manevrem a po nem (plus vyse zminene parametry manevru), tak muzeme dopocitat hmotnost stanice (vetsinou je to s pomerne velkou odchylkou, ale u takoveto stodoly to neni zas tak velky problem).


admin - 22/2/2007 - 13:31

Nešlo by to jednodušeji? Třeba měřením akcelerometru při tomto motorickém manévru?


avitek - 24/2/2007 - 09:58

citace:
Nešlo by to jednodušeji? Třeba měřením akcelerometru při tomto motorickém manévru?


Samozřejmě, že se měří zrychlení, působené chodem motorů. Problém je ale v přesnosti, protože tah motoru (motorů) závisí na mnoha parametrech (např. i takové prkotiny, jako je počáteční teplota spalovací komory, tlak v systému dodávky KPL, konkrétní nastavení ventilů v cestách paliva a okysličovadla, což vede ke změně směšovacího poměru a tím i změně hodnoty sepcifického impulsu a tím i ke změně tahu). Nicméně je to jediná možná cesta.

To, co se získá z rovnice

m = F/a

je tzv. dynamická hmotnost kosmického tělesa. Např. nejposlednější hodnota pro ISS je pro okamžik 2007-02-23 12:31:48 UTC:

479216,60 liber tj. 217369,0 kg

Jinak astronauti na palubě ISS mají "váhy", kterými si měří hmotnost svého těla. Jedná se o pružinové zařízení, které se rozkmitá a z kmitů se sanoví hmotnost jejich těla (všimněte si na stránkách ISS ve SPACE-40, že toto dělají pravidelně, naposledy to dělali dne 2007-02-05, příští budou dělat v pondělí 2007-02-26).


avitek - 24/2/2007 - 10:17

Princip pružinového měřiče hmotnosti spočívá v tom, že pro sílu, které je třeba pro stlačení nebo protažení pružiny platí pro značný rozsah výchylek pružiny z rovnovažné polohy lineární závislost (přímá úměrnost)

F = -K.x

(síla F má opačný směr než výchylka x, proto tam to mínus, K je konstantní vlastnost pružiny, odvozená z modulu pružnosti materiálu).

Ta síla musí být komplenzována setrvačnými silami, působenými tělesem o hmotnosti m, tedy

F = m.a = m.(d2x/dt2) = -K.x

Výsledkem řešení této diferenciální rovnice je rovnice harmonického kmitání, kde vyjde pro frekvenci kmitání f (resp pro periodu kmitu T)

f = 1/T = √[K/(2π.m)]

Frekvence f je tedy nepřímo úměrná druhé odmocnině z hmotnosti m (to je křeslo+astronaut+částečně i pružina). Čím je astronaut hmotnější ("víc váží"), tím s ním "pružinové křeslo" pomaleji kmitá. Samozřejmě to rovnou vyhodnocuje mikroprocesor, takže na displeji se přímo odečte hmotnost těla astronauta.

P.S. Omlouvám se, že toto vysvětlení není moc "laické", ale snad pár rovnic nevadí ... a nikoho to nezabije



[Upraveno 24.2.2007 poslal avitek]


JuDu - 12/3/2007 - 21:05

Dobry vecer,

Chcel by som vam polozit jednu otazku: ako je vyriesene pracovne naradie v kozme ? Pocul som o specialnej 'vrtacke', ktora sa pouziva na utiahnutie/uvolnenie skrutiek. Ak je to klasicke riesenie, tak na druhej strane treba vyvinut patricny protimoment, aby kozmonaut ostal na mieste. Je to vyriesene inak ? Su tam haky alebo vrtacku treba o nieco 'opriet' ?

Dakujem


Ervé - 14/3/2007 - 07:29

Pokud vím, řeší se to rotací části vrtačky/šroubováku opačným směrem, aby se momenty vyrovnali. Kosmonauti jsou většinou ještě opření, případně připojení, aby mohli působit silou bez odplavání nebo roztočení.


ales - 14/3/2007 - 10:59

> Pocul som o specialnej 'vrtacke', ktora sa pouziva na utiahnutie/uvolnenie skrutiek. Ak je to klasicke riesenie, tak na druhej strane treba vyvinut patricny protimoment, aby kozmonaut ostal na mieste. Je to vyriesene inak ? Su tam haky alebo vrtacku treba o nieco 'opriet' ?

Možná, že některé kosmické "vrtačky" mají opatření na zlepšení stability, ale myslím že na ISS je to vždy tak, že kosmonauti s nářadím pracují jen v poloze, kdy se mohou dobře "zapřít" a nástroj přitom pevně držet (aby vyrovnali moment, který vzniká ne rotací části "vrtačky", ale "třením" šroubu nebo vrtáku v materiálu). Při vrtání nebo šroubování se kosmonauti na ISS vždy drží druhou rukou pevně konstrukce stanice, nebo jsou zaklesnuti nohama na plošince PFR (nikdy nešroubují v okamžiku, kdy jen volně "plavou" v prostoru bez opory).

Takže se domnívám, že "vrtačka" je vždy "opřena" o kosmonautovu ruku a celý kosmonaut v tu chvíli musí být solidně "opřen" o konstrukci stanice (teoreticky by kosmonaut mohl být "opřen" o reaktivní zádovou manévrovací jednotku, ale nepoužívá se to).


Jirka - 14/3/2007 - 11:23

citace:
Pokud vím, řeší se to rotací části vrtačky/šroubováku opačným směrem, aby se momenty vyrovnali.

Nejde jen o rotaci hlavy vrtačky, ale i o treci sily. Tudiz by opacny moment musel byt hoooodne velky a zavisly hlavne na velikosti momentu trecich sil, ne jen na rotaci hlavy vrtacky. To by vyzadovalo nezavisly gyroskop. Bylo by pak zajimave sledovat co by to s kosmonautem udelalo kdyby takovym roztocenym gyroskopem chtel pohnout nebo ho odsaturovat.


avitek - 15/3/2007 - 18:09

Zřejmě jde diskuze o univerzálním "šroubováku" PGT (Pistol Grip Tool). Skutečně nemá kompenzaci kroutícího momentu, ale je to jinak značně sofistikované nářadí, řízené mikroprocesorem. Pomocí tohoto mikroprocesoru se dá nastavit kroutící moment (tedy "síla" utahování nebo povolování šroubů či matic), dá se nastavit předepsaná počet otáček. Na hlavici se dají připojit různé nástavce (případně i prodloužení), ale v podstatě je určen na standardní šrouby a matice 3/8". Původně byl vyvinut pro opravářské expedice k Hubbleovu dalekohledu, a později byl modernizován pro potřeby montáže ISS. Nejedná se o vrtačku.


H - 16/3/2007 - 03:26

kolik je tech šroubováku? já myslim že jich maj jen pár kusů asi 8 Ks či tak nějak


avitek - 16/3/2007 - 08:00

citace:
kolik je tech šroubováku? já myslim že jich maj jen pár kusů asi 8 Ks či tak nějak


Jen 8 kusů je určeno pro výcvik prací v NBF (Neutral Buyoancy Facility, to je ten bazén v JSC v Houstonu). Kolik je letových kusů se mi ještě nepodařilo zjistit.


JuDu - 28/3/2007 - 19:36

Dobry vecer,

Ak kotrolujem dlzku obehu ISS a dlzku 'normalnej' vychadzky do kozmu, tak dojdem k zaujimavej otazke: co robi kozmonaut v case, ked sa stanica nachadza v tme ? Nemozu tam byt silne reflektory + ked tam nie je atmosfera, nie je tam ani rozpyl svetla (atmosferou). Len tak 'existuje' v skafandre, pocuva hudbu a komunikuje cez radio ?

Dakujem


J2930 - 28/3/2007 - 19:58

citace:
...nie je tam ani rozpyl svetla (atmosferou). Len tak 'existuje' v skafandre, pocuva hudbu a komunikuje cez radio ?...


Mezi Zemi a Sluncem te atmosfery take moc neni... a preci sem svetlo doleti.... a prisvecuje si treba reflektorem na skafandru. Pracuje.
Napr. video:http://spaceflight.nasa.gov/gallery/video/shuttle/sts-115/qtime/sts115_fd04_02.mov
http://spaceflight.nasa.gov/gallery/video/shuttle/sts-115/qtime/sts115_fd10_01.mov


Martin Kolman - 28/3/2007 - 20:02

Kdyz jsem minule sledoval vycházku (to byla ta s "třepáním" při skládání solárního panelu) tak sem získal ten pocit, že:
Dělají za umělého světla, tzn. astronauti mají takové ty halogenky na přílbě po stranách, možná budou i nějaké reflektory na stanici.
V jednou případě ovšem, když nebyl k dyspozici přenos videa přes satelit
TDRS, nechali astronauty prostě čekat a dívat se na Zemi
To jsou ovšem moje dojmy pouze z jediné EVA co sem zatím trochu souvysle sledoval, praxe může být jiná....


ales - 28/3/2007 - 22:07

> ... vychadzky do kozmu ... co robi kozmonaut v case, ked sa stanica nachadza v tme ? Nemozu tam byt silne reflektory ...

Proč by tam nemohly být reflektory? Jsou tam!

Viděl jsem v NASA TV (na netu) několik EVA a konstatuju, že i ve stínu Země kosmonauti na povrchu stanice pracují úplně normálně a bez problémů. Svítí si přitom reflektory na přilbě skafandrů, ale na některých místech ISS a v nákladovém prostoru raketoplánu jsou i pevně zabudované reflektory vnějšího osvětlení. Nasvícení je tak dobré, že TV záběry z přilbových kamer jsou ve stínu Země docela dobré a dokonce i záběry z větší vzdálenosti z kamer na povrchu ISS nebo z raketoplánu jsou přijatelné (clona objektivu je hodně otevřena, ale obraz je poměrně kvalitní).

Odpověď na původní otázku tedy podle mne zní: Kosmonauti při EVA mohou normálně pracovat i ve stínu Země, protože mají "pracoviště" dostatečně osvětleno reflektory.


avitek - 28/3/2007 - 22:21

Ještě na okraj práce ve tmě: Ruské skafandry typu Orlan-M neměly původně ve výbavě žádné přílbové reflektory. Tak se dohodli s NASA, udělaly se drobné úpravy a teď se v případě, že se leze ven z modulu Pirs v ruských skafandrech, se na jejich přílby montují ty americké reflektory a kamery.

Dříve se skutečně v době "orbitální noci" ruští kosmonauti flákali a odpočívali. Problém časových harmonogramů ruských výstupů do volnoho prostoru (VKD) taky spočíval v tom, že kritické fáze práce se mohly dělat jen v době přeletů ISS nad územím bývalého SSSR, kdy bylo možné spojení s řídicím střediskem CUP přes ruské pozemní stanice. Dnes je to řešeno tak, že spojení Rusům zajišťují taky Američani (ale i tak je tam kvůli výpadku jedné TDRS asi 20minutová mezera na každém oběhu).


J2930 - 30/3/2007 - 12:24

Rosvicene prilbove reflektory. Jsou docela dobre polohovatelne...kdyz uz to tady zahlcuju obrazky:

http://spaceflight.nasa.gov/gallery/images/shuttle/sts-116/ndxpage4.html


Jano - 31/3/2007 - 00:41

>> ..ale i tak je tam kvůli výpadku jedné TDRS asi 20minutová mezera na každém oběhu
Znamená to, že počas tých 20' nemá ISS vôbec žiadne spojenie so Zemou, alebo len toľko, že vtedy používa nejaké úzkopásmovejšie cez inú družicu?


avitek - 31/3/2007 - 10:22

citace:
>> ..ale i tak je tam kvůli výpadku jedné TDRS asi 20minutová mezera na každém oběhu
Znamená to, že počas tých 20' nemá ISS vôbec žiadne spojenie so Zemou, alebo len toľko, že vtedy používa nejaké úzkopásmovejšie cez inú družicu?


Žádná jiná družice nemůže TDRS zastoupit. Na některých obězích je v té mezeře (která je přibližně nad Indickým oceánem) mohou zaskočit ruské pozemní stanice, ale to jen v případě, že dráha ISS je hodně "severně", aby přelétala přes území bývalého SSSR. Při jižnějších přeletech se dá v nouzi použít spojení v pásmu UHF přes pozemní stanici USAF na ostrově Diego Garcia v Indickém oceánu, případně pozemní stanice poblíž australského města Perth. Ale většinou to není třeba.


Petr - 31/3/2007 - 15:08

citace:
citace:
>> ..ale i tak je tam kvůli výpadku jedné TDRS asi 20minutová mezera na každém oběhu
Znamená to, že počas tých 20' nemá ISS vôbec žiadne spojenie so Zemou, alebo len toľko, že vtedy používa nejaké úzkopásmovejšie cez inú družicu?


Žádná jiná družice nemůže TDRS zastoupit. Na některých obězích je v té mezeře (která je přibližně nad Indickým oceánem) mohou zaskočit ruské pozemní stanice, ale to jen v případě, že dráha ISS je hodně "severně", aby přelétala přes území bývalého SSSR. Při jižnějších přeletech se dá v nouzi použít spojení v pásmu UHF přes pozemní stanici USAF na ostrově Diego Garcia v Indickém oceánu, případně pozemní stanice poblíž australského města Perth. Ale většinou to není třeba.



To je zajímavé. Zůstala vůbec někde v kontinentálních státech nějaká pozemní stanice ještě z dob Apolla, nebo to od 80.let všechno běží jen a pouze přes TDRS ?


avitek - 31/3/2007 - 17:29

citace:
Zůstala vůbec někde v kontinentálních státech nějaká pozemní stanice ještě z dob Apolla, nebo to od 80.let všechno běží jen a pouze přes TDRS ?


Ano jako záložní spojení na VHF/UHF na White Sands Missile Range a na Wallops Flight Station. Jednou měsíčně se testuje spojení mezi ISS a těmito stanicemi.


Jano - 31/3/2007 - 17:45

>>Žádná jiná družice nemůže TDRS zastoupit. ...... Ale většinou to není třeba.

Takže behom tej štvrťhodinky, keď na nich 'big-bradá z hjústnu' nevidí, tam hore môžu robiť neplechu (napr. rozprávať si ftipy o Griffinovi ..), asi ako Apollonauti nad odvrátenou stranou Mesiaca

Btw, spojovacie družice so silno eliptickou dráhou sú už definitívne minulosťou?


avitek - 31/3/2007 - 21:25

citace:
... Btw, spojovacie družice so silno eliptickou dráhou sú už definitívne minulosťou?


Ale ano, stále se používají, ale naprosto ne pro spojení s raketoplány ani s ISS. Většinou jde o komunikace mezi dvěma body na Zemi, ležícími ve vysokých zeměpisných šířkách. Proto je používají FRusové, protože na severu Sibiře jsou stacionární družice moc nízko nad obzorem (nebo vůbec pod obzorem) a nedají se použít.

Američani používali HEO-družice ke spojení se špionážními družicemi na polárních drahách (systém SDS).


JuDu - 4/4/2007 - 20:13

Dobrý večer,

Napadla ma teraz takáto otázka:
Predstavme si hypotetickú situáciu: astronaut sa zobudí po spánku na stanici a zistí že nebol pripútaný. Teraz je v strede obytného priestoru a nemá na dosah žiadny pevný bod - držadlo, stenu, nič.
Teraz hlavná otázka: ako prinúti svoje telo, aby sa pohol nejakým smerom ? Skúsi si odhdzovať svoje šaty a pohybovať na základe akcie-reakcie ?

Vďaka


martalien - 4/4/2007 - 20:53

citace:
ako prinúti svoje telo, aby sa pohol nejakým smerom ? Skúsi si odhdzovať svoje šaty a pohybovať na základe akcie-reakcie ?

Vďaka


Kolem je vzduch - tak bude plavat jako ve vode - jak primitivni ale jake ucinne - no teda nic moc ale na pohyb to staci.
martalien


JuDu - 5/4/2007 - 09:06


Kolem je vzduch - tak bude plavat jako ve vode ...


V prostredi mikrogravitacie je to mozne ? Vzduch okolo astronauta kladie taky odpor, ze moze plavat ?

Ď.


martalien - 5/4/2007 - 12:12

citace:
taky odpor, ze moze plavat ?
Ď.


Je to jen o tom odporu. Pokud bude rukama jen mavat tam a sem tak se ani nehne. Pokud s nimi pri pohybu dopredu pujde podel tela nad hlavu tak aby spicky prstu smer ovali "nahoru" a pak prudce kmitne natazenyma rukama zpet do upazeni (dlane natoceny tak aby kladli co nejvetsi odpor) - bohate to na pohyb staci.


bejcek - 5/4/2007 - 12:14

Mám doma i na hvězdárně (velmi mizerná kvalita) video "Astrosmiles", kdysi to pustila ČT2?. Tam různé posádky STS předvádějí stav beztíže. Je tam celá řada situací pohybu lidí v beztíží. Např.: Astronaut Storry Musgrave je "zavěšen" v prostoru a začne rukama se jakoby odstrkovat od okolí (jako když se chcete otočit) a začne se skutečně otáčet a dostane se po jedné otočce do výchozí polohy.

Určitě se nějaké video dá najít někde na stránkách NASA. Nebo to někdo má nahrané lépe.


ales - 5/4/2007 - 12:50

Souhlasím s martalienem, že v orbitální stanici by mělo jít "plavat" vzduchem podobně, jako pod vodou (jen mnohem, mnohem pomaleji).

Navíc si myslím že vzhledem k tomu, že beztíže na stanici není úplně dokonalá, by mělo stačit pár minut počkat (i bez pohybu) a "mikrogravitace" kosmonauta sama "odnese" k některé ze stěn stanice.


J2930 - 5/4/2007 - 13:01

citace:
Navíc si myslím že vzhledem k tomu, že beztíže na stanici není úplně dokonalá, by mělo stačit pár minut počkat (i bez pohybu) a "mikrogravitace" kosmonauta sama "odnese" k některé ze stěn stanice.


A proto si myslim ze napriklad ke kotoulu v prostotu (saltu) lze uspesne pouzit i setrvacnost... staci v klidove poloze jen trhnou hlavou dozadu, nebo dopredu...


ales - 5/4/2007 - 13:10

citace:
A proto si myslim ze napriklad ke kotoulu v prostotu (saltu) lze uspesne pouzit i setrvacnost... staci v klidove poloze jen trhnou hlavou dozadu, nebo dopredu...

Tak tohle by podle mne fungovat nemělo. Všechny síly i setrvačnosti by po skončení "trhnutí" měly být stejné, jako na začátku, takže kosmonaut by mohl být v jiné poloze, ale bez pohybu (setrvačného). Případný zbytkový pohyb by měl být spíš důsledkem nevyrovnaného "opření" o okolní vzduch (jako při tom "plavání").

K rotační změně polohy těla by mělo stačit např. kroužit rukama (ve vhodném směru), ale to je něco jiného (než trhnutí).


J2930 - 5/4/2007 - 13:32

citace:
citace:
A proto si myslim ze napriklad ke kotoulu v prostotu (saltu) lze uspesne pouzit i setrvacnost... staci v klidove poloze jen trhnou hlavou dozadu, nebo dopredu...

Tak tohle by podle mne fungovat nemělo....K rotační změně polohy těla by mělo stačit např. kroužit rukama (ve vhodném směru), ale to je něco jiného (než trhnutí).


Hlava asi opravdu stacit nebude je to mala vaha... ale je to zaklad akrobacie. Napriklad na snowboardu kam de hlava (a treba i rameno) tam jde i prkno. Ve vzduchu pri volnem padu staci casto jen ta hlava... nohy se to snazi podvedome nasledovat. Rotace do strany jdou skoro to same jen je dobre si pomoci rukama a muzou byt klidne blizko tela. Proste zmena teziste, posunuti hmotnosti tam kam chci padat...

Mala tryska na boku lodi taky uvede do rotace velky predmet a casto staci jen nepatrny zazeh, vypusteni plynu, odhozeni neceho....


Jirka - 5/4/2007 - 13:37

citace:

Hlava asi opravdu stacit nebude je to mala vaha... ale je to zaklad akrobacie. Napriklad na snowboardu kam de hlava (a treba i rameno) tam jde i prkno. Ve vzduchu pri volnem padu staci casto jen ta hlava...
Mala tryska na boku lodi taky uvede do rotace velky predmet a casto staci jen nepatrny zazeh, vypusteni plynu, odhozeni neceho....


Pokud bys opravdu odhodil hlavu velkou rychlosti do dali, pak by to trupu nepochybne nejakou rotaci a translaci udelilo.

Proto bych kosmonautovi vskutku doporucoval nechat se odnest cirkulaci vzduchu, pockat na pusobeni mikrogravitace ci rizene vypoustet plyny - tedy v tom pripade ze by se nechtel znemoznit "plavanim".


J2930 - 5/4/2007 - 14:10

citace:
Pokud bys opravdu odhodil hlavu velkou rychlosti do dali, pak by to trupu nepochybne nejakou rotaci a translaci udelilo. Proto bych kosmonautovi vskutku doporucoval nechat se odnest cirkulaci vzduchu, pockat na pusobeni mikrogravitace ci rizene vypoustet plyny - tedy v tom pripade ze by se nechtel znemoznit "plavanim".


Zkuste si skocit na prkne tak 10-15 m a uvidite. Zakladem pohybu v kosmu je nedelat prudke pohyby a pokud mouno do niceho nebusit.. zvlaste pak v otevrenem kosmu nepripoutan. A prave ten prudky pohyb ktery je zapovezen zpusobi pomoci setrvacne energie pohyb.. nekde je video kde se kosmonaut drzi za kolena a bez prispeni rukou se roztoci... urcite je

Tahle diskuse tady uz asi probehla... nebo mam dejavu.


J2930 - 5/4/2007 - 14:36

tady mate nejaka srandovni videa s machanim rukama:
http://spaceflight.nasa.gov/gallery/video/living/html/fun.html
a na vzduch to nevypada. hlavne posledni sekundy videa: Members of the STS-98 crew......

taky se o tom psalo nekde v nejake knizce. Leonov? Nevim


ales - 5/4/2007 - 14:42

Diskuze o pohybech v beztíži tu už určitě kdysi probíhala.

Teorie (a logika) říká, že prostá změna těžiště by se měla projevit jen v prostředí s nějakou nerovnovážnou silou (gravitace, proud vzduchu, nebo i proud fotonů). Proto náklon hlavy může fungovat na Zemi, ale v beztíži by se to projevit nemělo.

"Bouchnutí" do něčeho se samozřejmě projeví podobně na Zemi i v beztíži, protože to je přímá kontaktní záležitost (u obou "těles" se změní jejich hybnost).

Samotný prudký pohyb v beztíži by snad neměl vyvolat trvalejší rotaci nebo translaci. Problém je ale v tom, že na stanici je vzduch a prudký pohyb způsobí vlastně "bouchnutí" do okolního vzduchu a to už se projevit může.

Video s roztočením kosmonauta jsem taky viděl, ale myslím, že se to dá vysvětlit interakcí s okolním vzduchem ("pádlování" rukama), rotací části těla (kroužení rukama [dokud krouží, tak se točí]) a nenulovou počáteční rotací (výrazné zvýšení rychlosti rotace skrčením [jako připažení krasobruslaře při piruetě]). To všechno ve zmíněném videu je.

Rád se nechám poučit, pokud jsou moje představy nesprávné. Lze vyvolat trvalejší rotaci těla nějakou jednorázovou změnou polohy těla (z klidu, v beztíži a ve vakuu)? Já myslím, že ne.


Archimedes - 5/4/2007 - 15:08

Verte tomu, ze Newtonovy zakony nejsou pro srandu...

Izolovana soustava nemuze pusobenim svych vlastnich vnitrnich sil posunout svoje celkove teziste ani zmenit svuj celkovy moment hybnosti. Takze se muzete natahnout/skrcit, ale teziste se neposune, muzete roztocit telo, ale jen pokud necim jinym tocite naopak, muzete zrychlit rotaci, ale jen pokud se sbalite, atd...

Kosmonaut unvitr stanice izolovana soustava neni, protoze se jeste porad muze opirat o vzduch. Ovsem kosmonaut bez APU na zadech, vyhozeny volne na orbite do vakua uz se takove izolovane soustave velice blizi.


J2930 - 5/4/2007 - 15:59

citace:
Lze vyvolat trvalejší rotaci těla nějakou jednorázovou změnou polohy těla (z klidu, v beztíži a ve vakuu)? Já myslím, že ne.


Pokud je tam nulova gravitace, nulova hustota okolni hmoty, pak nic nebrani ukonceni pohybu... rotace. Zadne vnejsi sily. Na stanici trvale rotaci brani vzduch - aerodynamickyodpor tela a microgravitace. Na zacatku toho videa leti kosmonaut vzduchem a krome dopredneho pohybu se snazi rotovat telem kolem sve osy. Podobne to dela snowboardista pri nejnovejsim fazovanem stylu skoku - pri skoku a rotacich ze skoku.


J2930 - 5/4/2007 - 16:41

citace:
Verte tomu, ze Newtonovy zakony nejsou pro srandu...


Leti proti sobe nekde mezi Jupiterem a Marsem 2 kosmickou 300 tun tezka druzice a kosmonaut. Co se stane s kosmonautem?


Adolf - 5/4/2007 - 17:17

V beztíži ale uzavřeném prostoru plném vzduchu musí fungovat i při tom plavání docela zajímavé efekty. Když se pohybujeme v tekutině, tvoříme kolem sebe víry. Takový vír nebývá nic slabého. Kdo střílí z luku či kuše, ví, jak zvláštně se dovedou např. chovat šípy. Když šíp letí kolem svahu, tak ho svah jako by odtlačuje. Jde o efekt víru, který má na sobě rotující šíp nabalen a jímž se odtlačuje od svahu. U rychle letící kulky se to neprojevuje, protože víření zůstává daleko vzadu a kulí již vazbu téměř nemá.

Člověk si asi snadno dovede představit, jak by rozšplouchal vodu v bazénku velikosti místnosti někde na kosmické stanici. To vyvolání vln na vodě je ve skutečnosti vyvoláním vírů s horizontální osou. Při hustotě vzduchu musí být snadné vyvolat v něm víry, při čemž je lze dělat tak, že do sebe vír bude akumulovat stále více energie a nikam neuteče – bude se opírat do stěny stanice. O takto roztočené víry, které mají stěnu v zádech, se pak lze jistě docela zřetelně opřít. Pokud jsou víry navíc docela cíleně vyvolávány kvůli homogenizaci vzduchu, musí při plovacích manévrech beztížných suchoplavců vytvářet docela zajímavou strukturu prostředí, kterou lze docela dobře využívat.


Jirka - 5/4/2007 - 17:26

citace:
U rychle letící kulky se to neprojevuje, protože víření zůstává daleko vzadu a kulí již vazbu téměř nemá.


Muj znamy odstrelovac mi cosi naznacoval o deformaci drahy kulky kvuli jeji rotaci, ale nejsem si jisty ze jsem to spravne pochopil a on mi nechtel rict podrobnosti, ze pry by pak na me musel pouzit sveho pritele skorpiona.


Derelict - 5/4/2007 - 17:42

citace:
Leti proti sobe nekde mezi Jupiterem a Marsem 2 kosmickou 300 tun tezka druzice a kosmonaut. Co se stane s kosmonautem?


Pripomina mi to ten vtip:
Vite co prvni proleti hlavickou musce pri narazu na sklo rychle jedouciho automobilu ? ;o))

No predsi jeji zadecek.


Adolf - 5/4/2007 - 17:50

citace:

Muj znamy odstrelovac mi cosi naznacoval o deformaci drahy kulky kvuli jeji rotaci, ale nejsem si jisty ze jsem to spravne pochopil a on mi nechtel rict podrobnosti, ze pry by pak na me musel pouzit sveho pritele skorpiona.


Rotující kulka vlastně letí po šroubovici navinuté na tu zjednodušenou balistickou křivku, co se zpravidla maluje jako její trajektorie. Leč kulka na sebe nenabaluje vír. Co rozvíří, nechává daleko za sebou a zůstává tak mimo dosah svého víru. U šípu je systém vír-šíp velmi úzce svázaný a šíp s blízkými tělesy ve svém okolí s velkým povrchem interaguje jako by na dálku prostřednictvím na sebe nabaleného víru.

Také v letu hmyzu se projevují zajímavé vírové efekty. Všichni slyšeli, že prý čmelák nemá podle zákonů aerodynamiky co létat. Ve skutečnosti to vlastně platí pro veškerý hmyz. Vztlak křídla hmyzu je daleko větší, než jaký má vytvářet jeho profil při obtékání. Jenže hmyz létá jinak. Roztáčí si kolem křídla dost stabilní víry a plave v interakci s cíleně vytvořenými víry.

Myslím, že v kabině za beztíže není kosmonaut odkázán na jednoduché plavání a může docela zajímavě čmelačit investujíc energii svého plovací pohybu do roztáčení užitečných vírů s oporou stěn.

Chtělo by to asi nějakého veterána z ISS, aby nám řekl, zda na stanicích lze fakt pořádně čmelákovat. Tipnul bych si ale že jo.


Adolf - 5/4/2007 - 17:57

citace:

Pripomina mi to ten vtip:
Vite co prvni proleti hlavickou musce pri narazu na sklo rychle jedouciho automobilu ? ;o))

No predsi jeji zadecek.


Pokud by do zadku kosmonauta narazila moucha 2. kosmickou, tak jsem přesvědčen, že mu kromě jiného uletí hlava, nezávisle na směru letu mouchy.


J2930 - 5/4/2007 - 18:21

Modifikace:

Leti nekde mezi Jupiterem a Marsem rychlosti 20 km/h 300 tun tezka druzice a smeruje na nehybneho ziveho kosmonauta. Co se stane s kosmonautem?


ales - 5/4/2007 - 19:03

citace:
Leti nekde mezi Jupiterem a Marsem rychlosti 20 km/h 300 tun tezka druzice a smeruje na nehybneho ziveho kosmonauta. Co se stane s kosmonautem?

Podle mne:
- družice narazí do kosmonauta
- kosmonaut se odrazí (pokud se nezachytí za konstrukci družice)
- nakonec se kosmonaut vzdaluje od družice (zhruba ve směru letu družice, relativní rychlostí pod 20 km/h)
- kosmonaut srážku asi přežije (pokud se mu nepoškodí skafandr)
- (družice pokračuje v letu mírně nižší rychlostí [odhadem o několik m/h = několik mm/s nižší])


Adolf - 5/4/2007 - 19:53

citace:
citace:
Leti nekde mezi Jupiterem a Marsem rychlosti 20 km/h 300 tun tezka druzice a smeruje na nehybneho ziveho kosmonauta. Co se stane s kosmonautem?

Podle mne:
- družice narazí do kosmonauta
- kosmonaut se odrazí (pokud se nezachytí za konstrukci družice)
- nakonec se kosmonaut vzdaluje od družice (zhruba ve směru letu družice, relativní rychlostí pod 20 km/h)
- kosmonaut srážku asi přežije (pokud se mu nepoškodí skafandr)
- (družice pokračuje v letu mírně nižší rychlostí [odhadem o několik m/h = několik mm/s nižší])


Pád z 1,57 m by fakt kosmonaut mohl i přežít. Co by asi obíhala družice touto rychlostí?


J2930 - 5/4/2007 - 20:07

citace:

Podle mne:
- družice narazí do kosmonauta
- kosmonaut se odrazí (pokud se nezachytí za konstrukci družice)
- nakonec se kosmonaut vzdaluje od družice (zhruba ve směru letu družice, relativní rychlostí pod 20 km/h)
- kosmonaut srážku asi přežije (pokud se mu nepoškodí skafandr)
- (družice pokračuje v letu mírně nižší rychlostí [odhadem o několik m/h = několik mm/s nižší])


Do kosmonauta narazi 300 tun ve 20km/h rychlosti. Druzice ma stale 300 tun i kdyz je v kosmu, v prostredi kde zdanlive neni gravitace. Pokracuje dal. Setrvacnost. Kosmonaut se asi moc vzdalovat od druzice nebude, kdyz druzice ze sve rychlosti mnoho neztrati... ma vyssi hmotnost. Nejaka energie se jiste spotrebuje v miste srazky...
ale treba to vsechno chapu blbe. :-)

Lidske telo neni homogeni rovnomerne rozmistena hmota kolem svych os. Nemyslim si ze kosmonaut mimo stanici bez dotyku s nejakym jinym predmetem nema moznost se treba otocit jen za pomoci sveho tela. Neotoci ani trup s hlavou a rukama vuci svym noham? Zkus se otocit stojic na jedne pate...


JuDu - 5/4/2007 - 20:29

Dobry vecer,
Dakujem kazdemu za svoje nazory.
Mam take tusenie, ze odpovede nedavaju presne riesenia ako vyriesit tuto situaciu. Podla mojho nazoru v kozme ak ste bez opory a nemate na dosah nic pevneho, tak mozete maximalne robit rozne pohyby, ale vase tazisko ostava na rovnakom mieste. Je jedno podla ktorej osy sa tocite.
Mozem tuto situaciu vyriesit tak, ze sa vyzleciem do naha a oblecenie+topanky si hodim o stenu ? Impulz to je.

Vdaka.


Martin Kolman - 5/4/2007 - 20:43

Nešlo by třeba vybavit posádku oné hypotetické velkoprostorové stanice malýmy větráčky na baterky ? Pod vodou to u potapěčů funguje a ve stavu beztíže a vzhledem k tomu jak malý odpor má vzduch roti vodě by to mohlo funogavat docela dobře Stačilo by možná velmi malé zařízení, např. velikosti rtěnky se sklopnou vrtulí, jaké používají modeláři na větroních. Pokud by se člověk dostal omylem moc daleko od stěn a nechtělo by se mu čekat až ho odnese proud vzduchu nebo ho někdo "zachrání", vytáhl by vrtulku a nechal se dotáhnout ke zdi


Jano - 5/4/2007 - 20:51

Mám takový vodojem, že pán Adolf dnes trochu zavíril túto debatu

>>"Vztlak křídla hmyzu je daleko větší, než jaký má vytvářet jeho profil při obtékání. Jenže hmyz létá jinak. Roztáčí si kolem křídla dost stabilní víry a plave v interakci s cíleně vytvořenými víry."

To síce nevylučujem, ale možno je to oveľa jednoduchšie (slovo vír evokuje rotáciu vzduchu; ja skôr predpokladám, že prevažuje jeho posuvný pohyb):
Hmyz, jednoducho, mávaním krídel ženie vzduch nadol, a tým pádom seba tlačí nahor. Pritom sila pri pohybe krídla nadol musí byť väčšia, než pri pohybe späť nahor. To sa dá dosiahnuť napr. tým, že počas mávnutia nadol má krídlo väčšiu rýchlosť a/alebo väčší koef.odporu. Čo vy na to?
Nebolo by od veci nájsť nejaké rýchlobežno-nakrútené zábery letiaceho hmyzu...


Derelict - 5/4/2007 - 21:40

citace:
Mám takový vodojem, že pán Adolf dnes trochu zavíril túto debatu
>>"Vztlak křídla hmyzu je daleko větší, než jaký má vytvářet jeho profil při obtékání. Jenže hmyz létá jinak. Roztáčí si kolem křídla dost stabilní víry a plave v interakci s cíleně vytvořenými víry."



Napsal to dobre. Hmyz pouziva dost specificky zpusob mavani na rozdil od ptaku. Jednak dochazi k naklapeni kridel, konce kridel mohou opisovat osmicky, elipsy, povrch kridla u nektereho hmyzu dokaze reagovat ...
Viry, lepe receno turbulence vznikaji u jakehokoliv pohybu v plynnem nebo kapalnem prostredi. Cim mensi je pomer objektu k turbulenci, tim lepe muze vyuzit jeji energii. Kdyz to prezene, muze ho semlet ;o)


ales - 5/4/2007 - 22:55

citace:
Mam take tusenie, ze odpovede nedavaju presne riesenia ako vyriesit tuto situaciu. Podla mojho nazoru v kozme ak ste bez opory a nemate na dosah nic pevneho, tak mozete maximalne robit rozne pohyby, ale vase tazisko ostava na rovnakom mieste. Je jedno podla ktorej osy sa tocite.
Mozem tuto situaciu vyriesit tak, ze sa vyzleciem do naha a oblecenie+topanky si hodim o stenu ? Impulz to je.

To, co popisuješ, by platilo ve vakuu a dokonalé beztíži. Ve vzduchu na stanici by zcela určitě fungovalo popisované "plavání" a to i kdyby byl kosmonaut nahý a neměl co odhodit. Pokud by mohl něco odhodit, tak by to samozřejmě fungovalo spolehlivě taky (a ani by to odhozené nemuselo narazit do stěny stanice). Řešení jsou tedy, myslím, jasná (usilovně "plavat" okolním vzduchem, nebo co největší rychlostí odhodit něco většího [těžšího] ze svých svršků).


Ervé - 6/4/2007 - 07:24

Když jsem tak u těch svršků: Pokud vím, tak na palubách všech dosud vypuštěných stanic chybělo důležité zařízení - pračka. Kosmonauti a astronauti se převlékají vždy do čistého. Například pro let na Mars je to ale hodně nevýhodné, testovala se nebo se plánuje testování praček pro stav beztíže? Nebo mi něco ušlo a už se používají?


ober - 6/4/2007 - 07:30

a co treba pouzit obycejne dychani, s tim ze nadech delam s hlavou otocenou na jednu stranu a vydech (resp. rychle vyfouknuti vzduchu z plic) s hlavou otocenou na opacnou stranu, samozrejme plati jen v pripade, ze nejsem "zaizolovany" ve skafandru


Michal Pavelka - 6/4/2007 - 08:53

Dobrý den,

omlouvám se, že Vás vyrušuji z této diskuse, ale mě zajímá jiná věc.

Jelikož jsem zdravotně postižený, tak mě zajímá, jestli nevíte, aspoň přibližně, kolik pracuje v NASA či ESA zdravotně postižených lidí? Popřípadě zda, některá z org., se nechystá "vypustit" takového člověka.

Díky


J2930 - 6/4/2007 - 12:53

citace:
Dobrý den,

omlouvám se, že Vás vyrušuji z této diskuse, ale mě zajímá jiná věc.

Jelikož jsem zdravotně postižený, tak mě zajímá, jestli nevíte, aspoň přibližně, kolik pracuje v NASA či ESA zdravotně postižených lidí? Popřípadě zda, některá z org., se nechystá "vypustit" takového člověka.

Díky


Pocty bude vedet asi jiny specialista. Ja vim jen ze Stephen Hawking by mnel letet na Space Ship Two. Ovsem to je jen skok na hranici vesmiru (suborbital). Jeste predtim by mnel ablolvovat parabolicke lety. Tedy kratkodobe stavy beztize v utrobach skoro obycejneho letadla...
napr.: http://www.spaceadventures.com/index.cfm?fuseaction=Other_Spaceflight_Experiences.Zero_G


Jirka - 6/4/2007 - 13:33

citace:
jestli nevíte, aspoň přibližně, kolik pracuje v NASA či ESA zdravotně postižených lidí? Popřípadě zda, některá z org., se nechystá "vypustit" takového člověka.


Pokud se zde mysli prace astronauta nebo kosmonauta, tak vzhledem k soucasnym vysokym pozadavkum na fyzickou kondici maji malou sanci i prumerne zdravi lide.
Hawkins je pravdepodobne protezovan kvuli reklame. I kratky suborbitalni let vsak pro nej predstavuje zdravotni riziko mnohem vetsi nez pro fyzicky dokonale zdraveho cloveka.
Prace v kancelari u pocitace samozrejme dovoluje plnohodnotnou praci i pro (telesne) hendikepovane lidi.


J2930 - 6/4/2007 - 15:05

citace:
Hawkins je pravdepodobne protezovan kvuli reklame...


Neni nijak protezovan, sam to chce. Vymyslel si to sam, nikdo z zadnou nabidkou za nim neprisel.


Adolf - 6/4/2007 - 15:49

citace:
Mám takový vodojem, že pán Adolf dnes trochu zavíril túto debatu

>>"Vztlak křídla hmyzu je daleko větší, než jaký má vytvářet jeho profil při obtékání. Jenže hmyz létá jinak. Roztáčí si kolem křídla dost stabilní víry a plave v interakci s cíleně vytvořenými víry."

To síce nevylučujem, ale možno je to oveľa jednoduchšie (slovo vír evokuje rotáciu vzduchu; ja skôr predpokladám, že prevažuje jeho posuvný pohyb):
Hmyz, jednoducho, mávaním krídel ženie vzduch nadol, a tým pádom seba tlačí nahor. Pritom sila pri pohybe krídla nadol musí byť väčšia, než pri pohybe späť nahor. To sa dá dosiahnuť napr. tým, že počas mávnutia nadol má krídlo väčšiu rýchlosť a/alebo väčší koef.odporu. Čo vy na to?
Nebolo by od veci nájsť nejaké rýchlobežno-nakrútené zábery letiaceho hmyzu...


Slovo vír nejenže evokuje rotaci vzduchu, ale o rotaci vzduchu skutečně jde. Tedy je to přesněji taková fraktálové víření z vírků poskládaných vírů ve vírech - turbulenci - jako ostatně vždy, když jde o víry, jak správně podotkl myslím Jan Dusátko. Hlavně jde při tom snad o vír nad křídlem, který zkouší toho hmyza nasát - tedy vytváří nad ním podtlak.

U víru funguje takový efekt, že masa rotující tekutiny na sebe nabaluje "vlákna" další tekutiny jak klubíčko. Zamíchám tedy trochou tekutiny a nabalím na ni velké kolo. Roztáčím tedy celou tuto velkou hmotu a interatuji s ní, s jejím rotačním momentem hybnosti a kinetickou energií tohoto velkého rototu. Mohu se o ní tedy pořádně "opřít". I okolní vzdušné proudy interagují s celým tímto vírem a opírajíse o něj jak o velké "křídlo". Z řízeného víru si tedy vyrobím jakousi další velkou součástku "hmyzo-letounu". Např. takový "naviják" vzduchu, který mě za nabalující se vzduchová vlákna vytahuje nahoru a absorbuje další a další moji energii. Efektivně při tom interaguji s daleko větší vzduchovou hmotou než u prostého obtékání profilu.

Tohohle využívá nejen hmyz. Existuje také takový zvláštní princip plachetnice - ta nemá jen pasivní plachtu, ale jako plachta funguje svislý rotující válec. Válec rotuje velice rychle - proto s sebou strhává dost rychle i okolní vzduch a nabaluje ho do docela málo turbulentního víru. Protože jádro víru je rychlé, nabalování vírového "klubka" je velice efektivní a vír se vytvoří velmi veliký. Potom se ale vzduch neopírá jen o průřez plachty, ale do velké míry o celý ten gigantický vír - virtuální plachtu.

Samožřejmě, že u maličkých "letounků" jako hmyz s výhodnými poměry povrchu a objemu atp. se tahle forma aeronomiky využije snáz než u rozměrů velkých létajících objektů.

Teď momentálně po ruce bohužel žádnou animaci hmyzích křídel nemám.

Snad si později vzpomenu a po něčem se poohlédnu.


Adolf - 6/4/2007 - 15:55

Já bych v té kosmické lodi s velkými prostory asi všude natahal spleť záchytných provazů, aby člověk vždy na nějaký dosáhl, než abych přidělával na kosmonauty vrtulku. Nevypadá to tak věděcky, ale funkční by to jistě bylo. I ve tmě by to mohlo posloužit k nouzové orientaci a zároveň by v tom mohla být tažena některá kabeláž.


J2930 - 6/4/2007 - 16:42

citace:
Já bych v té kosmické lodi s velkými prostory asi všude natahal spleť záchytných provazů..


okolik by se zvedla hmotnost takove stanice? Tahat za kabely!!! No fuj.


J2930 - 6/4/2007 - 16:43

citace:
Existuje také takový zvláštní princip plachetnice - ta nemá jen pasivní plachtu, ale jako plachta funguje svislý rotující válec


Jako pokus to bylo i na jednom tankeru.


petr - 6/4/2007 - 16:55

citace:
Já bych v té kosmické lodi s velkými prostory ...


... to ještě bude trvat, než budou lodě táááááák veliké, že v dosahu obyvatele nebude nic uchycení/odražení vhodného!!!


Lukavský - 6/4/2007 - 18:01

citace:
a co treba pouzit obycejne dychani, s tim ze nadech delam s hlavou otocenou na jednu stranu a vydech (resp. rychle vyfouknuti vzduchu z plic) s hlavou otocenou na opacnou stranu, samozrejme plati jen v pripade, ze nejsem "zaizolovany" ve skafandru

Já bych si uvnitř lodi se vzduchem tipl na tento reaktivní pohyb. Na podobném principu se pohybují sépie a chobotnice (samozřejmě k tomu používají vodu). Není asi ani třeba otáčet hlavou, stačí pomalu nadechnout a pak vzduch prudce vyfouknout. Zkuste si fouknout do dlaně nebo odfouknout papíry. Myslím že by to i 100 kg kosmonauta uvedlo do pomalého pohybu.


martalien - 6/4/2007 - 18:23

citace:
Není asi ani třeba otáčet hlavou, stačí pomalu nadechnout a pak vzduch prudce vyfouknout


Akorat musite foukat tak aby vektor tahu prochazel Vasim tezistem. V opacnem pripade budete zadychany a bezmocne rotujici kolem sveho teziste..


Adolf - 6/4/2007 - 19:42

citace:

okolik by se zvedla hmotnost takove stanice? Tahat za kabely!!! No fuj.


Dnes v době kevlaru, kompozitů na bázi uhlíkového vlákna aj. by nemuselo jít nutně o vyřazené svářečské kabely. Mohlo by jít o velmi jemná a lehká vlákna utkaná do kšandy, aby byla pro lidskou ruku ergonomičtější.


Adolf - 6/4/2007 - 19:48

citace:

Akorat musite foukat tak aby vektor tahu prochazel Vasim tezistem. V opacnem pripade budete zadychany a bezmocne rotujici kolem sveho teziste..


Myslím, že by nezůstalo u rotace na místě, nýbrž že by tím došlo ke kombinaci s plaváním, takže poté, co by hlava s funícím nosem či našpulenou pusou opsala několik oblouků nějakého typu cykloidy, dofuněl by se k nějaké stěně.


Jano - 6/4/2007 - 20:02

Predmet veľkej plochy, použitý ako krídlo resp. veslo, t.j. na opieranie sa o vzduch, niečo ako majú baleríny tancujúce na lane. Niektorým zverom zas narástli veľké blany medzi pažami a trupom a používajú to pri skákaní/plachtení medzi stromami. No a vieme, aký jazyk má chameleón - podobná vystreľovacia vec by mohla poslúžiť aj na pritiahnutie sa k niektorej stene.


JuDu - 20/4/2007 - 12:46

Dobry den,

Moze mi niekto jednoducho vysvetlit, ako urcim, ako vysoko mozem/mam umiestnim satelit okolo Zeme ? Mozem 500 Kg druzicu dat na orbit vo vyske 1000/2000/3000 Km, alebo co urcuje hornu a dolnu hranicu moznej vysky ? Preco je stanica ISS umiestnena prave vo vyske cca. 380 Km ? Ak by som mal nejaky silny tahac, tak to mozem dopravit do dvojnasobnej vysky (samozrejme potom bude aj o x percent dlhsia doba obehu) ? Da sa nejakym raketovym motorom satelit tak urychlit, ze bude kruzit vo vyske 36000 km nad Zemou a bude mat obeznu dobu 1/2 dna ?

Vdaka


Jano - 20/4/2007 - 13:50

"Da sa nejakym raketovym motorom satelit tak urychlit, ze bude kruzit vo vyske 36000 km nad Zemou a bude mat obeznu dobu 1/2 dna?"

Tej výške zodpovedá prirodzená obežná dobe 1 deň (alebo 23:56). Ak by to mal stihnúť za pol dňa, potreboval by 4-násobné dostredivé zrýchlenie, t.j. by ho ten motor musel (nepretržite, stále) tlačiť dolu 3-násobkom jeho tamojšej váhy (t.j. so zrýchlením asi 0,07g , ak dobre počítam).
Ostatné otázky snáď zodpovedia iní.


ales - 20/4/2007 - 14:27

Pokusím se odpovědět, jak to chápu já.

> ako urcim, ako vysoko mozem/mam umiestnim satelit okolo Zeme ?
> Mozem 500 Kg druzicu dat na orbit vo vyske 1000/2000/3000 Km, alebo co urcuje hornu a dolnu hranicu moznej vysky ?

Horní hranice teoreticky samozřejmě není omezena, protože družici/sondu lze urychlit až na meziplanetární dráhu. Horní hranice použitelných stabilních oběžných drah kolem Země je někde ve výši řádově 200000 km, protože výše už se silně projevuje rušivý vliv Měsíce a dráha se stává nestabilní ("samovolně" se mění).

Dolní hranice je dána aerodynamickým odporem atmosféry. Čím níže, tím je hustota atmosféry vyšší a tím roste i aerodynamické brzdění družice. Minimum je někde ve výši cca 200 km protože pod touto hranicí by se už družice udržela jen pár oběhů (bez motorického manévrování) a pak by "spadla" na Zemi.

> Preco je stanica ISS umiestnena prave vo vyske cca. 380 Km ?

Vyšší dráhy jsou hůře dosažitelné (je třeba větší množsví paliva pro start) a hlavně s výškou dráhy rychle roste okolní radiace. Ve výškách pod 500 km je radiace ještě přijatelná, ale např. ve výškách 2000 - 10000 km jsou tzv. Van-Allenovy radiační pásy, kde je radiace mnohonásobně vyšší (myslím, že proto, že zde magnetosféra Země zachytává většinu nabitých částic z kosmu). Ve výškách pod 300 km už zase příliš roste spotřeba paliva na kompenzaci aerodynamického brzdění hustšími vrstvami atmosféry (i ISS se několikrát za rok motoricky "postrkuje" na vyšší dráhu). Proto je zvolená dráha ISS kompromisem mezi výše uvedenými protichůdnými potřebami a bývá ve výši 350 - 400 km.

> Ak by som mal nejaky silny tahac, tak to mozem dopravit do dvojnasobnej vysky (samozrejme potom bude aj o x percent dlhsia doba obehu) ?

Samozřejmě to lze. Ale roste výše zmíněná nebezpečná radiace a také spotřeba paliva na přílet ze Země (i odlet k Zemi).

> Da sa nejakym raketovym motorom satelit tak urychlit, ze bude kruzit vo vyske 36000 km nad Zemou a bude mat obeznu dobu 1/2 dna ?

Kruhová dráha ve výši 36000 km nad povrchem Země má oběžnou dobu cca 1 den. (Proč by měla mít oběžnou dobu půl dne?)
Této skutečnosti využívají geostacionární družice, jejichž dráha vede přesně nad rovníkem ve výši cca 36000 km (východním směrem) a protože oběžná doba je právě jeden den, tak zdánlivě "stojí" nad jedním bodem rovníku (a například telekomunikační antény mohou být stále nasměrovány jen jedním směrem).


ales - 20/4/2007 - 15:36

> ako urcim, ako vysoko mozem/mam umiestnim satelit okolo Zeme ?

Ještě jsem zapoměl napsat, že dráha družice se volí především podle účelu družice (telekomunikace, navigace, meteorologie, snímkování Země, orbitální stanice, ...). Každý účel má jiné nároky na oběžnou dráhu. Podle zvolené dráhy se pak postaví družice tak aby příslušné podmínky vydržela a splnila (radiace, osvětlení Sluncem, dosah radiokomunikace, manévrovací systém, příkon ze solárních článků, ...). Také nosič se musí vybrat tak, aby byl schopen dostat družici na zvolenou dráhu (v kombinaci s pohonem vlastní družice).


JuDu - 20/4/2007 - 20:16

Dobry vecer,
dakujem kazdemu za odpoved, viem ze draha geocentralna je vo vyske okolo 36000 Km. Prave preto chcem, aby v takej vyske nebola obezna doba druzice 24 hodin, ale 'len pre moju zvedavost' polovica tohto. Kam asi treba smerovat vektor tahu motoru ? Ak neratam obrovske mrhanie energie, je to mozne ?
Dakujem.


ales - 20/4/2007 - 21:51

citace:
... viem ze draha geocentralna je vo vyske okolo 36000 Km. Prave preto chcem, aby v takej vyske nebola obezna doba druzice 24 hodin, ale 'len pre moju zvedavost' polovica tohto. Kam asi treba smerovat vektor tahu motoru ? Ak neratam obrovske mrhanie energie, je to mozne ?
Teoreticky je to možné a pěkně to tu už popsal Jano.

Pokud je na začátku družice na geostacionární dráze (tedy letí východním směrem ve výši cca 36000 km na rovníkem rychlostí cca 3 km/s), pak by bylo nejprve třeba zvýšit okamžitou rychlost na dvojnásobek (aby oběžná doba byla poloviční), tedy na cca 6 km/s. Protože odstředivá síla roste s druhou mocninou oběžné rychlosti (F=m.v^2/r), tak by tato síla vzrostla na čtyřnásobek původní hodnoty. Aby družice zústala ve stále stejné výšce, musela by ty tři "nadbytečné" jednotky odstředivé síly trvale kompenzovat (vyrovnávat) motoricky. Tah "motoru" by tedy musel směřovat stále přesně do středu Země (kolmo ke směru letu). Klasický raketový motor by tedy musel mít trysku namířenu přesně "od Země". Velikost odstředivého zrychlení a=v^2/r je pro geostacionární dráhu cca a=3000^2/42000000=0,215 m/s^2 ("r" je třeba počítat od středu Země). Motorický systém družice by pak musel vyvinout trojnásobné zrychlení, tedy cca 0,65 m/s^2, což je těch cca 0,07g jak uvedl Jano. Za jediný den (24 hodin, cca 2 oběhy) by tak motorický systém musel být schopen dodat družici rychlost na úrovní delta_V cca 56 km/s, což je pro současné pohony zatím prakticky nedosažitelné.

Snad se někde nepletu. Omlouvám se za "nižší srozumitelnost" mého příspěvku.
[Upraveno 20.4.2007 poslal ales]


Jano - 20/4/2007 - 22:15

Dúfam, že raz sa podarí skonštruovať iónové motory, z ktorých budú "spaliny" vylietať rýchlosťou na úrovni stotín až desatín c (napr. v CRT obrazovke sa elektróny pohybujú pri urýchľovacom napätí rádovo 10kV rýchlosťou asi 0,2 c). V takom prípade by sa snáď aj uvedené delta_v dali dosahovať aj s relatívne malou spotrebou hmoty.

Dokonca ma teraz napadla idea "medziplanetárneho náporového motora" (ak nie náporového, tak zberačského). Raketa letiaca priestorom by zberala hmotu a tú používala ako palivo (ionizovať, vystreliť) namiesto aby si musela viezť veľké zásoby so sebou. Počul už niekto o niečom podobnom alebo je to úplná novinka (prípadne úplná blbosť ) ?

Ešte späť k JuDuovej téme: ak už plytvať, tak ako trochu užitočnejšie by mi pripadalo napr. zavesenie geostacionárnej družice niekde nad 10-15 stupeň šírky (pre používateľov z polárnych oblastí, pre ktorých sú bežné rovníkové g.s.družice clonené obzorom). V takom prípade by musel motor prekonávať snahu gravitácie ťahať družicu k rovníku a potrebná sila (rovnobežná s osou Zeme) by bola len na úrovni asi 0,005 g.


ales - 20/4/2007 - 22:37

> ... idea "medziplanetárneho náporového motora" ... je to úplná novinka (prípadne úplná blbosť) ?

Ano, už jsem někde četl úvahy o podobných pohonech (urychlujících mezihvězdnou hmotu). Takže to není úplná novinka, a určitě ani blbost.

> ... trochu užitočnejšie by mi pripadalo napr. zavesenie geostacionárnej družice niekde nad 10-15 stupeň šírky (pre používateľov z polárnych oblastí, pre ktorých sú bežné rovníkové g.s.družice clonené obzorom). V takom prípade by musel motor prekonávať snahu gravitácie ťahať družicu k rovníku a potrebná sila (rovnobežná s osou Zeme) by bola len na úrovni asi 0,005 g.

Tyto úvahy už jsem také někde viděl (asi v nějaké knize) a teoreticky by to snad mohly zvládnout některé druhy fyzikálních pohonů bez paliva, jako např. elektrodynamický tether, nebo sluneční plachetnice. Ovšem nároky na potřebnou kvalitu konstrukce jsou zatím také nedosažitelné (poměr příkon/hmotnost nebo plocha/hmotnost).


Wartex - 21/4/2007 - 06:34

S dostatecne silnym tahem po neomezenou dobu lze samozrejme v gravitacnim poli Zeme manevrovat jakkoliv.

Vzhledem k tomu, ze takovy pohon nemame, pohybuji se satelity krome faze navedeni na drahu a pripadnych pozdejich korekci zcela pasivne. Jejich drahy (a casovy vyvoj techto drah v case) je potom kompletne urcen vnejsimi podminkami, jako je (podle vyznamnosti) grav. pole Zeme a jeho nehomogenity, odpor zbytkove atmosfery, vliv Mesice, Slunce, Jupitera, tlak slunecniho zareni (pro satelity placate jako plachta), interakce s magnetickym polem ...

Kdyz vse zjednodusime na ciste Keplerovsky pohyb v idealne tvarovanem centralnim grav. poli bez ostatnich vlivu, pak jsou parametry zcela urceny hmotou Zeme (eventualne, pokud vas zajimaji i cifry na 150. miste za desetinnou teckou, i hmotou druzice). Z vysky drahy pak jednoznacne a nemenne vyplyva obezna doba, naveky kostantni presny pohyb po elipse (resp. kuzelosecce).

Je to stejne, jako polozit si otazku, zda je mozno vystrelit naboj z dela pod uhlem 45 stupnu, aby doletel 10km, ale dopadl za 2 sekundy. To je vzajemne v rozporu a pokud by naboj nemel vlastni (velmi silny) manevrovaci motor, tak to neni mozne.

Jake by presne musely byt parametry onoho hypotetickeho manevrovaciho motoru, a jak by musel byt v jakem okamziku smerovan jeho tah, aby bylo dosazeno libovolne trajektorie, uz nastinil kolega vyse.

Jednou to snad bude mozne a koraby ze StarWars se stanou realnymi :-)


Derelict - 21/4/2007 - 07:38

citace:
> ... idea "medziplanetárneho náporového motora" ... je to úplná novinka (prípadne úplná blbosť) ?

Ano, už jsem někde četl úvahy o podobných pohonech (urychlujících mezihvězdnou hmotu). Takže to není úplná novinka, a určitě ani blbost.



Ano, tyto a dalsi motory byly rozpracovavany jako teoreticke moznosti - viz napr. http://en.wikipedia.org/wiki/Bussard_ramjet " target=_blank> http://en.wikipedia.org/wiki/Bussard_ramjet Urcita vylepseni by bylo nepouzivat mechanicky sberac, ale pouze elektromagneticke pole (dalsi vylepseni je pouzit pouze jeho cast, teoreticky pomoci hypoteticke latky - magnetickych monopolu), nebo pole gravitacni, ale to uz je prilis velka fantazie.


Jano - 21/4/2007 - 14:29

>>en.wikipedia.org/wiki/Bussard_ramjet
Hehe, človeka vždy poteší, keď zistí, že (nezávisle) vymyslel niečo, na čo prišli aj múdre hlavy

>>Jednou to snad bude mozne a koraby ze StarWars se stanou realnymi
No, dúfam, že niektoré scifi-veci sa radšej nestanú reálnymi. Napr. cestovanie v čase alebo nadsvetelnou rýchlosťou - bol by z toho chaos.


Paldík - 21/4/2007 - 18:22

Ne Star Wars, ale STAR TREK!


Wartex - 22/4/2007 - 14:28

citace:
Ne Star Wars, ale STAR TREK!


O StarTreku nebyla rec. Tam z celeho cestovani zbyl jen warp-pohon a misto manevrovani blizko u planet "paprsek", eventualne mindfake v simulatoru.

Rec byla o Star Wars, kde vsechny lodi manevruji zcela evidentne BEZ jakehokoliv ohledu na vnejsi gravitacni pole. Leti proste kam chteji, a kdy chteji, na setrvacnost kaslou, atmosferou prochazeji rychlosti, jakou uznaji za vhodnou ... atd. atd.

Pohon, ktery dokaze sam sobe DLOUHODOBE udelit u Zeme zrychleni alespon kousicek pres 1g (aby zbylo aspon na malinky payload) ... to je to, oc tu bezi.


Derelict - 22/4/2007 - 15:24

citace:

Pohon, ktery dokaze sam sobe DLOUHODOBE udelit u Zeme zrychleni alespon kousicek pres 1g (aby zbylo aspon na malinky payload) ... to je to, oc tu bezi.


Predevsim - pri te nejmene namahave moznosti. Co takhle vytvorit pohon, ktery nebude mit nijak extremne velky vykon, ale bude dlouhodobe staly. Jenom aby stacil na prekonani areodynamickeho odporu. K tomu kridla a zacit krouzit okolo Zeme, dokud se nedosahne unikove rychlosti. Jestli to dobre vidim, je to neprakticky, ale teoreticky pouzitelny napad, lze na to vyuzit nejakou variaci jiz zmineneho Bussardova sberace. Problemy by byly shodne jako pri konstrukci scramjetu, energeticka narocnost vyssi, hlavne problemy s odvodem tepla. Docela by mne zajimaly pozadovane vztahy - rychlost, velikost nosne plochy, areodynamicky odpor a pozadovany vykon motoru ;o)))


Wartex - 22/4/2007 - 17:48

citace:
K tomu kridla a zacit krouzit okolo Zeme, dokud se nedosahne unikove rychlosti.


Prislusne vypocty urcite nejsou nezajimave. Muselo by to byt skutecne nevsedni letadlo, aby dokazalo pruzne menit svuj tvar od buclatych velkych kridel pro pomaly rozjezd v huste atmosfere, po spicaty tvar a sevrene deltakridlo pri prekonani rychlosti zvuku ve vysve jednotek kilometru, pres waverider pri prechodu na hypersonickou rychlost ve vysce kolem 20km.

Tam by se ovsem dostalo mile letadlo do lehkeho rozporu, jak udrzet vztlak a pritom se neprehrat, a zaroven nerozlamat dragem. Prislusne simulace a varianty, spolu s pozadavkem na pevnost konstrukce by me take zajimaly :-)

a mozna, ze se do nich skutecne pustim ;-)


Dodor - 22/4/2007 - 18:16

Ty návrhy už i existuji Orbiter Airship ATO

http://en.wikipedia.org/wiki/Orbital_airship

http://www.jpaerospace.com/


Derelict - 22/4/2007 - 18:20

citace:
citace:
K tomu kridla a zacit krouzit okolo Zeme, dokud se nedosahne unikove rychlosti.


Tam by se ovsem dostalo mile letadlo do lehkeho rozporu, jak udrzet vztlak a pritom se neprehrat, a zaroven nerozlamat dragem. Prislusne simulace a varianty, spolu s pozadavkem na pevnost konstrukce by me take zajimaly :-)

a mozna, ze se do nich skutecne pustim ;-)


Hmm... co takhle spoluprace ;o) Zmena kridla a charakteristiky v podzvukove a nadzvukove oblasti by se daly ustat, horsi je to s hyporsonickou rychlosti. To co by mne ale skutecne zajimalo by byl motor. Takze jsou tu tri zakladni problemy:

1. Motor
2. Kridlo s menitelnou geometrii (a soucasna menitelna geometrie je proti tomu odpoledni cajicek)
3. Mechanicke a fyzikalni vlastnosti materialu tohoto zarizeni

Dale je tu jeden velice zajimavy problem, kterym to zacina - profil drahy, charakteristiky namahani atd.


Wartex - 22/4/2007 - 19:28

[Quote]Hmm... co takhle spoluprace ;o)




Je to zajimavy problem, ale priznejme si, bez vyhledu na realny vysledek.

Mate-li prebytek volneho casu (unlike me) a prebytek chuti neco udelat, pojdte s nami spolupracovat na czCube ... lidi je malo, prace hodne a vysledek o dost realistictejsi ;-)


Adolf - 22/4/2007 - 19:43

citace:

Hmm... co takhle spoluprace ;o) Zmena kridla a charakteristiky v podzvukove a nadzvukove oblasti by se daly ustat, horsi je to s hyporsonickou rychlosti. To co by mne ale skutecne zajimalo by byl motor. Takze jsou tu tri zakladni problemy:

1. Motor
2. Kridlo s menitelnou geometrii (a soucasna menitelna geometrie je proti tomu odpoledni cajicek)
3. Mechanicke a fyzikalni vlastnosti materialu tohoto zarizeni

Dale je tu jeden velice zajimavy problem, kterym to zacina - profil drahy, charakteristiky namahani atd.



Mám obavy, že takto vznikaly takové ty zaručené návody na pokrok jako zadání pro raketoplán.


Derelict - 22/4/2007 - 19:55

citace:

Mám obavy, že takto vznikaly takové ty zaručené návody na pokrok jako zadání pro raketoplán.


Ktery ? STS nebo komplex Buran/Energia ?
Raketoplan nemam rad, je to prilis komplikovany stroj. Ale diky nemu jsou dnes docela zajimave vysledky. Za par desitek let se k nim mozna vratime, podstatne chytrejsi prave diky tezce nabytym znalostem. Vyvoj jde ve vlnach - chvili rakety, chvili letadla ... Kdo vi co prijde.
Raketoplan je takovy Jara Cimrman ... mistr slepych ulicek.


Adolf - 23/4/2007 - 16:29

citace:

Ktery ? STS nebo komplex Buran/Energia ?



STS! Buran byl reakcí na STS a jakž takž zavčas z něj moudře vycouvali.

Jednou možná budou raketoplány, jednou možná budou i ty nepředstavitelné obludy splňující nároky na celou tu škálu letových prostředků pro různé rychlosti a vrstvy atmosféry, které se tu diskutovaly. Zatím je to ale víc nákladné technické eskamotérství než technicky a ekonomicky optimalizované řešení.


Derelict - 23/4/2007 - 22:43

citace:
citace:

Ktery ? STS nebo komplex Buran/Energia ?



STS! Buran byl reakcí na STS a jakž takž zavčas z něj moudře vycouvali.

Jednou možná budou raketoplány, jednou možná budou i ty nepředstavitelné obludy splňující nároky na celou tu škálu letových prostředků pro různé rychlosti a vrstvy atmosféry, které se tu diskutovaly. Zatím je to ale víc nákladné technické eskamotérství než technicky a ekonomicky optimalizované řešení.


Nemohu si pomoci, ale komplex Buran/Energia na mne delal lepsi dojem ;o))) Skoda ho, zvlaste te tezke rakety.


Jano - 24/4/2007 - 23:56

Laická otázka:
Bol už niekedy človek na retrográdnej obežnej dráhe? Alebo na polárnej (mohol sa na vlastné oči pozrieť zhora na Ant/Arktídu)? Resp., počas doterajších pilotovaných letov aká bola najväčšia inklinácia?
Ďakujem.


Luděk - 25/4/2007 - 07:50

citace:
Laická otázka: Bol už niekedy človek na retrográdnej obežnej dráhe? Alebo na polárnej (mohol sa na vlastné oči pozrieť zhora na Ant/Arktídu)? Resp., počas doterajších pilotovaných letov aká bola najväčšia inklinácia? Ďakujem.




Největší inklinace byla při letu Vostok-5 - V. Bykovskij, 68° (1963). Američané při letu Atlantis STS-36 v roce 1990 létali na dráze 62°. Ti mají také nejmenší inklinaci, 28°. Číňané létají na 42°, Saljuty, Mir a ISS na 52°, Skylab létal na 50°. Na retrográdní nebo polární dráze žádný pilotovaný let neproběhl.


Ervé - 26/4/2007 - 15:27

Retrográdní dráhy jsou ideální pro DPZ jak civilní, tak i vojenské. Jenomže tohle zvládnou líp družice, takže lety s posádkou nejsou potřebné. Když STS-59 mapoval Zemi radarem, létal na dráze se sklonem 57°, takže polární oblasti byly mimo dosah. Na dráhu s větším sklonem než 62° nemůžou z KSC startovat kvůli dopadu SRB a nádrže do obydlených oblastí.


avitek - 27/4/2007 - 10:35

Před havárií Challengeru se proto počítalo se starty raketoplánů také ze základny Vandenberg AFB (Western Test Range) v Kalifornii na polární dráhy. Byla tam poslavena rampa SLC-6, která se nyní používá pro některé komerční nosné rakety (samozřejmě po úpravách).

Co se týče retrográdních drah: S výjimkou Izraele, u kterého je to nutnost, se na výrazně retrográdní rakety nic nevypouští. Meteorologické polární družice, jako jsou americké družice - civilní NOAA a vojenské DMSP - a ruské družice řady Meteor nebo čínské FY - jsou sice vypouštěny na retrográdní dráhy, ale se sklonem pouze cca. 97 stupňů k rovníku. Stejně tak jsou na tyto dráhy vypouštěny družice pro dálkový průzkum Země (DPZ). Vtip těchto jen o pouhých 7 stupňů retrográdních drah je v tom, že jejich rovina zachovává vůči Slunci prakticky stále stejnou orientaci (stačí jen drobné korekce); proto se těmto drahám říká heliosynchronní. Díky tomu snímkují povrch Země 2x za den, vždy za stejného osvětlení terénu pod sebou (např. dopoledne a pak o 12 hodin později po západu Slunce před půlnocí). Jednou přitom přelétají pozorované místo od severu na jih, o půl dne později opačným směrem.

Izraelci musejí střílet nosné rakety jen směrem na západ, nad Středozemní moře, jinak by jim stupně nosných raket padaly na území arabských sousedů a to každému dojde, že není možné - sousedi by si to mohli vykládat jako raketový útok. Proto družice Offeq mají sklon přibližně 143,5 stupně, což je vlastně sklon 36,5 stupně, ale proti směru otáčení zeměkoule.


JuDu - 30/4/2007 - 12:34

Ahojte,

V suvislosti s problemom 'vesmirneho smetia' ma napadla otazka: ak je dopredu dane, na akej obeznej drahe aku rychlost musi mat vesmirne teleso a viac ako 99% satelitov ma rovnaky smer obehu, tak preco su vzajomne rychlosti take velke ?
Mozem ja mat v rovnakej vyske dve satelity s roznou dobou obehu ? Podla mojho nazoru, ak nemam zdroj pohonu, tak asi nie, ze ?
Problem je, ze niektore satelity nekruzia po kruhovej drahe ale po eliptickej, nie ?

Dakujem


Dodor - 30/4/2007 - 12:49

citace:

Problem je, ze niektore satelity nekruzia po kruhovej drahe ale po eliptickej, nie ?

Dakujem


Přesně tak a tudíž můžou mít na stejném bodě rozdílný směr a velikost rychlosti (vector).


Jirka - 30/4/2007 - 13:42

Navic sklon techto drah nebyva stejny, tudiz do sebe muzou narazet z boku.

Pokud vystrelite jeden satelit pod uhlem 45stp k rovniku a dalsi satelit vystrelite taky pod uhlem 45stp, ale na druhem konci sveta a o pul obehu pozdeji tak ty dva satelity do sebe muzou narazit pod uhlem 90stp a mit tedy stretavaci rychlost 7.5 km/s.

Dva satelity proletajici nad Zemskymi poly (polarni draze) do sebe muzou klidne narazit i celne (15km/s). Staci jen vystrelit dva satelity smerem k severnimu polu ve stejnou chvili ze dvou protilehlych lokaci, nebo druhy satelit vystrelit ze stejne lokality, ale cca o pul dne pozdeji.


ales - 30/4/2007 - 14:35

> ... preco su vzajomne rychlosti take velke ? Problem je, ze niektore satelity nekruzia po kruhovej drahe ale po eliptickej, nie ?

Rozdílná výstřednost u eliptických drah by vedla jen k relativně malým kolizním rychlostem. Mnohem větší vliv mají rozdíly ve sklonu a hlavně v rovině drah, jak už tu napsal Jirka.

Ovšem i mne překvapilo, že většina možných srážek by měla mít kolizní rychlost cca 14 km/s a více, což podle mých výpočtů znamená srážku pod úhlem nejméně 125° (tedy už docela hodně "proti sobě"). Vysvětlením je zřejmě to, že původní družice FY-1C měla (bohužel) sklon cca 99°, takže i většina trosek má tento sklon (který je mírně "retrográdní", tedy tedy letí trochu "proti směru letu většiny družic"). Z toho plyne, že i družice na drahách se sklonem třeba jen 30° už se mohou, při nepříznivé vzájemné poloze rovin drah, srazit s troskami FY-1C pod úhlem přes těch 125° a tedy s kolizní rychlostí přes 14 km/s.

Obecně jsou ale potenciálně "v ohrožení" všechny družice Země jejichž výška dráhy je nižší než cca 1000 km (alespoň v perigeu), protože v této výšce (a níže) teď létá většina trosek FY-1C.

O FY-1C více na http://www.lib.cas.cz/www/space.40/1999/I025A.HTM


JuDu - 17/5/2007 - 09:27

Ahojte,
Teraz velmi leti otazka PRO, mna zaujimaju tieto veci:

Ak si predstavime, ze radar ma max. dosah 6700 km (pocital som s hodnotou 6000 km) a raketa leti s rychlostou 15 Mach (zobral som 18000 km/s), tak ak zaregistruje raketu, signal radara leti tam aj spat 0.04 s. Za ten cas raketa posunie o 360 m (za cas 0.02 s - cas kym dorazi odrazeny signal od rakety do radaru). Ak viem, ze v case ked sa mi dorazi signal do radaru raketa uz je prec o 720 metrov (moment vyslania - moment prijatia), tak ako sa to kompenzuje pri navadzani antirakiet ?
Je to ako sledovat svetlo so vzdialenich hvized - mozno hviezda uz neexistuje, je to minulost.
Dakujem


Wartex - 17/5/2007 - 09:48

Opakovani matka moudrosti:

Prave jste vyhmatnul klicovy problem navadeni na velmi rychle cile. Dokonce i kdyz vezmete finalni navadeni IR snimacem v EKV, tak pri vzdalenosti 100 km od hlavice (cca 7-10 sekund od zasahu), cestuje to IC vlneni od hlavice ke snimaci 0.3 ms, za kterouzto dobu urazi hlavice po draze cca 2.3 metru, cili zhruba jednu ci dve konske, pardon hlavicove delky.

Takze z hlediska EKV hlavice stejne jako palba z kulometu na letadlo - kus pred :-) a predpokladem je dost dobra predikce drahy. Cili pasivni hlavice, vysoke vakuum, dobry model GP atd ...

Radar samozrejme vidi stary obraz, proto navadi s presnosti desitek ci stovek kilometru, neni to v principu radar strelecky, ale orientacni. Jeho sila je v rozliseni a zrejme v jeho dalsich vlastnostech, aby (udajne) odlisil hlavice od balonu a zbytku z posledniho stupne a dokazal poskytnout data k predani do EKV pro finalni rozhodnuti, co je jeho cilem.


Wartex - 17/5/2007 - 10:02

Mel bych ovsem jiny problem, ktery mi vrta hlavou, pokud je zde nejaky odbornik na radiolokaci.

Hovori se fenomenalnim rozliseni onoho XBR radaru. Jak je takoveho rozliseni dosazeno? Na jakou vzdalenost udavana hodnota nekolika malo desitek centimetru plati?

Jak toto souvisi se sirkou svazku toho AESA pole radarove anteny? Je zrejme, ze tato technologie umi generovat svazek o uhlove sirce zlomky stupnu, ale pro rozliseni rekneme 20cm na vzdalenost 5000km mi vychazi,
ze by uhlova sirka svazku mela byt neco pres 2 miliontiny stupne, tj. asi 8 mas. Je to realne u mikrovlnneho svazku?

Kde delam v uvaze chybu?


Ervé - 18/5/2007 - 10:26

Myslím že takové rozlišení je možné jen na kratší vzdálenosti, tedy na dobu, kdy už protiraketa dohořela a EKV se blíží k cíli - řekněme na vzdálenost 1-2000 km. Hodnotu rozlišení Pentagon nikdy nezveřejní z bezpečnostních důvodů.


Wartex - 18/5/2007 - 10:44

citace:
Myslím že takové rozlišení je možné jen na kratší vzdálenosti, tedy na dobu, kdy už protiraketa dohořela a EKV se blíží k cíli - řekněme na vzdálenost 1-2000 km. Hodnotu rozlišení Pentagon nikdy nezveřejní z bezpečnostních důvodů.


Mne osobne se zatim zda, ze onen udaj o "rozliseni" se tyka spise presnosti urceni polohy kontaktu v prostoru, a to jeste z matematickeho vyhodnoceni cele rady odezev na jednotlive impulsy. Tim nasledne lze zpracovat drahu telesa a s takovouto presnosti predikovat jeho budouci polohu (v pasivni casti letu).

Coz je samozrejme v rozporu s tvrzenim, ze radar (resp. system vyhodnocujici jeho data) nasledne predava killeru TOM - cili mapu objektu leticich "spolecne" s vyznacenim cile.

Konkretni TTD radaru a jeho schopnosti samozrejme necekam, jde mi o to, zda je lze shora a zdola kvalifikovane odhadnout na zaklade zname frekvence, rozmeru anteny, zkusenosti s jinymi AESA radary, fyzikalnimi podminkami v atmosfere atd.


Adolf - 18/5/2007 - 10:57

citace:
Mel bych ovsem jiny problem, ktery mi vrta hlavou, pokud je zde nejaky odbornik na radiolokaci.

Hovori se fenomenalnim rozliseni onoho XBR radaru. Jak je takoveho rozliseni dosazeno? Na jakou vzdalenost udavana hodnota nekolika malo desitek centimetru plati?

Jak toto souvisi se sirkou svazku toho AESA pole radarove anteny? Je zrejme, ze tato technologie umi generovat svazek o uhlove sirce zlomky stupnu, ale pro rozliseni rekneme 20cm na vzdalenost 5000km mi vychazi,
ze by uhlova sirka svazku mela byt neco pres 2 miliontiny stupne, tj. asi 8 mas. Je to realne u mikrovlnneho svazku?

Kde delam v uvaze chybu?


Zdaleka nejsem odborníkem na radiolokaci. Ovšem kam sahají mé chabé znalosti: radar určuje polohu v podstatě v polárních souřadnicích. Jeho nejpřesnější rozlišení při tom vychází z měření vzdálenost, rozlišení polohy vyplývající z úhlového rozlišení je pak řádově jinde. Velmi přesných měření vzdálenosti je dosahováno kódování signálu. Osobně mi připadá měření s přesností desítek cm u cíle tisíce km vzdáleného neodpovídající účelu a ilustrující spíš skutečnost, že přílišná přesnost je na úkor relevance. Takovouto přesnost bych očekával od senzorů interceptoru ne od naváděcího lokátoru.


Jirka - 18/5/2007 - 11:35

http://en.wikipedia.org/wiki/Imaging_radar

Radar dela vlastne obrzek za pomoci fotonu o frekvenci 7 az 12.5 GHz a vlnove delce cca 3cm. Naproti tomu ma cervene (viditelne) svetlo vlnovou delku cca 740 nm a frekvenci 405 THz. To je docela rozdil, ale myslim ze to docela dobre korensponduje s tim maximalnim deklarovanym maximalnim rozlisenim 10cm. To je urcite dostatecne k odliseni hlavice od jakehokoliv jineho predmetu s jinym tvarem, pripadne k detekci rotace ci translace zpusobene manevrovanim, ci zmene tvaru zpusobene nafukovanim nejake atrapy.
Infracervene svetlo pouzivane v senzorech interceptoru a v planovane konstalaci satelitu na LEO, ktere maji prave pomoci s rozlisovanim falesnich cilu ma delku 750 nm az 1 mm. Americani pracuji na na senzorech schopnych pracovat ve vice vlnovych delkach schopnych sledovat distribuci teplot cile. Od 3 do vice nez 12 mikrometru. Rozliseni by tedy melo byt vice nez dostatecne.

Nasel jsem clanek s prekvapive mnoha informacemi:
http://mae.pennnet.com/articles/article_display.cfm?article_id=103676


Jirka - 18/5/2007 - 11:38

V tom clanku tvrdi, ze maximalni rozliseni XBR je golfovy micek na vzdalenost nekolika tisic mil. To by odpovidalo tem 3centimetrum. To je vsak asi teoreticke rozliseni. Kazdopadne je to i zrejme jedina moznost jak detekovat kosmicke smeti na LEO a dava to i odpoved jaka maximalni velikost je momentalne detekovatelna.


Wartex - 18/5/2007 - 12:34

citace:
V tom clanku tvrdi, ze maximalni rozliseni XBR je golfovy micek na vzdalenost nekolika tisic mil. To by odpovidalo tem 3centimetrum. To je vsak asi teoreticke rozliseni. Kazdopadne je to i zrejme jedina moznost jak detekovat kosmicke smeti na LEO a dava to i odpoved jaka maximalni velikost je momentalne detekovatelna.


To michate nekoilk veci dohromady.

Je rozdil mezi schopnosti detekce, to je vec vykonu vysilace, sumu a citlivosti prijimace, odrazivosti cile a dalsich faktoru.

... a mezi rozlisenim, tj. schopnosti nezavisle radiolokacne "proskenovat" dva sousedni body v prostoru. Samozrejme teoreticky je rozliseni limitovano vlnovou delkou zareni, ale tady se bavime o tisicich kilometru, urcujicim faktorem z hlediska polohy bude jiste dosazitelne sirka a rozbihavost mikrovlnneho svazku.

Adolfe, mate pravdu v tom, ze merit vzdalenost (vlastne merit cas) je nejpresnejsi vec, co lze. Navic lze zrejme studovat tvar vraceneho impulsu, jeho spektrum a dovozovat leccos z ruznych minidopplerovskych posunu v zavislosti na case, to si umim predstavit, tam ma matematika a elektronika siroky vybeh.

Senzory na EKV a naroky, ktere zpracovani jejich signalu klade na elektroniku a jeji vykon, je na jinou, nepochybne stejne zajimavou diskusi.


Jirka - 18/5/2007 - 12:46

citace:
citace:
V tom clanku tvrdi, ze maximalni rozliseni XBR je golfovy micek na vzdalenost nekolika tisic mil. To by odpovidalo tem 3centimetrum. To je vsak asi teoreticke rozliseni. Kazdopadne je to i zrejme jedina moznost jak detekovat kosmicke smeti na LEO a dava to i odpoved jaka maximalni velikost je momentalne detekovatelna.


To michate nekoilk veci dohromady.

Je rozdil mezi schopnosti detekce, to je vec vykonu vysilace, sumu a citlivosti prijimace, odrazivosti cile a dalsich faktoru.

... a mezi rozlisenim, tj. schopnosti nezavisle radiolokacne "proskenovat" dva sousedni body v prostoru.


No ja jsem prave myslel ze to bylo receno. Minimalni velikost detekovatelneho objektu 3cm - na mesich objektech nedochazi k odrazu svetla s vetsi vlnovou delkou. A rozliseni detailu o velikosti cca 10cm. To znamena ze kdyz na hlavici bude hrbolek s velikosti mensi nez 10cm tak ho nelze rozeznat. (Tomu by odpovidaly i ty zverejnene fotky, ktere se potulovaly uz v tom zverejnenem odkazu na b... listy.)
Od rozeznani vetsich detailu jsou prave ty infracervene senzory. Na satelitech nebo interceptorech.


JuDu - 20/5/2007 - 11:07

Ahojte,
Na internete som nasiel dost vela fotiek (zamerne nekvalitnych) o tom, ako vyzera konstrukcia americkej jadrovej strely. Preco je obal jadrovej strely chladeny plynom (asi tekuty dusik)? Aby mal taku teplotu ako pozadie ?
Ako su riesene chladiace radiatory v kozme ? Tam nefunguje chladenie prudenim (konvekcia), tak sa tam spolieha len na ziarenie ? Ako je to ucinne oproti normalnemu sposobu ? Treba tam vacsiu plochu, vacsi tlak alebo nuteny obeh ?

Dakujem.


Wartex - 20/5/2007 - 11:39

citace:
Ahojte,
Na internete som nasiel dost vela fotiek (zamerne nekvalitnych) o tom, ako vyzera konstrukcia americkej jadrovej strely. Preco je obal jadrovej strely chladeny plynom (asi tekuty dusik)? Aby mal taku teplotu ako pozadie ?
Ako su riesene chladiace radiatory v kozme ? Tam nefunguje chladenie prudenim (konvekcia), tak sa tam spolieha len na ziarenie ? Ako je to ucinne oproti normalnemu sposobu ? Treba tam vacsiu plochu, vacsi tlak alebo nuteny obeh ?



Odpovim nakolik staci moje vedomosti:

1. chlazeni jaderne naloze - pokud je jako stepny material pouzito plutonium (nejcastejsi pripad) je chlazeni nutne, protoze kovove plutonium se vlastnim rozpadem samovolne zahriva - a ne malo. Zalezi samozrejme na velikosti kusu, pomer hmotnosti k povrchu, ale radove dekagramy plutonia v ingotu se samovolne zahreji do cerveneho zaru (700-800 st.?). Tohoto principu samovolneho ohrivani vyuzivaji zdroje tepla na sondach - at uz jako tepelny zdroj pro spravnou tepelnou pohodu elektroniky (myslim ze napr. rovery), nebo zdroj tepla pro RTG termoclankove zdroje.

2. chlazeni v kosmu - na palube objektu lze vyuzit konvekce, ruzne systemy vedeni tepla (heatpipes, obeh chladiva a podobne), ale zbavit se odpadniho tepla lze jen radiaci. Vyzareny vykon radiatoru je umerny jejich plose a emisivite materialu chladice. Pokud je stanice ci sonda triose stabilizovana, musi se konstrukce snazit vystavovat Slunci plochy s co nejmensi absorpitivitou, a zaroven na stinne strane mit plochy s co nejvetsi emisivitou, pripadne celou soustavu vhodne regulovat. Pro predstavu, hlinikova desticka vystavena Slunci v kosmu na orbitu Zeme ma rovnovaznou teplotu, kdy se prijem ze Slunce plus nejake to albedo Zeme vyrovna s tepelnym zarenim diky rozpaleni desticky, zhruba na 400 stupnich Celsia.


Bryansk - 21/5/2007 - 14:25

Jelikož se blíží konec programu raketoplánů, je možnost k bilancování. Je tedy možné porovnat, co bylo v odborném tisku proklamováno před prvním startem 12.4.1981 /přesně 20 roků po Gagarinovi/ a čeho bylo dosaženo za 27 let programu. Program "Space Shuttle" předpokládal původně -"Nejlacinější, častou, bezpečnou" dopravu nákladů do kosmu. Počítalo se s 50 starty ročně, každý měl stát 5 milionů USD. "Klasické" rakety, vzhledem k tomu, jak jsou "drahé" a "neefektivní" měly být v USA zcela zrušeny - vše měl obstarat raketoplán. Podívejme se ovšem na realitu - raketoplán odstartoval za 27 let 117 x, tedy v podstatě průměrně 4 x za rok a cena za 1 start se vyšplhala přes 500 milionů USD.Raketoplán se ukázal být tím zatím nejnebezpečnějším kosmickým plavidlem, v jeho troskách nalezlo smrt zatím 14 kosmonautů a je dost možné , že 8.6. exploduje ještě Atlantis. Klasické rakety zažívají bouřlivou renesanci a nikoho zatím nenapadlo, zkoušet kopírovat US raketoplán. Z původních 50 startů ročně se tedy staly jen 4 a z ceny 5 mil. USD za start se stalo 500. Faktem zůstává, že raketa Proton vynáší do kosmu 25 tun při startovní váze 950 tun, raketoplán vynese stejně při 2000 tunách hmotnosti a ještě se při tom riskuje se životy kosmonautů. /i Kdyby bylo rozhodnuto dopravit do kosmu "fůru hnoje" , bude ji zřejmě doprovázet sedmička kosmonautů./ Tento nepovedený program ,budiž výstrahou a poučením dalším zemím, jak v kosmu rozhodně nepostupovat a jak "neprodělat kalhoty na nesmyslech".


Luděk - 21/5/2007 - 15:42

citace:
...že 8.6. exploduje ještě Atlantis.
citace:




To by bylo radosti, že?


Luděk - 21/5/2007 - 15:44

citace:
...a nikoho zatím nenapadlo, zkoušet kopírovat US raketoplán.
citace:




A co Buránek?


Martin J. - 21/5/2007 - 16:25

citace:
citace:
...a nikoho zatím nenapadlo, zkoušet kopírovat US raketoplán.
citace:




A co Buránek?


Buran = propaganda
V case propagandistickych subojov nemohli byt Rusi horsi.
Buran znamenal technologicku vyspelost. Nemaju vsak prachy na zivenie takeho nezmyslu.

Aj na Mesiaci boli Rusi prvy. Jedna ich sonda letela tam a priniesla vzorky spat. Vedia technologicky takyto let zvladnut. Nemaju na to ekonomicky.


Bryansk - 21/5/2007 - 17:53

citace:
citace:
citace:
...a nikoho zatím nenapadlo, zkoušet kopírovat US raketoplán.
citace:




A co Buránek?


Buran = propaganda
V case propagandistickych subojov nemohli byt Rusi horsi.
Buran znamenal technologicku vyspelost. Nemaju vsak prachy na zivenie takeho nezmyslu.

Aj na Mesiaci boli Rusi prvy. Jedna ich sonda letela tam a priniesla vzorky spat. Vedia technologicky takyto let zvladnut. Nemaju na to ekonomicky.
Plně souhlasím. Buran už před 20 roky ukázal, že SSSR bez problémů dokáže vyvinout tak složité plavidlo, jako je Space Shuttle , žádný problém po technické stránce. Ovšem ekonomika SSSR neposkytovala dost finančních prostředků na pravidelný provoz takového stroje a tak byl program "odpískán". Dost možná, že Rusové, když pozorují potíže s raketoplánem americkým - tak ani nelitují. Já osobně více lituji výtečné rakety Energija , ta měla dostat šanci - ovšem po zrušení Buranu pro ni nebyl dost velký náklad.


dodge - 21/5/2007 - 18:43

citace:
citace:
...že 8.6. exploduje ještě Atlantis.
citace:




To by bylo radosti, že?



Jak jsem řekl - je to ....[Upraveno 21.5.2007 poslal admin]


Tomáš_Kovařík - 21/5/2007 - 20:38

citace:
...že 8.6. exploduje ještě Atlantis.
citace:



To už trochu přeháníte! Už jste sem napsal moc zvrhlých příspěvků na to, aby se dalo věřit, že jste normální! Už sem prosím nepište!!!


xmariox - 21/5/2007 - 20:48

Tak to bylo fantasticke shodnoceni projektu space shuttle
odborne vecne

a to stim Atlantisem je perlicka na zaver

Chlapce teda.....


ales - 21/5/2007 - 22:10

Ke "statistice" pro bilancování raketoplánu ještě dodávám, že pět amerických raketoplánů dohromady uskutečnilo 114 úspěšných kosmických misí (více, než jakýkoliv jiný typ lodi ... celkový počet pilotovaných kosmických startů ještě nedosáhl čísla 250). Do kosmu raketoplány dopravily přes 690 kosmonautů (některé z nich opakovaně), což je dvakrát více, než všechny ostatní kosmické lodi dohromady (ty dopravily do kosmu cca 320 kosmonautů, také některé opakovaně). Na raketoplánech bylo uskutečněno tisíce experimentů a raketoplány hrají podstatnou úlohu při stavbě stanice ISS.

Po bitvě je každý generál a prohlásit raketoplán za "omyl" je příliš jednoduché. Je faktem, že původní očekávání se u raketoplánů nepodařilo naplnit, ale kdyby se to prakticky nevyzkoušelo, tak to nikdo nemohl vědět jistě. Podle mého názoru je dobře, že to američané vyzkoušeli a výsledky rozhodně nejsou jen negativní. Kvůli principiální nebezpečnosti je zřejmě opravdu třeba provoz současné flotily raketoplánů ukončit, ale věřím, že časem se podaří najít způsob, jak s lidmi létat na oběžnou dráhu Země (LEO) a zpět bezpečně a pohodlně ale přitom relativně levně. Uvidíme, co v tomto směru přinese např. ruský Kliper.

P.S.: Za "laickou otázku" v tomto případě považuju "dotaz" jestli americký raketoplán byl/je "nesmyslem".


Bryansk - 21/5/2007 - 22:22

Pro Mário - veškerá fakta , která jsem uvedl, myslím čísla - "plně sedí" a jsou nenapadnutelná. Plánované parametry Space Shuttle jsem vytáhl z časopisu L + K , myslím, že to byl ročník 1977. Tedy = 50 startů za rok, cena startu plánovaná 5 mil USD, po zavedení raketoplánu se skutečně v USA měly zrušit klasické rakety, v raketoplánech skutečně zahynulo 14 lidí, raketoplán je při startu při stejné nosnosti na orbit skutečně víc jak 2 x těžší jak Proton apod. , takže nechápu, co by tu mohlo být k smíchu !!! Leda blbost někoho druhého, který se nevyzná ve faktech. Jeden start je prostě 100 x dražší, než se počítalo - to může někomu připadat jako úspěch ??? Mně tedy ne ! Je to naprosto stejné, jako kdyby Škoda auto oznámila zákazníkům , že za 2 roky uvede na trh nový model Fábie a že bude stát 300 000 Kč , pak by se zjistilo, že bude stát 100 x tolik, neboli 30 milionů !! - já bych tedy chtěl vidět, kdo by si takový "koráb" za 30 milionů koupil ! A pokud jde o bezpečnost startu Atlantisu, riziko je skutečně velké. Malé srovnání - raketoplánů bylo původně pět, uskutečnily celkem dohromady 116 startů a zbyly 3 .Tedy ztráty = 40 % všech strojů. Malé srovnání - představte si na vesnici pět sousedů, všichni mají auta, kterými denně dojíždějí do práce do města. Za měsíc se jede do práce 22 x tudíž těchto pět aut uskuteční 116 jízd právě za měsíc !! Tedy 1. května začnete jezdit do práce a do 30. května budete muset jít dvěma svým kamarádům na pohřeb !! - toto srovnání je opět nenepadnutelné a nevykládejte mi něco o "složitosti "letu do vesmíru a obyčejné cestě do práce autem - to s tím nemá vůbec nic společného !!!Já osobně bych měl "husí kůži" , kdybych za pouhý jeden měsíc přišel o dva kamarády ze čtyř / = polovina !!!!/ vinou technických závad v jejich autech !!! Tedy já si havárii Atlantisu nepřeji, pouze se domnívám, na základě výše uvedeného srovnání, že riziko létat v těchto strojích je vysoké !!! Riziko raketoplánů také vynikne při srovnání se Sojuzy /opět mi nic nevykládejte o "jednoduchosti" Sojuzua "složitosti" raketoplánů./ Jak je nám všem dobře známo, poslední smrtelná nehoda v kabině Sojuz se odehrála 30.6.1971, kdy v kabině Sojuzu 11 zahynuli Dobrovolskij, Volkov a Pacajev. Tedy Sojuz má za sebou 36 let !!! bez smrtelné nehody a to je dost.


Andy - 21/5/2007 - 22:33

citace:
... raketoplán je při startu při stejné nosnosti na orbit skutečně víc jak 2 x těžší jak Proton apod......


Hmmm, a kolik tun z Protonu se vrací zpět na Zemi? A Proton vynáší i astonauty? Mícháte jablka s hruškami...


Bryansk - 21/5/2007 - 23:03

Pro Andy : Já jsem srovnával pouze a jenom pouze náklady na jeden start u nosiče Proton a raketoplánu - při podobné nosnosti na orbit je cena u raketoplánu kolem 500 mil. USD a u Protonu se pohybuje i pod hranicí 100 mil USD - vynést do kosmu 1 kg nákladu jde tedy Protonem 5 x laciněji, jak raketoplánem a vícenásobná použitelnost orbitálního stupně raketoplánu cenu startu jak vidno nedokáže snížit, neboť samotná příprava tohoto stupně na další starty je velmi zdlouhavá a drahá. Proton se vyrábí "sériově" , tuším, že v roce 2000 odstartoval asi 15 x za rok !! a to jeho cenu dále snižuje. Je to dost podobné, jako s vratnými skleněnými lahvemi, laik by se domníval, že mnohonásobná použitelnost zvítězí , ale jak sám vidíte , není tomu tak. U PET lahví odpadá jakákoli následná manipulace - nepotřebujete instalovat a opravovat drahé automaty na odběr skleněných lahví, nemusíte posílat auta, aby vám je stáhla z obchodů, nemusíte provozovat linku, která lahve čistí louhem sodným a platit tak další zaměstnance, atd. atd. Proton je jako "PETKA" - použij a zahoď - jak vidno, funguje to. Musím poznamenat, ža sám jsem příznivcem vratných lahví, a to třeba i plastových.


Adolf - 22/5/2007 - 00:43

Osobně považuji raketoplán koncepčně za omyl i přes všechnu užitečnou práci, kterou vykonal. Obtížnost uhnutí z tohoto omylu na druhou stranu je dobrou ilustrací toho, jak účinná může být rozpočtová kotva, neboť právě v tomto byl na druhou stranu až extrémně účinný.

Když člověk nastupuje k progresivní firmě jako samostatný odborník, chtějí od něj při pohovoru vědět, jaké neúspěchy utrpěl v poslední době. Kdo se nemůže pochlubit kvalitními neúspěchy, tomu dost poklesne oceňovací skór. Nedostatek neúspěchu je projevem měkkých ambicí a nízké produktivity při tvorbě výrazně průlomových řešení. Zárukou stabilního úspěchu je jen stagnace.

Taky bych se někdy chtěl chlubit tak monumentálními neúspěchy jako je raketoplán. Myslím, že na tento neúspěch mají v NASA důvod být právem hrdí.


DODOR - 22/5/2007 - 06:52

citace:
Jeden start je prostě 100 x dražší



Zase neříkáte pravdu. Sám píšete že 5mil USD je zroku 1977. A 500 mil USD z roku 2007 tzn. logika věci říká že to nejsou ty samé peníze!

A co se týše raketoplánu srovnáváte jablka z hruškami Sojuz můžete srovnat jen z Mercury nebo s Geminy. A i to s obtížemi (spíše Vostok, Voschod). Protože to co umí raketoplán Sojuz nikdy umět nebude.

PS:

Ne že by se mi raketoplán nějak zamlouval.


avitek - 22/5/2007 - 07:56

citace:
... Proton se vyrábí "sériově" , tuším, že v roce 2000 odstartoval asi 15 x za rok !! a to jeho cenu dále snižuje. ...


Čtrnáckrát.

1997 ... 9
1998 ... 7
1999 ... 7
2000 ... 14
2001 ... 6
2002 ... 9
2003 ... 5
2004 ... 8
2005 ... 7
2006 ... 6

Za tu dobu neměl však Proton žádnou fatální havárii, pouze jednou selhal poslední stupeň (2006-006A - Badr 1, 2006-02-28, stupeň Briz-M) s totální ztrátou užitečného zatížení.

Roční průměr tedy dělá za posledních deset let necelých 8 startů (7,8) za rok. Tady je vidět, jak ošidné je mávání statistikami a jak se jimi dá "veřejné mínění" ovlivňovat.

Citát: "Mohu důvěřovat jen té statistice, kterou si zfalšuji sám."
W. S. Churchill


Bryansk - 22/5/2007 - 08:10

Pro DODORA : Souhlasím s tím, že od roku 1977 v USA došlo ke znehodnocení dolaru inflací a že to tedy dnes není "ten samý dolar". Nemám informace o poklesu hodnoty USD za 30 let, ovšem něco vypovídá srovnání, které jsem nedávno četl - program Apollo stál kdysi zhruba 20 miliard USD, na dnešní peníze by to prý bylo asi 80 miliard. Takže zhruba se dá říci, že dřívejších 5 milionů, můžeme navýšit na dnešních 20. Ale i tak si myslím, že rozdíl 20 mil. , co bylo očekáváno a 500 za 1 start, se kterými program končí je neúnosně veliký rozdíl. Mám obavy, aby podobný vývoj nepostihl i samotnou stanici ISS , neboť jak známo, Rusové vypustili první základový modul již před 9 lety /1998/ a stanice se stále ještě staví a bude stavět. Ruská stanice Mir počala vypuštěním základového moudulu 19.2.1986 a skončila 23.3.2001 v Tichém oceáně, když množství závad, které bylo nutno odstraňovat dosáhlo neúměrné výše.Mir tedy existoval 15 let, ISS existuje už 9 let a její stavba stále ještě probíhá - mají její základní moduly ve srovnání s Mirem garantovánu mnohem vyšší životnost na oběžné dráze ???, než je 15 let ??


bejcek - 22/5/2007 - 09:02

citace:
Nedostatek neúspěchu je projevem měkkých ambicí a nízké produktivity při tvorbě výrazně průlomových řešení. Zárukou stabilního úspěchu je jen stagnace.

Taky bych se někdy chtěl chlubit tak monumentálními neúspěchy jako je raketoplán. Myslím, že na tento neúspěch mají v NASA důvod být právem hrdí.



Ano,souhlasím, je to velmi přesně řečeno a neznám technický projekt, který po realizaci splní naprosto přesně to co se očekávalo a jeho vývoj + dolaďování je naprosto bez problémů. V budoucnu se mnohé z STS hodí.


Ervé - 22/5/2007 - 10:15

Základní blok Miru měl garantovanou životnost 3 roky, Mir dosáhl vrcholu svých možností v červnu 1990 (Mir+Kvant1+2+Kristall). Mir by zanikl někdy kolem roku 1995, pokud by nedošlo k dohodě s NASA o letech STS. Za americké peníze byl dokončen Spektr (50% elektřiny) a Priroda. Byla to oboustranně výhodná smlouva. Raketoplány dovezly vybavení, zásoby a odvezly spousty nákladu. Po odletu posledního STS už Mir končil, let Belly a Haignéra byl posledním větším úspěchem.
ISS má mít životnost 15 let, reálné vidím 25 let, přitom od roku 2018 to bude víc o údržbě než o vědě. Pokud od roku 2009 bude mít 6člennou posádku, stihne se udělat spousta práce. Pokud ATV, HTV a Progressy budou létat, neměl by být problém s dopravou nákladu na stanici. Problém bude s návratem nákladu na Zem.
Při návratu STS s MPLM (zásobovacím modulem) se na Zem vrací asi 6 tun nákladu, bez MPLM asi 2 tuny. 2/3 budeme považovat za odpad, který by se dal vhodným roztříděním (týden práce posádky) oddělit a nechat shořet v Progresech a BO Sojuzů (i když ty jsou plné už teď). Pořád zůstávají 2 tuny a 0,7 t nákladu. Bezpilotní Sojuz by na ISS přivezl náklad (každý tak 1 tunu - 40% nosnosti Progressu), zpátky na Zem by dostal 500 kg nákladu (v kabině bez sedaček, vybavení a zásob). 1 start STS s MPLM tak nahradíte 1 letem Sojuzu s posádkou (i když Sojuz vynese jen 3 lidi), 4 lety Sojuzu bez posádky a 3 starty Progressu (5*40+3*35=305 mil. USD, 8 startů). STS s modulem nahradí 1 Sojuz, 1 bezpilotní Sojuz, 2 Progressy a 1 Proton (40+40+35+35+90 mil.USD = 240 mil.USD). Rusové od roku 2009 budou mít kapacity pro výrobu 4 Sojuzů ročně, ale ne víc, všechny budou pro posádky, ATV a HTV nedokážou náklad vracet. Klipr je v nedohlednu, takže opět zbývá americký Orion.


Jiří Hošek - 22/5/2007 - 11:30

citace:
1 start STS s MPLM tak nahradíte 1 letem Sojuzu s posádkou (i když Sojuz vynese jen 3 lidi), 4 lety Sojuzu bez posádky a 3 starty Progressu (5*40+3*35=305 mil. USD, 8 startů).
N.Sevasťjanov sdělil 16.5.2007, že doprava 1 kg nákladu na ISS lodí Progress M stojí 22000 - 25000 USD. Při hmotnosti nákladu 2,566 t, který dopravila na stanici loď Progress M-60, vychází celkové náklady na cca 60 mil. USD.

Roskosmos uzavřel nedávno s NASA kontrakt na stavbu šesti lodí Sojuz TMA a tří lodí Progress M v celkové hodnotě cca 1 mld. USD. Pokud tedy tři progressy á 60 mil. USD stojí 180 mil. USD, pak šest sojuzů stojí zbylých 820 mil. USD, tj. cca 135 mil. USD za každý.

Místo Vašich 305 mil. USD mi tedy vychází 855 mil. USD.


Jiří Hošek - 22/5/2007 - 12:09

citace:
Pokud tedy tři progressy á 60 mil. USD stojí 180 mil. USD, pak šest sojuzů stojí zbylých 820 mil. USD, tj. cca 135 mil. USD za každý.

Ještě dodatek. Možná je těch 135 mil. USD za Sojuz kalkulováno včetně "letenek" pro tři kosmonauty á 25 mil. USD.
Tzn. rozklíčování 135 mil. = 60 mil. + (3 x 25 mil.)
Ale to si jen domýšlím.


Jiří Hošek - 22/5/2007 - 13:37

Ten kontrakt byl v hodnotě 715 mil. USD.
Nepodařilo se mi najít podrobnější informace než na:
http://www.sciencedaily.com/upi/index.php?feed=Science&article=UPI-1-20070410-12174400-bc-us-nasa.xml

Moc se z toho rozklíčovat nedá, součástí kontraktu je například to, že raketoplán dopraví v roce 2010 na stanici ruský "Docking Cargo Module".
Jinak z diskusí na síti se zdá, že ruský modul MLM má další zpoždění (původně 2009) a DCM má být postaven právě proto, aby byl dolní uzel Zarji obsazen dříve než bude ke stanici připojen připojen Node 3.


Bryansk - 22/5/2007 - 18:50

V přehledu startů družic se v těchto týdnech objevuje jedna zajímavost. Čína vypustila úspěšně během jednoho měsíce tři družice / 11.4., 13.4. a 13.5./ - další dvě -Sinosat 3 a vojenská družice jsou avizovány na nejbližší týdny. Toto tempo jistě nebude udrženo v dalších měsících, přesto si myslím, že tento stav o něčem vypovídá. Právě dnes bylo ve Washingtonu zahájeno jednání delegací USA - Čína o ekonomických otázkách - jednání je vedeno na nejvyšší úrovni, ze strany USA je to ministr financí a také šéf FEDU pan Bernanke. Cílem jednání je přinutit konečně Čínu, aby nechala výrazně posílit svou národní měnu proti USD a to alespoň o 40 až 50 %. Loni totiž dosáhl deficit USA v obchodě s Čínou zoufalých rozměrů 232 miliard USD , takový deficit v obchodě s jednou zemí zatím nikdo nikdy neviděl. Takže, co můžeme očekávat v nejbližších letech ? Čínský HDP dosud stoupal o 10 % ročně a jelikož zřejmě bude muset uvolňovat Čína kurz své měny asi o 5 % ročně , bude tedy v dolarovém vyjádření činit růst čínského HDP kolem 15 % ročně - a to je už opravdu zajímavá cifra. USA porostou o 2 až 3 %. Čínské kosmonautice se tak bude dostávat zřejmě finančních prostředků vrchovatě. Na ostrově Hainan pokračuje výstavba čtvrtého čínského kosmodromu pro rakety velikosti Ariane- 5.Řada pisálků ovšem čínské plány přehání a musím se smát, když čtu, že v roce 2018 /tedy za 10 let/ budou Číňané chodit po Měsíci - zřejmě tito lidé ani nejsou schopni přeložit anglické texty. Za 10 let může mít Čína leda tak malou "orbitální stanici" složenou z několika jejich lodí Šen - žou a může začít s využíváním nosiče velikosti Protonu /CZ- 5 /, který by jim umožnil vybudovat stanici podobnou Miru - ovšem se zpožděním 30 let za Ruskem. Faktem je, že čínské "makroekonomické výkony" jsou fascinující - loni stoupl export o 0,21 bilionu USD, letos o dalších 0,24 bilionu - to znamená, že za 4 roky 2006 až 2009 stoupne čínský export o 1 bilion USD. Pro srovnání, USA trvalo dosažení exportu zboží ve výši 1 bilion USD přesně 230 let, od jejich založení v r . 1776 do roku 2006. Čína to zvládne za 4 roky - není co dodat, ještě to bude zajímavé.


Zbycho - 22/5/2007 - 19:06

citace:
V přehledu startů družic se v těchto týdnech objevuje jedna zajímavost. Čína vypustila úspěšně během jednoho měsíce tři družice / 11.4., 13.4. a 13.5./ - další dvě -Sinosat 3 a vojenská družice jsou avizovány na nejbližší týdny. Toto tempo jistě nebude udrženo v dalších měsících, přesto si myslím, že tento stav o něčem vypovídá. Právě dnes bylo ve Washingtonu zahájeno jednání delegací USA - Čína o ekonomických otázkách - jednání je vedeno na nejvyšší úrovni, ze strany USA je to ministr financí a také šéf FEDU pan Bernanke....


Hmm, a otázka? Občas si připadám, jak na ekonomickém fóru.


admin - 22/5/2007 - 21:18

2 Bryansk: Umíte číst? Toto vlákno se jmenuje "Laické otázky". Pokud jste nějakou napsal, netýkal se kosmonautiky. V tom případě bych Vás poprosil o přesunutí svých dotazů na nějaký ekonomický server.
Děkuji.


Bryansk - 22/5/2007 - 22:42

citace:
2 Bryansk: Umíte číst? Toto vlákno se jmenuje "Laické otázky". Pokud jste nějakou napsal, netýkal se kosmonautiky. V tom případě bych Vás poprosil o přesunutí svých dotazů na nějaký ekonomický server.
Děkuji.
Můj poslední příspěvek sice není uveden jednoznačně jako otázka : "Chci vědět věc XY", ale myslím, že se lehce dá z textu vyčíst o co jde - neboť kosmonautika je jenom o výkonu ekonomik a tedy i množství peněz, které do kosmonautiky mohou státy uvolnit. Čekal jsem tedy , že reakce na můj článek bude typu : Dnešní výdaje na kosmonautiku v Číně jsou ...... USD, odborníci se shodují, že za 10 let budou asi.....USD a že to Číně umožní nebo neumožní provést tyto druhy výzkumných operací v kosmu ..... Nevím, jak se bavit o perspektivách vesmírného výzkumu jednotlivých zemí, aniž bych znal směřování jejich ekonomik , bez těchto znalostí mi pak každá diskuse připadá jako plácání do vody.Článek jsem uvedl v duchu , že pět čínských družic za 2 měsíce , to stojí za diskusi - na to by mi ekonomové nikde neodpověděli.


geo - 23/5/2007 - 03:04

Pane "Bryansk", pokuste se prosím netlačit na pilu. Děkuji. Jirka


Jiří Hošek - 23/5/2007 - 17:54

citace:
Já osobně více lituji výtečné rakety Energija , ta měla dostat šanci - ovšem po zrušení Buranu pro ni nebyl dost velký náklad.
Ale vždyť Energija DOSTALA ŠANCI. Systém Energija/Buran měl takovou podporu, že náklady na jeho vývoj se nakonec staly jednou z příčin ekonomického kolapsu SSSR. To byl průšvih mnohem větších rozměrů než v případě amerického raketoplánu.
Máte snad na mysli DRUHOU ŠANCI pro Energiji jako samostatnou raketu? Litujete, že veškeré pozemní zařízení nezůstává v pohotovosti již téměř dvě desetiletí v očekávání druhé šance?


Ervé - 24/5/2007 - 15:48

Za 10 let může mít Čína orbitální stanici srovnatelnou s Mirem, pokud bude chtít a pokud plánovaná velká raketa CZ-5 bude úspěšná. Zatím ale vidím u Číňanů spíš utlumování pilotovaných letů. Bezpilotní lety Shenzhou byly po 13 měsících, SZ-4 letěl po 8 měsících, pilotovaný SZ-5 po 10 měsících, SZ-6 po 24 a SZ-7 má letět po 36 měsících. Čína se soustřeďuje na aplikovanou kosmonautiku, ze které je okamžitý a významný užitek. Vždyť i sonda k Marsu má být hodně maličká - při této velikosti stěží něco zůstane na vědecky hodnotné přístroje, bude to test, jestli už takovou misi Číňani dokážou uskutečnit. Zatím jedou v pilotovaných letech na půl plynu, v dálkových vědeckých sondách se teprve rozjíždějí, ale budoucnost je otevřená.


Jirka - 24/5/2007 - 17:17

Taky mi prijde ze Cina neprojevuje dostatek motivace v pilotovanych letech. Proto bych si dovolil pochybovat ze prichazi v uvahu vystavba stanice ve stylu Miru. Pokud by si ovsem moduly nezakoupili v Rusku. Ta jejich sonda k Marsu asi opravdu nestoji v dnesni dobe za rec, ale zajimavy by mohl byt ten lunarni rover.


Hawk - 24/5/2007 - 20:53

Je třeba si přiznat smutnou pravdu, že pilotované lety valný reálný přínos nemají, resp. platí "za hodně peněz málo muziky". Čína si to zjevně uvědomuje a proto hodlá udržovat pouze nezbytné minimum pro fungování svého pilotovaného programu.


Ervé - 25/5/2007 - 07:32

Pro větší přínos z pilotovaných letů je podle mně nutné změnit přístup, a to jsem čekal od Číny - stanice a la Mir, ale s výraznou automatizací obsluhy, třeba i iontovými motory pro dlouhodobé manévrování, pilotovanými lety jen pro obsluhu, opravy, výměnu vybavení a lékařské experimenty. Dva pilotované lety ročně s délkou 20 dní až 2 měsíce. Shenzhou přestavěná a la Progress pro dopravu zásob a spec. vybavení. Nezbývá než doufat.


Derelict - 28/5/2007 - 11:49

citace:
Máte snad na mysli DRUHOU ŠANCI pro Energiji jako samostatnou raketu? Litujete, že veškeré pozemní zařízení nezůstává v pohotovosti již téměř dvě desetiletí v očekávání druhé šance?


Take mne konec Energie mrzi. Priznam se, ze se mi libila, zajimalo by mne srovnani realnych provoznich nakladu napr. se soucasnymi Protony. Navic, videl jsem kdesi koncepty, jak by bylo mozne vyuzit komplex Energa (bez Buranu) jako nosic. Pri jejim vykonu by mohla na obeznou drahu dotahnout trosku materialu, mozna by to urychlilo stavbu ISS.
Co nevim, jestli by bylo mozne pridat nejaky naklad automaticky dopravit k ISS podobne jako Progresy ... prave zde bych videl nejvetsi problem.


Jiří Hošek - 29/5/2007 - 09:43

citace:
Take mne konec Energie mrzi. Priznam se, ze se mi libila, zajimalo by mne srovnani realnych provoznich nakladu napr. se soucasnymi Protony.
Porovnejte si prosím následující čísla:

Energija: Flyaway Unit Cost $: 764.000 million. in: 1985 unit dollars
http://www.astronautix.com/lvs/energia.htm

Proton: Launch Price $: 70.000 million. in: 1994 price dollars.
http://www.astronautix.com/lvs/pro86101.htm

Náklady na znovuzprovoznění rakety Energija se odhadují na 10 miliard dolarů, viz str. 19:
http://mek.kosmo.cz/novinky/clanky/valmez/sk2005.pdf

Roční rozpočet Roskosmosu je necelá miliarda dolarů
http://www.ajgl.cz/space/Agencies.htm


Jiří Hošek - 30/5/2007 - 17:53

citace:
citace:
Za 10 let může mít Čína orbitální stanici srovnatelnou s Mirem, pokud bude chtít a pokud plánovaná velká raketa CZ-5 bude úspěšná.
... Proto bych si dovolil pochybovat ze prichazi v uvahu vystavba stanice ve stylu Miru. Pokud by si ovsem moduly nezakoupili v Rusku.

Stanice ve stylu Saljutu 6 a 7:
http://forum.nasaspaceflight.com/forums/get-attachment-big.asp?action=view&attachmentid=14265

Ministanice z orbitálních modulů Shenzhou VIII a IX bez využití rakety CZ-5:
http://forum.nasaspaceflight.com/forums/get-attachment-big.asp?action=view&attachmentid=20416


Ervé - 31/5/2007 - 14:29

Rozumná stanice se dá poskládat pomocí startů CZ-2E (8,5 t) nebo upravené CZ-3B (11 t na LEO). Ze samotných orbitálních modulů nic normálního neposkládáte. Čína potřebuje modernější rakety s vyšší nosností, proto postaví CZ-5, i když spíš v lehčích verzích. Doufám, že SZ-9, možná i SZ-8 nebudou jen obyčejné Shenzhou, ale spíš něco jako Pirs - velký přetlakový modul místo malé návratové kabiny a malé orbitální sekce. Další možnost, kterou ale Čína odmítla, je možnost vynesení základního 20 t modulu stanice ruským Protonem a bočních, specializovaných modulů pomocí CZ-2E/CZ-3B.


Jiří Hošek - 31/5/2007 - 17:19

citace:
Doufám, že SZ-9, možná i SZ-8 nebudou jen obyčejné Shenzhou, ale spíš něco jako Pirs - velký přetlakový modul místo malé návratové kabiny a malé orbitální sekce.

O 8-tunové kosmické laboratoři s jedním spojovacím uzlem, vynesené pomocí jediné rakety CZ-2F, se psalo cca do roku 2005. Poté se místo toho začalo psát o laboratoři složené z OM lodí SZ-8 a SZ-9.
V poslední době jsem četl zvěsti, že Čína zřejmě nebude schopna uskutečnit EVA v roce 2008 a spojení v roce 2010 z důvodu absence pozemních trenažérů.


Hawk - 31/5/2007 - 20:30

Je otázka zda Čína nenechá orbitální stanice plavat a nezaměří se přímo na lunární program. Amíci konečně také přešlí přímo od letů na LEO k lunárnímu programu, Skylab přišel až po skončení Apolla.
Jinak elegantní alternativou k velkému 20t modulu je nafukovadlo a k tomu těžkou nosnou raketu nepotřebujete.


Ervé - 1/6/2007 - 07:39

Vzhledem k dlouhé době realizace relativně jednoduchých pilotovaných letů na LEO je Měsíc hodně vzdálený. Ten nedostatek trenažérů je podle mně nesmysl. Jak dlouho trvá postavit bazén a maketu lodi? Když jsou peníze tak 3 měsíce. Čína buď let chce uskutečnit, a pak to zvládne, nebo se jí do toho nechce, nedá na to peníze a začne se vymlouvat.
Nafukovadla nejsou řešením, jen trochu pomůžou. 2/3 hmotnosti 20 t bloku stanice připadá na vědecké vybavení, spojovací zařízení, komplex palubních systémů, zásoby, zdroje energie a pohonné látky. To žádné nafukovadlo nevyřeší. U lehčích bloků bude poměr nezbytných systémů ještě vyšší, obzvlášť, když se používají kyslíko-dusíkové atmosféry s tlakem 101 kPa. Přitom v přetlakových kabinách letadel je udržován tlak jen asi 70kPa - jaký to má vliv na hmotnost konstrukce nemusím říkat.


Jiří Hošek - 1/6/2007 - 09:10

citace:
Ten nedostatek trenažérů je podle mně nesmysl. Jak dlouho trvá postavit bazén a maketu lodi? Když jsou peníze tak 3 měsíce. Čína buď let chce uskutečnit, a pak to zvládne, nebo se jí do toho nechce, nedá na to peníze a začne se vymlouvat.
Jen připomínám, že před 15 měsíci Čína odložila let SZ-8 z roku 2007 na září 2008 s odkazem na nepřipravenost skafandrů.


JuDu - 1/6/2007 - 14:02

dobry den,

Urobil som par prepoctov, mozete mi to skontrolovat, prosim ?
Ak satelit vo vyske 36000 km nad Zemou je nad Europou (nulovy stranovy posun), a ak zoberieme polomer Zeme ako 6000 km, tak pod akym uhlom bude 'vidiet' satelit objekt, ktory sa nachadza v europe (48 stupnov)? Ja som dostal velmy cudne cislo.

Dakujem


ales - 1/6/2007 - 17:03

citace:
Urobil som par prepoctov, mozete mi to skontrolovat, prosim ?
Ak satelit vo vyske 36000 km nad Zemou je nad Europou (nulovy stranovy posun), a ak zoberieme polomer Zeme ako 6000 km, tak pod akym uhlom bude 'vidiet' satelit objekt, ktory sa nachadza v europe (48 stupnov)? Ja som dostal velmy cudne cislo.

Protože jste neuvedl žádné svoje konkrétní výsledky, tak není co zkontrolovat, ale mohu uvést svoje odhady a výpočty.

Velmi hrubým odhadem lze říci, že pokud družice vidí objekt přímo pod sebou na rovníku shora (a budu tomu říkat úhel 90°) a objekt blízko pólu vídí zboku (a budu tomu říkat úhel (0°), tak objekt na rovnoběžce 48°s.š. (např. Bratislava) bude z GEO družice vidět zhruba pod úhlem 90°-48°, tedy 42° (z opačného pohledu je to úhel, pod kterým je družice z Bratislavy vidět nad obzorem). To by platilo, pokud by družice byla "v nekonečnu".

Přesnější výsledek dostanu, pokud opravím odhadnutý úhel o hodnotu úhlu, o který vidím družici jinde, než kdyby byla v nekonečnu. Vystačím snad s goniometrií pravoúhlého trojúhelníku. Budu používat poloměr Země 6000 km, i když přesnější by bylo cca 6370 km. Nejprve vzdálenost Bratislavy od spojnice "střed Země" - družice (sZ-d). a = 6000 . cos(48°) = cca 4460 km. Průmět tohoto bodu na spojnici sZ-d je ve vzdálenosti b = 6000 . cos(48°) = cca 4000 km od středu Země. Spojnice sZ-D měří celkem cca 42000 km (6000+36000), takže opravný úhel x = cotg(4460/(42000-4000)) = cca 6,7°. Takže GEO družice (na poledníku Bratislavy) by měla být z Bratislavy vidět cca 35,3° nad místním obzorem (a pod stejným úhlem je vidět z družice povrch Země v Bratislavě).

Pokud otázka směřovala k tomu, pod jakým úhlem je např. Bratislava vidět z GEO družice oproti směru z družice k rovníku, tak je to výše uvedených 6,7° (tedy můj "opravný úhel" [pro nepřesný průměr Země 6000 km]).

Snad jsem se nikde zásadně nespletl (to se klidně mohlo stát, takže případně prosím o opravu). Možná to lze spočítat i snadněji, ale takhle mi to připadalo vyhovující.


JuDu - 1/6/2007 - 19:53

citace:

Protože jste neuvedl žádné svoje konkrétní výsledky, tak není co zkontrolovat, ale mohu uvést svoje odhady a výpočty.



Dakujem za Vasu pomoc, ja som dostal podobne vypocty. Dosadil som do vzorcov udaje o polomeru Zeme 6370 km, vyska satelitu 36000 km a 48 stupnov.
Satelit vidi moju polohu 7,08 stupnov od roviny rovnika. Ja vidim satelit od vodorovnej roviny pod uhlom 39,92 stupnov. Vase hodnoty sedia.

Dakujem


Lukavský - 3/6/2007 - 19:04

Nevíte prosím, co se zobrazuje na velkém číslicovém panelu v pravé horní části obrazu webkamery NASA č.4 (VAB/různé), která právě snímá vstupní vestibul před Atlantisem? Mohla by to být % vlhtosti a pokud ano proč jsou tak důležitá, že mají vlastní velký displej?


Bryansk - 4/6/2007 - 09:31

V prvních 5 měsících 2007 došlo v počtu úspěšných startů kosmických raket k pozoruhodné situaci : 1. Čína - 6 , Rusko - 6 , 2. USA - 4. Jaká je výše rozpočtů kosmických agentur jednotlivých zemí na letošní rok ? - myslím USA, Rusko, Čínu, Indii, Evropu, Brazílii, Izrael.


bejcek - 4/6/2007 - 10:14

citace:
Nevíte prosím, co se zobrazuje na velkém číslicovém panelu v pravé horní části obrazu webkamery NASA č.4 (VAB/různé), která právě snímá vstupní vestibul před Atlantisem? Mohla by to být % vlhtosti a pokud ano proč jsou tak důležitá, že mají vlastní velký displej?


Ve VAB je služba starající se o vlhkost vzduchu - stará se o její snižování. Budova je tak veliká, že při zvýšená vlhkosti mohou vypadávat srážky ze vzduchu (mrholí). Vzhledem k blízkosti moře ta vlhkost vzduchu bývá vysoká.


ales - 4/6/2007 - 10:23

citace:
V prvních 5 měsících 2007 došlo v počtu úspěšných startů kosmických raket k pozoruhodné situaci : 1. Čína - 6 , Rusko - 6 , 2. USA - 4. Jaká je výše rozpočtů kosmických agentur jednotlivých zemí na letošní rok ? - myslím USA, Rusko, Čínu, Indii, Evropu, Brazílii, Izrael.

Informace o rozpočtech kosmických agentur se snaží udržovat V.Ajgl na http://www.ajgl.cz/space/Agencies.htm .

Ovšem statistiky startů jsou z pohledu rozpočtů VELMI ošidné, protože řada komerčních a vojenských družic, včetně jejich nosičů, vůbec není placena z těchto agenturních rozpočtů, ale z jiných peněz (soukromých, nebo vojenských). Určit, které starty a družice jsou hrazeny ze státních rozpočtů, si osobně netroufám. Je to dost neprůhledné a dokonce i Čína zřejmě vypustila např. NigComSat-1 za peníze Nigérie a jiné starty zase mohly jít z vojenského rozpočtu. Těžko říci. V každém případě jsou čínské kosmonautické výsledky docela solidní a určitě je tu nikdo nezpochybňuje. Těším se na další vývoj nejen v čínské kosmonautice.


Hawk - 9/6/2007 - 08:06

Jsou vůbec ekonomicky dostupné technologie, které by umožnili kosmickým stanicím při správné údržbě vydržet v kosmickém prostoru třeba 100 let. Ptám se v souvisloti na již zaniklé projekty relativně megalomanských orbitálních konstrukcí. Když si to pak člověk spojí třeba s MIRem, který musel jít do kytek už cca po 15 letech...


Ervé - 11/6/2007 - 07:46

Saljut 6 provozován s posádkami 4 roky. Saljut 7 s posádkami 4 roky.
Mir 15 let. ISS patrně 15-25 let (od roku 2000, zatím 7 let).
Myslím že trend je jasný. Životnost 100 let je zatím nedosažitelná kvůli nedostatku peněz - gyroskopy, solární panely, ale i vědecké přístroje s dlouhou životností by se musely vyvinout a na to nejsou peníze. Hmotnost takových dílů by byla větší, vývoj náročný a navíc vždy je určitá míra nespolehlivosti, takže po x letech by se musely vyměňovat velké celky, vědecké přístroje by zastaraly, pokud bychom je draze nevyměňovali. Prostě je to zbytečné a bez raketoplánů to bude ještě mnohem náročnější.


Patek Luboš - 11/6/2007 - 16:03

citace:
Saljut 6 provozován s posádkami 4 roky. Saljut 7 s posádkami 4 roky.
Mir 15 let. ISS patrně 15-25 let (od roku 2000, zatím 7 let).
Myslím že trend je jasný. Životnost 100 let je zatím nedosažitelná kvůli nedostatku peněz - gyroskopy, solární panely, ale i vědecké přístroje s dlouhou životností by se musely vyvinout a na to nejsou peníze. Hmotnost takových dílů by byla větší, vývoj náročný a navíc vždy je určitá míra nespolehlivosti, takže po x letech by se musely vyměňovat velké celky, vědecké přístroje by zastaraly, pokud bychom je draze nevyměňovali. Prostě je to zbytečné a bez raketoplánů to bude ještě mnohem náročnější.


Záleží na vlastní koncepci stanice. Mir byl postaven bez pomoci STS, nebo jemu podobnému stroji. Ale ISS byla od počátku komcipována pro maximální využití STS při stavbě a provzu (pouze ruský segment /mimo vědecko-energetické platformy/ se měl budovat pomocí klasických raket). Takže, pokud bude nástupce ISS koncipován pro stavbu bez STS a jemu podobných prostředků, tak se také bez nich obejde.


Ervé - 12/6/2007 - 07:46

Jistě že se obejde, protože bude muset. Vzhledem k větším přetížením a vibracím při startu běžných raket bude všechno vybavení robustnější. Pokud japonský HTV bude fungovat, bude možné velké a rozměrné bloky vybavení vynášet na stanici a také se jich zbavovat. Rusové možná konečně postaví nákladní loď pro Proton/Angaru. Jinak se budou muset odhazovat celé moduly. Při vracení nákladu na Zem se dají použít bezpilotní verze např. Sojuzu (doveze cca 500 kg nákladu, tedy desetinu toho, co STS), lepší bude Orion.
Mir mohl fungovat tak dlouho, protože byl dvakrát výrazně omlazen - v letech 1989/90 připojením Kvantu 2 a Kristallu a v letech 1994/5 připojením Spektru a Prirody.
Plánuje se vynášení Progressů a Sojuzů raketami Sojuz 2 a 3? Větší nosnost zanmená víc nákladu.


Jiří Hošek - 12/6/2007 - 08:08

citace:
Jsou vůbec ekonomicky dostupné technologie, které by umožnili kosmickým stanicím při správné údržbě vydržet v kosmickém prostoru třeba 100 let. Ptám se v souvisloti na již zaniklé projekty relativně megalomanských orbitálních konstrukcí. Když si to pak člověk spojí třeba s MIRem, který musel jít do kytek už cca po 15 letech...
Jestli jste měl na mysli kilometry dlouhé kosmické elektrárny či kosmická města, tak to je spíš záležitost sci-fi než reálné kosmonautiky 21.století.


Jiří Hošek - 12/6/2007 - 08:17

citace:
... a v letech 1994/5 připojením Spektru a Prirody.

Drobné upřesnění: šlo o roky 1995 a 1996.


hroch - 12/6/2007 - 10:15

Zdravim! Jedna hodne laicka otazka. Dohadujeme se jake realne zvuky, pri praci na obezne draze, astronaut slysi? (radiovou komunikaci do toho nepocitaje). Prenaseji se nejake zvuky (treba v podobe vibraci)pres naradi, rukavice, ruku,...., az k usim astronauta?


Hawk - 12/6/2007 - 12:38

citace:
Jestli jste měl na mysli kilometry dlouhé kosmické elektrárny či kosmická města, tak to je spíš záležitost sci-fi než reálné kosmonautiky 21.století.


Měl jsem na mysli poněkud "skromnější" vizi stanice typu Space Base:

"V roce 1968, kdy vrcholily přípravy k letu na Měsíc, musel NASA opět rozhodovat o dalších perspektivních cílech. O rok později byl navržen projekt obří kosmické základny Space Base se stočlennou osádkou. Měla sloužit i jako základna meziorbitálních tahačů s nukleárními motory pro kyvadlovou dopravu mezi Space Base a stálou základnou na Měsíci. Bylo jasné, že taková obří základna si vyžádá velkokapacitní dopravní obsluhu. Proto se souběžně s návrhem kosmické stanice zrodila myšlenka kosmického raketoplánu Space Shuttle."

http://www.vesmir.cz/clanek.php3?CID=138


Jiří Hošek - 12/6/2007 - 14:25

Díky za odpověď, už jsem se ve Vašem dotazu zorientoval.

Design Life: 10 years.
http://www.astronautix.com/craft/spaebase.htm


Martin Kolman - 12/6/2007 - 21:25

citace:
....na obezne draze, astronaut slysi?....


Hlavně zřejmě slyší běh "klimatizace" která ve skafandru udžuje podmínky pro život. Tyto přístroje mívají řadu ventilátorů a pump, takže mu to vevnitř asi dost hučí (například i na palubě ISS je celkem slušný "kravál" od všech běžících přistrojů). Slyší samozřejmě i svůj vlastní dech.


David - 16/6/2007 - 17:41

Dobrý den,
mám dotaz, přemýšlím jak to vlastně ve vesmíru je s teplem. vše je tam obklopeno vakuem (a vakum je vyborný tepelný izolant), přesto je vše obaleno tepelnou izolací?

z diskuze jsme postřehl že si kosmonauti stežovaly, že jim mrznou konečky prstů(kam se to teplo ztrácí)?, přesto se na stanici montují velké radiátory, které odvádějí přebytečné teplo?

předpokládám že jedínný možný úbytek tepla ve vesmíru je vyzařováním, ale i přesto si myslím že by měl být tepla přebytek, vznikající dopadem slunečního světla..

co třeba nářadí? fotoaparat mají venku zabalený do tepelné izolace ( zde je asi problém že by na něm klesala teplota pod limitní hodnoty ) ale Aku šroubovak izolovaný řekl bych není, spíše na opak, je tmavé barvy aby lépe vyzařoval teplo ?

za případné odpovědi předem děkuji.


Archimedes - 16/6/2007 - 18:14

Chápete to vcelku správně, ale je nutné si uvědomit, že na oběžné dráze je obvykle družice střídavě na plném slunci a v úplné tmě (s tím, že občas si může nějakou část sama zastínit a že do toho má co mluvit i odraz od Země), takže někdy je potřeba chladit a někdy potřeba se ohřát, což se s tepelnou izolací reguluje lépe. U kosmonauta k tomu přistupuje i to, jak rychle a přesně zvládá jeho skafandr reagovat na podobné výkyvy.


JuDu - 18/6/2007 - 18:44

Dobry vecer,

Chcel by som polozit otazku, ako je to s napatovou hladinou 'nuly' vo vesmire. Kedze nie je nijaka referencia Zeme, tak odkial sa berie nula ? Pred pripojenim raketoplanu sa naboje vyrovnaju ako ? Predsa ked sa dokuje, iskra nepreskoci. Ak by to bol vrtulnik, tak by mohol spustit najprv lano na povrch stanice, ale vo vesmire ? Povrch stanice je z vodiveho materialu.

Dakujem

Juraj


Archimedes - 18/6/2007 - 20:44

"Zemneni" se dela pres okolni ridke plazma pomoci tzv. plazmovych kontaktoru, ktere lokalne zvysuji hustotu nabitych castic a tim vytvareji dostatecne velke "konktakni plochy". Takze potencial stanice se dorovnava vuci potencialu okolniho plazmatu.


zippi - 19/6/2007 - 10:29

zdravim, pozeral som zabery z lucenia posadky STS a ISS a medzi tym sa tam objavi zabery (pohlad z okna STS na ISS) ked okolo okna prelietavalo vela "smetia"
viem ze je vo vesmire smetie, ale az tolko? mne to pripadalo tak ako keby tam jemne snezilo aj velkost castic sa podobali velkosti vlociek, i ked kamera moze skreslovat.
to je tak vzdy? teda je to trvaly jav? alebo nahodou prelietavali cez take uzemie...
vdaka


ales - 19/6/2007 - 12:34

> ... (pohlad z okna STS na ISS) ked okolo okna prelietavalo vela "smetia" ... ako keby tam jemne snezilo

Ten záběr se mi nepodařilo najít, ale je skoro jisté že toto "smetí" pocházelo z ISS nebo z připojeného raketoplánu (protože mělo malou relativní rychlost vůči ISS). Šlo zřejmě o "tekutý odpad" vypouštěný občas do okolního prostoru, kde krystalizuje do jemných vloček. Svoje okolí si tak ISS "zaneřáďuje" sama, ale vzhledem k poměrně nízké hustotě toho "sněhu", vše bezpečně zanikne v hustých vrstvách atmosféry během několika týdnů.

Rozhodně nešlo o klasické "kosmické smetí", tedy zbytky družic nebo posledních stupňů raket, protože relativní rychlost takových objektů bývá vůči ISS tak vysoká (nejméně stovky metrů za sekundu), že nemohou být pozorovány jako "sněžení" v tandemu s ISS.


zippi - 19/6/2007 - 13:03

videl som to na NasaTv dnes ráno (chvílu pred tým ako som písal príspevok) bol to zostrih z predošlého dňa, chvílu po tom ako sa rozlúčila posádka, bolo tam viac záberov z STS...
Boli to malé častice, možno ako drobný sneh, a vyzeralo to tako sneženie (rýchlost pohybu).
Može to byť aj prach ktorý sa nachádza volne v priestore? Alebo ten je taký riedky ze ho nieje možné tak vidieť?
Slnko svietilo približne oproti, čo asi zoslilňovalo ten efekt.


avitek - 19/6/2007 - 14:30

citace:
videl...
Boli to malé častice, možno ako drobný sneh, a vyzeralo to tako sneženie (rýchlost pohybu)...


Včera se vypouštěla odpadní voda z raketoplánu, a s největší pravděpdobnosstí to mohlo být právě ono. Nedíval jsem ne na tnto pořad, ale časem si ho stáhnu.


avitek - 19/6/2007 - 14:36

Podle letového plánu na 11. den letu (2007-06-18) mělo se provádět vypouštění odpadní vody v době od 12:50 do 14:00 UTC (14:50 - 16:00 SELČ).


Hawk - 19/6/2007 - 20:48

Kdysy zde probíhala diskuse o pobytu člověka ve vakuu, této problematice se věnuje část článku zde:
http://www.distantworlds.wz.cz/DisWorlds1-2/Zivot/Meze.htm


zippi - 20/6/2007 - 12:28

spomínané "smetie" ci "sneženie" je vidieť na tomto videu http://labrador.freepgs.com/2m/STS-117%20-%20FLIGHT%20DAY%2011%20HIGHLIGHTS.ASF 43,9 MB resp. http://www.space-multimedia.nl.eu.org/index.php?option=com_remository&Itemid=63&func=fileinfo&id=791 v čase videa 12:50 až 13:20


ales - 20/6/2007 - 12:43

Podíval jsem se na "FD-11 Highlights" a myslím, že to jsou opravdu následky vypouštění odpadní vody do kosmu (odpovídá to i letovému plánu). V okolí raketoplánu ani ISS rozhodně takto "nesněží" běžně ale jen při těchto "příležitostech" (při vypouštění odpadu).


JuDu - 23/6/2007 - 14:22

Dobry den,

Chcel by som polozit otazku okolo pneumatik pre raketoplan. Su pneumatiky plno nafuknute aj vo vesmire ? Su sachty kolies vzduchotesne ? Je vypracovany velmi-velmi nepravdepodobny scenar, ze by sa mal rakatoplan pristat na 'brucho' ?

Dakujem


Adhara - 23/6/2007 - 20:51

Chcela by som položiť jednu nezvyčajnú otázku, odpoveď potrebujem čo najskôr. Čo by sa stalo, ak by niekto čisto teoreticky odpojil klimatizačný systém v orbiteri? Ak by sa zastavila cirkulácia vzduchu? Čo by to urobilo s teplotou, tlakom a vlhkosťou vzduchu, za aký čas by sa takéto podmienky stali nezlúčiteľné so životom?


:-(( - 23/6/2007 - 22:38

citace:
...odpoveď potrebujem čo najskôr...


???????????????????????????????????????????????????????????


O čom to je? Novinárom sa nechce študovať, tak hľadajú "informácie" a DRBY na verejných fórach? Kozmonautika nie je dosť zaujimavá, pokiaľ sa niečo nepokazí alebo pokiaľ sa nevytvorí ilúzia ohrozenia života?

???????????????????????????????????????????????????????????


ales - 23/6/2007 - 23:03

citace:
Chcel by som polozit otazku okolo pneumatik pre raketoplan. Su pneumatiky plno nafuknute aj vo vesmire ? Su sachty kolies vzduchotesne ? Je vypracovany velmi-velmi nepravdepodobny scenar, ze by sa mal rakatoplan pristat na 'brucho' ?

Myslím, že pneumatiky raketoplánu jsou určitě nafouknuté i ve vesmíru (nevidím důvod, proč by neměly být). Podvozkové šachty zřejmě nejsou vzduchotěsné (nevidím důvod, proč by měly být). Nevím, jestli je vypracován scénář přistání raketoplánu "na břicho", ale když by se mu nevysunul podvozek, tak by mu prostě nic jiného nezbylo (nemá možnost udělat cokoliv jniného). Jsem přesvědčen, že takové přistání by posádka přežila, ale raketoplán by asi byl dost silně poškozen.


ales - 23/6/2007 - 23:14

citace:
Chcela by som položiť jednu nezvyčajnú otázku, odpoveď potrebujem čo najskôr. Čo by sa stalo, ak by niekto čisto teoreticky odpojil klimatizačný systém v orbiteri? Ak by sa zastavila cirkulácia vzduchu? Čo by to urobilo s teplotou, tlakom a vlhkosťou vzduchu, za aký čas by sa takéto podmienky stali nezlúčiteľné so životom?

Nevím, jestli je k výše uvedenému někde k dispozici odborný rozbor, ale dovolím si sem napsat svůj hrubý odhad následků. Pokud by kompletně a nevratně vypadl systém ventilace, klimatizace a termoregulace, tak by se prostě postupně zhoršovala kvalita atmosféry v orbiteru. Teplota by zřejmě rostla, vlhkost asi také. Tlak by se snad dal udržovat ručně doplňováním kyslíku z tlakových lahví. Dýcháním posádky by se zvyšovala koncentrace oxidu uhličitého ve vzduchu. Přesto si myslím, že i za takovýchto podmínek by posádka mohla přežít několik hodin a snad až několik dní (mělo by to asi stačit k tomu, aby raketoplán mohl bezpečně přistát [pokud by mu fungovaly systémy potřebné pro přistání]). Pokud máte někdo přesnější informace, nebo jiné odhady, tak mne, prosím, opravte.


Adhara - 23/6/2007 - 23:16

citace:
citace:
...odpoveď potrebujem čo najskôr...


???????????????????????????????????????????????????????????


O čom to je? Novinárom sa nechce študovať, tak hľadajú "informácie" a DRBY na verejných fórach? Kozmonautika nie je dosť zaujimavá, pokiaľ sa niečo nepokazí alebo pokiaľ sa nevytvorí ilúzia ohrozenia života?

???????????????????????????????????????????????????????????



Takže stručne: Nie som novinár. Nenávidím novinárov, ktorí vytvárajú v každej misii katastrofické scénare. O kozmonautiku sa zaujímam už dlhé roky, prečítala som toho veľmi veľa o raketoplánoch a tiež som o nich veľa napísala. Ale toto je taká špecifická otázka, že som na ńu nenašla nikde odpoveď. Aspoň nie v slovenčine alebo češtine... a iné jazyky neovládam.

Bola by som veľmi rada, keby mi už niekto napísal aj niečo iné okrem nadávok, vyrývania a výsmechu. Napríklad odpoveď. Podľq možností aj podloženú nejakým tým zdrojom (v krajnom prípade anglickým).

ˇˇDakujem za uvítanie."


Adhara - 23/6/2007 - 23:24

citace:

Nevím, jestli je k výše uvedenému někde k dispozici odborný rozbor, ale dovolím si sem napsat svůj hrubý odhad následků. Pokud by kompletně a nevratně vypadl systém ventilace, klimatizace a termoregulace, tak by se prostě postupně zhoršovala kvalita atmosféry v orbiteru. Teplota by zřejmě rostla, vlhkost asi také. Tlak by se snad dal udržovat ručně doplňováním kyslíku z tlakových lahví. Dýcháním posádky by se zvyšovala koncentrace oxidu uhličitého ve vzduchu. Přesto si myslím, že i za takovýchto podmínek by posádka mohla přežít několik hodin a snad až několik dní (mělo by to asi stačit k tomu, aby raketoplán mohl bezpečně přistát [pokud by mu fungovaly systémy potřebné pro přistání]). Pokud máte někdo přesnější informace, nebo jiné odhady, tak mne, prosím, opravte.


Ďakujem za odpoveď. Ešte jedna zvláštna otázka - pracuje klimatizačný systém ISS na podobnom princípe ako ten v raketopláne? Bolo by možné v prípade núdze tieto dva systémy medzi sebou prepojiť? Resp. prepojiť tak, aby jeden úplne nahrádzal druhý? (Trochu ma inšproval ten výpadok stabilizačných systémov počas STS-117.)


Tomáš_Kovařík - 23/6/2007 - 23:37

Nevím, jestli je vypracován scénář přistání raketoplánu "na břicho", ale když by se mu nevysunul podvozek, tak by mu prostě nic jiného nezbylo (nemá možnost udělat cokoliv jniného). Jsem přesvědčen, že takové přistání by posádka přežila, ale raketoplán by asi byl dost silně poškozen.




pokud vím, tak raketoplán má za takové nouzové situace možnost přistát na vodě, kde je schopný několik hodin "plavat" nad hladinou. To by byla určitě lepší varianta než přistání bez podvozku na asfaltobetonu, i když nevím, jak by potom raketoplán zachránili, aby jim nezmizel pod hladinou...


xmariox - 23/6/2007 - 23:44

Nevim to jiste ale myslim ze jsem nekde cetl ze v pripade ze se nevysune podvozek jsou tam nejaka pyrotechnicka zarizeni a ty jej vysunou...


leemer - 24/6/2007 - 00:02

citace:
Nevím, jestli je vypracován scénář přistání raketoplánu "na břicho", ale když by se mu nevysunul podvozek, tak by mu prostě nic jiného nezbylo (nemá možnost udělat cokoliv jniného). Jsem přesvědčen, že takové přistání by posádka přežila, ale raketoplán by asi byl dost silně poškozen.


pokud vím, tak raketoplán má za takové nouzové situace možnost přistát na vodě, kde je schopný několik hodin "plavat" nad hladinou. To by byla určitě lepší varianta než přistání bez podvozku na asfaltobetonu, i když nevím, jak by potom raketoplán zachránili, aby jim nezmizel pod hladinou...




no jenom poupravim, ze to neni nekolik HODIN, ale jenom par MINUT


marcellino4 - 24/6/2007 - 00:14

k problematike klimatizacie:

niekde som cital ze 3 celnna posadka by mala byt shopna prezit na ISS bez obmeny a zasahov do atmosfery cca jeden tyzden. toto tvrdenie sa opieralo o fakt ze objem priestorov na ISS je velmi velky. no neviem co je na tomto udaji pravda.

a este jedna informacia ktora by s tym mohla suvisiet: po columbii udajne padlo rozhodnute aby zasoby na ISS vystacili na 90 dni pre 7 clennu posadku raketoplanu. udajne to bolo overene aj nejakou simulaciou ze by to fakt bolo mozne v pripade nudze uskutocnit.


Jano - 24/6/2007 - 03:26

".. raketoplán má za takové nouzové situace možnost přistát na vodě .."
V okamihu vysúvania podvozku zostáva do touch-down už len asi 10-15 s. Takže ak sa nevysunie, tak už nemajú veľmi ako hľadať nejakú vodu - na KSC by to síce mohli rýchlo ztočiť do toho paralelného kanálu a pristáť na hlave nejakému aligátorovi, na Edwards ale žiadna voda nie je.
"To by byla určitě lepší varianta než přistání bez podvozku na asfaltobetonu.."
Prašť jak uhoď (V Bratislave pred pár rokmi na leteckom dni úspešne pristála na bruchu ruská stíhačka, keď sa jej nevysunul podvozok)
Ale, svojho času ma napadlo, ako vylúčiť potrebu podvozku v orbiteri: Na dráhe by bolo pripravené niečo ako veľké sánky, resp. ploché vozidlo rozmeru okolo 20x30 m, ktoré by tesne pred príletom shuttlom rozbehli na jeho rýchlosť a raketoplán by pristál na ňom, pričom zabrzdenie by už malo na starosti to vozidlo. Prípadne by ho mohli "odchytiť" ešte v lufte. Nesmejte sa! - radšej pridajte nejaký zlepšovák, nech sa to o pár rokov môže začať používať (aspoň pre jeho nasledovníkov).


Jano - 24/6/2007 - 03:40

oprava: .. tesne pred príletom shuttlu ..
doplnenie: a kludne by to mohlo byť koľajové vozidlo - koľaje namiesto dráhy (podobne ako tie sánky, na ktorých testovali na človeku 30g)


nickdo - 24/6/2007 - 09:34

podvozek by se otevřel tak jako tak, odstřelej ho pyropatrony. Tak že je vlastně nemožný aby se neotevřel. A kdyby měl přistávat na břicho, asi by to hlavně odvesly dlaždice, který se stejně vyměňujou. Podle mě by, ale generálka trvala skoro do doby ukončení letů raketoplánů v 2010


ales - 24/6/2007 - 10:27

citace:
Ešte jedna zvláštna otázka - pracuje klimatizačný systém ISS na podobnom princípe ako ten v raketopláne? Bolo by možné v prípade núdze tieto dva systémy medzi sebou prepojiť? Resp. prepojiť tak, aby jeden úplne nahrádzal druhý? (Trochu ma inšproval ten výpadok stabilizačných systémov počas STS-117.)

Protože složení atmosféry v ISS i v raketoplánu je stejné, tak předpokládám, že i "klimatizační systémy" budou obdobné. "Propojit" by se určitě daly pouhým otevřením průlezů mezi ISS a raketoplánem, případně navíc pružnou hadicí s ventilátorem pro lepší cirkulaci vzduchu. Takovéto řešení by určitě v pohodě vydrželo několik dní.

Je třeba ale připomenout, že si prakticky nedovedu představit něco jako "totální výpadek klimatizačního systému", protože jednak jsou některé jeho díly zálohovány, jednak lze závadu opravit, a také lze navíc většinou vymyslet řešení, jak využít jiných zařízení na palubě ke "klimatizačnímu účelu" (viz. např. Apollo 13). Proto tyto "úvahy" berme opravdu jen jako "akademické", protože za normálních okolností k tomu nikdy nedojde a při krizi se bude muset reagovat na okamžitou situaci na palubě, kterou předem nikdo nedokáže ani odhadnout.


Adhara - 24/6/2007 - 15:47

citace:


Je třeba ale připomenout, že si prakticky nedovedu představit něco jako "totální výpadek klimatizačního systému", protože jednak jsou některé jeho díly zálohovány, jednak lze závadu opravit, a také lze navíc většinou vymyslet řešení, jak využít jiných zařízení na palubě ke "klimatizačnímu účelu" (viz. např. Apollo 13). Proto tyto "úvahy" berme opravdu jen jako "akademické", protože za normálních okolností k tomu nikdy nedojde a při krizi se bude muset reagovat na okamžitou situaci na palubě, kterou předem nikdo nedokáže ani odhadnout.


Veď na začiatku som spomínala, že je to len čisto teoretické - ani ja neverím, že by k niečomu podobnému mohlo dôjsť. Len ma tak napadlo, ako dlho by vydržala posádka nažive v raketopláne, ktorý by z nejakých dôvodov skrátka vôbec nemohol pristáť. Keď sa však tak nad tým zamýšľam, nedošli by im skôr batérie ako kyslík?

Našťastie, po tých opatreniach, ktoré urobili po Columbii, nehrozí, že by takáto situácia nastala.


Adolf - 24/6/2007 - 22:37

Co se týče ohledně toho totálního výpadku klimatizace si dovoluji vyslovit hypotézu, že při totálním dalšími opatřeními nekompenzovaném výpadku, pokud by nedocházelo ani k rozhývávání vzduchu v kabině nějakým míchacím pohybem ani k přemisťování osob, by prvním se životem neslučitelným účinkem byl vzestup koncentrace C02 v blízkosti člověka, který by promíchávání pouhou difúzí nestihlo odstranit. Zatímco třeba tepelný účinek výpadku klimatizace by závisel např. na trajektorii pohybu a z toho polynoucí sluneční expozici, což by z odhadu činilo záležitost s mnoha vstupními parametry, tak to zadušení CO2 při nepromíchávání a pokojové teplotě by šlo vypočítat asi nejjednodušeji, kdyby na to někdo měl chuť.


Ervé - 25/6/2007 - 14:59

Při výdechu uniká vzduch nenulovou rychlostí, nádech také, takže k víření dochází, i když člověk spí (nebo je v bezvědomí). Člověk na zvýšený CO2 reaguje - zrychlené dýchání, bolesti hlavy, nevolnost - takže by trvalo několik hodin, než by účinek byl nebezpečný. Vydýchat 1 m3 při spánku trvá cca 3 hodiny.
Až se poletí k HST, bude asi v nákladovém prostoru plošina EDO se zásobami kyslíku a vodíku pro palivové články (a dýchání), takže v případě velkých potíží by raketoplán v minimálním režimu spotřeby vydržel dost dlouho (30 dní?), aby mohla startovat záchranná výprava. Dosavadní lety ale myslím dobře ukazují, že poškození sice vzniká, ale je minimální a opravitelné, takže snad záchranný let nebude nikdy potřeba.


Jaro - 26/6/2007 - 16:08

Prepacte,

ze to pisem tu, ale neviem si dat rady s jednym problemom a to, ze niektore stranky v rustine mi nabiehaju bez azbuky s roznymi cudnymi znakmi - napr. stranka Lavockina alebo ruska verzia energie. Portal novosti alebo stranka roscosmosu nabiehaju v poriadku. Nevedel by mi niekto poradit?

Napr.:

 Ïàðèæå çàâåðøèëàñü ðàáîòà 47-ãî Ìåæäóíàðîäíîãî ñàëîíà àâèàöèîííîé è êîñìè÷åñêîé ïðîìûøëåííîñòè "Ëå Áóðæå-2007", îäíîãî èç âåäóùèõ àýðîêîñìè÷åñêèõ ñàëîíîâ ìèðà.

ÔÃÓÏ "ÍÏÎ èì. Ñ.À. Ëàâî÷êèíà" - ðàçðàáîò÷èê è èçãîòîâèòåëü àâòîìàòè÷åñêèõ êîñìè÷åñêèõ àïïàðàòîâ, ïðèíÿëî àêòèâíîå ó÷àñòèå â ýêñïîçèöèè ñàëîíà ñðåäè âåäóùèõ ðàêåòíî-êîñìè÷åñêèõ ïðåäïðèÿòèé Ðîññèè.


Vítězslav Novák - 26/6/2007 - 17:55

V prohlížeči View (Zobraz) Encoding (snad znaková sada) a podívat se, co je nastaveno. Je dobré mít zaškrtnuto Auto-Select, pak by měl většinu rozpoznat sám. Když ne, tak zkoušet různé Cyrillic nebo Unicode. Ten by měl být bez problémů.


Jaro - 27/6/2007 - 00:42

Dakujem velmi pekne. Vidim, ze to je velmi jednoduche, ale aj tak ste velmi pomohli.


Vítězslav Novák - 27/6/2007 - 13:03

To nic. Teoreticky by to mělo nabíhat samo, ale ne každá stránka má správně definovanou znakovou sadu.


Abraxis - 27/6/2007 - 21:45

Vim, ze to neni primo o kosmonautice, ale co bych se nezeptal ;-)

Mam takovy myslenkovy experiment a nevim, kde je chyba (musi tam byt :-)) - prosim o napady.

Vychozi predpoklad - teorie relativity omezuje zasilani informaci na maximalne rychlost svetla. Ted si predstavme 3 tyce dlouhe pro nazornost 1 svetlny rok. Prostredni tyc je jako "referencni". Podle toho zda posuneme dopredu/dozadu levou nebo pravou tyc, tak zasleme informaci (0 nebo 1). Naivni myslenka je, ze se oba konce pohnou najednou - tedy zasilame informaci na vzdalenost 1 sv.roku okamzite. Coz je blbost. Kde je ta chyba?

Napada mne jen jedno vysvetleni - tyce nejsou "hmota" ale molekuly provazane mezimolekularnimi vazbami. Tyto vazby maji "pruznost", ktera omezuje sireni zmen na max. rychlost svetla.

Dik za komentare :-)


MIZ - 27/6/2007 - 21:51

citace:
... Napada mne jen jedno vysvetleni - tyce nejsou "hmota" ale molekuly provazane mezimolekularnimi vazbami. Tyto vazby maji "pruznost", ktera omezuje sireni zmen na max. rychlost svetla.

Jak říkával jeden z mých profesorů: No vidíte, vždyť vy to víte! Tak proč to neřeknete hned?
Ještě si dovolím upravit, že tyče hmota jsou a hmota je takto pružná, i když se tak na první pohled třeba netváří.


Jano - 28/6/2007 - 02:39

Tá informácia by sa dokonca ani nešírila rýchlosťou svetla, ale len rýchlosťou zvuku v danom materiáli. Isteže, gravitačná a možno aj nejaká elmag. vlna spojená s ťuknutím na začiatok tyče by odtiaľ hneď letela rýchlosťou svetla, ale by bola extrémne slabá ... chcel som povedať slabá v porovnaní s intenzitou tej zvukovej vlny, ale vlastne aj tá by už po pomerne krátkej dobe asi dissipovala v materiáli tyče.

Pripomína mi to vetu z jedného pop-ved filmu, že Saturn ma malú hustotu a preto by v obrovskom pohári vody plával. No, kým by sa 'horná' polovica planéty dozvedela, že na 'dolnú' už tlačí voda, ... A aký by bol tlak vody v tej hĺbke Rsat? A čo by bolo zdrojom grav.poľa pre takú konšteláciu? Niektoré príklady sa asi nemajú chytať za slovo

Ešte dodám, že nikdy som nemal rád tvrdenia, že v nevákuu sa svetlo šíri pomalšie. Je to mierne zavádzajúce: interakcia sa predsa hocičím šíri presne rýchloťou svetla, to zdanie spomalenia elmag.vĺn v materiáloch je dané iba tým, že do 'hry' sa okrem pôvodného zdroja vzruchu zapájajú aj všetky ostatné častice, ku ktorým už dorazila pôvodná vlnoplocha, a až celý ten ansámbel interakčné vzruchy vysielajúcich častíc dohromady vyrába ten výsledok, že výsledná fázová rýchlosť vlny je menšia ako c.
Dúfam, že som tu nezamenil fázovú s grupovou rýchlosťou - už pár rokov som sa s nimi nestretol. Hm, a teraz mi hlava hovorí, že grupová by asi vždy mala byť rovná c, ale ....; hm, dneska by som skúšku z EM asi nespravil


TH - 28/6/2007 - 11:00

citace:

Napada mne jen jedno vysvetleni - tyce nejsou "hmota" ale molekuly provazane mezimolekularnimi vazbami. Tyto vazby maji "pruznost", ktera omezuje sireni zmen na max. rychlost svetla.



Tie medzimolekulove vazby, to su vlastne elektrostaticke sily, teda elektromagneticke pole. Ked potiahnete prvu molekulu v tyci, nasledujuca molekula v tyci sa to dozvie tak, ze sa to k nej presiri vyssim potencialom elektromagnetickeho pola. No a to sirenie el.mag vzruchu medzi molekulami prebieha rychlostou svetla.


Adhara - 28/6/2007 - 12:09

Ďalšia laická otázka ohľadom raketoplánov: Môžu palubné počítače raketoplánu zobrazovať telemetrické údaje, ktoré prijíma MCC v Houstone? Mohla by sa posádka raketoplánu dozvedieť údaje napr. o teplote, aerodynamickom namáhaní počas zostupu, atď. aj bez komunikácie s riadiacim strediskom?


Abraxis - 28/6/2007 - 16:46

citace:
Tá informácia by sa dokonca ani nešírila rýchlosťou svetla, ale len rýchlosťou zvuku v danom materiáli. Isteže, gravitačná a možno aj nejaká elmag. vlna spojená s ťuknutím na začiatok tyče by odtiaľ hneď letela rýchlosťou svetla, ale by bola extrémne slabá ... chcel som povedať slabá v porovnaní s intenzitou tej zvukovej vlny, ale vlastne aj tá by už po pomerne krátkej dobe asi dissipovala v materiáli tyče.



Jen pro upresneni - ja nemel na mysli sirit informaci "zvukem" (tuknutim), ale zmenou pozice ("zasunutim") tyce.


ales - 28/6/2007 - 22:06

citace:
Ďalšia laická otázka ohľadom raketoplánov: Môžu palubné počítače raketoplánu zobrazovať telemetrické údaje, ktoré prijíma MCC v Houstone? Mohla by sa posádka raketoplánu dozvedieť údaje napr. o teplote, aerodynamickom namáhaní počas zostupu, atď. aj bez komunikácie s riadiacim strediskom?

Odpověď bohužel neznám přesně, ale protože podle např. http://spaceflight.nasa.gov/shuttle/reference/shutref/orbiter/avionics/dps/gpc.html jsou GPC počítače raketoplánu přímo napojeny na telemetrii (která je teprve následně odesílána rádiem do MCC), tak se domnívám, že všechny dostupné údaje by měly být na GPC počítačích zobrazitelné. Normálně se ovšem určitě na GPC zobrazují jen souhrnné údaje potřebné a vhodné pro řízení. Sledovat měnící se hodnoty ze stovek snímačů, které jsou na raketoplánu, je pro posádku určitě hodně obtížné a nepřehledné, takže se to asi běžně nedělá. (opakuji ale, že výše uvedené úvahy jsou jen mým názorem bez jasného "důkazu", takže pokud někdo máte přesnější informace, napište je sem, prosím)


Adolf - 30/6/2007 - 18:39

citace:

Jen pro upresneni - ja nemel na mysli sirit informaci "zvukem" (tuknutim), ale zmenou pozice ("zasunutim") tyce.


Tou tyčí se při zatlačení na jeden konec šíří elastický kmit rychlostí zvuku v příslušném materiálu a vzdálenější části tyče jsou odtlačeny tímto elastickým kmitem. Je-li tyč velmi tuhá a zrychlení malé, tak si toho nevšimneme. Např. ale když vystřelíme šíp z luku, tak právě díky těmto kmitům nevadí opeření šípu, neboť kmitající šíp opeřeným koncem uhne a neotře se o luk. U velkých konstrukcí - např. u příhradových nosníků jeřábů je každý pohyb ramene spojen s výraznými kmity konstrukce. Kdo si kdy vlezl na jeřáb, užil si těch vemi patrných kmitů hodně. Nic není tuhé, vše je ve skutečnosti kmitající gel - úplná huspenina a síla se uvnitř přenáší jen jako akustická či tlaková vlna.

Ovšem "ďábelské" úvahy o okamžitém působení na dálku je možno realizovat poněkud jinak - např. přes kvantové vázené stavy. Např. je emitována dvojice fotonů procesem, při němž oba fotony mají zákonitě opatře orientovanou polarizaci. Kadý odletí na jinou stranu. Nepotká je žádná absorbce či jiný proces, který by zhroutil jejich společnou vlnovou funkci. Uletí každý tak světelný rok, když se jednomu fotonu připlete do cesty polarizační hranol. Projde jím, takže jeho polarizace se pootočí. Leč v tom okamžiku je pootočen provázaný foton tak, že je pořád antiparalelní.

Dokonce se při tom dá zlobit o trochu víc. Ten hranol bude mít přepínatelnou polarizaci a někdo u něj bude přepínat směr polarizace, jak se mu zlíbí. Chvilku poté, na druhém konci budeme snímat polarizaci dopadlých fotonů z dvojite o světelné roky dál. Myslíme si tedy, že jsme tím marnivě přepínaným hranolem zapřínili stav detektoru polarizace 2 ly daleko o vteřinu později. Může to ale být ještě ošemetnější. Poletíme superrychlou raketou hnanou třeba plynem z gamazáblesku. Tyto plyny létají třeba i rychleji než 99,99 % rychlosti světla. V tom případě se projeví relativita současnosti. Pozorovateli v té rychlé raketě se pří vhodné orientaci vektoru rychlosti bude jevit dopad fotonu na detektor polarizace dříve než průchod provázeného fotonu hranolem. Takže podle klasické kauzální logiky by tedy dopad fotonu na detektor měl způsobit, že operátor u přepínače polarizace hranolu se rozhodl hranol o 2 ly dál přepnout.


Adolf - 30/6/2007 - 19:01

Mám také laickou otázku. V katalogu kosmických sond se uvádí parametry dráhy. Je tam tedy uvedeno na jaké elipse - přesně platné v okamžiku epocha se nachází. Nevidím tam ale, žádnou z poloh k určitému okamžiku. Nemůžu si tedy za předpokladu stability dráhy a Keplerových zákonů vypočítat zdánlivé polohy příslušného tělesa na obloze ke zvolenému okamžiku a poloze porozovatele. Bylo by možné si nějaký takový počáteční bod pro výpočet poloh také někde najít?


Jano - 1/7/2007 - 01:51

Už dávnejšie som sa tým nezaoberal, ale myslím, že v TLE je obsiahnutá nielen info o dráhe, ale aj o polohe na nej v nejakom čase. Možno pomôže niečo z tohoto:
http://en.wikipedia.org/wiki/TLE
http://celestrak.com/columns/v04n03/
http://amsat.org/amsat/keps/kepmodel.html

Niekoľko medziplanetárnych sond by mal poskytovať systém Horizons - http://ssd.jpl.nasa.gov/?horizons , a to dokonca vo forme [x,y,z](t); možno raz pridajú aj družice Zeme.

O počítaní dráh sa debatilo aj na http://kosmo.cz/modules.php?op=modload&name=XForum&file=viewthread&fid=3&tid=854&page=5


DH - 1/7/2007 - 08:12

citace:
Už dávnejšie som sa tým nezaoberal, ale myslím, že v TLE je obsiahnutá nielen info o dráhe, ale aj o polohe na nej v nejakom čase.


Je to tak. Pozici telesa podle intergrace pohybovych rovnic popisuje 6 integracnich konstant (5 urcuje tvar a orientaci elipsy v inercialni soustave, tj. poloosa, vystrednost, inklinace, argument perigea, raan) a jeden urcuje polohu hmotneho bodu na teto elipse, tj. mean anomaly.

Mean anomaly je aktualni uhel pruvodice od "pruvodice perigea". Cili aktualni poloha na elipse. A kdyz se k tem 6 prida jeste parametr sedmy, tj. cas, ve kterem tech 6 platilo, vime, ve kterem case ona poloha na elipse platila.

TLE toto vsechno obsahuje a jeste par informaci navic, ktere nejsou potreba pro bezne vypocty, ale vyuziva jich nativni algoritmus pro TLE, tj. SGP/SDP prediktor.


Jano - 1/7/2007 - 12:18

>>A kdyz se k tem 6 prida jeste parametr sedmy, tj. cas, ve kterem tech 6 platilo, vime, ve kterem case ona poloha na elipse platila.

Predpokladám, že ak by bola dráha telesa čisto keplerovská (žiadna precesia, odpor vzduchu apod.), tak by ten siedmy parameter nebol potrebný.
Inak, 6 čísel vo forme r0,v0 (v oboch prípadoch samozrejme vektory) mi pripadá sympatickejších, než tie, čo sa dávajú do TLE.

Ešte pridám svoju otázku, či niekto vie, prečo bolo topografické meranie SRTM realizované len ako jedna misia raketoplánu a nie ako družica pracujúca samostatne niekoľko mesiacov až rokov (vypustená buď klasicky, alebo zo Shuttlu). Resp., prečo je takou až teraz vypustený TerraSAR-X.


DH - 1/7/2007 - 14:06

citace:
Predpokladám, že ak by bola dráha telesa čisto keplerovská (žiadna precesia, odpor vzduchu apod.), tak by ten siedmy parameter nebol potrebný ...


To predpokladate spatne. 6 cisel popisuje polohu v souradne soustave. Sedmym je cas, kdy toto plati. V zasade je jedno, jestli tech 6 cisel budou kepleriany nebo kartezske vektory udane ve slozkach v jakesi soustave (kterou musite vzdy jeste nadefinovat, v jake je mate).

Nepravidelnosti v pohybu dvou teles (ne idealne centralni GP Zeme diky zplosteni, vlivu GP SLunce, Mesice, Jupitera a vubec vseho, zbytky atmosfery, tlak slunecniho zareni, GP vody v oceanech ktera se presouva s prilivem a odlivem a X dalsich mene vyznamnych vlivu) zpusobuji, ze vysledny pohyb nema s Keplerovskym spolecneho vubec nic.
Nastesti nesjou odchylky tak velke, aby se nedaly povazovat za pouhe "perturbace" v keplerianech (navic hodne odchylek ma krome slabe trendove i vyznamnou periodickou slozku) a slusnym zpusobem je modelovat.

Nejlepsi model, povazovany za referencni, je SGP/SDP, bud ve verzi 4 nebo 8. Je primo konstruovan na to, aby do nej vstupovaly parametry z TLE, dokonce soucasti te originalni knihovny jsou i rutiny pro nacitani textoveho TLE (s chybami, ale diky za ty dary). To je mozna duvodem, proc se forma TLE pouziva nejcasteji.

Ostatne, asi budete souhlasit, ze vedet, jaka je inklinace a RAAN (a zhruba vystrednost a poloosa, nebo jeste lepe rozdvojena na apogeum a perigeum) je lidskemu nazornemu mysleni a predstave blizsi nez jakasi hausnumera v stovkach kilometru a kilometrech za sekundu.


Adolf - 2/7/2007 - 16:54

Pánové, děkuji za zajímavé odkazy a doplňkový výklad!

Dovolím si ještě jednu laickou otázku: Jaký je správný terminologický překlad výrazu ASCENDING NODE?

Ovšem, ač se mi dostalo hojnosti informací, k původně zamýšlenému účelu posloužit nemohou. Přiznám se, že jsem si chtěl vytvořit udělátko v Excelu, které po natažení parametrů dráhy bude počítat polohu a zdánlivou polohu na obloze k určenému místu a času. Mělo to vycházet z té nejjednodušší aproximace přesně keplerovského pohybu a z tvaru zeměkoule jako koule. Tímto jste mi to rozmluvili.


DH - 2/7/2007 - 19:00

citace:
Pánové, děkuji za zajímavé odkazy a doplňkový výklad!

Dovolím si ještě jednu laickou otázku: Jaký je správný terminologický překlad výrazu ASCENDING NODE?

Ovšem, ač se mi dostalo hojnosti informací, k původně zamýšlenému účelu posloužit nemohou. Přiznám se, že jsem si chtěl vytvořit udělátko v Excelu, které po natažení parametrů dráhy bude počítat polohu a zdánlivou polohu na obloze k určenému místu a času. Mělo to vycházet z té nejjednodušší aproximace přesně keplerovského pohybu a z tvaru zeměkoule jako koule. Tímto jste mi to rozmluvili.


Ascending node = vzestupny uzel, doslova. Je to bod, ve kterem obezna elipsa protina rovinu rovniku (ty jsou dva) a zaroven v nem satelit proleta z JIHU na SEVER "zespoda nahoru". Samozrejme diky ne zcela idealnimu pohybu je poloha tohoto bodu (brano v inercialni soustave) pokazde mirne jina. Rektascenze tohoto bodu (uhel mezi pruvodicem tohoto bodu a pruvodicem jarniho bodu je potom RAAN).

Polohu a zdanlivou polohu na nebeske sfere vam spocita spousta freeware programu, Nova, Orbitron ... Pokud chtete zkusit ten, ktery pouziva nase czCube, zkuste www.triada.wz.cz ...


Adolf - 2/7/2007 - 19:51

citace:

Polohu a zdanlivou polohu na nebeske sfere vam spocita spousta freeware programu, Nova, Orbitron ... Pokud chtete zkusit ten, ktery pouziva nase czCube, zkuste www.triada.wz.cz ...


Děkuji za další informace a odkaz na program, který to udělá za mě. Asi ho občas použiji. Nicméně ono účelem mělo být víc než mít takový program, udělat si cvičeníčko, které mi pomůže si věci trochu osvojit. Když se s něčím seznamuji, nejlépe to udělám tak, že si aspoň zjednodušeně cvičně něco spočítám nebo naprogramuji. Zároveň si tím i osvěžím některé školní znalosti. Tento účel měla mít i tahle hračka. Profesionální kalkulátor, ze kterého uvidím jen uživatelské rozhraní a výsledky, mě už tolik neuspokojí a budu si muset vymyslet jinou hračku.

K tomu nešťastnému termínu: Z popisu ve Winkině jsem pochopil, co to ten ascending node věcně je. Ale jako člověka, který si trochu přivydělává překlady, mě štve, že neznám český terminologický ekvivalent, i když text o takovéto problematice jsem nikdy nepřekládal a nejspíš ani nebudu. Žádný další výraz, k němuž bych neznal český ekvivalent, jsem tam nenašel. Překládat jen ze znalosti slov, ze kterých je víceslovný výraz složen, může být velice ošemetné. Pokud bych musel u tohoto výrazu provést překlad odhadem (bez znalosti ověřeného ekvivalentu), asi bych použil vrchol vzestupu.


DH - 2/7/2007 - 20:38

citace:
Nicméně ono účelem mělo být víc než mít takový program, udělat si cvičeníčko, které mi pomůže si věci trochu osvojit.


Skvely zpusob sebevzdelani, ale poustet se do toho from scratch neni efektivni.

Je celkem zbytecne, abyste stravil vikend nad odvozovanim a dospel k tomu, ze neexistuje analyticky vzorec, ktery vam ze znamych keplerianu a casu po dosazeni vrati slozky polohoveho vektoru a slozky vektoru rychlosti. To, ze jednim z kroku je reseni Keplerovy rovnice, coz se ve vsech keplerovskych prediktorech provadi numericky Newtonovou metodou na 10 kroku, nemusite objevovat. To uz objevili jini.

"Mala" napoveda: zacina se polohou na dvourozmerne elipse, ktera ma danou vystrednost a je umistena v pocatku souradnic, hlavni poloosa na X+. K tomu se pouzije mean anomaly (z ni resenim one rovnice true anomaly) a pomoci zname polosy (nebo mean motion, coz je jiny zpusob vyjadreni tehoz se zapoctenim grav. konstanty Zeme). Ziskame slozky polohoveho vektoru, s tim, ze Z-ova je nula.

Pak nastupuje transformace prostorovymi rotacemi o uhly arg.perigea, inklinace, raan. Nejprve elipsu pootocime kolem osy Z o uhel arg.perigea. V teto fazi mame obeznou elipsu v rovine rovniku, ale spravneho "tvaru". Nasledne ji otocime kolem X o inklinaci. Tim ziskame elipsu v prostoru, spravne sklonenou, ale se vzestupnym uzlem presne v jarnim bode. No a zaverecna rotace kolem Z o uhel RAAN posune ten vzestupny uzel tam, kde ma byt. Voila, vysledny polohovy vektor po transformacich je presne to, co hledame.

Kdyz to pote porovname s referencnim modelem (treba Nova) zjistime, ze je to k nicemu. Realny pohyb skutecne neni Keplerovsky ani omylem. Zjistime neco o perturbacich a implementujeme jejich prvni priblizeni. Stale nestaci a porad je nase druzice desitky km od mista, kde by po par tydnech mela byt. Nakonec stahneme a rozchodime referencni model SGP, aniz se ho pokusime pochopit - stovky a stovky goniometrickych rovnic a vztahu ani pochopit nejdou

A uplne ve finale, spokojeni s tim, jak se nas model shoduje s Novou a STK, dosadime Molniju ... a frustraci z toho, ze jak v Nove, tak v nasem algoritmu se po peti obezich ponori pod povrch Zeme, pekne klidne vydychame.


Adolf - 2/7/2007 - 21:14

To DH: To zní jako vlastní zážitek.


Tomáš_Kovařík - 24/7/2007 - 19:55

Dobrý den.
Chtěl bych se zeptat, jakým způsobem se zažehují bloky SRB při startu raketoplánu. Vím sice, že elektricky, ale jak konkrétně, aby bylo dosaženo maximálního tahu okamžitě v T-0s (nebo začíná zážehová sekvence o něco dříve před startem?)
Za případné odpovědi děkuji.


avitek - 24/7/2007 - 23:24

citace:
... Dovolím si ještě jednu laickou otázku: Jaký je správný terminologický překlad výrazu ...


Českou terminologii v kosmonautice upravuje norma ČSN 31 0000. Jednu její kapitolu (k níž mám autorské právo), která se týká kosmonautiky a částečně i raketové techniky a astronomie jsem už před drahným časem umístil na stránky SPACE-40. Je přístupná přrs úvodní text SPACE-40, nebo přímo přes URL

http://www.lib.cas.cz/space.40/NORMA.HTM

Ke každému českému termínu je uveden i jeho anglický ekvivalent (ekvivalenty) a stručná definice významu tohoto termínu.

Protože jsou připojeny český i anglický rejstřík, může posloužit i jako obousměrný anglicko-český a česko-anglický slovník.


ales - 25/7/2007 - 08:00

citace:
Chtěl bych se zeptat, jakým způsobem se zažehují bloky SRB při startu raketoplánu. Vím sice, že elektricky, ale jak konkrétně, aby bylo dosaženo maximálního tahu okamžitě v T-0s (nebo začíná zážehová sekvence o něco dříve před startem?)

Podle http://en.wikipedia.org/wiki/Space_Shuttle_Solid_Rocket_Booster#Ignition (nebo podle http://spaceflight.nasa.gov/shuttle/reference/shutref/srb/ignition.html ) jsou SRB zažehovány pomocí menších motorů na TPL (igniterů) tak, že ten nejmenší motorek je zapálen elektricky a svým plamenem zapaluje větší motorky, až nakonec hlavní zažehovací motorek (umístěný v přední části SRB) vyšlehne plamen skrz celý vnitřní kanál zrna SRB, čímž ho zažehne. Celý tento proces proběhne v řádu stovek milisekund, takže ten první elektrický impuls zřejmě skutečně proběhne přesně v T-0s (prodlevu do naběhnutí plného tahu a vzletu zřejmě běžně nezaregistrujeme).

Podle diskuzního příspěvku na http://yarchive.net/space/shuttle/srb_ignite.html to vícestupňové zapalování probíhá takto:
The NASA Standard Initiator (NSI, essentially a detonator) ignites a pyro booster pellet of BKNO3, which starts an ignitor initiator (a small rocket engine) about 18cm long. The ignitor initiator in turn ignites the main ignitor motor, another, larger, rocket engine about 91cm long. Its flame shoots down the entire length of the SRB for a hundred milliseconds or so. The igniter contains 64 kg of solid propellant -- reliable ignition requires a big flame!

Ve volném překladu:
NASA Standard Initiator (NSI, v podstatě rozbuška) zažehne pyropatronu z BKNO3, která nastartuje další spouštěč (malý raketový motor) dlouhý 18cm. Ten dále [svým plamenem] spustí hlavní zažehovací motor, což je další větší raketový motor dlouhý 91cm. Jeho plamen prošlehne dolů celou délkou SRB na dobu několika stovek milisekund [čímž zapálí SRB]. Tento poslední zažehovač obsahuje 64 kg tuhé pohonné látky -- spolehlivé zapálení vyžaduje velký plamen!


Misák - 25/7/2007 - 13:51

Dobrý den, mohl by mě někdo prosím vysvětlit princip manévru gravitačního urychlení? Všude najdu jen popis že to je manévr sloužící k urychlení sondy bez použití motorů. Že dojde k urychlení sondy při přibližování k planetě je mě jasné, ale neměla by sonda tento přírůstek rychlosti zase ztratit po průletu při odpoutání od planety?


DODOR - 25/7/2007 - 14:07

citace:
Dobrý den, mohl by mě někdo prosím vysvětlit princip manévru gravitačního urychlení? Všude najdu jen popis že to je manévr sloužící k urychlení sondy bez použití motorů. Že dojde k urychlení sondy při přibližování k planetě je mě jasné, ale neměla by sonda tento přírůstek rychlosti zase ztratit po průletu při odpoutání od planety?


Ztratí ale jen částečně.

1. Planeta se pohybuje vůči sondě také tzn. doba přibližování je delší než doba oddalování. Při oddálení se také mění vektor. (zrychlení působí odlišnou dobu).

Laicky:Když se osoba A pohybuje rychlostí 1m/s a vy jí doháníte (pod úhlem 0stupňů) rychlostí 2m/s ze vzdálenosti 1000m trvá to 1000s. No a v okamžiku když osobu A doženete obrátíte se o 90stupňů a začnete se navzájem vzdalovat jenže těch 1000m už bude trvat pouhých 700s.

Samozřejmě napožítám vzejemnou akceleraci/dekceleraci gravitačním polem v případě sonda planeta. Ale princip je asi jasny






ales - 25/7/2007 - 14:28

citace:
Dobrý den, mohl by mě někdo prosím vysvětlit princip manévru gravitačního urychlení? Všude najdu jen popis že to je manévr sloužící k urychlení sondy bez použití motorů. Že dojde k urychlení sondy při přibližování k planetě je mě jasné, ale neměla by sonda tento přírůstek rychlosti zase ztratit po průletu při odpoutání od planety?

Podrobný popis (česky) je na http://cs.wikipedia.org/wiki/Gravita%C4%8Dn%C3%AD_man%C3%A9vr .

Podstatné je asi toto: "Gravitační manévr okolo planety mění rychlost sondy vůči Slunci, ale zachovává její rychlost vůči planetě samotné, jak podle zákona zachování energie musí."

Takže vůči planetě je rychlost na příletu i na odletu opravdu stejná. Vůči Slunci sonda manévrem může "převzít" část rychlosti, kterou planeta obíhá kolem Slunce. Jiným směrováním a časováním manévru ale sonda může naopak "zpomalit", změnit směr svého letu, nebo změnit rovinu své dráhy.


avitek - 25/7/2007 - 17:14

Podstatně podrobnější a hlavně přesnější grafická schémata jsem publikoval již v knize "Malá encyklopedie kosmonautiky" v roce 1982 (spolu s Petrem Lálou). Už také před časem jsem to naskenoval a dal do SPACE-40:

http://www.lib.cas.cz/space.40/NEBMECH/GRAVMAN.HTM


Misák - 26/7/2007 - 14:34

Velmi děkuji za odpovědi i za zajímavé odkazy na toto téma..


ROBO - 21/8/2007 - 01:35

Jeden drobny dotaz laika ... (rekneme ze v dnesni dobe uz "t" jako cas neni az zas takova konstanta - viz. energeticky vykon cernych der atd... mozny posun v "t" a to plus minus ) teoreticky

dotaz

rekneme ze t-24h tzn vcera , kdy vlastne zeme se pohybuje ve vesmiru t-24h je zasazena "necim" ccä kometou letici napr. svetelnou rychlosti ... muze toto ovlivnit nas v caste t+24h .. rekneme ze z pohledu extremnich energetickych vlivu ... teoreticky ano ?

mozna hloupi dotaz .. ale zeptat se mohu ... ze

Diky


Dodor - 21/8/2007 - 14:14

jaký koliv hmotný předmět letící rychlostí světla bude mít nekonečnou hmotnost.

PS: což je nemožné.


Jinak ano jakákoliv odchylka rychlosti předmětu vztažená k zemi má za následek změnu plynutí času pro tento předmět vztažený k času na zemi.

Viz. testy atomových hodin na palubě letadla.

tady to je i s animací:

http://www.spszl.cz/~vascak/moje/fyzika_ve_flashi/dilatace.php


Wartex - 21/8/2007 - 15:30

Abychom kolegovi neudelali jeste vetsi gulas ...

1. cas neni konstantni v tom smyslu, ze rychlost jeho plynuti je zavisla na soustave, v niz se meri

2. tim se mysli v STR vzajemna rychlost merene a merici soustavy

3. v obecne teorii relativity jde o komplexni vliv rozlozeni hmot, energii (a tedy i grav. poli) tim je urcena deformace casoprostoru a tedy i vysledky prostorovych a casovych mereni

4. co je relativni, je soucasnost a soumistnost udalosti, ne udalost sama. Naraz komety do Zeme je udalosti ktera nastala a nasledne bude vsemi pozorovateli vnimana jako stejna, jen ruzne casove nebo mistne deformovana. Bude-li to udalost okamzita a bodova bude vsemi popsana stejne.


ROBO - 21/8/2007 - 21:35

dekuji za reakce ...

slo mi spise o to ze pokud naraz do zeme probehne ted , vidime reakci .. ale rekneme mozna to bude znit divne ,ale on ten naraz probehne diky rozdilu casu jak ho vnimame my a jak ho vnima teleso dopadajici na zem Rozdilne a to tak .. ze naraz v nasem case se odehraje VCERA , tj my nevnimame zadny nasledek ale ze vznikajici energie muze nejaka reakce prijit kterou budeme vnimat pritomne ... a to je na co jsem se ptal .. je to mozne ?

vysvetleni .. proto aby vsechno ve vesmiru fungovalo - energie a tak tak vime ze funguje pritomne a ze bude fungovat zitra .. tj musi fungovat i vcera jinak by se vse zbortilo jak domecek z karet :-)


Wartex - 22/8/2007 - 08:35

To, co popisujete, pokud vam tedy rozumim je otazka kauzality. Pricina vzdy predchazi nasledkum a to bez ohledu na vztaznou soustavu. Zadne relativisticke teorie na principu kauzality nic nemeni. Pokud vnimam nasledky udalosti, musel jsem ve sve soustave vniman DRIVE i samotnou udalost.

Bez ohledu na ruzne hypotezy a scifi teorie a myslenkove experimenty je princip kauzality neotresitelny - prosim nechytat na teto urovni za slovo.


Adolf - 23/8/2007 - 16:08

citace:
To, co popisujete, pokud vam tedy rozumim je otazka kauzality. Pricina vzdy predchazi nasledkum a to bez ohledu na vztaznou soustavu. Zadne relativisticke teorie na principu kauzality nic nemeni. Pokud vnimam nasledky udalosti, musel jsem ve sve soustave vniman DRIVE i samotnou udalost.

Bez ohledu na ruzne hypotezy a scifi teorie a myslenkove experimenty je princip kauzality neotresitelny - prosim nechytat na teto urovni za slovo.


Za slovo chytat nechci. Princip kauzality při uvažování samotné TR mě neuvádí do rozpaků. Horší je to ale když do toho zatáhneme KT a vázané stavy s jejich „okamžitými“ důsledky. Mám dvojici antiparalelně polarizovaných fotonů ve vázaném stavu. Jednomu přistrčím do cesty polarizační hranol. Za mžik změřím polarizaci druhého vázaného fotonu, která bude správně antiparalelní k tomu, jak jsem polarizoval jeho vázaného druha. Příčina jeho polarizace je jasná, moje packa co jeho druhovi strčila do cesty hranol. Žádná záhada. Jenže tu máme relativitu současnosti.

Na mě ale kouká sci-fi pozorovatel v relativistickém hvězdoletu, který letí tak rychle a v takovém směru, že v jeho soustavě proběhne měření polarizace vázaného fotonu antiparalelního k tomu, co prolít mým hranolem, dříve, než má packa dloubla do hranolu, který jsem svému fotonu postavil do cesty. Protože příčina by měla předcházet následek, bude na to koukat zelený mužík z hvězdoletu tak, že měření polarizace antiparaelního vázaného fotonu k mému fotonu způsobilo, že vzápětí mou vrtkavou mysl napadlo strčit do hranolu a postavit ho jeho kvantovému bratříčkovi do cesty.

Tuhle „zlomyslnou“ otázku mi ale asi žádný z pánů fyziků nevysvětlí. Na tuto odpověď bych asi potřeboval toho zelenýho mužíka, co umí lítat od hvězdy ke hvězdě.


Archimedes - 23/8/2007 - 21:11

Vsechny experimenty s vazanymi stavy maji jedno spolecne - abych zjistil, jako polarizaci jsem vlastne nameril (coz je ona informace), musi dojit mezi obema mericimi body jeste k nejake dalsi komunikaci "klasickym" zpusobem. Teprve to je skutecny prenos informace a ten probiha maximalne svetelnou rychlosti, tudiz i sci-fi pozorovatel neuvidi zadny paradox.


Adolf - 23/8/2007 - 22:58

citace:
Vsechny experimenty s vazanymi stavy maji jedno spolecne - abych zjistil, jako polarizaci jsem vlastne nameril (coz je ona informace), musi dojit mezi obema mericimi body jeste k nejake dalsi komunikaci "klasickym" zpusobem. Teprve to je skutecny prenos informace a ten probiha maximalne svetelnou rychlosti, tudiz i sci-fi pozorovatel neuvidi zadny paradox.


K vhodně nasměrovanému sci-fi pozorovateli přiletí rychlostí světla dva signály - že jsem strčil do cesty jednomu paprsku polarizační hranol a že o kus dál někdo změřil polarizaci provázaného fotonu. Vzhledem k inerciální soustavě ale u něj bude jiná následnost událostí než v mojí soustavě a pro něj proběhne měření polarizace provázaného fotonu dříve, než já dloubnu do polarizačního hranolu.

Tuto situaci lze přeci teoreticky vykonstruovat.


Adolf - 23/8/2007 - 23:24

Zaboha tu nemůžu najít diskusi o pustošivém dopadu komety či co na Zemi. Jen podle paměti si vzpomínám, že tam kdosi uvedl, že by ji bylo lepší rozbít na menší kousky než nechat spadnout jeden velký kus.

Mně se to ale nezdá. Tedy možná ten velký kus by mohl vyvolat nějaké kvalitativně jiné ničivé účinky než kusy menší, ale když bych uvažoval homogennost dopadových účinků a očekával od každého projektilu vlastně tlakové pole ničivé atmosféry úměrné energii dopadu, mohl bych o tom uvažovat podobně jako o jaderném úderu. Při jaderném úderu se ale považuje za ničivě efektivnější použít více náloží menších ráží než jednu velkou pumu, neboť pak menší megatuny zničí větší plochu.

Poloměr zničené plochy bude zhruba úměrný třetí odmocnině energie impaktu. Zničená plocha energii jednoho impaktu na dvě třetiny. Celková zničená plocha tedy bude větší u více menších impaktů než u jednoho velikého při stejné celkové energii velkého impaktu a menších impaktů, nebo snad ne?


martalien - 23/8/2007 - 23:58

citace:
Celková zničená plocha tedy bude větší u více menších impaktů než u jednoho velikého při stejné celkové energii velkého impaktu a menších impaktů, nebo snad ne?


Zalezi na tom na jak male kousky ho rozbijete. Pokud budou dostatecne male, dojde vlivem treni o atmosferu k jejich odpareni. Musi se taky zohlednit ze je nepravdepodobne ze by padaly kolmo. V praxi v atmosfere uleti nekolik stovek ci tisicu kilometru. Velky objekt ale nestrati trenim moc kineticke energie a dopadne na zem v plne sile. Cim mensi objekt tim mensi je pomer jeho brzdici plocha a hmotnost - tedy pokud uvazujeme priblizne kulovy tvar - ne jehlovity .


Adolf - 24/8/2007 - 00:26

citace:
Zalezi na tom na jak male kousky ho rozbijete. Pokud budou dostatecne male, dojde vlivem treni o atmosferu k jejich odpareni. Musi se taky zohlednit ze je nepravdepodobne ze by padaly kolmo. V praxi v atmosfere uleti nekolik stovek ci tisicu kilometru. Velky objekt ale nestrati trenim moc kineticke energie a dopadne na zem v plne sile. Cim mensi objekt tim mensi je pomer jeho brzdici plocha a hmotnost - tedy pokud uvazujeme priblizne kulovy tvar - ne jehlovity .


Při té příležitosti mě napadá taková trochu děsivá myšlenka, jestli by nakonec nebylo nestrašnější potkat se s mračnem prachu, které by nás svléklo z atmosféry, že by z nás byl potom Mars.


Archimedes - 24/8/2007 - 11:51

citace:
K vhodně nasměrovanému sci-fi pozorovateli přiletí rychlostí světla dva signály - že jsem strčil do cesty jednomu paprsku polarizační hranol a že o kus dál někdo změřil polarizaci provázaného fotonu. Vzhledem k inerciální soustavě ale u něj bude jiná následnost událostí než v mojí soustavě a pro něj proběhne měření polarizace provázaného fotonu dříve, než já dloubnu do polarizačního hranolu.
Jenže samo měření polarizace a strčení hranolu do cesty nejsou v příčinném vztahu, jsou to nezávislé události. Tím že se experimentátor rozhodne měřit polarizaci, nijak neovlivní vaši možnost strčit současně do trasy hranol a naopak. Takže tyto dvě události můžou být viděny z různých inerciálních soustav v libovolném pořadí a je to tak naprosto v pořádku. V příčinné souvislosti jsou až vložení hranolu a _výsledek_ měření. Ten ale principiálně neexistuje, dokud mezi měříci místy neproběhne i klasická "světelná" komunikace. A ta kauzalitu zachovává spolehlivě.


Adolf - 24/8/2007 - 13:27

citace:
Jenže samo měření polarizace a strčení hranolu do cesty nejsou v příčinném vztahu, jsou to nezávislé události. Tím že se experimentátor rozhodne měřit polarizaci, nijak neovlivní vaši možnost strčit současně do trasy hranol a naopak. Takže tyto dvě události můžou být viděny z různých inerciálních soustav v libovolném pořadí a je to tak naprosto v pořádku. V příčinné souvislosti jsou až vložení hranolu a _výsledek_ měření. Ten ale principiálně neexistuje, dokud mezi měříci místy neproběhne i klasická "světelná" komunikace. A ta kauzalitu zachovává spolehlivě.


Nádhera! Mám dva polarizační hranoly a číhám na foton. Mužík v relativistickém kosmoletu na mě kouká a vidí i možné polarizační směry mých hranolů. Vidí i budík, co změří polarizaci provázaného fotonu. Spatří měření polarizace, které je u mě až následné po mé akci s hranolem. U něj je však dřív nejen v důsledku pořadí, v jakém k němu doletí signál, ale i v důsledku plynutí času v jeho inerciální soustavě. Vidí naměřenou polarizaci a kouká na mě, jak dumám nad hranoly, co s nimi podniknu. Ví při tom, že nemusím udělat vůbec nic, a pak foton proletí s antiparalelní polarizací, kterou jsem nijak neovlivnil. Ví, že mu můžu stračit do cesty hranol, co mám po pravé ruce, jehož účinek je v souladu a naměřenou polarizací. Ví ale také, že ať jsem vrtošivý, jak chci, napadnout mě strčit tam levý hranol, který otočí polarizaci jinak, nemůže. Je tedy událost mého vrtošivého počínání u hranolů nezávislá na pro něj předchozí události měření polarizace?


Archimedes - 24/8/2007 - 14:15

Jenže mužík neuvidí napřed výsledek ve smyslu "byla naměřena pravotočivá polarizace" ale jen "byla naměřena _nějaká_ polarizace", což nevylučuje jakoukoliv manipulaci s hranoly. To, jaká byla, se obě měřící místa dozví až po vzájemné "klasické" komunikaci. To je záludnost měření vázaných stavů - obě měřicí místa naměří "něco", ale co přesně, k tomu potřebují "klasicky" nesenou informaci z druhého místa.


Adolf - 24/8/2007 - 15:32

Mužíkovi se snad zaostří obrázek polarimetru teprve, až k němu doletí i můj obrázek, jak šťouchám do hranolu, i když k němu fotony od polarimetru přiletí dřív?


Archimedes - 24/8/2007 - 15:58

Nezaostří se, protože se nemá co zaostřovat. Ve chvíli, kdy měření proběhne, je tím měřením známa jen část informace (podle konkrétního problému) například v podobě "polarizace je buď taková nebo taková" - tím měření skončilo, nic víc z něj nevyleze v žádné vztažné soustavě. Která možnost se realizovala, se dá s jistotou tvrdit až po klasické komunikaci mezi oběma měřícími místy.

snad je to líp a ve zkratce vysvětleno třeba tady
http://www.scienceworld.cz/sw.nsf/0/E303A09D7BFE9276C1256E9700492123?OpenDocument&cast=1
http://en.wikipedia.org/wiki/Quantum_teleportation


Adolf - 24/8/2007 - 16:47

Tak teď abych si v těch braketech dopočítal ještě mužíka.


Adolf - 25/8/2007 - 10:15

Archimede,

jestli tomu zatím dobře rozumím, tak vlnová funkce si počká se zhroucením na potenciální signál z celého propletence.


Archimedes - 26/8/2007 - 10:28

Tak tohle už je opravdu na nějakého teoretika :) Já to chápu tak, že ke kolapsu dojde sice hned, ale u vázaných stavů prostě není výsledek z jedné "půlky" jednoznačný. Pokud mezi námi není nějaký zkušený kvantovkář, bude další debata jen filosofické plácání :)


Adolf - 27/8/2007 - 22:46

citace:
Tak tohle už je opravdu na nějakého teoretika Já to chápu tak, že ke kolapsu dojde sice hned, ale u vázaných stavů prostě není výsledek z jedné "půlky" jednoznačný. Pokud mezi námi není nějaký zkušený kvantovkář, bude další debata jen filosofické plácání


Jen jestli ti teoretici by neměli být aspoň dva, aby to teoretické plácání bylo vepsáno do správných operátorů.

V experimentu vyzkoušet vázaný stav, kde by mezi měřením na obou koncích vázaného stavu byla významná prodleva přesahující čas samotného měření by byl asi taky voříšek. Takže klid v duši, že kauzalita je v pohodě, asi tak honem mít nebudu.


Lorin - 29/8/2007 - 17:49

Chtěl bych se vás zeptat na něco o kosmonautice.Zatím tomu moc nerozumím,ale myslíte že by šlo na sondu namontovat sadu balonů.Ty by se při spatáteční cestě atmosférou odstřelili ,balony byse začal zahřívat tak by se nafoukly.A tím by se spomalil sestup družice.Nevím jestji by to nějaký material vydržel a jestli by to mělo dosta tečný účinek.


avitek - 29/8/2007 - 18:03

citace:
Chtěl bych se vás zeptat na něco o kosmonautice.Zatím tomu moc nerozumím,ale myslíte že by šlo na sondu namontovat sadu balonů.Ty by se při spatáteční cestě atmosférou odstřelili ,balony byse začal zahřívat tak by se nafoukly.A tím by se spomalil sestup družice.Nevím jestji by to nějaký material vydržel a jestli by to mělo dosta tečný účinek.
Něco podobného již dvakrát (ale zatím neúspěšně) zkoušeli Rusové ve spolupráci s Němci. Jednalo se o nafukovací konstrukci tvaru kužele, unitř kterého bylo umístěno vlastní těleso které se mělo vrátit na zemi. Zkoušeli to pro návrat urychlovacího stupně Fregat. Projekt měl označení IRDT, což je zkratka, znamenající Inflatable Re-entry and Descent Technology.

Obrázek je např. na stránce

http://news.bbc.co.uk/1/hi/sci/tech/637184.stm

Nad podobnou technologií uvažovali také v NASA /Langley Research Center); tam se to jmenovalo Inflatable Re-entry Vehicle Experiment (IRVE) a mělo se to zkoušet na sondážní raketě Terrier-Orion koncem roku 2005, ale jak to dopadlo nevím.


Archimedes - 30/8/2007 - 09:55

IRDT nebylo tak uplne neuspesne - pri prvnim testu alespon jeden ze dvou nakladu obstojne prezil (i presto, ze se brzdici kuzel nafoukl jen zcasti) a byl nalezen. Probehly jeste dalsi tri testy, ale byly problemy s nosnou raketou (Volna) a taky s tim, ze pokud selze pouzdru pri sestupu radiomajak, na Kamcatce je tezke neco najit.

Strucne v cestine (spolu s dalsimi starsimi, ale nerealizovanymi projekty navratu z orbity) na:
http://www.kosmo.cz/data/klub/seskok.ppt

hlavni vyrobce IRDT:
http://www.2r2s.com


JuDu - 11/9/2007 - 16:14

Dobry den,

Mal by som na vas otazku okolo drahy telesa. Ide o vyskum Marsu a Venuse - hlavne Sovietky zvaz. Druzice boli posielane priamo (start so Zeme a priame navedenie na vypocitanu drahu) a nie najpr na parkovaciu drahu okolo Zeme a potom a druhym startom smerom k planetam. Ma toto riesenie nejaku zasadnu vyhodu ?

Dakujem


Lůca - 12/9/2007 - 16:47

Ahoj vsichni
Ja bych mela taky otazku, bylo by mozne poskytnout ve stavu beztize prvni pomoc? A jak?


Tomas - 12/9/2007 - 17:25

Stejně jako na Zemi.


lůca - 12/9/2007 - 17:40

diky


Laik - 12/9/2007 - 21:29

- dychani z ust do ust by nemel byt problem
- masaz srdce problem je, bez solidniho vzepreni a opreni pacienta to nejde
- nebezpeci zapadnuti jazyka je mensi
- nebezpeci duseni zvratky je vetsi
- autotransfuze nefunguje


JuDu - 24/10/2007 - 19:06

Ahojte,

Mam otazku okolo orientacie kozmickej lode pri starte. Ako je zname, lod Sojuz ma pevne manontovane trisky. Ked potrebuje naklonit raketoplan (pocas letu), tak zvysujete tah na jednej strane, ale ako by ste zabezpecovali otacanie lodi ? Je to tam jedno ?

Dakujem


Adolf - 22/1/2008 - 17:32

Mám takovou provokativní otázku: Čím přispěla kosmonautika k rozvoji kartografie u nás od dob Josefa II?

České území je až na výjimky – Ostravsko, Mostecko, Sokolovsko a Praha – zmapované pomocí map pocházejících z kartografie Marie Terezie a Josefa II. Rozsáhlá pozdější mapování celých krajin provedly téměř výhradně důlní společnosti, které to ze svých pochopitelných důvodů provedly i pro stát. Ostatní pozdější mapování probíhala obvykle jen v lokálním měřítku, kdy jsou zapotřebí k realizaci staveb – např. dopravní infrastruktury. To se však nedělá celá krajina ale např. jen úzký řez kolem liniové stavby. Když se provádí kartografie kolem staveb, provede se postaru – měřeními na místě, a to i v případě silnic aj. Když potřebují přeci jen proměřit kus krajiny, jsou rozmrzelí, protože je to drahé a dělá se to zpravidla hlavně pomocí leteckých snímků 200 Kč / za snímek. Dělá se to ale málo. Kartografie osvícených panovníků konce feudalismu je postačující i v době, kdy je na orbitě tlačenice satelitů.

Má na tuto situaci a případnou možnost její změny někdo ze zdejších zasvěcenějších nějaký názor?


Milan - 23/1/2008 - 13:37

citace:
Mám takovou provokativní otázku: Čím přispěla kosmonautika k rozvoji kartografie u nás od dob Josefa II? ...
To se však nedělá celá krajina ale např. jen úzký řez kolem liniové stavby. Když se provádí kartografie kolem staveb, provede se postaru – měřeními na místě...


Zdání klame - ono se sice zdá, že geodeti běhají po poli s trasírkami a nivéláky - ale mají na nich umístěné GPS přístroje - takže tady je spojení s kosmonautikou celkem jasné a jednoznačné :-)
Už se místa neodměřují od nivelačřních bodů v terénu (mimochodem, kdy jste kdo naposledy viděl na horizontu stojící konstrukci nivelačního bodu - často zaměňovanou za minirozhlednu, nebo posed myslivců), ale podle GPS souřadnic.

Milan


Jano - 24/1/2008 - 15:28

>> Když se provádí kartografie kolem staveb, provede se postaru – měřeními na místě ... Když potřebují přeci jen proměřit kus krajiny ... dělá se to zpravidla hlavně pomocí leteckých snímků ... Dělá se to ale málo.

>> ono se sice zdá, že geodeti běhají po poli s trasírkami a niveláky - ale mají na nich umístěné GPS přístroje

Popri leteckej fotogrammetrii a používaní GPS mi na um ešte prichádzajú napr. technológia LIDAR (z vyvýšených stanovíšť alebo z lietadiel) a spomedzi satelitov napríklad:
-SRTM @STS99(feb.2000): výškopis skoro všetkej pevniny, s rozlíšením asi 30m a nepresnosťou asi 10-15m,
-LandSat7: so 16dennou periódou opakovania a rozlíšením 15/30m dokáže snímkovať celú pevninu,
-TerraSAR-X: novší nástupca SRTM, s rozlíšením až do 1m
-CartoSat: indický satelit, čb-snímky s rozlíšením 1 m
-Ikonos, QuickBird a ďalšie, s rozlíšením až okolo 0,5 m

Viaceré z menovaných sa týkajú aj strednej Európy.
Otázkou je len, či sa aj naozaj využívajú pre serióznu kartografiu, alebo len pre aplikácie typu GoogleMaps, ktoré sú typu "no warranty", t.j. môžu a nemusia byť presné/aktuálne.
Na druhej strane, pozemnými a leteckými prostriedkami je zrejme možné dosiahnuť lepšiu detailnosť a presnosť, než satelitnými - družice sa skôr hodia (resp.: sú jedinými možnými alebo efektívnymi) na mapovanie rozsiahlych odľahlých oblastí.


Jirka - 29/1/2008 - 14:40

Mel jsem znameho, ktery delal mapy Gronska (nebo snad Islandu?). Myslim ze pouzivali vetsinou letecke snimky a podle nich zakreslovali takove ty hlavni objekty jako cesty, vodni toky, domy atd. Urcite meli ale k dispozici i nejakou zkladni sit promerenych bodu.


Ladze - 24/4/2008 - 15:39

1)
Co je dnes na vypuštění rakety/raketoplánu nejdražší?

- vyprojektování?
- výroba stroje?
- obsluha (na zemi/ve vesmíru) ?
- pohoné hmoty?

víte někdo kolik to je v %?

2)
Kolik ušetřím, když použiju pro vynesení (malé, menší) družice klasickou raketu nebo B-52 a pegasus nebo jim podobný systém i kdyby byl zatím jen na výkresech. 10%, 1/5, 1/3 nebo jen několik procent ceny?

Jestli to někdo znáte odpověď nebo jen máte zajímavý názor budu vděčný.


JuDu - 11/5/2008 - 20:33

Dobry vecer prajem,

Nova etapa, nova kacirska myslienka :-)
Takze: ako urcia (a ci urcia/potrebuju vobec) ludia na ISS vodorovnu polohu ? Kedze je tam mikrogravitacia, tak bublina vo vodovahe bude plavat v celej tekutine. Pri pripajani lodi podla vsetkeho potrebuju vediet len vychylku os dvoch lodi a ci je to suosove. Funguje v mikrogravitacii vobec napr. laserovy vodomer ? Gyroskop ano, ale po case to treba zastavit-nastartovat-nakalibtovat. Polohu urcuju podla hviezd ?

Dakujem za porozumenie ...

Juraj


MIZ - 12/5/2008 - 15:40

Taky mají "Gde Proboha Su?", ne?


ales - 12/5/2008 - 20:29

citace:
... ako urcia (a ci urcia/potrebuju vobec) ludia na ISS vodorovnu polohu ? Kedze je tam mikrogravitacia, tak bublina vo vodovahe bude plavat v celej tekutine. Pri pripajani lodi podla vsetkeho potrebuju vediet len vychylku os dvoch lodi a ci je to suosove. Funguje v mikrogravitacii vobec napr. laserovy vodomer ? Gyroskop ano, ale po case to treba zastavit-nastartovat-nakalibtovat. Polohu urcuju podla hviezd ? ...
Podle stránky http://www.boeing.com/defense-space/space/spacestation/systems/guidance_navagation_control.html se navigační systém ISS skládá z několika částí. Ruský systém využívá navigační družice GLONASS a také optické sledování hvězd, Slunce a horizontu Země. Americký systém je založen na navigačních družicích GPS a je doplněn několika přesnými laserovými měřicími gyroskopy. Ruský a americký systém se vzájemně zálohují a doplňují (vyměňují a kontrolují si data). Výsledná informace o aktuální poloze a orientaci ISS je pak dostupná v interní počítačové síti stanice, takže ji mají k dispozici nejen kosmonauti, ale i všechny zapojené experimenty a přístroje (navigační informace slouží např. i pro natáčení solárních panelů a radiátorů).


x - 7/7/2008 - 16:36

Je schopen prezit clovek v kosmickem vakuu (ci alespon ve vakuove komore) nejakou omezenou dobu bez nasledku ??

Ja jsem slysel, ze ano - proste lidske telo je natolik odolne,ze pretlakem 1 atmosfery se nerozpadne, zmrznuti ve vakuu nehrozi - je to izolant, vareni krve z duvodu nizkeho tlaku nastane snad az za delsi dobu.
Proste ta kratka doba co staci s dechem - nesmi se prijit o tlak vzduch v plicich tak vydrzi.


dubest - 7/7/2008 - 16:52

citace:

Proste ta kratka doba co staci s dechem - nesmi se prijit o tlak vzduch v plicich tak vydrzi.

Něco na toto téma: http://www.ian.cz/detart_fr.php?id=736&nadpis=Zhluboka


David - 10/7/2008 - 08:37

Náhlá dekomprese do vakua je okamžitá smrt, krev se začne vařit - tady by měl diskutovat Vlado - dekomprese je jeho parketa a ne se zabývat nesmysly.


avitek - 10/7/2008 - 09:56

citace:
Náhlá dekomprese do vakua je okamžitá smrt, krev se začne vařit - tady by měl diskutovat Vlado - dekomprese je jeho parketa a ne se zabývat nesmysly.


Jste na omylu. Američané v 60. létech minulého století zkoušeli explozivní dekompresi na šimpanzech a dospěli k závěru, že pokud před dekompresí dýchali čistý kyslík, tedy za situace podobné, jako když je člověk ve skafandru (při normálním vzduchu hrozí dusíková embolie), přežili bez zřetelných následků.

Viz např.

Accession Number : AD0479738

Title : THE EFFECT ON THE CHIMPANZEE OF RAPID DECOMPRESSION TO A NEAR VACUUM.

Descriptive Note : Contractor rept. 1 Jul 64-31 Jan 65,

Corporate Author : AEROMEDICAL RESEARCH LAB (6571ST) HOLLOMAN AFB NM

Personal Author(s) : Koestler, Alfred G.

Report Date : NOV 1965

Pagination or Media Count : 106

Abstract : Eight chimpanzees, used in nine separate tests, were decompressed from 179 mm Hg (100% oxygen) to less than 2 mm Hg in 0.8 seconds and remained at this altitude from 5 to 150 seconds. After recompression to 179 mm Hg (again breathing 100% oxygen), the subjects were kept at this altitude for 24 hours. Performance by all animals, on a complex operant schedule presented during and following rapid decompression, reached a baseline level of performance within a four hour post-decompression period. No central nervous system damage (as measured by behavior) could be detected. Cortical EEG, ECG, and respiration were recorded before, during, and following decompression. Visual analysis of recorded physiological parameters was conducted and correlations with performance were attempted. EEG fast activity (10-12 cps) always preceded the end of the period of total behavioral impairment, while total behavioral recovery followed the return of normal EEG patterns. All subjects showed slight neutrophilia, increased transaminase, and facial edema which returned to normal within 72 hours after decompression. All subjects survived in good health and no lasting effects of rapid decompression to a near vacuum could be detected. (Author)

Plný text této studie je ke stažení jako pdf

http://ntrs.nasa.gov/archive/nasa/casi.ntrs.nasa.gov/19650027167_1965027167.pdf


Jásám - 10/7/2008 - 16:21

Otázka k balistickým raketám:
Kozmické rakety sa spravidla vystreľujú východným smerom, aby získali doplnkovú rýchlosť od rotácie Zeme.
Má podobné konanie nejaký väčší zmysel aj v prípade streľby balistickými raketami? Dosiahne sa streľbou "na východ" väčší dostrel?
Domnievam sa, že nie, pretože cieľ pred raketou "uteká" tiež východným smerom a pri streľbe na západ sa zasa pohybuje proti smeru letu strely, čím sa výhoda doplnkovej rýchlosti úplne kompenzuje.


Vítězslav Novák - 10/7/2008 - 18:11

Balistické rakety ve skutečnosti nejsou moc "balistické". Po balistické křivce létají náboje z děl - a tam je to celkem jedno.

Mezikontinentální "balistické" střely letí většinu své dráhy mimo atmosféru a po kepplerovské dráze. Tedy po elipse, jejíž perigeum je pod povrchem Země. Takže směrem na východ skutečně získají jistou rychlost, jenže u takových střel je podstatné, že musí doletět na konkrétní místo, ne na nějakou dráhu. Takže mezikontinentální střela musí mít dostatečnou nosnost, aby doletěla kam má i bez takové berličky. Když nemá - je to jen hračka.

U kosmických startů samozřejmě závisí na každém kile, tam se přídavná rychlost hodí hodně. V civilu se nemůže plýtvat tak, jako v armádě.

BTW - pochopitelně uvedené platí přibližně, poslední fáze letu hlavic probíhá po části balistické dráhy, navíc hlavice mohou samostatně manévrovat... po (zhruba - Země není hmotný bod) kepplerovské dráze letí rakety jen ve fázi letu setrvačností atd.

Když to chcete brát čistě kinematicky, tak musíte počítat s rozdílem mezi obvodovou a úhlovou rychlostí - lineárně se posouvající papír není vhodný model.


Jásám - 10/7/2008 - 18:48

Podstata problému mi je zrejmá, ale zaujíma ma práve problém smeru streľby a čí má zmysel pre diaľkové rakety uvažovať doplnkovú rýchlosť od rotácie Zeme.
Je zrejmé, že pre presné zamierenie na cieľ sa s rotáciou Zeme uvažovať musí, mne ide skôr o dosiahnutie nejakého dostrelu a v ktorom prípade (na východ - na západ) sa pri inak rovnakej rakete dosiahne väčší dostrel.


Derelict - 10/7/2008 - 19:42

citace:
Podstata problému mi je zrejmá, ale zaujíma ma práve problém smeru streľby a čí má zmysel pre diaľkové rakety uvažovať doplnkovú rýchlosť od rotácie Zeme.
Je zrejmé, že pre presné zamierenie na cieľ sa s rotáciou Zeme uvažovať musí, mne ide skôr o dosiahnutie nejakého dostrelu a v ktorom prípade (na východ - na západ) sa pri inak rovnakej rakete dosiahne väčší dostrel.


Mam k tomu take dotaz (otazka zda jeste balisticky). Vzdy jsem uvazoval tim zpusobem, ze raketa ma urcity dostrel. Smerem na vychod se "zvysi rychlost" pouze tim, ze se zeme pod raketou za dobu letu "pootoci", je to tedy pouze otazka vztazne soustavy.
S timto mi ale pripada zajimava jina otazka. Ma nejaky meritelny vliv nejenom rychlost rotace Zeme a rychlost obehu okolo slunce ? Nebo tyto vlivy ma smysl pocitat pouze u startu k jinym telesum ?


Alchymista - 10/7/2008 - 21:02

Pokiaľ sa na to pozrem z pohľadu kozmickej techniky: Prvá kozmická rýchlosť je určená z hmotnosti Zeme a výšky obežnej dráhy družice - vlastne vzdialenosti od stredu Zeme ako hmotného bodu. Táto rýchlosť teda nie je žiadnym spôsobom závislá na rotácii Zeme.
Preto sa družice vypúšťajú zväčša východným smerom - v smere rotácie Zeme - čo umožňuje získať "zadarmo" na rovníku rýchlostný prírastok 460m/s - o to môže byť raketa slabšia alebo družica ťažšia.

Medzikontinentálne rakety ale zasahujú ciele spojené so súradnicovým systémom rotujúcej Zeme.
Pokiaľ sa strieľa východným smerom, získa raketa v závislosti na mieste štartu nejaký rýchlostný prírastok, a teda jej rýchlosť bude o čosi vyššia, ale zároveň sa jej cieľ tiež pohybuje východným smerom - dráha k cieľu sa počas jej letu "predlžuje".
Pokiaľ sa strieľa západným smerom, rýchlostný prírastok je "záporný", ale cieľ na povrchu zeme sa pohybuje východným smerom, teda proti smeru letu rakety.

Vychádza mi to tak, že pre medzikontinentálnu raketu nie je dôležité zvažovať východný čí západný smer streľby - v súradnicovej sústave rotujúcej Zeme sú oba smery (približne) rovnocenné a v oboch smeroch bude s rovnakou raketou dosiahnutý (približne) rovnaký dostrel. Odlišnosti by mohli byť v presných tvaroch dráh pre dosiahnutie rovnakého diaľkového dostrelu.

Rýchlosť pohybu Zeme na obežnej dráhe okolo Slnka (cca 30km/s) má podľa mojej predošlej úvahy význam uvažovať len pri letoch k iným planétam (druhá kozmická rýchlosť a vyššie), pokiaľ som si všimol, všetky dráhy k vonkajším planétam - Mars a ďalej - sú orientované v smere obehu Zeme okolo Slnka. [Upraveno 10.7.2008 poslal Alchymista]


Emil - 11/7/2008 - 17:50

Zajímalo by mne zda sondy které opouštějí sluneční soustavu následují i poslední stupně jeich nosných raket, na jakých jsou tyto poslední stupně drahách, vzdálenostech, jak jsou velké a hmotné, ev. zda nesou též nějaké " poselství".
Ví to někdo, děkuji.


Emil - 11/7/2008 - 18:07

Ještě bych dotaz doplnil o poslední stupeň nosné rakety sondy Ulyses, při odletu od Země měl 15,6 km/s mířil k Jupiteru, je známa jeho konečná dráha - mohl teoreticky též pži průletu kol Jupitera zíslat únikovou rychlost ?


JS - 16/7/2008 - 10:18

citace:
Ještě bych dotaz doplnil o poslední stupeň nosné rakety sondy Ulyses, při odletu od Země měl 15,6 km/s mířil k Jupiteru, je známa jeho konečná dráha - mohl teoreticky též pži průletu kol Jupitera zíslat únikovou rychlost ?


Tak nevim co se mysli tou konecnou drahou. Zrejme bude znama draha v momente odpoutani druzice (vzdy tam byva nejaky impulz treba pomoci pruzin, nebo motorickeho manevru), ale vzhledem k tomu, ze neni moznost jakekoliv komunikace s urychlovacim stupnem a drahy podlehaji rusivym gravitacnim vlivum, tak pochybuju ze muze byt draha stupne znama s nejakou vetsi presnosti.
Pokud vyslete druzici k Jupiteru, tak k odpoutani urychlovaciho stupne dojde jeste nekde blizko Zeme. Je tak jen velmi mala sance, ze by vyhorely stupen opravdu dokazal proletet v tesne blizkosti Jupitera. S nejvyssi pravdepodobnosti tak skonci na protahle draze s periheliem v blizkosti Zeme a apoheliem za drahou Jupitera. Po par stovkach ci tisicich let se muze usadit v pasu asteroidu nebo v nekterem libracnim bode.
To same se stalo s urychlovacimi stupni Voyageru, Pionyru nebo New Horizons. Zrejme se zasekly nekde kolem drahy Jupitera. Nikdo nemuze vedet kde presne.


Tomáš_Habala - 16/7/2008 - 10:32

citace:
To same se stalo s urychlovacimi stupni Voyageru, Pionyru nebo New Horizons. Zrejme se zasekly nekde kolem drahy Jupitera. Nikdo nemuze vedet kde presne.

Urychlovací stupeň Star 48B sondy New Horizons dorazil k Jupiteru o něco dříve než sonda. Jelikož ale neměl možnost měnit trajektorii, neproletěl optimálním koridorem kolem planety. Získaný náskok na prvním úseku dráhy proto po gravitačním manévru sondy u obří planety rychle ztratí a oběžnou dráhu Pluta nedosáhne dříve než 2015-10-15.


JS - 16/7/2008 - 11:04

citace:
citace:
To same se stalo s urychlovacimi stupni Voyageru, Pionyru nebo New Horizons. Zrejme se zasekly nekde kolem drahy Jupitera. Nikdo nemuze vedet kde presne.

Urychlovací stupeň Star 48B sondy New Horizons dorazil k Jupiteru o něco dříve než sonda. Jelikož ale neměl možnost měnit trajektorii, neproletěl optimálním koridorem kolem planety. Získaný náskok na prvním úseku dráhy proto po gravitačním manévru sondy u obří planety rychle ztratí a oběžnou dráhu Pluta nedosáhne dříve než 2015-10-15.


Prvotni urychleni New Horizons bylo zrejme velice presne. Kdy doslo k poslednimu kontaktu se Star 48B? Vyfotila ho New Horizons, nebo to jsou jen odhady na zaklade puvodni drahy New Horizons?


Tomáš_Habala - 16/7/2008 - 11:11

citace:

Prvotni urychleni New Horizons bylo zrejme velice presne. Kdy doslo k poslednimu kontaktu se Star 48B? Vyfotila ho New Horizons, nebo to jsou jen odhady na zaklade puvodni drahy New Horizons?


Poslední impuls na cestu k Plutu udělil sondě raketový motor Star-48 krátce po startu. Motor se pak oddělil a pokračoval po vlastní trajektorii, která se příliš nelišila od dráhy, po které se pohybovala sonda. Vzhledem k preciznímu navedení na meziplanetární dráhu bylo nutno uskutečnit opravy změnou rychlosti sondy o pouhých 18 m/s. Předletové kalkulace přitom počítaly s korekcemi 5x až 10x většími. Motor byl naposledy pozorován v den startu 2006-01-19 a jeho dráha tudíž nemohla být stanovena příliš přesně. I toto krátké sledování však stačilo k tomu, aby se zjistilo, že Star-48 dorazí k Jupiteru 2007-02-27 přibližně šest hodin před tím, než se k planetě přiblíží New Horizons. Těchto šest hodin je však příliš mnoho, aby urychlovací stupeň prolétl správným místem nad planetou a byl urychlen k Plutu. Počínaje Jupiterem se tak dosud blízké dráhy sondy a jejího urychlovacího motoru začaly rychle rozcházet.


JS - 16/7/2008 - 11:33

O sest hodin driv? Co to znamena? Zrejme to, ze proletel Jupiterovu drahu o sest hodin driv. Pochybuju, ze by se mohl k Jupiteru priblizit vice nez New Horizons. To by bylo opravdu nehorazne stesti.


Tomáš_Habala - 16/7/2008 - 11:36

citace:
O sest hodin driv? Co to znamena? Zrejme to, ze proletel Jupiterovu drahu o sest hodin driv.


Ano, o sest hodin driv proletel Jupiterovu drahu.


David - 18/7/2008 - 06:09

Nejde o to, zda stupně doletěly k cílům cest vlastní sondy, ale o to, zda vnikly dostatečně hluboko do gravitačního pole Jupitera, aby zapůsobil gravitační " prak" a ony získaly únikovou rychlost pro opuštění Sluneční soustavy. U sondy N.H. se to zdá zcela jasné, 18 m/s a šest hodin rozdílu znamená že stupeň soustavu zřejmě opustí. Záleží ovšem na tom, zda oněch 18 m vlastní sondě chybělo, nebo byla rychlejší než plán.
Stejně by se dalo odvodit, jak dopadly stupně ostatních sond, pokud by bylo jasné, jak probíhaly jednotlivé korece vlastních son, tedy zdy byly sondy urychlovány, či brzděny , o kolik a kdy, tedy v který okamžik letu.
Rozdíl v okamžiku oddělení není velky cca lm/s, takže stupeň téměř přesně sleduje dráhu sondy před prvou korekcí a tak by mělo být zcela přesně známo v jaké vzdálenosti a kdy kolem Jupitera proletěl a z toho lze jednoduše odvodit zda získal požadovanou rychlost, tedy zrychlení.
Pokud se někdo z Vás zabývá astrodynamikou, měla by to pro něj být hračka.
Myslím že tolik těles zatím Sluneční soustavu neopouští a v dohledné době neopustí, aby bylo možno poslední stupně nosných raket pominout.


JS - 18/7/2008 - 10:53

Pokud me pamet neklame, tak mel N.H. 3 kosmickou rychlost uz po startu. Prulet kolem Jupitera tedy jen zajistil rychlejsi prilet k Plutu ke kteremu by po jine trajektorii doletel stejne.
Myslim, ze jeho urychlovaci stupen kvuli odchylce sice protnul drahu Jupiteru drive, ale prilis daleko od nej na to, aby byl vyzname urychlen.
Ostatni sondy mely mnohem mensi pocatecni rychlost snad krome Voyageru a Pioneeru. Ty myslim nejdrive leteli taky k Jupiteru a tudiz mohli mit dostatecnou pocatecni rychlost aby jejich urychlovaci stupne taky opustili slunecni soustavu.
Naopak treba sonda Cassini nejdrive proletela kolem Venuse a tudiz jeji urychlovaci stupen bude ztraceny nekde uvnitr obezne drahy Zeme.
Marne premyslim nad tim, jaky uzitek by tato informace mohla prinest. K cemu je takovy vyhorely stupen bezcilne bloudici nekde v kosmu dobry?
Na LEO jej lze vyuzit ke cvicnemu dokovani a u Mesice s nim lze prastit do nejakeho krateru, jinak si ale nejsem vedom, ze by kdy byl k necemu vyuzit.


David - 18/7/2008 - 17:55

Vlastní sondy nesly tzv. " mezihvězdný dopis", dle něhož možný nálezce mohl identifikovat konstruktéry sondy. Stejnou pravděpodobnost " nálezu" mají poslední stupně a je jen škoda, že též nenesou " dopisy ", tím by se pravděpodobnost nálezu zvýšila o více než lOO%. Jisté je, že pokud by někdo někdy v hlubinách kosmu nalezl některý z posledních stupňů, zcela jistě by rozeznal jeho umělý původ, poznal by účel k němuž byl postaven a podle dráhy by určil i přibližně oblast z níž pochází. Proto považuji i tyto zdánlivě nyní již neužitečné výtvory lidského génia za hodné pozornosti. A´t chceme, nebo nechceme pokud opustí Sluneční soustavu, stanou se posly inteligence, která je postavila.
Proto ten " zájem ", nebo se mýlím ?


Alchymista - 18/7/2008 - 21:16

A z analýzy konštrukcie zrejme celkom dobre odhadol aj technologickú úroveň civilizácie ktorá ho vypustila.
Veď taký raketový stupeň obsahuje pomerne pestrú zbierku zariadení a konštrukčných prvkov: predovšetkým samotný raketový motor (motory) na kvapalné palivo a zrejme aj na tuhé palivo, nádže - nezabúdať na ich tepelné a iné izolácie a nátery, vysokovýkonné čerpadlá, rôzne silové mechanizmy hydraulické a elektrické, rôzne rozvody, a tiež polovodičové elektronické bloky, nejaké elektrické zdroje, výbušniny v autodeštrukčnom systéme atď. => Celkom dobrý výber ukážok vrcholovej technológie našej civilizácie v oblasti metalurgie, obrábania kovov, chémie organických a anorganických látok, elektrotechniky a elektroniky, samozrejme vrátane ukážok inžinierskeho myslenia a prístupu k riešeniu technických problémov.

Na druhej strane - pre civilizáciu, ktorá by bola schopná takýto stupeň vo vesmíre nájsť a zachytiť by to bol zrejme len historický exkurz do jej vlastnej minulosti. [Upraveno 18.7.2008 poslal Alchymista]


David - 19/7/2008 - 05:14

To je celkem jasné, ale mě zajímá, a proto se ptám, jestli je někdo schopen fundovaně odpovedět, ZDA se poslední nstupně nosných raket na mezihvězdnou pouť vúbec dostaly, případně na jaké jsou dráze, v jaké vzdálenosti od Slunce a kam směřují.
Takový je můj dotaz, děkuji.


JS - 21/7/2008 - 11:07

citace:
A z analýzy konštrukcie zrejme celkom dobre odhadol aj technologickú úroveň civilizácie ktorá ho vypustila.
Veď taký raketový stupeň obsahuje pomerne pestrú zbierku zariadení a konštrukčných prvkov: predovšetkým samotný raketový motor (motory) na kvapalné palivo a zrejme aj na tuhé palivo, nádže - nezabúdať na ich tepelné a iné izolácie a nátery, vysokovýkonné čerpadlá, rôzne silové mechanizmy hydraulické a elektrické, rôzne rozvody, a tiež polovodičové elektronické bloky, nejaké elektrické zdroje, výbušniny v autodeštrukčnom systéme atď. => Celkom dobrý výber ukážok vrcholovej technológie našej civilizácie v oblasti metalurgie, obrábania kovov, chémie organických a anorganických látok, elektrotechniky a elektroniky, samozrejme vrátane ukážok inžinierskeho myslenia a prístupu k riešeniu technických problémov.

Na druhej strane - pre civilizáciu, ktorá by bola schopná takýto stupeň vo vesmíre nájsť a zachytiť by to bol zrejme len historický exkurz do jej vlastnej minulosti. [Upraveno 18.7.2008 poslal Alchymista]


Pokud nejsem v omylu, tak predpokadam ze sanci dostat se na mezihvezdnou drahu mely jen posledni stupne Pionyru, Voyagery a New Horizons. Vsechno to byly male stupne na tuha paliva. Neprekvapilo by me pokud byly stabilizovany pouze rotaci a mozna ani nemely zadny navigacni system. Ten byl pravdepodobne na stupni centaur, ktery zustal nekde na vysoke Zemske obezne draze, nebo Heliocentricke draze blizko Zeme.
Rozhodne tedy nemaji zadna turbocerpadla, hydraulicke systemy, nejakou zvlastni avioniku, nebo autodestrukcni system. Taky vzdalenost kterou muzou uletet je extreme mala v porovnani s mezihvezdnymi vzdalenostmi. Elektromagneticke vysilani ze Zeme je urcite mnohem snaze zachytitelne, nez najit objekt o hmotnosti par desitek kilogramu mezi ostatnim smetim pohybujicim se v mezihvezdnem prostoru. V Oartove oblaku a v blizkem mezihvezdnem prostoru je podobnych objektu mnoho miliard. Mimozemstane, by museli mit opravdu mimoradne stesti aby si takoveho stupne vsimli a zaroven ignorovali takovou hezkou planetu jen par svetelnych let daleko.


David - 22/7/2008 - 08:14

Mezihvězdný dopis má o mnoho řádů menší pravděpodobnost než lahvová pošta z pustého ostrova, ale jedná se technický problém. Mne zajímá, zda se tato tělesa vůbec dostala na mezihvězdnou dráhu, neboť lidských výtvorů, které to dokázaly, není zrovna přehršle.
Vycházím z faktu, že poslední stupeň nosné rakety sleduje téměř přesně / rozdíl cca -l m/s /dráhu vlastní sondy před první korekcí a tato dráha je jistě přesné známa, jinak by se korekce nedala provést.
TEDY můj laický dotaz zní, zda je někdo z návštěvníků schopen tuto dráhu určit, nebo alespoň určit odchylku od cíle a hodnotu provedené korekce a jestli sondu urychlila, nebo zpomalila. Z toho by se dalo dovodit, zda těleso vniklo dostatečně hluboko do Gravitačního pole Jupitera, aby získalo přídavnou rychlost. Též je podle mě důležité, zda průletová dráha vedla před, nebo za planetou.
Díky.


Emil - 26/7/2008 - 08:38

Vypadá to, že krom posledního stupně sondy N.Horizons, žádný již svoji sondu do mezihvězdného prostoru nenásledoval, dokonce ani poslední stupeň sondy Ulyses, který měl 15,6 km/s a prolétl kolem Jupitera.
Nic jiného jsem z dosavadní diskuse nevyčetl a jsem poněkud zklamán, neboˇdle mého odezněly pouze laické názory.
Nemohl by odpovědět někdo fundovaný ?
Díky.


ales - 28/7/2008 - 09:46

citace:
Vypadá to, že krom posledního stupně sondy N.Horizons, žádný již svoji sondu do mezihvězdného prostoru nenásledoval, dokonce ani poslední stupeň sondy Ulyses, který měl 15,6 km/s a prolétl kolem Jupitera.
Nic jiného jsem z dosavadní diskuse nevyčetl a jsem poněkud zklamán, neboˇdle mého odezněly pouze laické názory.
Nemohl by odpovědět někdo fundovaný ?
Díky.
Obávám se, že otázka (ohledně současné polohy urychlovacích stupňů vzdálených sond) je příliš těžká. Z informací dostupných na internetu lze s pravděpodobností na 99% říci, že na únikové dráze mimo Sluneční soustavu je v tuto chvíli jen 6 umělých kosmických těles, z toho je 5 sond (Pioneer 10, Pioneer 11, Voyager 1, Voyager 2 a New Horizons) a navíc ještě urychlovací motor sondy New Horizons, která byla jako jediná navedena na únikovou dráhu ze Sluneční soustavy hned po startu ze Země (ostatní sondy byly navedeny jen na heliocentrickou dráhu s apocentrem za drahou Jupiteru). Polohu aktivních sond lze sledovat např. na serveru Heavens Above - http://www.heavens-above.com/solar-escape.asp .

Poloha pasivních těles (urychlovacích stupňů) se asi běžně na internetu nezveřejňuje a také výpočet jejich drah je velmi obtížný a relativně nepřesný, protože jednak nejsou k dispozici dostatečně přesné počáteční údaje (alespoň já je neznám) a jednak byly heliocentrické dráhy těchto těles ovlivněny poměrně blízkými průlety kolem Jupiteru. Problém vidím i v tom, že pasivní umělá tělesa ve vzdálenostech Jupiteru zřejmě nelze rozumným způsobem sledovat (a tak upřesnit jejich dráhu).

Domnívám se tedy, že přesnou aktuální polohu urychlovacích stupňů vzdálených sond zná jen velmi málo lidí na světě a na toto fórum zřejmě nikdo z nich nechodí. Na některé otázky prostě není snadné odpovědět ...


David - 28/7/2008 - 18:02

dík, pane Aleši, ale mě by postačilo znát hodnoty prvé korekce jednotlivých sond,a zejména jejich kladný či záporný smysl, z toho by se dalo usoudit zda bylo těleso sondy urychleno či nikoli.
Dále znát okamžik a vzdálenost vlastních sond kdy a v jaké vzdálenosti by minuly Jupitera bez prvé korekce.
Tato údaje by někde měly být, neboť bez nich nelze dráhu korigovat.
Pokud by tyto údaje byly známy bylo by relativně snadné odvodit i dráhu posledního stupně nosné rakety, neboť v okamžiku oddělení je jeho typická rychlost -1m/s vůči vlastní sondě, ergo kladívko se pohybuje prakticky po totožné dráze jako vlastní sonda před prvou korekcí.
Děkuji předem.
P.S. Bylo by dobré se zařadit mezi těch několik ... ., proto doufám, že nastíněný problém někoho hlouběji zaujme, myslím někoho ve věci fundovaného, vždyť se tu mnohdy " mlátí prázdná sláma" příkladně nad zcela scestnými výplody pana Vlada.


Tomáš_Habala - 29/7/2008 - 10:35

citace:
dík, pane Aleši, ale mě by postačilo znát hodnoty prvé korekce jednotlivých sond,a zejména jejich kladný či záporný smysl, z toho by se dalo usoudit zda bylo těleso sondy urychleno či nikoli.
Dále znát okamžik a vzdálenost vlastních sond kdy a v jaké vzdálenosti by minuly Jupitera bez prvé korekce.


Tak ja ešte raz napíšem o New Horizons.

2006-01-19 19:44:55 UT - Oddelenie sondy od urýchľovacieho stupňa Star 48B
2006-01-28 dráhová korekcia TCM-1a Δv=-5 m/s
2006-01-30 dráhová korekcia TCM-1b Δv=-13.3 m/s
2006-02-01 sonda sa nachádzala približne 15000 km za Star 48B
2006-02-15 dráhová korekcia TCM-2 zrušená
2006-03-09 dráhová korekcia TCM-3 Δv=1.16 m/s
2007-02-28 gravitačný manéver pri Jupiteri Δv=+4 km/s, Star 48B priletel k Jupiteru o 6 hodin skôr, vzdialenosť medzi sondou a stupňom bola cca 400 000 km.
2015-07-14 najväčšie priblíženie New Horizonsu k Plutu
2015-10-15 Star 48B pretne dráhu Pluta, od sondy bude vzdialený 200 mil km.

Stupeň Centaur zostal trvalou družicou Slnka. Jeho dráha zasahuje hlboko do pásma asteroidov. Parametre dráhy sú:

veľká poloosa: 300.46694 mil. km
výstrednosť : 0.51053830
sklon: 5.7429941°
obežná doba: 2.85 roka

V diskusii k New Horizonsu zaznel názor, že Slnečnú sústavu možno opustil aj posledný stupeň sondy Pioneer-lO.Tam bolo po oddelení urobené len zrýchlenie + l4 m/s, takže stupeň pri Jupiteri meškal za sondou cca 12 hodín, čo by malo znamenať posunutie bodu prieletu o 800 000 km, čo by mohlo vydať potrebné 2 km/s urýchlenia.


jan.tak - 29/7/2008 - 11:01

To mě napadlo, připravuje se vůbec ještě nějáká sonda (myslím jestli je už na rýsovacích prknech nebo k nim mela blýzko:)co by mohla opustit sluneční soustavu? O ničem takovým nevím sem si kdysi poslal na NH svoje meno na CD a je to dobrej pocit, že letí uplně dopryč. A vypadá to jako na dlouho poslední šance něčeho takovýho. Už jen ten přílet 2015 k plutu! co vím co tou dobou bude)


Emil - 29/7/2008 - 13:33

Dík Tomášovi, ale NH je celkem jasný, ten Sluneční soustavu zcela jistě opustí. Mne by však zajímaly obdobné hodnoty korekcí u ostatních sond vč. Ulyssesu, pak by se dala jejich dráha zhruba odvodit. Ještě by mne zajímalo, zda stupeň NH bude prolétat před nebo za Plutem ve smyslu jeho pohybu okolo Slunce ?
Děkuji.


Roman - 29/7/2008 - 13:45

Japonská sonda s nákladem kometárního prahu se vrací na Zemi, mohl bych vědět kdy má přistát ?


jan.tak - 29/7/2008 - 14:02

citace:
Japonská sonda s nákladem kometárního prahu se vrací na Zemi, mohl bych vědět kdy má přistát ?


jestli myslíš Hayabusu tak ta se pokoušela něco odebrat na planetce Itokava, ale vůbec se neví jestli se jí odebrat něco posařilo, měli problémy jak s odběrem tak řízením sondy, neví se jestli doletí, měla v r.2010


ales - 29/7/2008 - 15:34

citace:
citace:
Japonská sonda s nákladem kometárního prahu se vrací na Zemi, mohl bych vědět kdy má přistát ?
jestli myslíš Hayabusu tak ta se pokoušela něco odebrat na planetce Itokava, ale vůbec se neví jestli se jí odebrat něco posařilo, měli problémy jak s odběrem tak řízením sondy, neví se jestli doletí, měla v r.2010
Detaily o sondě Hayabusa (česky) jsou např. na http://spaceprobes.kosmo.cz/index.php?&cid=83


Jano - 29/7/2008 - 17:39

Dráhy (napr. aj voči barycentru Slnečnej sústavy) hromady telies sa dajú zistiť aj pomocou služby JPL Horizons:
http://ssd.jpl.nasa.gov/?horizons
http://ssd.jpl.nasa.gov/?horizons_doc
Spomínajú tam aj "selected spacecraft", ciže vybrané sondy. Takže buď to už v zozname majú, alebo by možno aspoň vedeli poradiť, ako sa k presným informáciám o posledných stupňoch dá dopátrať.


David - 30/7/2008 - 16:20

Chtěl bych se zeptat Tomáše h. zda náhodou neví jaké hodnoty měly prvé korekce u ostatních předmětných sond / P-ll, V-l, V-2 a Ulysses/a zda byly kladné, či záporné.
Děkuji.


Tomáš_Habala - 30/7/2008 - 18:47

citace:
Chtěl bych se zeptat Tomáše h. zda náhodou neví jaké hodnoty měly prvé korekce u ostatních předmětných sond / P-ll, V-l, V-2 a Ulysses/a zda byly kladné, či záporné.
Děkuji.

To neviem. Skus sa opytat tu:
http://www.unmannedspaceflight.com/index.php?showtopic=4834&pid=112497&mode=threaded&start=#entry112497


Rudla - 31/7/2008 - 08:13

Dík, zkusím to, překonám-li jazykovou bariéru.


David - 31/7/2008 - 08:30

Dobrý nápad, dík.


JuDu - 14/11/2008 - 11:31

Dobry den,

Dufam ze nikoho nebudem velmi vyrusovat, ale nemate niekto rovnice na prepocet suradnic na oblohe (RA a Dec) na zemske suradnice (vyska, azimut) v zavislosti od miesta na Zemi a casu ?

Dakujem pekne


HonzaVacek - 14/11/2008 - 13:05

citace:
Dobry den,

Dufam ze nikoho nebudem velmi vyrusovat, ale nemate niekto rovnice na prepocet suradnic na oblohe (RA a Dec) na zemske suradnice (vyska, azimut) v zavislosti od miesta na Zemi a casu ?

Dakujem pekne


To co potřebujete, je transformace z rovníkových souřadnic na obzorníkové.

http://en.wikipedia.org/wiki/Horizontal_coordinate_system
http://www.grecner.cz/astro/alt-az-01.doc

Ale na internetu se toho dát najít víc. Můžete použít i některou z učebnic nebo skript sférické trigonometrie nebo astronomie. Např.: Základy astronomie a astrofyziky od V. Vanýska, kde je o souřadnicích celá jedna kapitola. [Upraveno 14.11.2008 poslal HonzaVacek]


JuDu - 20/11/2008 - 14:49

Ahoj Honza,

Dakujem za odpoved, idem rovno pocitat ...

Juraj


Vlado. - 20/11/2008 - 23:49

Laická otázka. Co mohlo vytvořit na Měsíci útvary zvětralého pohoří ? Když tam základní erozivní síly (voda ,atmosféra ) schází. Sopeční a intaktní činnost takové oblé útvary neudělá.
Děkuji za každé vysvětlení.

ttp://www.lpi.usra.edu/resources/apollo/images/browse/AS17/140/21496.jpg
http://www.lpi.usra.edu/resources/apollo/images/browse/AS15/89/12176.jpg
http://www.lpi.usra.edu/resources/apollo/images/browse/AS15/87/11793.jpg
http://www.lpi.usra.edu/resources/apollo/images/browse/AS15/90/12188.jpg


Ervé - 21/11/2008 - 08:24

Nejsem geolog, ale láva deroucí se k povrchu vytvoří krásnou oblinu i na Měsíci. I když se kůrou neprotlačí, zvedne povrchovou vrstvu, pak už jen záleží na tom, kolik a jakých meteorů/asteroidů danou oblast zasáhne a přetvoří. Nezapomínat na vliv tepelného namáhání (den-noc), výrony plynů a lunotřesení (slapový vliv Země).


David - 21/11/2008 - 12:49

Zajímalo by mne v jaké vzdálenosti proletěla sonda Ulysses kolem Jupitera a jakou hodnotu měly provedé korekce dráhy po startu, myslím tím, zda sondu brzdily, či urychlily.
Děkuji.


yamato - 21/11/2008 - 13:22

citace:
Nejsem geolog, ale láva deroucí se k povrchu vytvoří krásnou oblinu i na Měsíci. I když se kůrou neprotlačí, zvedne povrchovou vrstvu, pak už jen záleží na tom, kolik a jakých meteorů/asteroidů danou oblast zasáhne a přetvoří. Nezapomínat na vliv tepelného namáhání (den-noc), výrony plynů a lunotřesení (slapový vliv Země).


Ja tiez nejsom geolog, ale este by som dodal ze ostre stitovite pohoria ake pozname zo zeme vznikaju pri tektonickych pohyboch. Mesiac ziadne take pohyby nema, takze ani nie je dovod aby mal ine pohoria nez take ake ma. Ale mal by sa vyjadrit geolog.


Petr Tomek - 21/11/2008 - 14:29

citace:
Nejsem geolog, ale láva deroucí se k povrchu vytvoří krásnou oblinu i na Měsíci. I když se kůrou neprotlačí, zvedne povrchovou vrstvu, pak už jen záleží na tom, kolik a jakých meteorů/asteroidů danou oblast zasáhne a přetvoří. Nezapomínat na vliv tepelného namáhání (den-noc), výrony plynů a lunotřesení (slapový vliv Země).


Tady bych drobně polemizoval, záleží totiž dost na množství vody které láva obsahuje. Také nejsem geolog, ale tohle si z malého vulkanologického muzea na Puy-de-Dôme pamatuji. Výbuchy sopek totiž způsobuje především silně přehřátá pára, která se uvolní, když na lávu přestane působit tak silný tlak geologických vrstev (tj. dostane se třeba jen kilometr pod povrch země). Proto je také láva na průřezu plná bublinek. Ve vakuu by se ale chovala jinak, protože by pára měla v podstatě neomezenou možnost rozpínání. Vzniklo by tedy asi něco jako pemza ale s poměrně velkými bublinami, které by občas praskly. Ale je také možné, že by vznikly bubliny tak tenké, že by se rozsypaly už během stydnutí. Pokud si zase představíme lávu bez obsahu vody, byly by zase mnohem méně pravděpodobné vývěry (vznikají změnou objemu).


Petr Tomek - 21/11/2008 - 14:42

Nicméně ta otázka je dobrá:
Co mohlo vytvořit na Měsíci útvary zvětralého pohoří, když tam základní erozivní síly (voda ,atmosféra ) schází?

To co je vidět v pozadí totiž nevypadá jako pohoří zvětralá vodou.
Chybí jí erozní rýhy, jako např:
http://images.wildmadagascar.org/pictures/tana_flight/madagascar_erosion_aerial_view_3.JPG

a nevypadá to ani jako větrná eroze
http://www.foldabikes.com/CurrentEvents/Story/Photos/SouthAmerica/stone.jpg

Vlastně to nejvíc připomíná duny
http://hongkongmilktea.files.wordpress.com/2008/02/desert-inner-mongolia-edit.jpg

ovšem chybí ten hřeben ohrnutý větrem.

nejjednodušší vysvětlení je, že uvnitř každého kopce je velký kámen nebo několik balvanů a to co je nad nimi je prostě prach a písek z dopadů meteoritů. Ovšem daly by se vymyslet i lepší věci.


Alchymista - 21/11/2008 - 15:39

Ma povrchu Mesiacu je niekoľko typov objektov, ktoré vznikali v rôznych obdobiach vývoja Mesiacu a rôznymi procesmi.
Je tam "pôvodný" terén starý miliardy rokov - bol tvarovaný veľkými dopadmi, seizmickými a inými prejavmi veľkých dopadov, plus tektonikou mladého mesiacu a sčasti možno aj jeho dočasnou atmosférou. Tento povrch je pokrytý troskami (balvanmi) z veľkých dopadov, predovšetkým z obdobia "veľkého bombardovania" po ztuhnutí mesačnej kôry. Prakticky celý tento pôvodný povrch je pokrytý vrstvou regolitu - "mesačnej pôdy" miliardy rokov drvenej a preorávanej dopadmi meteoritov a mikrometeoritmi všetkých veľkostí.
Na tomto povrchu sú potom balvany z mladších dopadov, opäť pokryté väčšou či menšou vrstvou regolitu, sčasti vzniknutou na mieste a sčasti premiestnenou z iných miest povrchu pri dopadoch ďalším meteoritov.

Nedá sa povedať, že povrch Mesiacu nie je tvarovaný eróziou - erózia tam tiež pôsobí, je to ale erózia iného typu, než ju poznáme na Zemi. A predovšetkým, pôsobí v časových merítkach, pre ktoré na Zemi asi neexistuje ekvivalent, dokonca aj pohyb pozemských kontinentov je v porovnaní s procesmi utvárania povrchu Mesiacu rýchly.


Jirka - 21/11/2008 - 15:44

http://en.wikipedia.org/wiki/Image:Moon-craters.jpg

NO nevim, ja na obrazcich z Mesice vidim vetsinou jen utvary typicke pro kratery, popripade velmi stare planiny vznikle vylevem magmatu, prikryte nadrcenym prachem.
Je mozne ze ten prach prodelal nejakou metamorfozu pusobenim slunecniho vetru a kosmickym zarenim.
Na zemi vetsina mineralu vznikla za spoluucasti vody, coz jim casto dava tvrdost betonu. Na Mesici ci jinych telesech s minimem vody jsou vsak vlastnosti mineralu naprosto rozdilne.
Docela rad bych provedl experiment, kdy bych lunarni prach pokropil vodou (za atmosferickeho tlaku). To jsem zvedav co by se stalo.


Alchymista - 21/11/2008 - 16:00

Čo som čítal, mesačný prach sa chová dosť odlišne od prachu "pozemského".
Napríklad obsahuje veľmi vysoký podiel častíc ostrohranných, takže údajne dosť pripomína, po stránke mechanických vlastností, brúsne materiály (voľné brusivá). Je to evidentne dôsledok toho, že po vzniku sa zrná príliš nepohybovali. Pozemský prach má naopak vysoký podiel guľovitých či oblých častíc, čo je zasa dôsledok pohybu a omieľania zŕn pri ich neustálom transporte vodou a vetrom.

Zmes mesačného prachu a vody by zrejme dosť pripomínala brúsnu pastu a nedá sa tiež vylúčiť, že by časom reagovala nejak podobne ako cement - určite by v nej prebiehali nejaké chemické reakcie zložiek prachu s vodou a tiež procesy rozpúšťania a rekryštalizácie.


Vlado. - 21/11/2008 - 21:48

citace:
Nejsem geolog, ale láva deroucí se k povrchu vytvoří krásnou oblinu i na Měsíci. I když se kůrou neprotlačí, zvedne povrchovou vrstvu, pak už jen záleží na tom, kolik a jakých meteorů/asteroidů danou oblast zasáhne a přetvoří. Nezapomínat na vliv tepelného namáhání (den-noc), výrony plynů a lunotřesení (slapový vliv Země).
Já taky nejsem geolog. Myslím si že při vulkanické činnosti nebudou vnikat pupence ale láva (magma) po výronu tuhne ,sopka roste , v případě přetečení přes hranu kráteru se vytváří lávové pole (moře) Viděl jsem to na N Zélandu ,Reunionu , atd. Jaký podíl má sopečný prach na Měsíčním prachu nebo regolitu?
Tepelné namáhání (den a noc) na lávové kameny bez obsahu vody nemá vliv .
Při pohledu na sypné uhly kráterů tam moc velké lunotřesení nebývá.
citace:
Ja tiez nejsom geolog, ale este by som dodal ze ostre stitovite pohoria ake pozname zo zeme vznikaju pri tektonickych pohyboch. Mesiac ziadne take pohyby nema, takze ani nie je dovod aby mal ine pohoria nez take ake ma. Ale mal by sa vyjadrit geolog.
Já si myslím že i Měsíc prošel tektonickou tvorbou (vrásněním).
citace:
nejjednodušší vysvětlení je, že uvnitř každého kopce je velký kámen nebo několik balvanů a to co je nad nimi je prostě prach a písek z dopadů meteoritů. Ovšem daly by se vymyslet i lepší věci.
http://www.lpi.usra.edu/resources/apollo/images/browse/AS17/140/21496.jpg

Odkud ten kamen přilítnul ,že se nerozbil ,že neudělal kráter ,že není zaprášen jako okolí ,že není vidět stopa po kutálení ?


Alchymista - 21/11/2008 - 22:16

citace:
Odkud ten kamen přilítnul ,že se nerozbil ,že neudělal kráter ,že není zaprášen jako okolí ,že není vidět stopa po kutálení ?

1) Balvan sa pri dopade rozbil - oba veľké balvany na zábere zrejme pôvodne tvorili jeden kus, zrejme aj s kameňmi v ich okolí
2) Na bližšom balvane je vidieť pekných pár vedier prachu a regolitu - regolit na balvane má takmer rovnakú farbu ako povrch terénu
3) Kráter sa nevytvoril, pretože na tento balvan to nemal dostatočnú dopadovú rýchlosť, sám bol vyrazený z povrchu Mesiaca pri dopade meteoritu a vzniku kráteru - pri podrobnom preskúmaní by sa dalo dokonca odhadnúť aj to, z ktorého smeru a akou rýchlosťou dopadol.

A že sa nekotúľal? A prečo by sa mal?


Vlado. - 21/11/2008 - 23:41

http://www.lpi.usra.edu/resources/apollo/images/browse/AS17/141/21596.jpg

http://www.lpi.usra.edu/resources/apollo/images/browse/AS17/141/21596.jpg
Chce to velkou představivost aby to dopadlo jak to dopadlo
Tak velký balvan i při 1/6 gravitaci letící bůhví odkud , musí do měkkého regolitu udělat nějaký ten důlek.
Podle toho pohoří v pozadí to bude nějak čerstvé ,ani se ještě nestačil usadit kosmický prach.


Vlado. - 21/11/2008 - 23:45

http://www.lpi.usra.edu/resources/apollo/images/browse/AS17/141/21597.jpg
Pro porovnání velikosti.


Derelict - 22/11/2008 - 00:47

citace:
http://www.lpi.usra.edu/resources/apollo/images/browse/AS17/141/21597.jpg
Pro porovnání velikosti.



Vlado, ja uz nevim. Podvrh je jenom Apollo, nebo cely Mesic?

Lava je obecne dostatecne tepla smes hornin (cca 1000C, ale zavisi to na velkem mnozstvi parametru), obsahujicim nejenom vodu, ale i plyny. Siricitany, uhlicitany a dalsi, vazane v hodninach (priznam se, ze bych docela rad videl vylev lavy na mesici, rozhodne by to mohlo byt zajimave divadlo. Zajimavejsi nez vylev lavy na povrchu a mozna stejne zajimave jako vylev lavy pod hladinou). Protoze chemicke slozeni Mesice je jak se zda podobne Zemi, rozdil vznikne az pri kontaktu s vakuem.

Chladnuti materialu vyzarovanim bude branit tvorbe vetsich krystalickych celku na povrch, ale je pravdepodobne ze ve vetsich hloubkach by mohly byt. Protoze i Mesic byl zhave teleso, musel projit fazi chladnuti kdy se vytvori krusta, ktera diky slapovym silam Zeme musela prochazet jevy, ktere by se daly popsat jako tektonika. V tuto chvili a po nejakou dobu byl Mesic dostatecne cinnny, aby mohl mel aktivni tektonickou cinnost ve vsech formach. Vcetne vrasneni a sopek.

Tepelne namahani ma vliv na jakykoliv material. Rozdil dne a noci dost vyrazny na projevy tepelne dilatace i u homogennich teles. Vnitrek telesa zustane teplejsi, vnejsi cast (kura) chladne rychleji a tak vznika tepelne namahani dostatecne k tomu, aby se casem teleso rozpadlo, nebo alespon znacne popraskalo. Zalezi pouze na tepelne vodivosti(bazalty, regolit???), gradientu tepelneho spadu (rozdil cca 400C, neznama doba, odhad 12h) a soucinitelich tepelne roztaznosti. Dostupne informace nejsou dostatecne uplne pro provedeni alespon zakladnich vypoctu, takze se je to nazor proti nazoru.

Na Zemi jednak erozi pomaha, jednak maskuje erozi cela biosfera. Naproti tomu na Mesici neni zadna ochranna vrstva atmosfery a magnetosfery. Chybi zde eroze pomoci vetru a vody, objevuje se zde ale radiacni starnuti (nebo spise eroze), eroze zpusobena primarnimi a sekundarnimi dopady mikrometeoritu a meteoritu, postupnym uvolnovanim plynu z "kaps" vytvorenych pri nataveni materialu (dopad meteoritu a kdysi i vylevy lavy), ale i slapovymi silami pusobicimi na vetsi celky a jiz zminenym tepelnym namahanim. Casem zde pribude, jak doufam, erozivni pusobeni lidskou cinnosti.


David - 22/11/2008 - 08:42

Nerad ruším, ale Vlado má své vlastní vlákno a na můj dotaz jste zapoměli. Myslím ohledně sondy Ulysses.Dovoluji se proto opět připomenout.
Děkuji.


Vlado. - 22/11/2008 - 11:52

Měsíc je pravý ! Čestné soudružské i důchodcovské.

citace:
Tepelne namahani ma vliv na jakykoliv material. Rozdil dne a noci dost vyrazny na projevy tepelne dilatace i u homogennich teles. Vnitrek telesa zustane teplejsi, vnejsi cast (kura) chladne rychleji a tak vznika tepelne namahani dostatecne k tomu, aby se casem teleso rozpadlo, nebo alespon znacne popraskalo. Zalezi pouze na tepelne vodivosti(bazalty, regolit???), gradientu tepelneho spadu (rozdil cca 400C, neznama doba, odhad 12h) a soucinitelich tepelne roztaznosti. Dostupne informace nejsou dostatecne uplne pro provedeni alespon zakladnich vypoctu, takze se je to nazor proti nazoru.
Ze vším souhlasím , ale podstatný pro destrukci tepelně namáhaného materiálu je čas působící tuto změnu .
Na rovníku to bude 29,5 dne tj. 708 hodin při rozdílu teplot +123 -230 st C To já nepovažuji za tepelné namáhání.
Za tepelný šok na Měsíční kameny považuji pohyb stínu kosmonauta po povrchu. To by ti kameny musely praskat před očima .
Dále si myslí ,že Zem na Měsíc nepůsobí slapovými silami, nebo se jedná o vázanou rotaci.Slapově působí slunce v kombinaci se Zemí


Alchymista - 22/11/2008 - 12:31

Tak toto je už celkom jasný doklad nevzdelanca.
Dráha Mesiaca okolo Zeme nie je kruhová, ale eliptická - s perigeom 363 104 km a apogeom 405 696 km. Navyše rovina dráhy nie je v rovine ekliptiky, ale má sklon 1,54° a osa rotácie je od roviny dráhy sklonená od 3,60° do 6,69° (kolíše). Dôsledkom sú librácie alebo librace
Zemské slapy na Mesiaci sú relatívne slabšie ako mesačné slapy na Zemi, ale to je okrem iného dôsledkom toho, že na Mesiaci nie je voda, ktorá by sa mohla relatívne ľahko premiestňovať a zároveň je librácia pomerne malá oproti rotácii Zeme.


Derelict - 23/11/2008 - 14:26

citace:
Měsíc je pravý ! Čestné soudružské i důchodcovské.
citace:
Tepelne namahani ma vliv na jakykoliv material. Rozdil dne a noci dost vyrazny na projevy tepelne dilatace i u homogennich teles. Vnitrek telesa zustane teplejsi, vnejsi cast (kura) chladne rychleji a tak vznika tepelne namahani dostatecne k tomu, aby se casem teleso rozpadlo, nebo alespon znacne popraskalo. Zalezi pouze na tepelne vodivosti(bazalty, regolit???), gradientu tepelneho spadu (rozdil cca 400C, neznama doba, odhad 12h) a soucinitelich tepelne roztaznosti. Dostupne informace nejsou dostatecne uplne pro provedeni alespon zakladnich vypoctu, takze se je to nazor proti nazoru.
Ze vším souhlasím , ale podstatný pro destrukci tepelně namáhaného materiálu je čas působící tuto změnu .
Na rovníku to bude 29,5 dne tj. 708 hodin při rozdílu teplot +123 -230 st C To já nepovažuji za tepelné namáhání.
Za tepelný šok na Měsíční kameny považuji pohyb stínu kosmonauta po povrchu. To by ti kameny musely praskat před očima .
Dále si myslí ,že Zem na Měsíc nepůsobí slapovými silami, nebo se jedná o vázanou rotaci.Slapově působí slunce v kombinaci se Zemí



Vlado,
podivej se na mostni konstrukce. Betonove mosty vcetne naspu jsou "homogenni" (hodne v uvozovkach, ale bavim se o meritku), protoze pouzite materialy jako beton a udusany nasep na obou stranach budou mit obdobnou tepelnou roztaznost. A podivej se na takove ty mezirky, co sou na obou stranach. Chrani se tim pred tepelnou roztaznosti pri zmenach teplot. Napriklad zima a leto. Nehommogennim utvarem v tomto slova smyslu jsou napriklad koleje. Z fyziky na druhem stupni zakladni skoly si pamatuji jejich prodluzovani a poznamku meho ucitele ("Proto v lete maji ceske drahy zpozdeni").
Protoze horniny na Mesici jsou take homogenni jenom z pouzitim vetsiho meritka, bude zde dochazet k obdobnym vlivum. Bohuzel plasticita vetsiny materialu klesa s teplotou, je rozdil v schopnostech materialu pri teplotach okolo 100 stupnu a vyse, a je dost dramaticky rozdil pri teplotach -100 stupnu a nize. Zkusil jsi nekdy klepnout do oceli, podchlazene napr. kapalnym dusikem ? Staci kozena rukavice aby te ochranila od kontaktu a rukou prerazis za normalnich okolnosti tvarny plech (a ne slaby ;o) ). Byly to hloupe hratky, ale dostatecne "inspirativni". Prachu to nevadi, ale vetsi monoliticke celky budou za techto okolnosti praskat nejenom z duvodu tepelne roztaznosti, ale doslo mi, ze i slapovymi silami, lunotresenim, nebo i zvukem impaktu siricim se horninami. Jde pouze o silu dostatecnou k prekonani pevnostnich hranic materialu. A material je s klesajici teplotou cim dal tim krehci ....


Vlado! - 23/11/2008 - 17:42

To taky chápu . Mostní konstrukce jsem řešil , ba závěsy a ložiska vyráběl. Obdivuji důvtip tvého učitele .
Mně se nejednalo o trhlinách v masívech ,ale o zvětrávání povrchu pohoří ,sopek , hran impaktů a kameni jim vyražených. No prostě ,jak udělat Nízké Tatry z Vysokých.
Dopluji můj předchozí příspěvek v rychlosti ochlazování . Při pohybu slunečního stínu po povrchu Měsíce dochází ochlazování 0,5st (uhlových) /hod , na Zemi to je 30x rychleji
Doporučuji pokus .Zahřát lávový kamen na 130st C vložit ho do termosky a ponořit do tekutého dusíku .Povyrovnání teplot (cca 24h - to není 708h) kamen bude studený ale celý.


martinjediny - 23/11/2008 - 19:08

citace:
To taky chápu . Mostní konstrukce jsem řešil , ba závěsy a ložiska vyráběl. Obdivuji důvtip tvého učitele .
Mně se nejednalo o trhlinách v masívech ,ale o zvětrávání povrchu pohoří ,sopek , hran impaktů a kameni jim vyražených. No prostě ,jak udělat Nízké Tatry z Vysokých.
Dopluji můj předchozí příspěvek v rychlosti ochlazování . Při pohybu slunečního stínu po povrchu Měsíce dochází ochlazování 0,5st (uhlových) /hod , na Zemi to je 30x rychleji
Doporučuji pokus .Zahřát lávový kamen na 130st C vložit ho do termosky a ponořit do tekutého dusíku .Povyrovnání teplot (cca 24h - to není 708h) kamen bude studený ale celý.



Sorry, ale to je !
Jaka rychlost zmeny teploty?! Chladnutie je jasne proste sa vyziari a to uz pri obycajnom radiatore je cca 30% odovzdavaneho tepla, takze nic zanedbatelne.

Zasadny omyl je ale v rychlosti ohrevu. Do hlboko podchladeneho kamena zacne na plny vykon prazit netienene Slnko! Hned, okamzite, bez nabehu!
O takych podmienkach sa nam na Zemi ani nesnivalo a tekuty dusik je oproti tomu slaby jak caj. A pocet cyklov za miliony a miliardy rokov?


ales - 23/11/2008 - 20:46

citace:
... Mně se nejednalo o trhlinách v masívech ,ale o zvětrávání povrchu pohoří ,sopek , hran impaktů a kameni jim vyražených. No prostě ,jak udělat Nízké Tatry z Vysokých.
Vlado, na tuto otázku by mohl fundovaně odpovědět jen specialista na lunární povrch a myslím že je zřejmé, že mezi čtenáři zdejšího fóra nikdo takový není. Nelze prostě čekat perfektní odpověď na jakoukoliv zde položenou otázku.

Osobně mohu odpovědět jen jako laik se selským rozumem. Po chvíli hledání na internetu bych řekl, že zvětrávání povrchu Měsíce (a jiných kosmických těles bez atmosféry) solidně popisují např. texty na http://en.wikipedia.org/wiki/Space_weathering nebo na http://www.pnas.org/content/101/18/6847.full . Vyplývá z nich, že povrch Měsíce nejvíce erodují a přetvořují dopady meteoritů, mikrometeoritů, tvrdého kosmického záření a částic "slunečního větru". Pro mne to je to zcela dostatečné vysvětlení. Pokud to někomu nestačí, nemám už co dodat (protože na to nejsem odborník).

citace:
... Dopluji můj předchozí příspěvek v rychlosti ochlazování . Při pohybu slunečního stínu po povrchu Měsíce dochází ochlazování 0,5st (uhlových) /hod , na Zemi to je 30x rychleji
Ale no tak. To si, Vlado, opravdu myslíš, že povrch Měsíce 14 dní pomalu chladne a pak se 14 dní pomalu ohřívá? Jak už napsal Martin, tak je zcela jasné, že na rozhraní světla a stínu (při východu a západu Slunce) dochází na Měsíci k mnohem rychlejší změně teploty v řádu stovek stupňů za pár desítek minut. Myslím, že relativní úhlová rychlost pohybu Slunce na to nemá zásadní vliv.


Vlado! - 23/11/2008 - 21:29

citace:
Zalezi pouze na tepelne vodivosti(bazalty, regolit???), gradientu tepelneho spadu a soucinitelich tepelne roztaznosti
Srozumitelně to vysvětlil Jandus.
citace:
Do hlboko podchladeneho kamena zacne na plny vykon prazit netienene Slnko! Hned, okamzite, bez nabehu!
U toho náběhu výkonu Slunce je třeba brát v úvahu uhel dopadu (denní dobu)
Něco by nám o tom mohli říct kosmonaute s ISS


martinjediny - 23/11/2008 - 22:21

citace:
citace:
Do hlboko podchladeneho kamena zacne na plny vykon prazit netienene Slnko! Hned, okamzite, bez nabehu!
U toho náběhu výkonu Slunce je třeba brát v úvahu uhel dopadu (denní dobu)
Něco by nám o tom mohli říct kosmonaute s ISS



1/ Reagoval som len na zakladne chyby v naivnej predstave o neskodnosti tepelnej dilatacie na Mesiaci a nepopisoval som problem komplexne
2/ nehovorim o bocnych stenach, o vrchnej a ani o zadnej stene z pohladu svetla, ale o strane privratenej k Slnku v danom okamihu
3/ Tak ako na cundri nam Slnko osvietilo celu tvar v kratkom okamihu(po prekonani prekazky na obzore) taky isty zasah dostanu aj balvany a "vysoke tatry" na Mesiaci
4/ Skus tie Alesove clanky, erozia je tam popisana daleko komplexnejsie a odbornejsie ako som toho schopny ja.


hloupa otazka - 3/12/2008 - 10:25

Mám dotaz, nevíte proč se užívá přepočet z radiánů na stupně, resp. proč má lidstvo definováno , že má kruh 360 stupňů? Jaký má ta číslovka 360 historicko vědecký důvod použití? Vím, hloupost, ale vrtá mi to hlavou Slyšel jsem že to souvisí něják s 12 měsícema za rok, tj počtem dní v roce, ale nejsem si s tím jistý. Diky


ales - 3/12/2008 - 10:41

citace:
Mám dotaz, nevíte proč se užívá přepočet z radiánů na stupně, resp. proč má lidstvo definováno , že má kruh 360 stupňů? Jaký má ta číslovka 360 historicko vědecký důvod použití? Vím, hloupost, ale vrtá mi to hlavou Slyšel jsem že to souvisí něják s 12 měsícema za rok, tj počtem dní v roce, ale nejsem si s tím jistý. Diky
Přesný důvod zavedení šedesátkové soustavy asi už dnes nikdo nezná (soustavu používali už Sumerové). Uvádí se jen řada různě pravděpodobných názorů - viz např. http://www.myty.info/view.php?cisloclanku=2007100001 .


JuDu - 18/1/2009 - 21:40

Dobry vecer,

Mohol by mi niekto vysvetlit, ako funguje u druzic/kozmickych telies orientovanie v kozme/v blizkosti Zeme ? Hlavne mi ide o fotoprieskumne druzice, ktore musia byt zamerane s velkou presnostou. Musia vediet kedy budu lietat nad urcitou krajinou. Ak odfotografuju nieco, ako sa urcia spatne presne suradnice ciela ? Maju na fotke nejake zname orientacne body ?
Napr. znama fotografia pocas studenej vojny, kde americania zvejernili fotku so spionazneho satelitu o zastupe ludi stojiace pred Leninovom mauzoleumom. Satelit dostal so Zeme presnu cas a to, kedy ma spustit natacanie na film ? Polohu so Zeme mozem optickymi pristrojmi/radarom urcit dostatocne presne, aby to bolo pouzitelne ?

Dakujem pekne


Alchymista - 18/1/2009 - 22:37

Družica určuje svoju priestorovú orientáciu spravidla optickým zameraním na významné vesmírne telesá - Slnko, vhodné jasné hviezdy a podobne. Orientáciu udržiavajú a upravujú raketové motory a gyroskopy, v menej náročných prípadoch sa používa pasívna orientácia. Priestorová poloha môže byť určená zo Zeme sledovaním dráhy družice opticky a radarom alebo priamo na družici napríklad zo signálov družíc GPS.
So znalosťou priestorovej orientácie družice, parametrov dráhy a polohy voči známym meracím stanovištiam je možné určiť orientáciu snímacieho zariadenie a okamihu snímania, aby sa záujmový objekt dostal do záberu. Pokiaľ nie je poloha záujmového objektu presne známa, snímajú sa napríklad pásy dlhé až stovky kilometrov, ktoré sa potom skladajú a objekt sa na záberoch vyhľadá a poloha určí zameraním voči známym objektom alebo z parametrov dráhy a orientáce družice a snímačov - tieto údaje sú súčasťou doplnkových údajov k jednotlivému záberu.

Presnosť určenia vlastnej priestorovej orientácie družice a snímačov a ich krátkodobá stabilita môže dosahovať až tisíciny stupňa, presnosť určenia priestorovej polohy družice z krátkodobých pozorovaní jednotky metrov, z dlhodobých pozorovaní aj menej ako jeden meter. Pokiaľ je úlohou družice sledovať dôležité záujmové územie, jej poloha sa obvykle určuje jednak krátko pred preletom a potom čo najskôr po prelete nad záujmovým územím a výsledky sú pripojené k záberom.
Dôležité snímkovania sa nerobia okamžite po vypustení družice, obvykle je družica nejaký čas v skúšobnej prevádzke, ktorá okrem iného slúži práve na presné určenie parametrov dráhy a kalibrovanie snímačov.


JuDu - 19/1/2009 - 08:02

Ahoj Alchymista,

Dakujem pekne za odpoved.

Juraj


bejcek - 19/1/2009 - 12:03

Oblíbenou hvězdou pro orientaci meziplanetárních stanic je Canopus ze souhvězdí Lodní kýl, které je na jižní obloze. Jeho hvězda Canopus je druhá nejjasnější hvězda oblohy (po Siriovi), má kolem sebe slabé hvězdy, proto bývá důležitým orientačním bodem.


Ervé - 19/1/2009 - 15:09

Jak družice pozná, že je to daná hvězda? Podle intenzity jasu, polohy ostatních bodových zdrojů (hvězd) nebo podle frekvence záření hvězdy?


Mirek Pospíšil - 19/1/2009 - 15:21

citace:
Jak družice pozná, že je to daná hvězda? Podle intenzity jasu, polohy ostatních bodových zdrojů (hvězd) nebo podle frekvence záření hvězdy?
Například takhle: http://www.ces.clemson.edu/~stb/ece847/fall2004/projects/proj22.doc


Alchymista - 19/1/2009 - 15:53

citace:
Jak družice pozná, že je to daná hvězda? Podle intenzity jasu, polohy ostatních bodových zdrojů (hvězd) nebo podle frekvence záření hvězdy?
Zrejme to prebehne v dvoch krokoch - v prvom sa družica zorientuje "nahrubo" napríklad podľa Slnka a Zeme, čo postačuje na dosiahnutie presnosti priestorovej orientácie povedzme +/- 10° (Slnko má uhlový priemer okolo 0,5°, u Zeme sa budú zrejme vyhľadávať kraje - rozhranie atmosféra/vesmír, v IR pásme je presnosť ich určenia povedzme 1°) Tým je vlastne družica nahrubo zorientovaná v jednej rovine, čo pre mnoho aplikácií celkom postačuje. V druhom kroku potom začne detektorom, približne nasmerovaným v prvom kroku, vyhľadávať určenú hviezdu alebo skupinu hviezd, zrejme podľa ich rozloženia na hviezdnej sfére (teda podľa vzájomnej polohy).
Ako je to presne urobené neviem, konštrukciu nepoznám (a celkom by mato zaujímalo), ale napadá ma niekoľko riešení - napríklad rastrová matrica snímačov a nejaký počítačový soft, čo to spracuje. V primitívnejšej podobe to môže byť napríklad malý ďalekohľad, v ktorom je clona s presne rozmiestnenými otvormi a za ňou skupina fotodetektorov - správna poloha je dosiahnutá, keď vydajú všetky fotodetektory signál, že zachytili svetlo hviezd so stanovenou intenzitou. Ako sa detektor do správnej polohy dostane, je vecou nejakého riadiaceho algoritmu, ktorý riadi polohovanie detektoru hviezd (samostatne alebo otáčaním celej družice).


JuDu - 23/4/2009 - 10:18

Ahojte,

Rozprudila sa tu diskusia, kolko vazi aerodynamicky stit a ako funguje zachranna raketa.
Napadla ma, preco nevyuzit tah tejto rakety, preco to nechat odpadnut bez prace.
Ked sa vypojuju motory prveho stupna a zapaluje sa dalsi stupen, nebolo by mozne zapnut tento motor ? Pred koncom prace motorov by sa cely stit odpojil, aj tak by to spolahlivo odletel. V case ked pracuje by zmiernil vypadok tahu (prepnutie 1. a 2. stupna).
Bolo by to nemozne ? Znizila by sa tym bezpecnost celej konstrukcie ?

Vdaka


Ervé - 24/4/2009 - 07:54

Taky jsme nad tím přemýšlel. Konstrukce záchranných systémů je podle mně řešena tak, že při zážehu hlavního motoru záchranné rakety utrhne kabinu i v případě, kdy selže oddělení kabiny od nosiče/přístrojové sekce. Je to bezpečnostní pojistka pro selhání pyrotechniky. Takže by jste pro toto využití museli vyztužit nosný systém mezi kabinou a raketou a kromě zvýšení hmotnosti by jste tím zhoršil bezpečnost, což nikdo neschválí. Výhoda nevyváží riziko. Navíc právě problémy s oddělováním 1. a 2. stupně jsou obvykle poslední kritickou situací, pro kterou je záchranný systém potřeba, takže ho nemůžete spotřebovat/odhodit dřív, než motor 2. stupně spolehlivě funguje na plný výkon.


martinjediny - 24/4/2009 - 09:22

citace:
Taky jsme nad tím přemýšlel. Konstrukce záchranných systémů je podle mně řešena tak, že při zážehu hlavního motoru záchranné rakety utrhne kabinu i v případě, kdy selže oddělení kabiny od nosiče/přístrojové sekce. Je to bezpečnostní pojistka pro selhání pyrotechniky. Takže by jste pro toto využití museli vyztužit nosný systém mezi kabinou a raketou a kromě zvýšení hmotnosti by jste tím zhoršil bezpečnost, což nikdo neschválí. Výhoda nevyváží riziko. Navíc právě problémy s oddělováním 1. a 2. stupně jsou obvykle poslední kritickou situací, pro kterou je záchranný systém potřeba, takže ho nemůžete spotřebovat/odhodit dřív, než motor 2. stupně spolehlivě funguje na plný výkon.

To je fakt, ale neslo by pouzit napr. motory, ktore sa pouzivaju na pribrzdenie pri pristavani na sus?


Alchymista neprihlásený - 24/4/2009 - 11:08

Asi nie - na záchranu je potrebné kabíne udeliť tak veľké zrýchlenie, aby sa v čase rádu jednotiek sekúnd dostala do vzdialenosti niekoľko sto metrov od rakety - na to je potrebný raketový motor značného výkonu s náplňou okolo jednej tony paliva pri hmotnosti celej poháňanej zostavy do desať ton. Delta v je okolo 100m/s a doba činnosti jednotky sekúnd.
Motory mäkkého pristátia sú oproti záchrannému systému relatívne veľmi slabé - ich úlohou je pribrzdiť rýchlosť pádu kabíny s hmotnosťou do 5 ton z rýchlosti okolo 10m/s na rýchlosť okolo 3m/s na dráhe pod 10 metrov - delta v je teda 5-10 m/s a doba činnosti zlomky sekundy. Hmotnosť náplne raketových motorov je niekoľko desiatok kilogramov paliva (menej než 100 kg).

Pokiaľ by to mal zabezpečiť jeden a ten istý systém, na záchranu bude na spodnej hranici bezpečnosti - nemusí dostať kabínu do potrebnej vzdialenosti a nemusí to stihnúť dostatočne rýchlo.
Na pristátie je zasa zbytočne silný - dokáže kabínu zdvihnúť, čo môže mať za následok "vyliatie" padáku, zdvihnutie kabíny do výšky desiatok či stoviek metrov, z ktorej kabína vzápätí padne prakticky voľným pádom, so všetkými následkami.
Navyše by mal takýto kombinovaný systém aj značnú hmotnosť, s ktorou by loď absolvovala prakticky celý let - na úkor užitočnej nosnosti.

Záchranný systém má u Sojuzu i u Apollo niekoľko raketových motorov, záchranu zabezpečuje jeden motor, oddelenie záchranného systému od rakety, keď prestane byť potrebný, druhý, obvykle značne slabší.

Juraj Duducz - keď si predstavíte, že druhý stupeň s palivom váží pár desiatok ton, zisk rýchlosti nebude veľký, možno niekoľko metrov za sekundu, skôr ešte menej, pretože zostava musí byť výrazne pevnejšia (a teda tažšia) a zostáva tu problém, ako použitý záchranný systém a aerodynamický kryt oddeliť.


x - 4/7/2009 - 00:53

Mate pro todle jine vysvetleni ci proste jen kachna ???? - tedy ze to neni to co tam pisi a tedy neni zatim nutne to co zde nize pisu:

http://technet.idnes.cz/warpovemu-pohonu-ze-sci-fi-star-trek-teorie-podle-fyziku-nebrani-php-/tec_vesmir.asp?c=A090626_141058_tec_vesmir_vse

Tomu uplne jiz presne nerozumim, ale jestli je tomu tak - takze se zda ze by to mohla byt ona dloho hledana dira o moznosti premisteni od jedne hvezde k druhe za mnohem kratsi dobu nez je rychlost svetla - a to i Z POHLEDU POZOROVATELE NA ZEMI - uplne kratsi doba zde nez je potrebna pro cestu rychlosti svetla k dane hvezde ze ZEME !!!!!!

Mozna na case bude ponekud vhodne - pokud nebude zcela vyloucene, ze se to da uskutecnit pri mnohem mensi energeticke narocnosti,ale i tak PROSTE SE TO NEVYLUCUJE - prehodnotit moznost cestovani vesmirem rychlosti ti vetsi nez svetlo - jinak z moznosti nic neni rychlejsi nez svetlo tedy ani cestovani vesmirem nad svetelnou rychlosti - tedy premistit se k jine hvezde drive, nez by tam ze Zeme doletelo svetlo -si budou mozna i celkem brzo delat verejne legraci v TV estradach.

A mozna i prehodnotit doposud tak udajne vyloucenou moznost pritomnosti jine civilizace zde, ktera nas zde skryte jak my skryte a neruse pozorujem vazna zvirata zkouma trba kvuli moznemu budoucimu otevremu kontaku a spolupraci.


Archimedes - 4/7/2009 - 01:35

Ten trik je schovaný v tom, že zatímco na lokální rychlost čehokoliv vůči čemukoliv je limit rychlost světla, nic nebrání tomu, aby se prostor sám rozpínal nebo smršťoval zcela libovolnou rychlostí. Čili nesnažit se PŘEMÍSTIT o jeden metr, ale o jeden metr ZVĚTŠIT (zmenšit) vzdálenost. Zní to divně, ale jsou to dvě různé věci.

Právě tak by fungovala ta "cestující bublina". Některé úvahy dokonce prý vedou na fígl, jak ji udělat zvenku menší (rozměru elementární částice) než zevnitř (celá loď), což by drasticky snížilo energetickou náročnost. Stejně tak už se teoretici zaobírali myšlenkou červích děr, které by spojovaly různá místa jak v prostoru, tak v čase.

Bohužel, všechny tyhle triky (aspoň co vím) mají zdá se jednu zásadní vadu - vyžadují použití značného množství hmoty se zápornou klidovou hmotností a tu zatím nikdo na hromádce neviděl....

Tolik, co k tomu jako neteoretik můžu tlumočit


HonzaVacek - 4/7/2009 - 02:06

citace:
Mate pro todle jine vysvetleni ci proste jen kachna ?


Kachna to úplně tak není, jde ale spíše o takovou matematickou hříčku z obecné teorie relativity a vytvořit něco takového jako warpovou bublinu, o které se v článku píše, zatím patří do oblasti sci-fi. Pokud pominu, že nikdo neví jak vůbec takovou bublinu vytvořit, tak v článku je jedna celkem zásadní nepřesnost. Píšou tam o energii 1045 J (Asi mělo být 10^45). Ale i to je sakra málo. Pro vytvoření bubliny o rozměrech cca 100 m je třeba enegetického ekvivalentu 10^65 kg, který je zapotřebí k vytvoření požadované metriky časoprostoru. To ale přesahuje odhadovanou hmotnost námi pozorovaného vesmíru. Navíc ta energie musí mít záporné znaménko, což je další problém, protože nikdo neví nejenom kde to vzít, ale co si pod tím vůbec představit.

O záporné energii se sice uvažuje v souvislosti s expanzí vesmíru a označuje se jako "temná energie", ale to je asi tak všechno co o ní zatím víme.

Alcubierrovým Warp Drivem se zabývali i jiní, jako třeba van Broeck nebo Krasnikov. Ti si ještě s metrikou bubliny pohráli a požadovaná energie jim vyšla podstatně menší. Už si to nepamatuju, ale mám pocit že -45 kg. Tam šlo o to, že rozměry té bubliny jsou jiné pro pozorovatele uvnitř a pro pozorovatele venku, kterému se bude jevit podstatně menší.

Tahle "warpová bublina" má ale jednu zajímavost. Uvnitř ní je časoprostor plochý, čili jinými slovy, ačkoliv by bublina "zrychlovala", uvnitř nebude žádné zrychlení. Takže "inerciální tlumiče" ze Star Treku by nebyly zapotřebí.

Něco o tom se lze dočíst třeba tady
http://arxiv.org/abs/gr-qc/0009013
http://arxiv.org/abs/gr-qc/9905084



HonzaVacek - 4/7/2009 - 02:11

Aha, koukám, že Archimedes mě předběhnul, zatímco jsem to smolil. No nic...


x - 4/7/2009 - 02:52

Dekuji za odpoved - ale potiz je v TOM, ZE I OD RADY DOPOSUD PRO MNE UZNAVANYCH ODBORNIKU JSEM SLYSEL:

Jakykoliv moznost presunu kamkoliv ze Zeme k jine hvezde rychlejc, nez by tam bylo schopne doletet svetlo ze Zeme je NUTNO ZASADANE ODMITNOUT - ODPOROVALO BY TO VSEM NASIM DOSAVADNIM ZNALOSTEM A VEDOMOSTEM - proste rychlost SVETLA JE KONECNA A NEEXISTUJE MOZNOST JAK TAM DOSTAT COKOLIV !!!! TAM RYCHLEJC NEZ RYCHLOSTI SVETLA !!!!

Prominte, ale to co zde rikate TOTO PROSTE PRIMO POPIRA - TATO MOZNOST ALESPON TEORETICKY EXISTUJE !!!!
A to je podstatne - rycholst svetla NENI PROSTE JIZ NENI DOGMATICKY NEPREKROCITELNOU HRANICI PRI PREMISTOVANI VESMIREM A TO JE TO HLAVNI !!!!


Alchymista neprihlasený - 4/7/2009 - 07:48

Prosím ťa, nekrič.
Ešte raz si prečítaj, čo napísal Honza Vacek.

citace:
Pro vytvoření bubliny o rozměrech cca 100 m je třeba enegetického ekvivalentu 10^65 kg, který je zapotřebí k vytvoření požadované metriky časoprostoru. To ale přesahuje odhadovanou hmotnost námi pozorovaného vesmíru. Navíc ta energie musí mít záporné znaménko, což je další problém, protože nikdo neví nejenom kde to vzít, ale co si pod tím vůbec představit.
Chápeš, čo to znamená?

Ak nie, tak len veľmi stručne - je to vesmír v stave zrodu, jeho BigBeng práve začal...


martalien2 - 4/7/2009 - 07:51

Mam dotaz. kdyz je ted Slunce mene aktivni, tak je v nem i plyn mene v pohybu. cim mene pohybu, tim dle teorie relativity mensi hmotnost. Mohlo by se to projevit i tak, ze se vlastne diky relativistickym efektum snizi hmotnost slunce? Tim by se i planety dostali na vzdalenejsi obezne drahy dokud se slunce zase nerozjede. Anebo je tento efekt zanedbatelny? Anebo je tato uvaha zcestna?


Archimedes - 4/7/2009 - 08:03

pro x:
1) Prosím šetřit CapsLockem.
2) Uznávané odborníky uznávejte dál, mají pravdu :) Jenže tenhle trik nijak nenarušuje limit pohybu čehokoliv rychleji než světlo vůči pozorovateli. Představte si, že mezi Vámi a dejme tomu televizorem se prostor natáhl o metr navíc. Ať bude měřit kdokoliv jakkoliv čímkoliv, vzdálenost mezi Vámi a televizorem narostla, ale přitom Vy vůči křeslu ani televizor vůči svému stolku jste se nepohnuli. Jako byste prostě vložil jen tak do luftu metr navíc. A podobně by to vypadalo, kdyby se prostor smrštil. A to je myšlenka toho "bublinového" pohonu. Je to tahání hory k Mohamedovi - místo rozpohybovávání lodě přeskládám kus prostoročasu okolo sebe. To ovšem vyažaduje ty obrovské energie (i ty záporné).

Proč se ale loď uzavřená v takové bublině může pohybovat libovolně rychle? Protože ona _nikam_neletí_. Ve své bublině stojí, čili tam je to bez problémů. A i když se bude prostor za lodí rozpínat nadsvětelnou rychlostí, není problém s teorií relativity. Ono totiž pozorovatel, od kterého se bublina (ale jen a pouze díky "přespočetným" přibývajícím metrům prostoru) vzdaluje, ji totiž vůbec neuvidí a nemůže s ní interagovat, protoče ani světlo tu bublinu nedohoní! Pro takového pozorovatele ta loď uvnitř bubliny prostě neexistuje - a neexistující se objekty se na nějaké rychlostní limity neohlíží :)

Zkuste si představit pružnou blánu, na které žijí placaté bytosti a v jejich 2D světě je rychlost světla 1cm/s. Pak blánu vezmete a prudce natáhnete. V té chvíli se všechny bytosti můžou vůči sobě vzdalovat nadsvětelnou rychlostí - jenže protože jejich pomalé světlo "nestíhá" to prudké natahování jejich vesmíru (blány), z pohledu jedné bytosti ostatní bytosti v tu chvíli neexistují - a objeví se někde dál ve chvíli, kdy vy přestanete za blánu tahat.


Alchymista - 4/7/2009 - 08:26

Edit: toto je na martalienovu otázku

Je to inak formulovaná otázka, či relativistická zmena (prírastok) hmotnosti pohybujúceho sa objektu má vplyv na gravitačné účinky jeho hmoty...
Neviem. Mám dojem, že pokiaľ by to platilo, hmotnosť prestane byť skalár (a to asi dosť dobre nemôže) a pohybujúce sa teleso bude mať v rôznych smeroch rôznu hmotnosť - inú rovnobežne so smerom pohybu, inú kolmo na smer pohybu.


Pokiaľ sa na otázku pozriem z inej strany:
Slnečné škvrny sú vlastne prejavom magnetických polí v slnečnej fotosfére a chromosfére. Tá ale predstavuje len malý zlomok hmoty Slnka a je absolutne ovplyvňovaná procesmi v konvektívnej vrstve. Hlavný pohyb sa odohráva pod "povrchom", a to predovšetkým ako výstupné a klesavé prúdy plazmy v podpovrchovej konvektívnej zóne (vrcholky týchto vzostupných prúdov plazmy sú viditeľné ako granule) a podobné prúdenie v hlbšie položenej oblasti žiarivej rovnováhy. Tu prebieha prenos energie od jadra smerom k povrchu. Pritom jadro zaberá menej než 2% priemeru slnka, ale predstavuje polovicu hmotnosti Slnka.
Hlavným motorom je tepelné prúdenie plazmy prenášajúcej energiu smerom od jadra k povrchu, magnetické pole je potom až dôsledkom, aj keď sa samozrejme navzájom ovplyvňujú.
Z tohoto pohľadu sú potom zmeny magnetickej aktivity slnečného povrchu vo vzťahu k hmotnosti Slnka a k obežným dráham planét bezvýznamné.

[Upraveno 04.7.2009 Alchymista]


alamo - 4/7/2009 - 10:34

práve som si spomenul, ako sa mi raz podarilo "zastaviť" debatu o cestovaní v čase.. (o relativistickej fyzike nemám ani šajn) vyšiel som s tvrdením, že cestovanie v čase, je síce teoreticky možné, ale v praxi mu bráni "energetická bariéra", prose na realizáciu cesty v čase by bolo treba toľko energie, koľko neobsahuje vesmír ako celok. čiže by ho bolo treba celý "spáliť", a ešte kúsok na vyše. v dôsledku čoho by nebolo kam cestovať.
a debata sa zastavila.. možno to bolo skôr tím, že ani ostatný v diskusii nemali o týchto veciach "ani šajnu"
ale ak by som mal pravdu (náhodou), neplatilo by to tak trochu, aj o tom "warpovom pohone"?


Alchymista - 4/7/2009 - 11:02

Pokiaľ vychádzam z toho, že fyzikálne teórie sú nepresné a neúplné, tak výsledok typu "premeníme všetku hmotu vesmíru na energiu a ono to môžno pôjde" znamená asi toľko, že použitá teória prekročila medze svojej platnosti alebo použiteľnosti.
Newtonova teória fungovala viac ako jedenapol storočia k plnej spokojnosti vedcov i okoloidúcich občanov. Až keď sa vedátori začali hlbšie zaoberať elektromagnetickým poľom a skúmať podstatu hmoty a žiarenia, začali im z Newtonovej teórie vychádzať nezmysly. Do poriadku to dala kvantová mechanika a Einsteinova teória relativity. Nie je ale žiadny dôvod sa domnievať, že tieto dve teórie sú konečné a úplné, časom sa určite objaví ďalšia, rozvinutejšia, teória, ktorá spojí kvantovú mechaniku a teóriu relativity ako svoje dobré aproximácie s jasne vymedzenou oblasťou platnosti, tak ako obe v sebe zahrňujú Newtonovu teóriu ako dobrú aproximáciu pre určitú oblasť použitia.


Derelict - 4/7/2009 - 11:16

Kdysi se tvrdilo, ze stroje tezsi vzduchu nemohou letat. Jeden cas se tvrdilo neco podobneho ironicky o cmelacich - ze nemuze letat, protoze proste nema dostatecne velkou plochu kridel. Protoze ovsem nevi nic o matematice, leta vesele dal. Postupem doby se prislo na chybu v metodice vypoctu, cmelak svymi kridly opisuje trochu jiny tvar, kde "nahoni" vztlak.
Debaty na tema warp pohonu, gravitacnich motoru, ramjetu nebo cehokoliv dalsiho jsou spravne. Jednoho dne treba prijdeme na zpusob, jak neco podobneho udelat energeticky mozne. Treba by nam na to dokonce stacila spravne vynalozena energie kterou spali slunce za rok. Nebo energie kterou produkuje Temelin. Nebo mozna jenom energie tuzkove baterie.
Normalni clovek ma v relativisticke fyzice problem s pochopenim proc je rychlost svetla maximalni rychlosti v ctyrrozmernem prostorocase. Proc je rychlost svetla shodna i pro pozorovatele, ktery rychlosti blizkou rychlosti svetla leti ke stejnemu bodu a na stejne primce jako vysilac, jenom ma na rozdil od vysilace opacny vektor. Pochopit tuto "relativne" jednoduchou zalezitost znamena udelat prvni premet v chapani, podvolit se matematice a zapomenout na selsky rozum. Ten relativisticky neni. Ale stale tu jsou dalsi oblasti, ktere jsou o navstevu Bohnic.
Podle meho otazka "Zda se muze objekt pohybovat rychlosti vetsi nez rychlost svetla" vychazi ze spatne formulace technetu. Jak tu Alchymista poznamenal, nejedna se o prekroceni rychlosti svetla. Rychlost svetla zustava stale horni mezi, ktera je dana vlastnostmi ctyrrozmerneho prostorocasu. Ale popsana technologie by znamenala "utrzeni" lokalniho kusu casoprostoru a jeho prenos jinam. Nebo spise jeho pomale "klouzani" po znamem vesmiru. Ani na prechodech mezi touto bublinou a okolnim vesmirem by nedoslo k prekroceni rychlosti svetla.
Dalsi alternativou by byly vyssi rozmery. Kdysi jsem narazil na debaty ohledne moznosti jejich vyuziti pro cestovani (fandove SCI-FI si vzpomenou na cervi diry ve StarGate nebo skoky v BattleStar Galactica), jinou jsou stroje casu pouzivajici exotickou hmotu nebo zapornou energii. Matematickych hricek je spousta, jsou to moznosti, ale omezuje nas dostupnost techto materialu ve znamem vesmiru ;o) Mozna by bylo lepsi rict, omezuje nas otazka, zda takove materialy vubec existuji.


Adolf - 4/7/2009 - 16:21

citace:
Chápeš, čo to znamená?

Ak nie, tak len veľmi stručne - je to vesmír v stave zrodu, jeho BigBeng práve začal...


Jasně nejprůchodnější cesta k nadsvětelné rychlosti v rámci našich znalostí je udělat si malým velkým třeskem soukromý vesmír vedle a lítat v něm.


x - 4/7/2009 - 16:40

Proste kazdy novinar a mnoho diskutujcich v kazde diskuzi a zrejme mnoho z tech odborniku - proste reklo: Nic tam nedorazi drive zadnym moznym zpusobem, nez by tam dorazilo svetlo - z toho je nutne vychazet pri cestovani vesmirem.

A dosazeni byt i jen polovicni rychlsoti svetla by vyzadovalo nesmirne energeticke naklady - vzorce jsou zcela jasne a plati pro kazdeho !!!!

A proto rikam - toto Tvrzeni proste neni jiz tak zcela pravdive - ja jsem pochopil, ze se o prekroceni rychlosti svetla nejedna - ale o moznost byt nekde u nejake hvezdy DRIVE nez by tam dorazilo ze Zeme svetlo a o tom to je !!! - ne o prekroceni rychlosti svetla pri vlastnim letu, ale byt tam presto drive nez tam prileti svetlo ze Zeme !!!

A pokud se neda teorii zcela 100% vyloucit moznost to delat pri mnohem nizsich energetickych narocich - bude opravdu vhodne prehodnotit moznosti cestovani kosmickych civilizaci a vyzkumnych sond vesmirem k jinym obydlenym planetam - proste to tak kategoricky v podstate nevylucovat jako doposud.
Ukazeni se mimozemstanu na verejnosti v TV vysilani napriklad, jako ze se prave se mam po dukladnem nasem skrytem pruzkumu rozhodli ukazat (neruseny jejich vyzkum nas muze byt ten duvod proc tu sou a presto se neukazuji - pdobne jako my zkoumame nerusene divoka zvirata) - bude mit za nasledek zrejme velmi znacne zesmeseni tech kdo byt i jen cestovani civilizaci - jedna k druhe tak doposud odmitali.


alamo - 4/7/2009 - 16:59

takže "oprava"
http://technet.idnes.cz/warpovemu-pohonu-ze-sci-fi-star-trek-teorie-podle-fyziku-nebrani-php-/tec_vesmir.asp?c=A090626_141058_tec_vesmir_vse
netreba spáliť "celý vesmír" stačí spáliť "Jupiter"
"Vytvoření Alcubierrovy bubliny by vyžadovalo minimální energii 1045 Joulů. Jen pro představu – to zhruba odpovídá množství energie, které je obsaženo v celé hmotnosti planety Jupiter. O něčem podobném se nám může jen zdát."
no tak na to že som si dokázal predstaviť "spálenie celého vesmíru, a ešte kúska navyše", musím povedať že spáliť Jupiter, je "malina"


Adolf - 4/7/2009 - 17:05

citace:
Pokiaľ vychádzam z toho, že fyzikálne teórie sú nepresné a neúplné, tak výsledok typu "premeníme všetku hmotu vesmíru na energiu a ono to môžno pôjde" znamená asi toľko, že použitá teória prekročila medze svojej platnosti alebo použiteľnosti.



Pod to povídání mezích použitelnosti současných fyzikálních modelů se mohu podepsat. Já bych ale vypíchl určitý aspekt současné fyzikální teorie, který nejen naznačuje, že za ní je něco daleko zajímavějšího, ale také, že veškeré fyzikální dění je plné nadsvětelných rychlostí a nemusí jít jen o rychlost kosmologickou.

Hlavním proudem interpretace kvantové teorie je dosud dost zvláštní a skoro bych řekl neurotická interpretace Kodaňská. Kodaňský kánon kvantové hermeneutiky nám předepisuje představy a kvantových fenoménech a co je na něm zvláštního, také zakazuje jako nejtěžší hřích představy o procesech skrytých pod kvantovou slupkou a tedy pokusy uvažovat o kvantovém popisu reality jen jako epifenoménu nad jemnějším modelem, který by byl LOKÁLNÍ a DETERMIINOVANÝ JEDNOZNAČNOU KAUZALITOU – tedy zapovídá uvažovat o skrytých parametrech a procesech, které by popisovaly souběžně probíhající změny nesoumístných stavů v rámci jevu popisovaného stejnou vlnovou funkcí jako interakci probíhající s nesmírnou rychlostí, která by musela být značně nadsvětelná. Kodaňci si radši libují v tom, že když je např. pár kvantově vázaných fotonů od sebe 10 km a jednomu fotonu otočíme polarizaci, načež se ze nezměřitelně krátkou dobu (oni tvrdí okamžitě), pootočí druhý foton, tak mezi nimi neproběhla žádná interakce. Z té okamžitosti nesleví ani o píď i přes jisté relativistické svízele s tím spojené. Kvantové fenomény jsou prostě nelokální a basta. Ovšem pokud ne, tak je subkvantová povaha všech procesů značně nadsvětelná.


Alchymista - 4/7/2009 - 21:16

Ku kvantovej teórii so skrytými parametrami:

citace:
http://www.aldebaran.cz/forum/viewtopic.php?t=961
Vojta Hála napísal
citace:
... Ale pravý důvod pro zamítnutí teorií se skrytými parametry je jinde. Dá se teoreticky dokázat, že v každé teorii se skrytými parametry bude triviálně splněna Bellova nerovnost. To je nějaké tvrzení o tom, jaké budou korelace mezi tím, co naměří pozorovatelé A a B v různých bázích. Když páry nejsou entanglované, ale jejich stav je předem jednoznačně určen, dopadne to v experimentu přesně tak. Bellovy nerovnosti jsou splněny. Jenže kvantová mechanika dává jinou předpověď té korelace pro propletené páry. Předpovídá, že bude větší, než co umožňují Bellovy nerovnosti. Když se takových propletených párů připraví hodně a spočítá se ta statistika, ukáže se jasně, že výsledek je blízký QM předpovědi a v každém případě narušuje Bellovu nerovnost. Je to tedy způsob, jak experimentálně dokázat, že Einstein se mýlil, když byl přesvědčen, že náhoda v QM vyjadřuje pouze naši neznalost.

Podrobnosti například v encyklopedickém hesle Bell's theorem.


Bude treba nájsť a uskutočniť taký experiment alebo jav, pri ktorom bude predpoveď kvantovej mechaniky zreteľne nesprávna/nepresná. Zatiaľ sa to nepodarilo...
[Upraveno 04.7.2009 Alchymista]


Adolf - 4/7/2009 - 23:17

citace:
Bude treba nájsť a uskutočniť taký experiment alebo jav, pri ktorom bude predpoveď kvantovej mechaniky zreteľne nesprávna/nepresná. Zatiaľ sa to nepodarilo...
[Upraveno 04.7.2009 Alchymista]


Nejde tu o fenomenolický popis pozorovatelných, který QM poskytuje přinejmenším s přesností, za jakou se s technickým zvládáním procesu měření ještě dlouho nemáme šanci dostat. Nezpochybňuji přesnost popisu QT.

Zpochybňuji Kodaňskou interpretaci. Přesněji řečeno - tuto doktrínu bych označil nejen za intepretaci, nýbrž i za zákaz uvažovat a povaze kvantových jevů jako o epifenoménu. Tyto teorie, k jejichž hlavním prosazovatelům patřila v počátcích profilace KI jako mainstreamu skupina ERP - Einstein, Rosen, Podolsky. KI se vyprofilovala jako určitá doktrína odolná proti skeptikům, což ji myslím po jejím sesazení historici vědy nezapomenou. Vypracovala si tedy nejen postupy jak věci interpretovat, ale i obranu proti možnostem interpretovat jinými způsoby, které tehdy nebyly zvládnuty. Obsahuje tedy i zákazy uvažovat o kvantových procesech lokálně a o skrytých parametrech, kromě změřených. Jde na to přes Bellovy nerovnosti a přes ultimátní von Neumanův důkaz, nemožnosti teorií se skrytými parametry. Tento důkaz byl, pokud vím, někdy v 90. či už 80. letech vyvrácen jako chybný, i když je od von Neumana.

Já teď nenajdou rychle odkazy na jeho vyvrácení. Když ale přišly zprávy, že byl vyvrácen, vlastně mě tak moc nezajímaly. Já mu totiž nikdy nevěřil. Ovšem přístup bez zamítnutí teorií skrytých parametrů umožňuje reintrpretovat QT jako deterministickou teorii deterministického chaosu, která dá stejné výsleky.

Jedním ze zákazů, které KI formulovala, platí i úvaha o kvantových feinoménech jako lokálních. Tedy když se v jednom bodě něco děje, přenese se příčina nějakou rychlostí z bodů jiných. QT je však nelokální. Jakmile bychom totiž chtěli chtěli uvažovat o kvantovém procesu, jako o lokálním, byl by to proces vysoce nadsvětelný. Nešlo by to přinejmenším bez složitého rozšíření teorie relativity. Ovšem jev by musle být procesně definován tak, že by současná QT musela být jeho epifenoménem, jinak by nic nevysvětloval. Kdyby takový proces byl vymyšlen, mohl by být navržen jako explanační model za statistickým modelem fenoménu, jaký poskytuje QT.

Nikdo tu nezpochybňuje relevantnost statistického modelu, který poskytuje QT. Zpochybněno je ale její použití jako explanačního modelu a do ní zabudované tvrzení, že nelze vytvořit jiný explanační model.

I když přijmeme KI s nelokálností, jsme v potížích se současností, která je relativní. Tyhle potíže nejsou však nic moc proti teoretickým potížím, připuštění lokálnosti v teorii. Proto tato interpretace založná na řekněme na ekonomické optimalizaci své obhájitelnosti, do sebe nacpala ochranu proti obtížným zpochybněním.

Ovšem požadavek nelokálnosti je extrémně silný a odporující přirozenému myšlení. Kdyby byla QT teorie reinterpretována jako lokální, byly by její podstatou nadsvětelné - tedy relativisticky těžko interpretovatelné procesy. QT by pak musela být jejím epifenoménem.

Popravdě moc bychom si tím nepomohli. To nadsvětelné by probíhalo někde uvnitř vlny řídící zákonitě podsvětelný proces. Ale měli bychom teoretický aparát, podle něhož bychom mohli uvažovat o fyzikálních procesech v širším kontextu.




Archimedes - 5/7/2009 - 01:18

Představa nelokality svázaných stavů je divná, ale zas tak moc mě neděsí, protože v měřených stavech není přenesena informace a teorie relativity se to tímpádem netýká.
U původní Kodaňské interpretace mě spíš irituje zeď mezi mikrosvětem (zkoumaný systém) a makrosvětem (měření). Ale i ta se zdá se bourá díky teorii dekoherence.

Protože ale kvantovku aktivně nedělám, na filosofování moc nemám nárok (jako prakticky všichni neteoretici )


x - 5/7/2009 - 03:36

Pokud se neda teorii zcela 100% vyloucit moznost to delat pri mnohem nizsich energetickych narocich.

Predevsim by mne zajimalo, jestli opravdu zcela spolehlive tedy 100% vyloucit, ze to lze delat pri mnohem mensich energetickych narocich - pokud toto lze - je pravda co rikate - teorie ma mez sve pouzitelnosti a toto se zda byt velmi dobre za hranici mozne realizovatelnosti.

Pokud to vyloucit nelze, ci je dokonce pravdepodobne ze pujdou opravdu radikalne snizit a to na celkem dostupnou hodnotu pak je opravdu na prehodnoceni moznosti cestovani vesmirem treba jen pro kosmicke sondy nadesel jiz ted cas a to podle toho jak moc bude pravdepodobne ze lze snzit eneregticke naroky.

Od "Je i mozne - neni to tak zcela vylouceno, ze by se pouzil tento zpusob pro jeji let a to z hlediska pozorovatele na Zemi nadsvetelnou rychlosti"
Az na "Zda se, ze se asi zrejme jiz podarilo najit zpusob jakym asi v budoucnu po vyreseni vsech obtizi, lze vysilat komicke sondy k jinym hvezdam a to z hlediska pozorovatele na Zemi nadsvetelnou rychlosti".


Archimedes - 5/7/2009 - 04:31

Tyhle všechny úvahy se potácí na hranici mezi vědou a scifi - jsou to spíš taková cvičení v domýšlení důsledků současných fyzikálních teorií.

Znalosti o cestování pomocí manipulace s prostorem jsou teď zhruba v takové fázi jako aerodynamika v době Newtonově. Máme nejobecnější teoretické principy, podle kterých by se daný jev měl řídit a můžem rozvíjet první fyzikálně podložené úvahy. Některé z nich se časem pro jisté specifické situace ukážou jako v hrubých rysech správné, ale ve většině z nich se najde nějaká zásadní bota daná nedokonalostí dnešních představ. O fyzice letu pomocí manipulace s prostorem víme asi tolik, co mohli v 17.století vědět o stavbě nadzvukových letadel.

Jenže je tu jeden rozdíl: na let nadzvukovou rychlostí postačí třeba i kanón a střelný prach, které bezpochyby existují. Pro cestování v pseudo-nadsvětelné bublině by byla potřeba hmota, o jejíž existenci se dá úspěšně pochybovat...


alamo - 5/7/2009 - 12:43

Archimedes :"..potácí na hranici mezi vědou a scifi - jsou to spíš taková cvičení v domýšlení důsledků.." "..hmota, o jejíž existenci se dá úspěšně pochybovat..."
tak to "vyšrovbujme" na maximum..
http://www.transhumanismus.cz/blog.php?time=061218#4029
"Nelineárnym myslením [transhumanismus.cz] si môžeme predstaviť, ako sa z dnešného „low-tech“ sveta v krátkej dobe môžeme dostať do „hi-tech“ sveta budúcnosti. Dneska chcem skúsiť opačný postup. Ukážeme si, ako by mohla vyzerať singularita. Ak totiž z toho zahliadnete čo i len nejasné obrysy, nejaké „nanoboty“ vám budú pripadať ako úplne normálna vec."
"Šoková úroveň meria hi-tech koncepty, o ktorých dokážete premýšľať bez toho, aby ste boli ohromený, vydesený, slepo nadšený – teda bez toho, aby vás uvedená predstava šokovala. Klasifikácia má význam, pretože pomáha odmerať, na aké informácie sú pripravený vaši poslucháči. Napr. ak preskočíte 2 úrovne, spôsobíte, že ľudia budú vyplašený, ak preskočíte 3 úrovne... budú naozaj vyľakaný."

"DÓ ÚTÓKÚ!!!"

grigar v svojej relácii okná vesmíru.., tvrdil že žijeme v "nenormálnom vesmíre". že stav nášho vesmíru je "nenormálny", podľa výpočtov mala "hmota" vypadať inak, ak si dobre spomínam protón, miesto protónu mala vzniknúť iná častica, čo by bola menšia a ťažšia.
nemala by s tím súvisieť aj tá "zmena kvantového stavu", teda uvedenie do "normálnosti", ktorá má potenciál rozložiť vesmír, na nanočastice?
to by už dalo, štyri variácie, formy hmoty, "normálnu" hmotu a k nej opačnú antihmotu. a našu bežnú "nenormálnu" hmotu, a k nej antihmotu s opačnou polaritou..
ak je teda teoreticky možné, vytvoriť antihmotu k našej súčasnej hmote, malo by byť možné vyrobiť, aj "normálnu hmotu"..

"..hmota, o jejíž existenci se dá úspěšně pochybovat..." to, akože, sú možné, aj ďalšie "variácie"? ktoré by bolo možné, teoreticky vyrobiť?
[Upraveno 05.7.2009 alamo]


neuromancer - 5/7/2009 - 13:06

Az teraz som si vsimol tuto debatu, a dost ma potesila koncne nieco k comu sa viem vyjadrit.

citace:
martalien2:Mam dotaz. kdyz je ted Slunce mene aktivni, tak je v nem i plyn mene v pohybu. cim mene pohybu, tim dle teorie relativity mensi hmotnost. Mohlo by se to projevit i tak, ze se vlastne diky relativistickym efektum snizi hmotnost slunce? Tim by se i planety dostali na vzdalenejsi obezne drahy dokud se slunce zase nerozjede. Anebo je tento efekt zanedbatelny? Anebo je tato uvaha zcestna?


No nieze je ten "efekt" zanedbatelny, ono ten "efekt" neexistuje. Vasa uvaha vychadza z interpretacie vzorca pre reletivisticku hmotnost. Mala rada (naozaj uprimne v dobrom), ja som tu uz par krat napisal, ale fyzike treba rozumiet a nie len verit. Vzorce treba brat ako formalny popis teorie. Takze v prvom rade treba porozumiet tej teorii. Dalsim krokom je, ze formalny popis moze priniest nieco s cim povodna teoria neuvazovala, o tom neskor.

V com je problem pri vasej uvahe, Vy uvazujete, ze ked sa nieco hmotne pohybuje nejakkou rychlostou v1 tak ma hmotnost m1 a ze ked sa bude pohybovat rychlostou v2 a ked plati ze v2>v1 tak aj hmotnost m2>m1. Tato cast uvahy je spravna. Dalej ale uvazujete, ze pri hmotnosti m2 budu gravitacne ucinky vyssie ako pri hmotnosti m1. Toto je tiez spravne, ale nejak ste zanedbali to co tu zmenu rychlosti sposobilo..

Vy totizto uvazujete len o zachovani hmotnosti, ale ta v teorii relativity neplati, musite k nej propocitat aj energiu. A to z toho dovodu, ze energia a hmota su tym istym elementom a transformacnou konstantou medzi tymito dvoma formami je rychlost svetla na druhu (ano slavny vzorec E=mc^2. Ked mate nejaky hmotny objekt s motnostou m1 a rychlostou v1 a chcete zrychlit na rychlost v2 (v2>v1), tak muste tomu telesu predat energiu. Jej cast sa premietne do zmeny rychlosti a cast sa premeni na hmotnost. Cim blizsie sa budete blizit rychlosti svetla tym viac energie, ktoru dodate sa bude menit na hmotnost a nie na rychlost. Az teoreticky pri rychlosti svetla sa bude vsetka dodana energia menit na hmotnost, to je aj dovod preco je rychlost svetla limitnou rychlostou pre hmotne objekty. Lebo ked aj dodate energiu, ktora by mala objekt zrychlit, tak tato enegria sa premeni len na hmotnost a nie na rychlost.

Energia aj hmota maju rovnaky ucinok na deferomaciu priestou. To znamena, ze energia ekvivalentna 1kg hmoty ma rovnaky gravitacny ucinok ako ten 1kg hmoty.

No a teraz spat k tomu vasemu slnku. Ak sa slnko "rozbehne" zacne plyn v slnku pohybovat rychlejsie, tak je to na zakalade toho, ze v jadre doslo pri jadrovej reakcii k premene casti hmotnosti na energiu, ktora rozpohybovala viac tento plyn (a jej mala cast sa premenila aj na vyssiu hmotnost castic tohoto plynu), ale nikde nedoslo k tomu, ze by sa nam hmota len tak objavila, len zmenila svoju formu, takze gravitacny ucinok sa nemeni.

No teda meni lebo cast tej energie bude vyziarena, takze slnko svoju hmotnost stracat bude ale tym sa budu jeho gravitacne ucinky znizovat a nie zvysovat. Takze vyssia slnecna aktivita bude mat za nasledok len rychlejsie stracanie energie a urcite nie zvysenie gravitacnych ucinkov a nejakoho spatneho pritiahnutia planet.


Derelict - 5/7/2009 - 13:28

citace:

"..hmota, o jejíž existenci se dá úspěšně pochybovat..." to, akože, sú možné, aj ďalšie "variácie"? ktoré by bolo možné, teoreticky vyrobiť?



Teoreticky ano. Vetsina strunovych teorii bere hmotu i energii jako urcite stabilni struny/(mem)brany ve vicerozmernem vesmiru. Otazkou je, kolik stabilnich struktur je mozne vytvorit ;o) Ale tato diskuse by se vice nez na kosmo hodila na aldebaran.

Zakladni popis jedne z adeptek na teorii vseho:
http://en.wikipedia.org/wiki/M-theory " target=_blank> http://en.wikipedia.org/wiki/M-theory

Velice hezke video spolecnosti Nova - mohu pripadne zapujcit. Ale melo by snad byt mozne sledovat ho i online.
http://www.pbs.org/wgbh/nova/elegant/ " target=_blank> http://www.pbs.org/wgbh/nova/elegant/

Nakonec, jako zaklady pro pochopeni doporucuji nasledujici odkaz:
http://www.dimensions-math.org/Dim_download2_E.htm " target=_blank> http://www.dimensions-math.org/Dim_download2_E.htm

Kde je krasne zobrazeno jak pochopit vicerozmerne prostory. Na dalsich dilech se intenzivne pracuje.


martinjediny - 5/7/2009 - 14:37

citace:
...- tedy premistit se k jine hvezde drive, nez by tam ze Zeme doletelo svetlo...
A mozna i prehodnotit doposud tak udajne vyloucenou moznost...

Ak uz by som aj zobral minisondu a aj aby som ju zvladol exoticky premiestnitk inej hviezde, co by tam robila? budem cakat stovky a miliony rokov, kym dorazi signal spat?


neuromancer - 5/7/2009 - 14:45

citace:
Jakykoliv moznost presunu kamkoliv ze Zeme k jine hvezde rychlejc, nez by tam bylo schopne doletet svetlo ze Zeme je NUTNO ZASADANE ODMITNOUT


No tiez by so odporucil setrit CapsLockom, to o com sa tu pise vobec nepopiera to co tu tvrdite. A pokusim sa vysvetlit preco. Ryhlost svetla je zatial stale limitujucou a vyzera to tak, ze este nejaku dobu aj zostane. Co si ale musite uvedomit je to co vlastne rychlost je. Rychlost hovori i tom za aku dobu prejdete urcitu vzdialenost. Takze ked sa chcete premiestnit z jedneho miesta vesmiru na ine zalezi od toho aku dobu a po akej drahe sa budete pohybovat. Kedze ani cas ani vzdialenost niesu absolutne veliciny je moznost ich nejakym sposobom ovplyvnit. Problem preco sa Vam javi, ze to o com sa pise je v rozpore s tym, ze nic sa nemoze pohybovat rychlejsie ako je rychlost svetla je, ze povazujete vzdialenost medzi dvoma bodmi vo vesmire za konstantnu (v danom case). No a to nieje pravda.

Dve miesta vo vesmire mozu mat dokonca niekolko vzdialenosti (je to tak aj ked to odporuje "selskemu" rozumu niektorych ludi co navstevuju tento portal ). Priestor medzi nimi moze byt totizto vplyvom velkeho mnozstva hmoty alebo energie deformovany. Takto je mozne pozorovat napriklad nejaku galaxiu 5x (aj viac krat) na "roznych" miestach, vravi sa tomu gravitacna sosovka. Pre zjednodusenie mozme ovazovat o dvoch takychto obrazoch. Ked v takejto galaxii exploduje napr. supernova mozeme pozorovat, ze najprv explodovala v prvom obraze a potom v druhom obraze tejto galaxie. Dovodom je to, ze svetlo ku nam letelo po dvoch roznych drahach, ktore neboli rovnake. Takze to nieje kvoli tomu, ze jedno svetlo bolo rychlejsie ako to druhe. Ale jedno dorazilo ku nam skor ako druhe. Takze ked trvdite, ze nieje mozne aby ste sa premiestnili medzi dvomi bodmi rychlejsie ako medzi nimi prejde svetlo, tak ktore myslite? To co dorazi ako prve alebo to co dorazi ako druhe? Ak by ste sa premiestnili rychlejsie ako to druhe bolo by to rozpore s teoriou relativity? Nebolo, lebo by ste isli po inej drahe kde by ste rychlost svetla neprekrocili. Kto tvrdi, ze sa muste premiestnovat po tej istej drahe ako pritelti to svetlo? Kto tvrdi, ze ta vzdialenost po ktorej dorazilo svetlo je najkratsia ked aj samotne svetlo moze letiet po viacerych drahach.

No takze predpokladam, ze uz teraz dokazete pripustit, ze svetlo z jedneho miesta vo vesmire moze dorazit na ine miesto napriklad po dvoch roznych drahach, pricom jedna moze byt kratsia ako druha.

No ak sa dokazeme po tento bod zhodnut, tak mozme ist na ten warpovy pohon. Ten neurobi to ze zdeformuje vsetok priestor medzi dvomi miestami tak aby sa tato vzdialenost zmensila naraz, ale robi to tak, ze si maly kusok priestoru (alebo casu) pred sebou zdeformuje tak, ze prejde kratsiu vzdialenost a za sebou ten priestor zase upravi. Problem je v tom, ze okolo takejto rakety musite vytvorit urcitu casovo priestorovu bublinu aby ste nedeformovali aj prietor samotnej rakety lebo by ju to mohlo destruktivne zdeformovat a bol by trochu problem v takejto lodi prezit. Takze warp pohon nieje v rozpore so vseobecnou teoriou ralativity lebo lokalne nevznika pohyb vacsou rychlostou ako je rychlost svetla, ale skracuje drahu. Na prvy pohlad je to jednoduche ale hacik je v tej deformacii priestoru a bubline. Na to je zda sa potrena obrovska energia v tom horsom pripade este aj nejaka zaporna energia. Ale vyzera to tak, ze warp pohon je teoreticky mozny, nezodpovedanou otazkou je vsak technologia hlavne jej ekonomicka narocnost.

Ktosi tu spomenul, ze warp pohon je skor taka matematicka hracka vseobecnej teorie relativity, ale tou bol kedysi aj Schwarzschildov polomer a aj samotny Einstein ho povazoval len za matematicku hracku. Dnes uz vieme, ze ked sa da na kopu urcite mnozstvo hmoty, tak gravitacna sila dokaze po case prekonat postupne vsetky sily a stlacit svoju hmotu pod tento polomer a vytvorit ciernu dieru.

Mozno raz prideme na energeticky nenarocny alebo energeticky sebestacny sposob ako uskutocnit takyto pohon.


Adolf - 5/7/2009 - 14:50

citace:

Ale tato diskuse by se vice nez na kosmo hodila na aldebaran.




I tady je zapomenuté vlákno Fyzika.


Adolf - 5/7/2009 - 15:21

citace:
Představa nelokality svázaných stavů je divná, ale zas tak moc mě neděsí, protože v měřených stavech není přenesena informace a teorie relativity se to tímpádem netýká.
U původní Kodaňské interpretace mě spíš irituje zeď mezi mikrosvětem (zkoumaný systém) a makrosvětem (měření). Ale i ta se zdá se bourá díky teorii dekoherence.




No, trochu jsem si tu už povídali, kam lze úvahy o vázaných stavech dotáhnout s relativistickými paradoxy, když se budu dívat na pokus, kdy bude pozorovatel v jiné inerciální soustavě sledovat experiment s průchodem jedné částice z dvojice polarizačních hranolem a v jeho soustavě bude předcházet dopad na fotografickou desku průchod hranolem.

Mně ovšem připadá přístup KI podobný přístupu Machistů (empiriokritikům) k daltonistům. Daltonisté se v té době úspěšně pokoušeli vypracovat částicovou interpretaci termodynamiky. Tedy vytvořit model podprocesů, jichž je termodynamika jen epifenoménem. Pro empiriokritiky to bylo velice nevědecké spekulace. Spor v té době měl velice konfliktní povahu, chudák géniální Bolzman - završitel interpretace termodynamiky jako částicové kinematiky - kvůli tomu spáchal sebevraždu utopením. Dnes tu sice rozumnou teorii subkvantových procesů, jichž by byla QT jen epifenoménem, nemáme. Ale už od počátku kvantovky v ní máme empiriokrickou doložku proti pokusům něco takového vůbec zkoušet.

Ovšem, jisté je, že kdyby se někdo pokusil stát Maxwellem či Boltzmanem kvantovky a udělat její reinterpretaci za základě subprocesů, neobejde se bez nadsvětelné rychlosti. Já se přiznám, že čekám na nového kvantového Bolzmana jako někdo na mesiáše, protože právě od něj čekám pořádnou vývojovou singularitu.


Derelict - 5/7/2009 - 16:32

citace:

... Ovšem, jisté je, že kdyby se někdo pokusil stát Maxwellem či Boltzmanem kvantovky a udělat její reinterpretaci za základě subprocesů, neobejde se bez nadsvětelné rychlosti. Já se přiznám, že čekám na nového kvantového Bolzmana jako někdo na mesiáše, protože právě od něj čekám pořádnou vývojovou singularitu.




Pravdou je, ze selsky rozum v teto oblasti nepomuze a pouzity matematicky aparat je bud znacne obtizny, nebo "nedokocneny". Ridit se ciste matematicky, kdy zname postupy jsou postaveny na KI pomuze jen castecne, protoze veskera interpretace je svazovana timto "axiomem". Otazkou je, zda KI je axiom, nebo pouze interpretace znamych zakonu.
Treba nekdy v budoucnu zjistime, ze struny/(mem)brany mohou prenaset informace ve vyssich rozmerech rychlosti, ktera nam v nami znamem casoprostoru muze pripadat nadsvetelna (pro tuto hypotezu nemam zadny podklad a cela M-teorie je zatim prilis svehlave ditko). Je mozne, ze se objevi jina interpretace, ktera nam ukaze jedine. V hodinovem strojku kteremu nerozumime jsou dalsi kolecka. Treba existuje determinismus, treba tu budou kvantove jevy u kterych neni mozne predpovidat vysledek. Kdo vi. Konec koncu teorie elektromagnetickeho pole pouziva ctyrrozmerny prostor, slucovani s dalsimi silami vyzaduje vyssi a vyssi rozmery a moderni fyzika zatim nedokaze dat jednoznacny vyklad, co to pro nas znamena. Nas model sveta neni uplny ani bezezsporny (Goedel by k tomu mohl rict sve).


x - 5/7/2009 - 22:40

citace:
citace:
...- tedy premistit se k jine hvezde drive, nez by tam ze Zeme doletelo svetlo...
A mozna i prehodnotit doposud tak udajne vyloucenou moznost...

Ak uz by som aj zobral minisondu a aj aby som ju zvladol exoticky premiestnitk inej hviezde, co by tam robila? budem cakat stovky a miliony rokov, kym dorazi signal spat?



Na to odpovim takto - musela by byt schopna pracovat plne autopmaticky a to jeste v neznamem prostredi.
Zjisti nejvhodnejsi planety pro podrobnejsi pruzkum a pak vysle k nim planetarni sondy co by privezla sebou. Ty by sebrali vzorky rostlin a zivocichu,pudy,ovzdusi,vody tamejsich rek (a i v horsim pripade i casti tel - vlasy,chlupy,kousky odrolene kuze, sliny ci i krev - i z tamejsich obyvatel te planety).
Ty sezbirane vzorky by se odeslaly automaticky v navratovych pouzdrech zpet k mezihvezdne sonde. A rovnez by se k ni odvysilaly veskere namerene udaje.
Ta mezihvedna sonda by vzorky premistila do sve navratove casti - a i do jeji pameti by spolehlive ulozila veskere porizene snimky a mereni co poridila ona sama tak i ony planetarni sondy a poslala by ho zpet do Slunecni soustavy k Zemi.
A doufala, ze jako odpoved ze Zeme prileti dali mezihvezdna sonda, ktera ji "rekne" co a jak dal ci by ona pripadne privezla pozemske bakterie pro umele vytvoreni planety pozemskeho typu - pokud by byli s vodou a zivinami jen "sterilni".

Ci jejim ukolem by bylo tam dat na nekterou z tech planet jen pozemske bakterie, pokud by zjitila sama, ze je "sterilni" - jen umele sireni zivota do vesmira pro budouci mozne osidleni takto umele "vytvorene" zivouci planety.


Adolf - 6/7/2009 - 01:04

citace:

Na to odpovim takto - musela by byt schopna pracovat plne autopmaticky a to jeste v neznamem prostredi.



Co?


alamo - 6/7/2009 - 01:23

x : "Na to odpovim takto - musela by byt schopna pracovat plne autopmaticky a to jeste v neznamem prostredi."

e!?.. to?.. ako že? .. oné?.. ale veď..
..
raz, dva, tri, štyri, päť, šesť, sedem, osem, deväť, desať...
nádych, výdych
takže.. odpoveď:
to už akosi vychádza samo zo seba, vieš pre ľudí s niečím ako predstavivosť, to nie je žiadny problém, predstaviť si čosi, čo sa volá "umelá inteligencia, schopná učiť sa"
okrem toho už pri vývoji takého pohonu, budeme potrebovať umelú inteligenciu, alebo aspoň, nejaké skutočne aktívne, prepojenie niekoľkých ľudských mozgov do jedného celku, teda osobnosti pretože, (a teraz, sa asi ako poriadny skeptik, prejavím ja) bežná ľudská myseľ na riešenie tejto úlohy stačiť nebude, a to nemám na mysli jedného "vedátora" ale celý vedecký tým, proste preto že si "abstraktné" pojmy, možno dokážu predstaviť ako jednotlivci, ale proste asi neexistujú možnosti ako by tie myšlienky mohli vyjadriť a zdieľať, bežnou "konverzáciou"..

počuj nepracuješ náhodou, na patentovom úrade? nenavrhoval si náhodou zrušiť ho, lebo je už zbytočný?

ps: prepáč ale spôsobil si tu pár ľuďom, väčší šok, než tebe táto debata.. [Upraveno 06.7.2009 alamo]


Deni - 20/7/2009 - 21:35

akou rychlosťou sa pohybuje vesmirna stanica okolo zeme


ales - 20/7/2009 - 21:44

citace:
akou rychlosťou sa pohybuje vesmirna stanica okolo zeme
Pohybuje se tzv. 1. kosmickou rychlostí pro výšku 350 km nad povrchem Země, tedy rychlostí cca 7700 m/s (7,7 km/s).


Li-sung - 20/7/2009 - 21:46

citace:
akou rychlosťou sa pohybuje vesmirna stanica okolo zeme

Podle Wikipedie má průměrnou rychlost 27743,8 km/h neboli 7,7066 km/s [Upraveno 20.7.2009 Li-sung]


zippi - 24/7/2009 - 10:53

ISS - online parametre najdes tu:
http://spaceflight.nasa.gov/realdata/tracking/index.html
rychlost ked som to pisal bola 7700,24


JuDu - 4/9/2009 - 10:23

Zdravym kazdeho,

Napadla ma otazka, ze preco sa zmeni klopenie rakety (vlastne uhol k vodorovnej rovine) z 90 stp. na 0 stp. postupne pocase vystupu raketoplanu (po urcitej vyske). Nebolo by to primitivnejsie a potom aj bezpecnejsie/lepsie kontrolovatelne, ak by to vyletelo len kolmo nahore pokial motory stacia a potom by jeden motor otocil lod o 90 stupnov, a udelil by nejaku minimalnu rychlost ?

Dakujem za pochopenie ...

Juraj


Tomas Habala - 4/9/2009 - 10:28

citace:

.. ak by to vyletelo len kolmo nahore pokial motory stacia a potom by jeden motor otocil lod o 90 stupnov, a udelil by nejaku minimalnu rychlost ?



Keby raketa vyletela len kolmo hore a vypla motory, chovala by sa ako kamen vyhodeny do vysky. Teda spadla by okamzite naspat na zem. To co telesa udrzuje tam hore, je prave ta vodorovna rychlost.


JuDu - 4/9/2009 - 11:03

Ahoj Tomas,

Ja to chapem, ze musi byt udelena aj dopredna rychlost (paralelne a povrchom) ale preco sa to robi postupne, preco nie je v urcitej vyske sklopena raketa a rychlost udelena tam ?
Ano, 'orbitalne skoky' boli sposob, ako sa napr. americania oboznamovali s vesmirom.

Dakujem

Juraj


avitek - 4/9/2009 - 11:18

"Lomený" způsob navádění na dráhu má velmi vysoké gravitační ztráty. Naopak z hlediska gravitačních ztrát by byl optimální pohyb hned od startu "horizontálně". Ale to by prodlužovalo dobu letu v nízkých výškách, kde je hustá atmosféra a proto by zase byly vysoké ztráty působené aerodynamickým odporem. Proto se startuje svisle vzhůru (u některých malých nosných raket, jako byla např. japonská raketa Lambda pod úhlem asi 80 stupňů k horizontále) a postupně se dráha zakřivuje.

Pozvolna se zakřivující dráha je optimalizací, při které je maximálně využito výkonu rakety k překonání jak gravitačních, tak aerodynamických ztrát. Při jednoimpulsním navádění na nízkou kruhovou dráhu kolem Země, tj. bez dlouhodobého přerušení chodu motorů rakety (jednotlivé stupně se zapalují téměř okamžitě po dohoření předešlého stupně) je závěrečná část dráhy během chodu motorů rakety dokonce taková, že výška po určitou dobu (desítky sekund) dokonce mírně klesá.


David - 4/9/2009 - 16:13

Družice na kruhové LEO neustále padá do středu Země a současně se pohybuje kolmo na tento směr pádu takovou rychlostí, že přesně o tolik o kolik " spadne" pod ní klesne povrch Země, díky jeho zakřivení. Výsledek je ten, že se pohybuje vodorovně s povrchem v konstatní výšce.Z toho plyne že stav " bez tíže" na LEO je ve skutečnosti permanentím, nekončícím pádem, pokud myslíme tíží gravitaci, resp, gravitační zrychlení směrem k barycentru.


jan.tak - 8/9/2009 - 16:15

Nevíte někdo mezi anglickým "meteorite" a "meteorite oxide"? myslím, že je to něco v chemii ale nejsem geolog, je "meteorite oxide" meteorit? Díky


avitek - 8/9/2009 - 16:41

citace:
Nevíte někdo mezi anglickým "meteorite" a "meteorite oxide"? myslím, že je to něco v chemii ale nejsem geolog, je "meteorite oxide" meteorit? Díky


Pokud je mi známo, tak pojem "meteorite oxide" se používá pro zoxidovanou povrchovou vrstvu železných meteoritů.


sulinar - 8/9/2009 - 18:05

Přesně tak, je to korozivní produkt při průchodem atmosféry, vyskytuje se u železných, někdy i kamenoželezných meteoritů.


jan.tak - 8/9/2009 - 18:39

citace:
Přesně tak, je to korozivní produkt při průchodem atmosféry, vyskytuje se u železných, někdy i kamenoželezných meteoritů.


Děkuji, a lze to tedy považovat za meteorit nebo ne? jednu takovou věc mám a řekli mi po analýze, že je to "meteorite oxide"
diky


alex - 8/9/2009 - 20:44

Nepatrí to tu ale je to celkom milé.

http://www.novinky.cz/koktejl/178447-vtipalek-virtualne-vystrelil-berlinskou-televizni-vez-do-kosmu.html


M: - 17/9/2009 - 12:20

Mam zmatok, v akom poradi leti HTV a ISS.
Vo vsetkych mnou objavenych trackeroch leti najprv HTV, potom ISS.

Napr. http://www.n2yo.com/?s=35817|25544
http://www.n2yo.com/?s=35817|25544

Ale niekolko ucastnikov tohto fora sa stale snazi pozorovat HTV az po ISS.(napriek mojmu nesmelemu protestu) Mam nieco ja zle nastavene? nieco som nepochopil? Ako vlastne letia?


ales - 17/9/2009 - 15:03

citace:
Mam zmatok, v akom poradi leti HTV a ISS.
Vo vsetkych mnou objavenych trackeroch leti najprv HTV, potom ISS.

Napr. http://www.n2yo.com/?s=35817|25544
http://www.n2yo.com/?s=35817|25544

Ale niekolko ucastnikov tohto fora sa stale snazi pozorovat HTV az po ISS.(napriek mojmu nesmelemu protestu) Mam nieco ja zle nastavene? nieco som nepochopil? Ako vlastne letia?

Problém je v tom, že HTV manévruje (mění dráhu), ale TLE parametry se obvykle aktualizují jen jednou za den, takže se starými parametry pak "trackery" počítají polohu jinam, než je ve skutečnosti.

Ve skutečnosti byla HTV-1 krátce po startu kousek "před" ISS, ale dost rychle jí "uletěla" a v dalších dnech ji "dohonila" téměř o celý oběh (takže v minulých třech dnech už byla HTV-1 "za" ISS). V posledních hodinách určitě HTV-1 přešla na dráhu bližší k ISS, takže ji "nepředhonila" (jak nepřesně "předpovídají" trackery), ale je blízko za ISS. Ovšem počasí v ČR pozorování nepřálo a HTV-1 jsem neviděl (a asi ani dnes neuvidím).


Lukavský - 17/9/2009 - 17:39

Přesně tak to je. Dřívější dráhy byly dosti odlišné a HTV ISS rychle předbíhal a proto byl při nezměněné dráze z jednoho místa pozorován jednou před a podruhé za ISS (změna o celý oběh). Současná dráha je jen o trochu rychlejší a HTV se jen pomalu přesouvá před ISS. Doporučuji spustit si například v Orbitronu simulaci po jednom oběhu, kde to je pěkně vidět. Další motorické manévry by již neměli být tak velké a dráha se bude měnit jen málo i když korekce pro připojení je nutná. HTV bude stále blízko ISS, ale motorické manévry provedené v době po aktualizaci dat stále znemožní přesné určení polohy.
Z posledních dat z dnešního rána by měl být HTV od nás pozorovatelný jen kousek před ISS (rozdíl cca 2 minuty). Kdyby to počasí dovolilo, měli by se obě tělesa rozlišit podle jasu (ISS je jasnější a měli by být vidět na obloze současně) bez ohledu na to, které je podle předpovědi vepředu. Naděje na pozorování je ale téměř nulová.


martinjediny - 17/9/2009 - 20:48

dik za info.
takze som sa stal obetou vlastnej predstavy o "sofistikovanej drahe" kedy sa HTV bude pomaly a neustale priblizovat k ISS, len s niekolkymi testovacimi a priblizovacimi manevrami
a nahody, ked som otvoril trackery (pocas doterajsieho priebehu tri krat)vzdy v tedy, ked sa nachadzalo HTV kusok pred ISS


JuDu - 31/10/2009 - 14:27

Dobry den,

Momenatalne som na internete prezeral snimky satelitov, ktore su teraz testovane a prekvapilo jeden fakt: niektore vodice su len jednoducho prilepene lepiacou paskou na bok satelitu, bez nejakeho upevnenia. Pokial dobre viem, tak kazdy satelit musi prejst skuskami pevnosti, kde testuju odolnost stroja pre rozne vybrane frekvencie. Nevadi to tam ?
Tie vodice su tam len pre merania v stadiu zostavenia satelitu ? Pred finalizaciou sa to odstranuje ?

Dakuejm pekne

Juraj


ales - 31/10/2009 - 17:35

citace:
Tie vodice su tam len pre merania v stadiu zostavenia satelitu ? Pred finalizaciou sa to odstranuje ?
Je velmi pravděpodobné, že jde skutečně jen o dočasnou kabeláž po dobu testování (letová kabeláž bývá upevněna důkladněji), ale záleží na konkrétním případu. O jaké snímky přesně jde? Prosím o odkazy.


JuDu - 1/11/2009 - 10:24

Dobry,

Konkretna snimka:
http://esamultimedia.esa.int/images/galileo/GIOVE-B-PM-DSC_1837_H.jpg - originalna stranka je http://www.esa.int/esa-mmg/mmg.pl?b=b&type=I&mission=Galileo&start=4 . Ide o test palubnych pristrojov, tak asi je to potrebne len na Zemi.
Ta folia so zlatou farbou ktora je na antenach, sa pred ozavretim sondy pod aerodynamicky kryt sa odstrani ? Maju tu taky system ako na bojovych lietadlach: cervene vysacky sa odstrania pred startom ?

Vdaka

Juraj


Alchymista - 1/11/2009 - 15:13

"Zlatá" fólia je tepelná ochrana a tá zostáva na sonde.
"Červené" sa odstraňuje aj na sondách -je to napríklad štvorcový kryt vľavo hore vedľa antény a dva malé kryty na čiernom bloku pod anténou.
Je pravdepodobné, že sa odstráni aj priehľadná žltá fólia pred anténou.


VYFUN - 2/11/2009 - 23:13

Chtěl bych se zeptat na takovou blbost ohledně gravitace. Co je tou silou co přitahuje tělesa k sobě?


jirka44 - 2/11/2009 - 23:55

citace:
Co je tou silou co přitahuje tělesa k sobě?


http://cs.wikipedia.org/wiki/Gravitace

Ono se to tak nějak vlastně neví....


Vyfun - 3/11/2009 - 10:15

Jj. To jsem taky četl. Já jen jestli mi něco neuniklo. Jak jsem se dostal k teorii strun začalo to být krapínek nepřehledné. Pořád jsem nemohl pochopit co ty tělesa přitahuje. Takže, každý s gravitací žije počítá s ní, ale v podstatě o vztahu mezi tělesy víme prd. :-)


ohara - 3/11/2009 - 10:27

Celkove je pojem sily trosku umely, viz jak to funguje treba u elektromag interakce z pohledu casticove fyziky , kde silu zprostredkovavji virtualni fotony ktere si mezi sebou interagujici hmota vymenuje. (u ostatnich interakci jsou to jine typy intermedialnich castic) V pripade gravitace se na to muzete divat tak ze silu vytvari gravitony jako intermedialni castice. Nebo asi lepsi zpusob, ze priroda realizuje nejmensi moznou akci. Protoze hmota telesa zakrivuje prostor. Muzete si predstavit planetu jako mic na gumove blane, pak sila ktera vas donuti pri pruletu kolem takoveho mice zahnout je jen zdanliva. Ve skutecnosti se pohybujete po nekratsi mozne trajektorii - provadite nejmensi moznou akci (pojem z teoreticke mechaniky), ktera je na krivem prostorocase takto ohla.


DH - 3/11/2009 - 12:07

citace:
Celkove je pojem sily trosku umely...ktera je na krivem prostorocase takto ohla.


Kouzlo je navic prave v tom, ze nejakou formou by mely platit obe varianty, geometricka makroskopicka Eisteinova a casticova mikroskopicka kvantova. Jak to ale je doopravdy (nebo aspon tak aby to vydrzelo test experimentem) nikdo nevi.


Alchymista - 3/11/2009 - 13:09

Takých elementárnych "vecí", o skutočnej povahe ktorých sa serioznejší a pravdovravnejší fyzici odmietajú baviť alebo odpovedajú "nevieme", je viac a gravitácia je len jednou z nich. Ďalšími sú napríklad čas, hmotnosť, elektrický náboj...
Ani sa im príliš nečudujem. V "normálnom" svete poviem - "toto je pevná hmota", príde fyzik so svojimi nástrojmi, lepšie sa prizrie a zistí, že tá moja "pevná hmota" je v skutočnosti takmer dokonalá prázdnota už na úrovni molekúl a atómov.


ohara - 3/11/2009 - 13:11

Ona je otazka nakolik jsme to schopny pouze jazykem nebo nejakou predstavou popsat. Vemte si treba elektron a jeho anticastici pozitron, muzete se na to divat jako na anticastici s opacnym nabojem a identickou hmotnosti, nebo jako na identicky elektron pohybujici se zpatky v case, z matematickeho hlediska je to uplne ekvivalentni. Je dost dobre mozny, ze popis sily pomoci intermedialnich castic nebo pomoci nejmensi akce v nejaky neeuklidovsky vicrozmerny geometrii je matematicky ekvivalentni, jen zatim nikdo neobjevil matematiku v ramci ktere by jsme to poznali, treba to bude strunova teorie.


Tomas Habala - 3/11/2009 - 13:16

citace:
...príde fyzik so svojimi nástrojmi, lepšie sa prizrie a zistí, že tá moja "pevná hmota" je v skutočnosti takmer dokonalá prázdnota už na úrovni molekúl a atómov.


Až na tie kvantové fluktuácie, ktoré naopak zapĺňajú priestor časticami do posledného miestečka tým usilovnejšie, čím sa fyzik lepšie a podrobnejšie prizerá...


Alchymista - 3/11/2009 - 13:50

Ale nie
IMHO virtuálne častice sú vlastne len a iba "matematická barlička", virtuálne sú práve preto, že sa nedajú "uvidieť" alebo "zmerať", existujú len v predstavách fyzikov, aby predpoklady teórie a fakty pozorovania súhlasili.
Obchádzať zákony a vytvárať častice alebo energiu "z ničoho" je možné, ale musí sa vždy vrátiť do ničoty a musí sa to urobiť tak šikovne, aby si to "kontrolný orgán" nestačil všimnúť.


wintermute. - 3/11/2009 - 13:54

citace:
Jj. To jsem taky četl. Já jen jestli mi něco neuniklo. Jak jsem se dostal k teorii strun začalo to být krapínek nepřehledné. Pořád jsem nemohl pochopit co ty tělesa přitahuje. Takže, každý s gravitací žije počítá s ní, ale v podstatě o vztahu mezi tělesy víme prd. :-)


No ak to chcete rozoberat na takejto urovni, tak to uz zdaleka nieje laicka otazka. Na to aby sa dala tato otazka zodpovedat, potrebujeme najskor odpovedat na inu otazku. A to je co dava castici hmotnost (teoreticky to vieme, tusime, uvidime ci sa teoria potvrdi), respektive este vseobecnejsie co je vlastne hmotnost, lebo nemusi ist ani o casticu. Existuju aj castice ktore kludovu hmotnost nemaju. Dalsia vec je co chapete pod pojmom sila. Ak by ste si vzali napr. vztah z Newtonovky a chceli spocitat akou silou posobi napr. slnko alebo zem na foton, tak to nespocitate, lebo foton nema kludovu hmotnost takze by nemal byt gravitacnou silov pritahovany. On ani nieje, on si vlastne stale leti rovno. Len priestor okolo hmotneho telesa je deformovany tak, ze to rovno je trosku krivo, v pripade ciernej diery je to rovno spat na nu. Na druhej strane ked pokrocime dalej a prejdeme k teorii relativity tak vieme, ze foton ma enegriu a energia je len inou formou hmoty, a tak isto ako hmota aj energia ma rovnake dosledky na priestor (a v neposlednom rade aj na cas). To, ze sa telesa pritahuju nazyvame silou preto, ze sme sformulovali teoriu, ktora s touto silou pracuje. Takze pojem sila chapte ako predstavu niecoho co sme pouzili na sformulovanie teorie, ktora dokaze vysvetlit nejaku mnozinu javov. Aj predstava o tejto sile sa meni tak ako sa vyvijaju teorie.

Fyziku musite chapat ako subor matematickych modelov, ktore sa snazia zmapovat spravanie prirody. Tieto modely nemusia (a ani niesu) uplne. To znamena, ze existuju javy, ktore podla nich vysvetlit nejde. Dokonca aj dnes mame teorie, ktore su platne pre velky rozsah dejov ale vzajomne si odporuju, kvantova teoria a teoria relativity (pri strunovej teorii je ich dokonca 11 ak sa to uz nezmenilo a na to aby sste ich dokazali spojit potrebujete pouzit niekolko dimenzii).

To preco sa telesa pritahuju je mozne vysvetlil vzdy len z hladiska konkretnej teorie a v konecnom dosledku to vobec tak byt nemusi. Napr. do kvantovej teorie nam gravitacna sila vobec nepasuje a pokusy spojit ich koncia po matematickej stranke kolapsom. Mozem vam len potvrdit to na co ste prisli, ze o tom v podstate vieme prd. Ked pochopime co robi hmotu hmotou, cas casom a priestor priestorom potom budeme vediet odpovedat aj na vasu otazku ale ked mam pravdu povedat obavam sa toho momentu ak vobec niekedy nastane. Mozno nas blizsie k tomuto poznaniu posunie aj planovany experiment v CERNe.


Tomas Habala - 3/11/2009 - 14:08

Virtualne castice robia meratelne efekty a daju sa pozorovat, ak vcas oddelime casticu a anticasticu. Napriklad vyzarovanie ciernej diery. Jedna virtualna castica sa na horizonte udalosti uvolni - druha pada naspat. Zaujimavejsi je vsak pripad Casimirovej sily, ktora sa da pekne vyuzit v mikrotechnologiach. Tam sa dokonca stava jednym z najhlavnejsich efektov:
http://www.equark.sk/index.php?cl=article&iid=1224&action=itemclick&tname=onhome&pr=click,default


Alchymista - 3/11/2009 - 14:08

wintermute - Dobre napísané.

Tomas Habala - hranica čiernej diery je hodne extrémne prostredie, matematicky tvorba reálnych častíc z virtuálnych vychádza, nie ja ale podopretá pozorovaním
Casimirov efekt - sú k nemu skutočne treba virtuálne častice? Alebo je tlak virtuálnych častíc len jedno z viacerých možných vysvetlení? [Upraveno 03.11.2009 Alchymista]


wintermute. - 3/11/2009 - 14:15

citace:
Ale nie
IMHO virtuálne častice sú vlastne len a iba "matematická barlička", virtuálne sú práve preto, že sa nedajú "uvidieť" alebo "zmerať", existujú len v predstavách fyzikov, aby predpoklady teórie a fakty pozorovania súhlasili.
Obchádzať zákony a vytvárať častice alebo energiu "z ničoho" je možné, ale musí sa vždy vrátiť do ničoty a musí sa to urobiť tak šikovne, aby si to "kontrolný orgán" nestačil všimnúť.


Kazda teoria vo fyzike je len "matematicka barlicka". Co sa tyka vytvarania castice alebo energie z nicoho. Ono tomu nieje tak uplne presne, a ani sa to do nicoty vratit nemusi a ani vzdy nevracia. Ako dalo by sa pisat o kvantovych javoch na horizonte udalosti ciernej diery, ale skuste si to ist k tej ciernej diere zmerat ci to je pravda .

Ale existuje aj jednoduchsi experiment vdacime za neho dvom panom. Jeden sa vola Casimir a druhy Lifsic. Tito dvaja pani urobili jednu predpoved, ohladom kvantovych fluktuacii. Zjednodusene, ze ak existuju kvantove kluktuacie, tak ked obmedzime priestor natolko aby nemohli vznikat castice roznych vlnovych dlzok ale len dlzok ktore su celocicelnym nasobkom vzdialenosti v tomto priestore (napr.vymedzeny pomocou dvoch nenabitych kovovych platni), tak potom musi vzniknut v tomto obmedzenom priestore nizsi tlak oproti zvysku. Prvy krat bol tento jav experimentalne potvrdeny v roku 1996.


wintermute. - 3/11/2009 - 14:30

citace:
wintermute - Dobre napísané.



Dakujem, aj ked pisat o naozaj fundamentalnych zakladoch tohoto sveta je priam nemozne lebo naozaj o tom nic nevieme.

citace:

Tomas Habala - hranica čiernej diery je hodne extrémne prostredie, matematicky tvorba reálnych častíc z virtuálnych vychádza, nie ja ale podopretá pozorovaním



Pravdepodobne ani podopreta pozorovanim nebude (ak ano tak Hawking dostane nobelovku), lebo ziarenie ktore vznika pri tomto jave je podstane slabsie ako reliktove ziarenie, takze ho asi nikdy nebudeme moct pozorovat.

citace:

Casimirov efekt - sú k nemu skutočne treba virtuálne častice? Alebo je tlak virtuálnych častíc len jedno z viacerých možných vysvetlení? [Upraveno 03.11.2009 Alchymista]


No ide o to, ako sa na to pozriete. Vznikla nejake teoria a na zaklade tej teorie niekto (Casimir) povedal, ze ak to tak je tak potom musi existovat nieco co sme doposial nepozorovali. A ono sa to experimentalne potvrdilo. Minimalne to tu teoriu podporuje. Ale nieje vylucene (skor by som povedal ze to je takmer iste), ze sa najde nova teoria, ktora bude v sulade s pozorovanym javom ale ho inak vysvetli.

Osobne napr. strunovu teoriu (teorie) moc nemam rad. Matematicky sa totizto znacne komplikuju a nevyzeraju tak "pekne". Vacsina teorii ci uz teoria relativity, kvantova reoria ... je popisanych "peknymi" rovnicami. Myslim ze Einstein (ale niesom si isty ci on) povedal nieco take, ze ked pracujete na teorii a zacnu vam vychadzat "pekne" rovnice, tak ste na spravnej ceste. Ale to uz ideme do subjektivnych pocitov. Osobne sa ma paci trochu iny smer vyvoja teorii ale to su uz na tuto temu (laicke otazky) velmi zlozite uvahy o prietore a case.


MIZ - 3/11/2009 - 14:40

citace:
Osobne napr. strunovu teoriu (teorie) moc nemam rad. Matematicky sa totizto znacne komplikuju a nevyzeraju tak "pekne". Vacsina teorii ci uz teoria relativity, kvantova reoria ... je popisanych "peknymi" rovnicami. Myslim ze Einstein (ale niesom si isty ci on) povedal nieco take, ze ked pracujete na teorii a zacnu vam vychadzat "pekne" rovnice, tak ste na spravnej ceste.

Ano, to je známá poučka. Ale právě rovnice strunových teorií údajně začnou vypadat mnohem lépe, pokud se nasadí do světa o 10 či 11 rozměrech. Je to taková dominantní příčina úvah o tom, že tolik rozměrů náš vesmír má celkem.


Alchymista - 3/11/2009 - 15:05

Casimirov efekt - z toho, čo som o ňom čítal, funguje len pri veľmi malých vzdialenostiach platní, rádu jednotiek nm (desiatky priemerov atomu) a je silne závislý na opracovaní a kvalite povrchov platní. Nepostačujú na jeho vysvetlenie napríklad Van der Waalsove sily?

Bolo by tu samozrejme pár problémov - Casimirov efekt je pozorovaný na vodivých metalických doskách, Van der Waalsove sily sú skôr záležitosť nevodičov a polárnych molekúl...
Lenže povrchová vrstva kovu v merítku nanometrov sa dá označiť za "vodič" len s istými výhradami, rovnako ako povrchová vrstva nevodičov v rovnakom merítku za "nevodič".


wintermute. - 3/11/2009 - 15:09

citace:

Ano, to je známá poučka. Ale právě rovnice strunových teorií údajně začnou vypadat mnohem lépe, pokud se nasadí do světa o 10 či 11 rozměrech. Je to taková dominantní příčina úvah o tom, že tolik rozměrů náš vesmír má celkem.


No ono to vtedy tie teorie ide spojit dokopy, nepovadal by som ze to potom vypada krajsie. A predstava 11tich dimenzii sa mi tiez moc nepaci. Ale ako som uz povedal, toto je uz o pocitoch. Tym nijak strunove teorie nespochybnujem respektive spomynanu 11 rozmernu M-teoriu. Avsak neriesia problem hmotnosti ako takej, respektive je tam viac neznamych ako v terajsej kvantovke a teorri relativity dokopy.

Ono ked uz me pri 11 tich dimenziach, tak tusim pri 12tich (uz si to nepamatam presne) dokazete spojit aj vseobecnu teoriu relativiity s maxvelovycmi rovnicami, prespektive vam vydu maxvelove rovnice ako specialny pripad 12 rozmernej vseobecnej teorie relativity. Len osobne si nemyslim, ze toto je ta spravna cesta.

Podla mna sa treba uberat inym smerom, nie pridavanim dimenzii. Problem je, ze potom sa zacnu tie dimenzie rozpadat, teda s rozpadom jednej (casu) ku ktorej rozpadu zatial vzdy doslo to zo sebou automaticky strhne aj tie tri dalsie (teda samotny priestor). Pri hladani novych teorii sa dost zameriavame na vysvetlenie podstaty hmoty (co je logicke, nejedna sa o tak abstraktnu vec, a omnoho lahsie sa s nou experimentuje), ale mozno cesta vedie cez vysvetlenie podstaty casu. Kedze na existenciu hmoty je potrebny priestor, ale bez casu samotny priestor existovat nedokaze (teda aspon zatial sme neprisli na to ako by dokazal). Ale to uz sme naozaj strasne daleko od Laickych otazok.


wintermute. - 3/11/2009 - 15:21

citace:
Casimirov efekt - z toho, čo som o ňom čítal, funguje len pri veľmi malých vzdialenostiach platní, rádu jednotiek nm (desiatky priemerov atomu) a je silne závislý na opracovaní a kvalite povrchov platní. Nepostačujú na jeho vysvetlenie napríklad Van der Waalsove sily?

Bolo by tu samozrejme pár problémov - Casimirov efekt je pozorovaný na vodivých metalických doskách, Van der Waalsove sily sú skôr záležitosť nevodičov a polárnych molekúl...
Lenže povrchová vrstva kovu v merítku nanometrov sa dá označiť za "vodič" len s istými výhradami, rovnako ako povrchová vrstva nevodičov v rovnakom merítku za "nevodič".


Teraz neviem ci rozumiem. Ide o to ci efekt sposobeny Casimirovym javom, pri pozorovani fluktuacii vakua nieje sposobeny niektorou s Van der Waalsovych sil?

No nemyslim, mozeme tu rozobrat vsetky tri, ale s takmer istotou mozem povedat ze to fungovat nebude.


Alchymista - 3/11/2009 - 15:34

citace:
No nemyslim, mozeme tu rozobrat vsetky tri, ale s takmer istotou mozem povedat ze to fungovat nebude.
Evidentne si v tejto oblasti kovanejší ako ja takže to beriem tak, že to už niekto van der waalsovými sila vysvetliť skúsil - a nefungovalo to.

Skúsim z inej strany
Zaoberal sa niekto pozorovaním casimirovho javu
- medzi nevodivými doskami?
- medzi doskami z nevodiča pokrytými extrémne tenkou vrstvou vodivého materiálu?
- nmedzi kovovými doskami pokrytými extrémne tenkou nevodivou vrstvou?
(myslím tým vrstvami tenšími ako je šírka medzery medzi platňami)


Tomas Habala - 3/11/2009 - 15:39

Najpresvedcivejsim argumentom pre Casimirovu silu je to, ze bola najprv vypocitana, teda predpovedana a az na zaklade tejto predpovede experimentalne potvrdena. Hladat teraz ine riesenie, akym bola Casimirova sila predpovedana by uz malo mensiu doveryhodnost. Muselo by na dosial neodmeranych desatinnych miestach robit odlisnu predpoved ako metoda virtualnych castic a potom to potvrdit experimentom.


wintermute. - 3/11/2009 - 16:54

citace:
Evidentne si v tejto oblasti kovanejší ako ja takže to beriem tak, že to už niekto van der waalsovými sila vysvetliť skúsil - a nefungovalo to.



No tiez niesom odbornik prave na pokus ktorym v 1996 experimentalne potvrdili. Viem ze bud koncom minuleho alebo tohoto roku bola potvrdena aj odpudiva sila ale to sa nerobilo s vakuom ale nejakou kvapalinou.

Ale pokial mi je zname Casimirova sila sa pohybuje radovo podstatne nizsie ako Van der Waalsove sily.

citace:

Skúsim z inej strany
Zaoberal sa niekto pozorovaním casimirovho javu
- medzi nevodivými doskami?
- medzi doskami z nevodiča pokrytými extrémne tenkou vrstvou vodivého materiálu?
- nmedzi kovovými doskami pokrytými extrémne tenkou nevodivou vrstvou?
(myslím tým vrstvami tenšími ako je šírka medzery medzi platňami)



Co sa tyka nevodivych dosiek tak tie vodive sa pouzivaju prave na odstranenie Van der Waalsovych sil. Ale pri dokazovani odpudivej sily sa tusim nepouzili kovove platne. Musel by som vyhladat popis experimentov, slo o dost narocne experimenty. Poznam len teoriu okolo vzniku tejto sily.

Silne pochybyjem, ze by bol uznany experiment kde by nevylucili vplyv inych znamych sil.


x - 7/11/2009 - 21:27

Zajimala by mne jedna otazka - asi odpoved trochu tusim - ale nevim zdali je spravna.
Lod pro cesty slunecni soustavou - pro kontrolu a doplneni zasob, vody a i "paliva" by ji bylo nutno natolik pribrzdit, aby zakotvila kolem obezne drahy - to beru jako vychozi predpoklad - kvuli moznostem dopravy zasob a provedeni i narocnejsich oprav.
Jaka - hlavne jak vysoka draha by to mela byt, aby musel byt brzdena co nejmene a presto byla na obezne draze Zeme - ne jen tedy prulet kolem Zeme - aby tam byla proste delsi dobu - treba i nekolik let z duvodu treba narocnejsi udrzby ci i montazi novych casti te lodi.
Nechtel bych jsem jit vyse nez na geostacionarni drahu 36 000km tedy pokud nebude nejaky vyznamejsi duvod.
Jaka draha je pro to nejlepsi - ciste dotaz ze zajimavosti - proste mene to zajima - jake jsou nejmensi ztraty rychlosti a tim i paliva (spise energie potrebne pro pohon) pro "zaparkovani" kosmicke lodi na obezne draze Zeme ???


martinjediny - 7/11/2009 - 21:43

Unikova rychlost je zavisla na vzdialenosti od taziska telesa, ktoremu chces uniknut. cim vyssie zaparkujes, tym menej sa narobis pri odlete.

Dalsia otazka je ala gravitacbny prak vid wiki. http://sk.wikipedia.org/wiki/Gravita%C4%8Dn%C3%BD_man%C3%A9ver
aby ta Zem pri odlete spravne zrachlila, resp. spomalila.

Nedavno mi M.K. odpovedal odkazom http://buzzaldrin.com/cycler

tam je zrejme len maly rozdiel medzi tym aby v pripade potreby stanica tancovala okolo Zeme i niekolko obehov a medzi odletom k Marsu. vid. nacrt. na stranke. (vlozit nejde) [Editoval 07.11.2009 martinjediny] [Editoval 07.11.2009 martinjediny]


ales - 8/11/2009 - 00:00

citace:
Lod pro cesty slunecni soustavou - pro kontrolu a doplneni zasob, vody a i "paliva" by ji bylo nutno natolik pribrzdit, aby zakotvila kolem obezne drahy - to beru jako vychozi predpoklad - kvuli moznostem dopravy zasob a provedeni i narocnejsich oprav.
Jaka - hlavne jak vysoka draha by to mela byt, aby musel byt brzdena co nejmene a presto byla na obezne draze Zeme - ne jen tedy prulet kolem Zeme - aby tam byla proste delsi dobu - treba i nekolik let z duvodu treba narocnejsi udrzby ci i montazi novych casti te lodi.
Nechtel bych jsem jit vyse nez na geostacionarni drahu 36 000km tedy pokud nebude nejaky vyznamejsi duvod.
Jaka draha je pro to nejlepsi - ciste dotaz ze zajimavosti - proste mene to zajima - jake jsou nejmensi ztraty rychlosti a tim i paliva (spise energie potrebne pro pohon) pro "zaparkovani" kosmicke lodi na obezne draze Zeme ???
Domnívám se, že čistě teoreticky by asi bylo nejvýhodnější "parkovat" u libračních bodů soustavy Země - Slunce (L1 nebo L2), protože z nich je nejsnazší odletět k libračním bodům u jiných planet. Potřebné delta-v je sice snad jen někde v řádu desítek m/s, ale ty body jsou dost daleko od Země (cca 1,5 mil. km, je docela těžké se k nim ze Země dostat, a také přeletové doby mezi planetami vychází relativně dlouhé (roky). Viz. např. http://en.wikipedia.org/wiki/Interplanetary_Transport_Network .

Z hlediska minimálního delta-v na příletu i odletu je výhodné využít vysoce eliptickou dráhu s velmi nízkým perigeem (např. 200 km nad Zemí) a velmi vysokým apogeem (např. 200000 - 400000 km). Potřebné delta-V pro manévry (v perigeu [platí pravidlo "manévruj co nejníž"]) jsou někde v řádu stovek m/s a přeletové doby mohou být v řádu stovek dní. Samozřejmě, že i na tuto výstřednou dráhu je docela těžké se dostat ze Země (potřebné delta-V pro přechod z LEO na výše uvedenou dráhu je cca 3000 m/s).

Z hlediska delta-v pro přelety mezi planetami kupodivu nejsou nijak výhodné vysoké kruhové oběžné dráhy kolem Země (např. geostacionární dráha), protože potřebná delta-v jak na příletu, tak i na odletu už vychází v řádu km/s (tisícovek m/s). Navíc i z LEO je potřeba delta-v přes 3000 m/s abychom se na vyskou kruhovou dráhu dostali (kvůli obsluze té "hypotetické lodi".

Nakonec může být celkově nejvýhodnější používat jako "obslužnou dráhu" klsickou LEO ve výši cca 300 - 400 km, protože je relativně dobře dostupná ze Země (pro přechod z nižší LEO stačí delta-v v řádu desítek m/s) a na příletu a odletu je třeba delta-v nejméně cca 4 km/s (4000 m/s). Na příletu je navíc teoreticky možno využít atmosféru Země a "zpomalit" o ni (aerocapture nebo alespoň aerobraking). Je třeba se uvědomit, že pro rozumné "létaní mezi planetami" stejně musí mít planetolet "zásobu rychlosti" alespoň několik km/s (čím větší, tím lepší). Proto pžedpokládám, že taková loď by neměla mít vězší problémy s přechodem na LEO a s odletem z LEO.

Moje odpověď bohužel není příliš jednoznačná, ale snad alespoň trochu ukazuje některé souvislosti naznačeného problému.


Ervé - 9/11/2009 - 10:25

Nejlepší pro pilotované lety je nízká dráha do 500 km s nízkým sklonem k rovníku, vysoký sklon zhoršuje nosnost raket na orbitu, při vysoké dráze silnější radiace, při apogeu nad 10 000 km prolétáte radiačními pásy, takže loď pak může být neobyvatelná nebo se zničenou elektronikou. Librační body jsou vhodné pro náklad - s dlouhou dobou letu.


wintermute. - 9/11/2009 - 20:42

citace:
Nejlepší pro pilotované lety je nízká dráha do 500 km s nízkým sklonem k rovníku, vysoký sklon zhoršuje nosnost raket na orbitu, při vysoké dráze silnější radiace, při apogeu nad 10 000 km prolétáte radiačními pásy, takže loď pak může být neobyvatelná nebo se zničenou elektronikou. Librační body jsou vhodné pro náklad - s dlouhou dobou letu.


Zase mystifikacia VARB? To akoze GPS druzice, druzice Glonas a Galileo ale aj druzice na GEO lietaju so znicenou elektornikou? Ano je tam vyssia radiacia ale vo vacsine pripadov elektronova. Tazsie ionty su len na spodnej hranici a tato zona nieje moc hruba. Viac by som sa bal erupcii na slnku ako radiacie vo VARB.


Ervé - 10/11/2009 - 08:34

Jenomže navigační a komunikační družice proletí nejnenezpečnějším VARB jen jednou, od začátku jsou stavěny odolné radiaci. U pilotované lodi na eliptické dráze, která by prolétala VARB x-krát (v průběhu montáže, plnění nádrží, nástupu posádky a kontroly před odletem) by to zkomplikovalo konstrukci a ohrozilo zásoby potravin. Dá se to vyřešit tak, že blok s posádkou, řídící elektronikou a zásobami se připojí až nakonec, těsně před odletem. Zbytku lodě by to tak moc vadit nemuselo.


Wintermute. - 10/11/2009 - 21:29

citace:
Jenomže navigační a komunikační družice proletí nejnenezpečnějším VARB jen jednou, od začátku jsou stavěny odolné radiaci. U pilotované lodi na eliptické dráze, která by prolétala VARB x-krát (v průběhu montáže, plnění nádrží, nástupu posádky a kontroly před odletem) by to zkomplikovalo konstrukci a ohrozilo zásoby potravin. Dá se to vyřešit tak, že blok s posádkou, řídící elektronikou a zásobami se připojí až nakonec, těsně před odletem. Zbytku lodě by to tak moc vadit nemuselo.


No rok by som tam kruzit nechcel, ale tyzden v pohode.


VYFUN - 11/11/2009 - 20:54

SKLON DRÁHY: Nula stupňů je na sever a devadesát je rovník?


ales - 11/11/2009 - 21:46

Přesně naopak. Sklon se udává k rovníku. Takže dráha se sklonem 0° je nad rovníkem (ve směru rotace Země) a dráha se sklonem 90° vede přes pól.
Viz např. http://cs.wikipedia.org/wiki/Sklon_dr%C3%A1hy


VYFUN - 11/11/2009 - 22:07

Dík. Kurňa, tak to jsem to měl předtím dobře. :-D


M: - 12/11/2009 - 09:18

Slnecna pec tu bola nedavno spominana v jednom vlakne.

http://sk.wikipedia.org/wiki/Slne%C4%8Dn%C3%A1_pec_v_Odeillo

Otazka:
Je mozne pri vyssej koncentracii slnecneho ziarenia dosiahnut v ohnisku pece teplotu
70 000°K, ak teplota povrchu Slnka je cca 6 000°K?


Ervé - 12/11/2009 - 09:42

Teoreticky je to možné, přidáváte další a další kW slunečního výkonu do jednoho místa působení, pro 70 000°C už by ale plocha panelů byla tak velká, že to je na Zemi obtížně realizovatelné, problém by byl i s ionizací vzduchu v okolí pece. Na Měsíci nebo na orbitě by to mělo jít snáz, ale dráž.


M: - 12/11/2009 - 10:10

citace:
Teoreticky je to možné, přidáváte další a další kW slunečního výkonu do jednoho místa působení, pro 70 000°C už by ale plocha panelů byla tak velká, že to je na Zemi obtížně realizovatelné, problém by byl i s ionizací vzduchu v okolí pece. Na Měsíci nebo na orbitě by to mělo jít snáz, ale dráž.


o.k. odhliadnime od "tazkosti" technickej realizacie sustredit zo zrkadla 1500km x1500km luce do ohniska 1x1mm.

odpovedal som podobne, ale bolo mi namietnute, ze potom by chladnejsie teleso ohrievalo teplejsie teleso.

bude teda II. zakon termodynamicky poruseny? Ja tvrdim ze nie, ale neviem to odborne vysvetlit.


Ervé - 12/11/2009 - 11:19

Posíláte fotony, nepřitlačujete 6000°C Slunce k tělesu. Foton nese energii (bez vlastní teploty), soustředím hodně fotonů s dostatkem energie. Pokud je objekt ve vakuu (neztrácí rychle teplo ani se neionizuje okolí), můžu ho ohřát na vysokou teplotu. Jsou dvě možnosti - 1. objekt se roztaví a odpaří 2. objekt se zahřeje na takovou teplotu, že vlastním zářením (vlákno v žárovce) dokáže vyzářit stejné množství energie (stejnou energii fotonů), kolik dostává - rovnovážný stav.


Alchymista - 12/11/2009 - 11:50

myslím, že druhý zákon termodynamiky by porušený nebol, už preto, že sa zachytáva energia z väčšej plochy a sústreďuje na plochu menšiu, čo vôbec nie je to isté ako ohrievanie teplejšieho telesa chladnejším (ostatne aj to sa dá - na tomto princípe pracujú všetky tepelné pumpy ako napríklad chladničky, klimatizácie... Nie je problém strčiť výparník chladničky do vody 3°C teplej v Dunaji a radiátor v byte bude mať kľudne 35-40°C - pri spotrebe do 100W budem čerpať z vody Dunaju teplo zodpovedajúce niekoľkým kilowatom)

Celkom ľahko možno spočítať energiu v slnečnej peci - na meter štvorcový na úrovni orbity Zeme dopadá každú sekundu približne 1,3kW energie - zo zrkadla 250x250 metrov postaveného kolmo k slnečným lúčom sa dá mimo atmosféru bez väčších problémov získať výkon viac ako 80MW a nie je problém ho sústrediť na plochu 1x1 meter - dosiahne sa tak 80MW na meter štvorcový. Pri zmenšení plochy ohniska primerane viac - so zbernou plochou 250x250 metrov a dvojzrkadlovým systémom by sa dalo žiarenie zaostriť prinajmenšom rovnako ako v Odeillo a dosiahnuť viac ako jeden gigawatt na meter štvorcový.
Známa slnečná pec v Odeillo dosahuje v optimálnom prípade 16-17MW na meter štvorcový pri ploche ohniska len 0,0625m2 (25x25cm) - sústreďuje niečo viac ako jeden megawatt a dosahuje teploty okolo 4000°C.


Tomas Habala - 12/11/2009 - 12:02

Otázka je, akým mechanizmom sa energia el.mag poľa fotónu premieňa na kinetickú energiu atómov. Myslím, že bez poznania tohto mechanizmu môžeme len strieľať od boku, či fotónami s určitou konštantnou energiou t.j. určitou teplotou môžeme zohriať teleso na vyššiu teplotu.


MIZ - 12/11/2009 - 12:10

Mikrovlnka?


Vyfun - 12/11/2009 - 12:38

Problém toho všeho je, že to bezvadně funguje jen při bezoblačné obloze. Dá se nějak podobně zužitkovat rozptýlené světlo (podmračné)?


M: - 12/11/2009 - 13:20

citace:
Problém toho všeho je, že to bezvadně funguje jen při bezoblačné obloze. Dá se nějak podobně zužitkovat rozptýlené světlo (podmračné)?

ee. Slnko ma u nas uz takmer rovnobezne luce, s ktorymi ide podtatne jednoduchsie narabat a umoznuje ich efektivne zaostrit.

takze hoci vykon tychto lucov je uz len 1,4kW, tak su rovnobezne a da sa s nimi pracovat dalej.
Ak zoberiem difuzne svetlo mrakov, tak kvazi rovnobezny bude len velmi maly podiel a teda su nezaostritelne.

Ale spat k otazke.
Foton nenesie teplo, ale energiu.
Stefanův-Boltzmannův zákon tiez nerozlisuje ci energia dopada na studene, alebo teple teleso. proste mu len foton svoju energiu odovzda a teleso ohreje.

Takze ak spravne chapem namietku TH, tak je otazka, ci foton vyziareny chladnejsim povrchom moze svoju energiu odovzdat teplejsiemu povrchu a tym ho ohriat?


Tomas Habala - 12/11/2009 - 13:28

Ano je to tak. Urcitej teplote telesa zodpoveda urcita vlnova dlzka vyzarovanej el. mag. vlny. Otazka je, ci tato vlna moze zohrievat ine teleso v podstate na lubovolnu teplotu.


Ervé - 12/11/2009 - 14:50

Samozřejmě že může, foton nárazem zanikne (nebo změní hybnost odrazem) a energii předá tělesu. Je jedno jakou má těleso teplotu.


wintermute. - 12/11/2009 - 17:06

citace:
odpovedal som podobne, ale bolo mi namietnute, ze potom by chladnejsie teleso ohrievalo teplejsie teleso.

bude teda II. zakon termodynamicky poruseny? Ja tvrdim ze nie, ale neviem to odborne vysvetlit.


No neviem preco sa toto tema vola laicke otazky, ono to tu padaju celkom zaujimave otazky, miestami dost tazke. Naposledy co pritahuje telesa k sebe.

No spat k teme. Druhy termodynamicky zakon. Jeden z fundamentalnych zakonov fyziky. Mozno niekoho prekvapi ale tento zakon nefunguje na 100%. Totizto sa nejedna o kvalitativny zakon ako je tomu pri prvom termodynamickom zakone. Jedna sa o pravdepodobnostny zakon. A funguje s pravdepodobnostou bliziacou sa k 100%. Moze sa totizto stat, ze s urcitou malou pravdepodobnostou dojde prenosu maleho mnozstva tepla s chaldnejsieho telesa na teplejsie (dokonca to umoznuje samotny 2. termodynamicky zakon). Jedna sa o drobne fluktuacie.

Problem skor vidim s jeho formulaciou (existuje ich niekolko). Treba si uvedomit kedy tie zakony vznikli a co vlastne opisuju. Dost casto sa stretavame s formulaciou:

Teplo nemoze samovolne prechazat z telesa studensieho na teleso teplejsie.

To sformuloval nejaky Rudo v roku 1850, konkretne Rudolf Julius Emanuel Clausius. Lenze prednim zil aj nejaky Nicolas Leonard Sadi Carnot ktory pomocou diferencialneho poctu (konkretne kruhoveho integralu totalneho diferencialu) sformuloval matematicky zapis pre druhy termodynamicky zakon. Pomocou totalneho diferencialu sa zaviedla neskor stavova velicina zvana Entropia. Velmi zjednodusene popisovana ako neusporiadanost. A pomocon nej sa da sformulovat druhy termdynamicky zakon o nieco lepsie.

No lenze spat k problemu slnecnej pece. Chyba je v tom, ze tu vobec nejde o druhy termodynamicky zakon. Totizto druhy termodynamicky zakon hovori o samovolnom prechode tepla. Teraz kde sme to samovolne zrusili? Pravdepodobne sa tu na fore nachadzaju aj radioamateri. tak to skuim vysvetlit cez tuto vednu disciplinu (lebo rozoberat teoriu elektormagnetickeho pola vseobencne by bola nuda a zbytocne zlozite) Pri vysielani totizto do prenosovej trasy mozu vstupovat dva typy prvkov. Aktivny (ktory signal zosilnuje) a pasivny (ktory ho utlmuje). Ovsem kazdy dobry radioamater vie, ze taka antena je prvok aktivny (dokaze produkovat zisk) ak to aj pri vysielani (urcite poznaju aj ludia co prevadzkuju wifi na 30dB anteny , telekomunikacny urad ma z toho radost potom ale psssst tu je vsetko v poriadku). No a sme pri mieste kde nastala chyba, totizo pri slnecnej peci pouzivate "antenu". Nejedna sa uplne o samovolny prenos. Dolezite je, ze neporusujete prvy termodynamicky zakon. Nazdromazdite energiu a nasledne ju pomocou anteny (zdrkadiel) sustredite na jedno miesto. Ked nejakemu telesu dodate energiu a ono ju nedokaze najakym sposobom akumulovat (napr. premienat na chemicke vazby alebo mechanicku energiu v krajnom pripadne na hmotu, ale to uz ideme do relativistickych javov), tak ju musi pekne naspat vyziarit vo forme tepla. No lenze danu energiu potrebujete vyziarit mensou plochou z akej ste ju zozbierali, takze sa to prejavi vyssou teplotou. Ked z tade vyhodite tu antenu, tak sa mozeme bavit o druhom termodynamickom zakone.

Som rad, ze sa tu zavse objavi tema do ktorej viem prispiet aj ja konstruktivne, mimo dokazovania urcitej osobe, ze o Fyzike nema ani sajnu.


Adolf - nepřihlášený - 12/11/2009 - 22:19

Já nechci mít moc ve fyzice binec, tak si to vysvětluji takhle:

- Světlo je těleso jako každé jiné, které má svou teplotu. Jako u všeho ostatního je teplota kinetická energie částice na stupeň volnosti, tedy teplota světla je dána jasně frekvencí. Při výměně tepla mezi světlem a tělesem by tedy světlo nemělo těleso ohřát na teplotu vyšší, než jsou nejteplejší složky jeho spektra.

- Světlo ale může fungovat i jinak než ohřívající látka. Může fungovat jako stlačující píst. Píst může zvednout teplotu látky kompresí, aniž by sám musel mít vysokou teplotu. Je-li charakter absorbovaného elekromagnetického záření takový, že způsobuje kolektivní zrychlení mas látky jedním směrem, vyvolává tak ohřev kompresí (mikrorázy) - jako je tomu u mikrovlnky. Když bychom ale mikrovlnkovali přehřátou páru, dosáhli bychom po čase rovnováhy tlaku emitovaných a přiváděných fotonů, a pak bychom už kompresí nic neohřáli.


Wintermute. - 12/11/2009 - 23:30

citace:
Já nechci mít moc ve fyzice binec, tak si to vysvětluji takhle:

- Světlo je těleso jako každé jiné, které má svou teplotu. Jako u všeho ostatního je teplota kinetická energie částice na stupeň volnosti, tedy teplota světla je dána jasně frekvencí. Při výměně tepla mezi světlem a tělesem by tedy světlo nemělo těleso ohřát na teplotu vyšší, než jsou nejteplejší složky jeho spektra.

- Světlo ale může fungovat i jinak než ohřívající látka. Může fungovat jako stlačující píst. Píst může zvednout teplotu látky kompresí, aniž by sám musel mít vysokou teplotu. Je-li charakter absorbovaného elekromagnetického záření takový, že způsobuje kolektivní zrychlení mas látky jedním směrem, vyvolává tak ohřev kompresí (mikrorázy) - jako je tomu u mikrovlnky. Když bychom ale mikrovlnkovali přehřátou páru, dosáhli bychom po čase rovnováhy tlaku emitovaných a přiváděných fotonů, a pak bychom už kompresí nic neohřáli.



Ok nic v zlom, skusim inak. Zoberme si dva fotony. Kazdy bude mat nejaku frekvenciu (pre jednoduchost uvazujme rovnaku) to znamena nejaku energiu a tym padom aj teplotu. Zatial sa myslim zhodujeme.

A teraz si predstavte, ze by existoval sposob ako dva fotony absorbovat a nasledne emitovat jeden foton s vyssou frekvenciou teda aj teplotou. (len pre uplnost takyto jav existuje a je aj dost pravdepodobne, ze ked sa zvladne prislusna technologia lacnejsie tak sa tento jav bude vyuzivat na zvysenie ucinnosti fotovoltalickych clankov). Takze dosiahnut vyssiu teploti je takto mozne dokonca aj bez toho aby sme zohrievali iny objekt. (ale toto sa netyka slnecnej pece, len som vam chcel spravit ten binec )

A teraz si predstavte, ze mate teleso a tomu dodavate energiu a to teleso nedokaze tuto energiu nejak akumulovat ani premienat na inu energiu, tak proste pridanim dalsej energie zvysujete teplotu (musi samotne teleso vyziarit viac energie). Prikladom najucinejsieho prenosu pomocou fotonov a zohriatie objektu je indukcny ohrev.

Ako da sa to vysvetlit aj na urovni jedneho atomu ale to uz fakt nebude patrit do laickych otazok.


MIZ - 13/11/2009 - 04:52

Já se nechci nějak hádat, ale mohli bychom se domluvit, že pojem "teplota" budeme používat pouze u částic a ne u fotonů? Tam by nám mohla stačit vlnová délka a energie [přičemž částečně částicové chování fotonům neberu]. Teplotu má těleso fotony vyzařující. Co vy na to? Je to jen takový návrh pro lepší srozumitelnost debaty.


Ervé - 13/11/2009 - 08:04

Zkomplikovali jste to tak, že už to žádný laik nepochopí. Takže to shrnu takto: sluneční pec může soustředit fotony tak, že dosažená teplota může být desetitisíce stupňů. Jednoduchým důkazem je laser nebo mikrovlnka, kde proud fotonů může odpařovat ocel nebo ohřát plazma na milióny stupňů (VASIMR), přitom samotné zařízení má pokojovou nebo jen mírně vyšší teplotu.


wintermute. - 13/11/2009 - 10:17

citace:
Já se nechci nějak hádat, ale mohli bychom se domluvit, že pojem "teplota" budeme používat pouze u částic a ne u fotonů? Tam by nám mohla stačit vlnová délka a energie [přičemž částečně částicové chování fotonům neberu]. Teplotu má těleso fotony vyzařující. Co vy na to? Je to jen takový návrh pro lepší srozumitelnost debaty.


Suhlasim, s teplotou fotonu tu zacal niekto iny, tak som sa toho len drzal aby som mu to nekomplikoval. Ale asi sa mi to nepodarilo.


wintermute. - 13/11/2009 - 10:37

citace:
Zkomplikovali jste to tak, že už to žádný laik nepochopí. Takže to shrnu takto: sluneční pec může soustředit fotony tak, že dosažená teplota může být desetitisíce stupňů. Jednoduchým důkazem je laser nebo mikrovlnka, kde proud fotonů může odpařovat ocel nebo ohřát plazma na milióny stupňů (VASIMR), přitom samotné zařízení má pokojovou nebo jen mírně vyšší teplotu.


Dakujem, ja som sa len snazil vysvetlit aspon trosku princip preco tumu tak je.


Tomas Habala - 13/11/2009 - 10:39

citace:
Ako da sa to vysvetlit aj na urovni jedneho atomu ale to uz fakt nebude patrit do laickych otazok.



Dik wintermute. Ja som sice laik, ale predsa len by ma zaujimalo, ako sa el.mag. energia fotonu meni na kineticku energiu atomu. Lebo teplo je kineticka energia castic.


wintermute. - 13/11/2009 - 10:52

citace:
[přičemž částečně částicové chování fotonům neberu]


No tu by sme sa asi neshodli, ja povazujem fotony jednoznacne za casticu. Totizto dnes uz vieme vysvetlit vsetky opticke javy pomocou toho, ze svetlo ma casticovu povahu. Ale vysvetlite mi pomocou vlny ako vznika ciastocny odraz svetla?

Len ako poznamka, som myslel, ze od doby co Feynman sformulovat kvantovu elektrodynamiku (za ktoru mimochodm dostal aj nobelovku) som si myslel, ze uz je jasne, ze foton je castica.


M: - 13/11/2009 - 11:37

citace:
... som si myslel, ze uz je jasne, ze foton je castica.

je mi jasne ze ma povahu aj casticovu aj vlnenia.
Fakt nechapem, ako moze naraz prejst napr. dvoma otvormi sucasne.
to je skor pre vlnenie.

Beriem to ako v sucasnosti najvyssi stupen poznania, ktoremu je potrebne venovat daleko vaic casu ako mam k dispozicii.

Takze sa staviam k tomu asi jak sekretarka k windowsu.

Ten prestup teploty ste zodpovedali super a dakujem.


Tomas Habala - 13/11/2009 - 11:40

Jednoducho povedane, kym foton leti, je vlna, ked narazi je castica.


Derelict - 13/11/2009 - 11:55

citace:
citace:
... som si myslel, ze uz je jasne, ze foton je castica.

je mi jasne ze ma povahu aj casticovu aj vlnenia.
Fakt nechapem, ako moze naraz prejst napr. dvoma otvormi sucasne.
to je skor pre vlnenie.

Beriem to ako v sucasnosti najvyssi stupen poznania, ktoremu je potrebne venovat daleko vaic casu ako mam k dispozicii.

Takze sa staviam k tomu asi jak sekretarka k windowsu.

Ten prestup teploty ste zodpovedali super a dakujem.



Kvuli tomu se rika, ze jakakoliv castice muze mit vlnovou podobu. S trochou nadsazky lze rict, ze existuje i vlnova podoba pro cloveka. Rozptyl lze pozorovat nejenom pro fotony, ale i elektrony a dalsi castice. Jenze popis jejich chovani je uz tak hluboko v mikrosvete, ze pres 100 let se to snazime pochopit a jsme stale jen na okraji poznani.
Pokud se chcete neco dozvedet o fyzice, osobne doporucuji Feynmanovy prednasky z fyziky. Dle meho se jedna o nejcitelnejsi dilo, je sice uz temer 50 let stare, ale myslim si, ze pro vetsinu lidi se jedna o vyborny zdroj pochopeni. Co se tyka moderni fyziky, pokud mate dostatecne znalosti matematiky, pak je mozne (prestoze s tim spousta lidi nebude souhlasit) vzit jako zdroj Formanka nebo Skalu - kvantova teorie, kvantova mechanika. Jinak dalsimi zdroji mohou byt nasledujici:
M.I.T. Open CourseWare - http://ocw.mit.edu/OcwWeb/web/home/home/index.htm " target=_blank> http://ocw.mit.edu/OcwWeb/web/home/home/index.htm
Cornell University Library - http://arxiv.org/ " target=_blank> http://arxiv.org/

V obojim pripade je nutna znalost anglictiny a matematiky, vrele doporucuji Feynmanovy prednasky jako zaklad. Navic, jsou v cestine.

http://www.neoluxor.cz/katalog/hledani/?searchstring=feynman&searchin=all " target=_blank> http://www.neoluxor.cz/katalog/hledani/?searchstring=feynman&searchin=all


Tomas Habala - 13/11/2009 - 12:12

Feynamanove prednasky z fyziky su vyborne ale dualnost castica-vlna je tam prezentovana v podstate len na jednom priklade (klasicka dvojstrbina).

Za velmi dobre pre laika s hlbsim zaujmom povazujem cyklus od Cejnara a Duska, ktory vychadzal kedysi vo Vesmire. Je tam uvedenych viac prikladov iducich az k samotnej podstate, ked sa naozaj zda, ze castice reaguju snad len na stav vedomia. Cloveku sa tam ta podivnost dualnej povahy rozprestrie pred ocami v plnej krase.

http://www-ucjf.troja.mff.cuni.cz/resource/outreach/vesmir1.pdf
http://www-ucjf.troja.mff.cuni.cz/resource/outreach/vesmir2.pdf
http://www-ucjf.troja.mff.cuni.cz/resource/outreach/vesmir3.pdf


wintermute. - 13/11/2009 - 12:12

citace:
Jednoducho povedane, kym foton leti, je vlna, ked narazi je castica.


Foton je castica, ked leti. Foton ked neleti tak neexituje. A letiet musi rychlostou svetla.


wintermute. - 13/11/2009 - 12:13

citace:

Dik wintermute. Ja som sice laik, ale predsa len by ma zaujimalo, ako sa el.mag. energia fotonu meni na kineticku energiu atomu. Lebo teplo je kineticka energia castic.


No pokusim sa, aj ked mi tu to zjednodusovanie moc nejde.

Takze v prvom rade by som bol rad keby sme sa zhodli na tom, ze foton je naozaj castica. Akurat sa musi stale pohybovat rychlostou svetla, ked ho len trochu spomalite prestane existovat. (len dodam ,ze foton sa naozaj vzdy pohybuje rychlostou svetla, nepliest si to s ryhlostou sirenia svetla ta mozem byt aj nizsia, ako je rychlost svetla).

Kedze sa hybe rychlostou svetla musite pouzit relativisticku fyziku. Vzladom na to, ze sa foton stale pohybuje musi mat energiu. Ta je vyhadrena vztahom E=hf, kde h je plankova konstatna a f je frekvencia. Vzhlaodm na to ze foton ma energiu viete mu priradit aj hmotnost tuto hmotnost ma ked sa pohybuje) a to podla ekvivalncie hmoty a energie E=mc^2. Kedze ma foton hmotnost a sa aj pohybuje tak vieme spocitat hybnost, akurat nema kludovu hmotnost takze zo vztahu pre relaitvisticku hmotnost nam cast, ktora pocita s kludovou hmotnostou vypadne. Dostaneme teda, ze hybnost fotonu p=E/c.

Takze sme sa dostali k tomu, ze si ten foton leti a ma pri tom urcitu hybnost. Pri zaniku odovzda tuto energiu elektornu, no a elektron je hmotna castica.


Tomas Habala - 13/11/2009 - 12:15

Moze castica letiet rychlostou svetla?


wintermute. - 13/11/2009 - 12:17

citace:
je mi jasne ze ma povahu aj casticovu aj vlnenia.
Fakt nechapem, ako moze naraz prejst napr. dvoma otvormi sucasne.
to je skor pre vlnenie.



Dovma otvomi sucastne v kvantovom stave prejde akakolvek castica. Mimochodom to iste co foton spravi napr. aj helive jadro. Takze dufam ze uz sa zhodneme na otm ze heliove jadro nieje vlna. Takze aj foton uz budeme pokladat za casticu.


wintermute. - 13/11/2009 - 12:22

Pri fotone sa dualita castica a vlna pouzivala na vysvetlenie niektorych javov, ktore sme pri castici vysvetlit nevedeli a vlna bol vhodny model.

Uz tomu tak nieje. Pri fotone ako castici vieme vysvetlit vsetky javy a nepotrebuejeme na to model vlny. Dokonca este viac. S modelom vlny nedokazete vysvetlit ciastocny odraz svetla, s modelom castice ho vysvetlit dokazeme. Avsak vediet spocitat (a pochopit) takyto pohyb pri ktorom vznika ciastocny odraz vyzaduje naozaj niekolko rocne studium kvantovej elektrodynamiky.


MIZ - 13/11/2009 - 12:23

citace:
Moze castica letiet rychlostou svetla?

Částice může letět rychlostí světla pouze má-li nulovou klidovou hmotnost. To splňuje pouze foton.
[Neutrinům byla tuším zjištěna nenulová klidová hmotnost, všem třem.]


wintermute. - 13/11/2009 - 12:23

citace:
Moze castica letiet rychlostou svetla?

Moze, su to castice, ktore nemaju kludovu hmotnost. Jednou z nich je foton.


MIZ - 13/11/2009 - 12:29

citace:
Pri fotone sa dualita castica a vlna pouzivala na vysvetlenie niektorych javov, ktore sme pri castici vysvetlit nevedeli a vlna bol vhodny model.

Uz tomu tak nieje. Pri fotone ako castici vieme vysvetlit vsetky javy a nepotrebuejeme na to model vlny. ...

Právě zmíněný dvojštěrbinový experiment dokazuje mj. vlnovou povahu fotonu: I jediný foton zanechá za dvojštěrbinou stopu vlnové interference. To částice neudělá.
EDIT: Totožně experiment ale dopadne i s "normální částicí" elektronem. I ten se chová také jako vlna.

Jinak je pravda, že vše má duální, tedy částicovou + vlnovou povahu a každé částici a z částic vytvořenému objektu lze přiřadit i vlnovou délku. Akorát pro objekty velikosti člověka ta vlnová délka vychází neměřitelně malá. [V příslušném vztahu vystupuje Planckova konstanta, což je číslo zlehčeně řečeno dost podobné nule. ] [Upraveno 13.11.2009 MIZ]


wintermute. - 13/11/2009 - 12:32

citace:
To částice neudělá.



Urobi to napr. aj heliove jadro, je to vlna? Vlna je len model ktorym sa vysvetlovalo spravanie, ktore sme pomocou modelu castice vysvetlit nevedeli. Uz to vysvetlit vieme vid. Kvantova elektrodynamika.


Tomas Habala - 13/11/2009 - 12:33

Mne sa na predstave fotonu ako castice nezda prave to, ze nema kludovu hmotnost a ze jediny jeho existujuci stav je pohybovat sa voci vsetkemu rychlostou svetla. Tie casticove vlastnosti zistujeme az ex-post, po zaniku fotonu. Jediny moment, ked sa fotom prejavi ako castica je moment jeho zaniku. Alebo sa mylim?


wintermute. - 13/11/2009 - 12:35

K vlne, ked my vysvetlite ako vznika ciastocny odraz svetla pomocou vlny, tak uznam ze foton je vlna. Zaroven asi poziadam aby Feynmanovy odobrali nobelovu cenu.


wintermute. - 13/11/2009 - 12:36

Za gramaticke chyby sa ospravednujem vidim ich po sebe ked si precitam moje prispevky ale pri pisani mi to unika. Je to vec, pri ktorej poprosim o toleranciu.


wintermute. - 13/11/2009 - 12:48

citace:
Mne sa na predstave fotonu ako castice nezda prave to, ze nema kludovu hmotnost a ze jediny jeho existujuci stav je pohybovat sa voci vsetkemu rychlostou svetla. Tie casticove vlastnosti zistujeme az ex-post, po zaniku fotonu. Jediny moment, ked sa fotom prejavi ako castica je moment jeho zaniku. Alebo sa mylim?


Nie, casticove vlastnosti ma po celu dobu. Ja chapem, ze sa na skolach vysvetluje dualita. Na stredenj a myslim, ze ani na vysokych skolach sa bezne kvantova elektordynamika neprebera, takze jediny sposob ako vysvetlit niektore javy je pouzit model vlny. Ale opakujem model vlny nieje spravny, nepokryva vsekty javy, dokonca ani niektore, ktore by ste cakali pri "vlnovom prejave" fotonu. Priklad je ciastocny odraz svetla, ten proste touto teoriou nevysvetlite. Naopak pri modeli castice dokazete vysvetlit vsetky doteraz zname javy, navyse kvantova elektrodynamika sa uz netyka len fotonov a elektoronov ale bola rozsirena aj na jadro atomu a aj na atomy ako take. Proste su dve teorie.

1. castica - pokryva vsetky zname javy
2. vlna - vysvetluje len malu cast javov


MIZ - 13/11/2009 - 12:49

citace:
citace:
To částice neudělá.

Urobi to napr. aj heliove jadro, je to vlna?

Nevím.
Dvojštěrbinový experiment má určitě nějakou horní hranici, po kterou to funguje. Mám takový dojem, že elektron apod. je někde blízko toho maxima. Heliové jádro je už řádově větší a vlnová část jeho charakteru je tedy již mnohem menší než třeba u toho elektronu. [Vlnová část charakteru zrnka písku je už zcela zanedbatelná. ]

Odraz světla: Neumím vysvětlit ani úplný, omlouvám se. A to jsem něco od Feynmana četl...


wintermute. - 13/11/2009 - 12:51

citace:
citace:
citace:
To částice neudělá.

Urobi to napr. aj heliove jadro, je to vlna?

Nevím.
Dvojštěrbinový experiment má určitě nějakou horní hranici, po kterou to funguje. Mám takový dojem, že elektron apod. je někde blízko toho maxima. Heliové jádro je už řádově větší a vlnová část jeho charakteru je tedy již mnohem menší než třeba u toho elektronu. [Vlnová část charakteru zrnka písku je už zcela zanedbatelná. ]

Odraz světla: Neumím vysvětlit ani úplný, omlouvám se. A to jsem něco od Feynmana četl...


Lenze nemozete argumentovat strbinovym experimentom pre vysvetlenie vlnovej teorie, kedze ten isty vysledok dostanete aj ked pouzijete namiesto elektoru alebo fotonu aj heliove jadro. Dvojstrbinovy experimnet vyvetluje bezchybne aj casticovy model popisany kvantovou elektordynamikou.


Tomas Habala - 13/11/2009 - 13:03

Ospravedlnujem sa cistym kozmonautom, este zostanem pri tejto teme, zaujima ma to.

Takze wintermute, mozes prosim popisat, aky je problem s ciastocnym odrazom?


MIZ - 13/11/2009 - 13:03

citace:
Mne sa na predstave fotonu ako castice nezda prave to, ze nema kludovu hmotnost a ze jediny jeho existujuci stav je pohybovat sa voci vsetkemu rychlostou svetla. Tie casticove vlastnosti zistujeme az ex-post, po zaniku fotonu. Jediny moment, ked sa fotom prejavi ako castica je moment jeho zaniku. Alebo sa mylim?

Vyhnu se té druhé části, kolem toho zániku. Foton se snad dá brát i jako kvantum energie a zánik pak není to pravé slovo...
Ano, foton je trochu výjimečný. Nulová klidová hmotnost a zároveň maximální možná rychlost pohybu jsou sobě vzájemně podmínkou a důsledkem - s nenulovou klidovou hmotností se prostě na rychlost světla nedá dostat.
Foton také postrádá některé kvantové charakteristiky. Je sám sobě antičásticí, tj. antifoton nelze rozlišit od fotonu.

Pokud elektron v atomu poklesne na nižší energetickou hladinu, vyzáří foton s odpovídající energií = vlnovou délkou. Pokud je atomem foton pohlcen, atom získá energii a některý jeho elektron vystoupá na vyšší energetickou hladinu. Atom ale pohltí jen fotony takové vlnové délky, pro jejichž energii má k dispozici vhodné energetické hladiny pro své elektrony. Elektrony [jako ostatně všechno ve vesmíru] se snaží zaujmout stav s nejnižší možnou energií, tj. přebytečnou energii zase formou fotonů vyzářit.


Alchymista - 13/11/2009 - 13:24

Len doplním - v niektorých prípadoch, u tvrdého žiarenia gama s veľmi malou vlnovou dĺžkou, môže byť fotón zachytený aj jadrom atómu, ktoré sa tak dostane do vzbudeného stavu. Podobne môže byť fotón vyžiarený z jadra atomu vybudeného napríklad po záchyte neutrónu alebo protónu.


Dualita častica-vlna fotonu (alebo elementárnych častíc všeobecne)
Do značnej miery to funguje tak, že pokiaľ sa zostaví nejaký experiment, ktorý by mal dokázať, že elementárna častica je napríklad "častica" (na to bol vlastne pôvodne určený dvojštrbinový experiment), dopadne to tak, že sa dokáže pravý opak.
Obrazne povedané, pokiaľ sa elementárnej častici dá v experimente jasná otázka "Si častica?" alebo "Si vlna?" jej "odpoveď" bude vždy záporná.
Druhým javom, spojeným s touto dualitou, je to, že s narastajúcou energiou sa elementárne častice chovajú stále viac ako "častice", a naopak, pri nizkej energii sa chovajú viac ako "vlny".


M: - 13/11/2009 - 13:38

citace:
... Atom ale pohltí jen fotony takové vlnové délky, pro jejichž energii má k dispozici vhodné energetické hladiny pro své elektrony. ...


A jeje. Takze ak chcem laserom odparit zelezo, tak potrebujem presnu vlnovu dlzku fotonov?

Este inak. tkaze ak chcem ohriat ziarenim jedneho telesa druhe, tak musim vyzarovat na "akceptovatelnej" vlnovej dlzke?
Takze jedno teleso ma pre rozne vlnove dlzky roznu emisivitu?
Absolutne cierne teleso pohlcuje "vsetko" ziarenie bez rozdielu?

Je teda priradenie koeficientu 0,96 pre asfalt bez udania rozsahu vlnovej dlzky nepresne?


M: - 13/11/2009 - 13:42

Pre ohrev je teda vhodne, aby plocha vyzarovania i prijimania ziarenia bola z rovnakeho materialu?

je tam nejaka tolerancia, alebo sa musi absorbovat presne kvantum enrgie?


MIZ - 13/11/2009 - 14:14

A jéje! Ne.
Popsán byl mechanismus pohlcení a vyzáření fotonu atomem.
Kovy jsou již složité atomy s mnoha elektrony ve složitém elektronovém obalu. Tedy s mnoha energetickými úrovněmi.
Další uplatňující se efekt je krystalový charakter kovů. Celý velký odlitek kovu může být jediným krystalem [známé jsou monokrystaly křemíku]. Navíc kovy vedoucí el. proud mají pak elektrony tzv. volné, tj. volně se pohybující, nevázané na konkrétní jádro. [V některých kovech neputují elektrony, ale naopak tzv. díry - volné pozice pro elektrony.] Tam pak zřejmě bude velmi mnoho variant možných energetických hladin.
To se asi už dostáváme od těch jednoduchých základů...


Alchymista - 13/11/2009 - 14:15

citace:
Takze ak chcem laserom odparit zelezo, tak potrebujem presnu vlnovu dlzku fotonov?
To zaiste nie.
citace:
ak chcem ohriat ziarenim jedneho telesa druhe, tak musim vyzarovat na "akceptovatelnej" vlnovej dlzke?
Nie.
citace:
Je teda priradenie koeficientu 0,96 pre asfalt bez udania rozsahu vlnovej dlzky nepresne?
správne, je to nepresné.

Narazili ste na jeden problém - MIZ popisuje zachytenie fotónu v elektrónovom obale jedného atomu. V prípade tuhej látky je to komplikovanejšie.

http://cs.wikipedia.org/wiki/Absolutn%C4%9B_%C4%8Dern%C3%A9_t%C4%9Bleso


Barrymore - 13/11/2009 - 14:51

Zdravím. Tahle diskuze ohledně fyziky mě zaujala a napadla mě otázka: Mějme částici (elektron) letící rychlostí 0.9c, která vletí do diamantu s indexem lomu 2.5. Co se bude dít? Částice má nenulovou hmotnost, takže její rychlost bude klesat postupně a ne okamžitě jako u fotonu. Takže by se logicky měla určitou dobu pohybovat nadsvětelnou rychlostí. Uvažuji správně?


Alchymista - 13/11/2009 - 15:14

Ano, uvažuješ správne. Viditeľným dôsledkom bude Čerenkovovo žiarenie

http://cs.wikipedia.org/wiki/%C4%8Cerenkovovo_z%C3%A1%C5%99en%C3%AD


Adolf - nepřihlášený - 13/11/2009 - 16:12

citace:
Ano, uvažuješ správne. Viditeľným dôsledkom bude Čerenkovovo žiarenie

http://cs.wikipedia.org/wiki/%C4%8Cerenkovovo_z%C3%A1%C5%99en%C3%AD



Je to správně v tom smyslu, že se bude pohybovat rychlostí vyšší než rychlost penetrace světla daným materiálem, nikoliv však, že by se elektron pohyboval rychlostí vyšší než rychlost šíření světla vakuem, což bych si myslel, že by mělo být chápáno jako nadsvětelná rychlost.


Adolf - nepřihlášený - 13/11/2009 - 16:30

citace:
Dvojštěrbinový experiment má určitě nějakou horní hranici, po kterou to funguje. Mám takový dojem, že elektron apod. je někde blízko toho maxima. Heliové jádro je už řádově větší a vlnová část jeho charakteru je tedy již mnohem menší než třeba u toho elektronu. [Vlnová část charakteru zrnka písku je už zcela zanedbatelná. ]




Dvojštěrbinový model, pokud vím funguje i u pěkně velkých molekulárních otesánků. Ovšem experimentálně ho připravit je čím větší částice, tím obtížnější. Ale i kdybychom byli superexperimentárory, tak to selze, když je objekt tak veliký, že délka jeho vlny je moc malá vůči jeho rozměrům.

Úplně největší humus v představivosti pak přichází, když začneme uvažovat kolektivní kvantové excitace - kvazičástice.

U kvazičástic mě kdysi napadla myšlenka, která by měla použitelnost v kosmonautice. Uvedl jsem ji tu už kdysi ale zapadla:

Lze vytvořit kondenzovanou fázi, ve které budou kvazičástice s takovou absorbční frekvencí, že by mohly mít velký účinný průřez při průchodu vysoce energetického záření. Tímto by možná šly vytvořit polovodičové lapače ionizujícícho zaření s vyšší efektivitou než vrstvy olova či vodíke bohatých látek pro různé typy radiace.

Jak to vidíte?


Adolf - nepřihlášený - 13/11/2009 - 16:46

Světlo netřeba z ničeho vyřazovat. Jsou to částice s kinetickou energií a - tedy teplotou. Elm. vlny přihřívají jiné objekty úplně stejně jako jiné objekty – přes svoji teplotu (světelnou teplotu) nebo kompresí. Když přiletí asteroid a vletí nám do atmosféry - ač je studený a i atmosféra je studená – bude nám tu zatraceně horko. Není to ohřev studeným tělesem, nýbrž ohřev kompresí pomocí studeného tělesa. Elektromagnetické vlny také dovedou zahřívat jak přenosem tepla z teplého tělesa na chladné, tak vytvořením tepla kompresí, což je ovšem třebas i indukční ohřev aj.


Alchymista - 13/11/2009 - 16:48

Keď človek občas číta o pokrokoch vo výskume vlastností rôznych nanomateriálov, napríklad uhlíkových nanotrubíc, tak ten nápad by nemusel byť zasa celkom márny.
Ovšem cena takéhoto materiálu by bola zrejme celkom šialená.


Barrymore - 13/11/2009 - 16:59

Díky za odkaz na čerenkovovo záření. Pojem jsem slyšel, ale o jeho podstatě jsem toho moc netušil.

citace:
Elektromagnetické vlny také dovedou zahřívat jak přenosem tepla z teplého tělesa na chladné, tak vytvořením tepla kompresí, což je ovšem třebas i indukční ohřev aj.


Nevím, nejsem kvantový fyzik, pouze radiotechnik, ale indukční ohřev bych zrovna kompresí nenazýval. Tam dochází k ohřevu vyvoláním el. proudu, což je v podstatě převod elmag. energie na kinetickou.


Adolf - nepřihlášený - 13/11/2009 - 17:05

citace:
citace:
Moze castica letiet rychlostou svetla?

Částice může letět rychlostí světla pouze má-li nulovou klidovou hmotnost. To splňuje pouze foton.
[Neutrinům byla tuším zjištěna nenulová klidová hmotnost, všem třem.]


Pokud vím, tak všem částicím s nulovou klidovou hmotnost se zkouší zjistit, zda nejde o částici s nepatrnou, ale přeci jen nenulovou hmotností, která se ve skutečnosti pohybuje nepatrně podsvětelnou rychlostí, včetně světla. V podstatě si asi fyzikové ani nedovedou představit důkaz, že částice má skutečně nulovou klidovou hmotnost, jen se měří, jaká je největší možná hmotnost částice podle zjištění z našich experimentů. Tyto hmotnost se tedy u neutrin, fotonů a gravitonů občas uvádí. Ale nevím, že by to opravdu u neutrin pozitivně zjistili, že tuto hmotnost neutrino má a ne jen, že je největší možná, ač to nevylučuji. Pokud to bylo fakt zjištěno, rád bych věděl, kde se to dočtu.

Co se týče klidové hmotnosti fotonů, jsou teď astrofyzikové trochu na větvi z jisté supernovy, od níž sem asi přišlo záření časové roztříděno podle vlnové délky světla. To je docela voda na mlýn vyznavačům klidové hmotnosti těchto částic.


Alchymista - 13/11/2009 - 17:34

http://hvezdarnauherskybrod.sweb.cz/

citace:
Einsteinova teorie stále platí

Jiný záblesk paprsků gama, zachycený tentokrát družicí FERMI přinesl potvrzení správnosti Einsteinovy teorie relativity. RGB 090510 zachycený 10.5.2009 vyslal směrem k Zemi dva fotony záření gama s rozdílnými energiemi. Oba fotony dorazily k Zemi ze vzdálenosti 7,3 miliardy světelných roků prakticky současně, přesněji s odstupem 0,9 sekundy, přestože se jejich energie lišila více než milionkrát. Navíc rozdíl v čase příletu byl, stejně jako rozdílná energie obou fotonů, zaviněn tím, že každý z nich vznikl v jiném místě jevu, který se nám prezentoval jako záblesk paprsků gama.

Tímto pozorováním dvou fotonů, které k nám putovaly přes polovinu vesmíru bylo potvrzeno, že skutečně platí tak zvaná Lorentzova invariance, tedy to, že rychlost fotonu se nemění s jeho energií. Tím byla současně potvrzena i Einsteinova speciální teorie relativity.

Tento poznatek je ale problémem pro moderní teorie gravitace snažící se o to, co se Einsteinovi nikdy nepovedlo, tedy sjednotit čtyři základní přírodní síly do jedné teorie. Podle nových pokusů o jejich sjednocení by Lorentzova invariance měla být v měřítku velmi malých vzdáleností narušena kvantovými gravitačními vlivy a tak by foton s vyšší energií měl být oproti fotonu s nižší energií zpomalen. A to se evidentně nestalo. Einstein tedy kraluje i nadále a vědci teoretičtí fyzikové mají další důvod k přemýšlení.
[Upraveno 13.11.2009 Alchymista]


Wintermute. - 13/11/2009 - 18:16

citace:

Dualita častica-vlna fotonu (alebo elementárnych častíc všeobecne)
Do značnej miery to funguje tak, že pokiaľ sa zostaví nejaký experiment, ktorý by mal dokázať, že elementárna častica je napríklad "častica" (na to bol vlastne pôvodne určený dvojštrbinový experiment), dopadne to tak, že sa dokáže pravý opak.
Obrazne povedané, pokiaľ sa elementárnej častici dá v experimente jasná otázka "Si častica?" alebo "Si vlna?" jej "odpoveď" bude vždy záporná.
Druhým javom, spojeným s touto dualitou, je to, že s narastajúcou energiou sa elementárne častice chovajú stále viac ako "častice", a naopak, pri nizkej energii sa chovajú viac ako "vlny".


Mozete mi prosim napisat o nejakom jave, pri ktorom nieje foton, alebo ina castica, casticou ale vlnou? Ja totizto uz taky dost dlho nepoznam.
Asi sa budem zase opakovat, ale foton aj akakolvek castica je stale len castica, teoria ohladom duality bola sformulovana preto, ze sme pre casticu nevedeli niektore javy vysvetlit. Od sformulovania kvantovej elektrodynamiky ich uz vysvetlit vieme a nepotrebujeme na to ziadnu vlnu. Jasne z kniziek to nezmazali, vsak ked Einstein sformuloval teoriu relativity, tiez nevymazali z ucebnic Newtonku, ale nikto uz dnes netvrdi ze sila posobi len tak okamzite na dialku a ze cas plynie vzdy rovnako. Proste foton je castica, ano niektore javy si mozete spocitat pomocou modelu vlny lebo ten matematicky model na urcitu skalu javov sedi a aj vypocet je jednoduchsi. Tiez si nebudete ratat cas za ktory dorazi vlak z prahy do bratislavy cez vseobecnu teoriu relativity.

Odpoviem aj na dalsie otazky ked sa dostanem k pocitacu, co bude asi az zajtra. Toto pisem z mobilu .


MIZ - 13/11/2009 - 19:12

citace:
citace:
... [Neutrinům byla tuším zjištěna nenulová klidová hmotnost, všem třem.]

...
Ale nevím, že by to opravdu u neutrin pozitivně zjistili, že tuto hmotnost neutrino má a ne jen, že je největší možná, ač to nevylučuji. Pokud to bylo fakt zjištěno, rád bych věděl, kde se to dočtu.

Četl jsem to jako objev před 3 nebo více lety. Pamatuji si byt, v kterém jsem to četl a tam už 3 roky nebydlím. Neutrina jsou podle této zprávy trojího typu, označena řeckými písmeny, mj. tuším tau, přičemž vzájemně mezi těmito třemi "verzemi" přecházejí. Byly tam uvedené i číselné hodnoty jejich energií. Na víc si nevzpomenu.
Pro opravdové zájemce to jistě půjde někde vyhledat.


HonzaVacek - 13/11/2009 - 19:35

Ta neutrina se označují jako e , mu a tau podle toho, jakých reakcí se zúčastňují a s kterými leptony se při reakci vyskytují. O hmotnosti neutrina e a tau se neví prakticky nic nicméně se naměřilo, že neutrino mu má přibližně o 0.07 eV větší klidovou hmotnost než neutrino e.


Adolf - nepřihlášený - 13/11/2009 - 22:01

citace:
Ta neutrina se označují jako e , mu a tau podle toho, jakých reakcí se zúčastňují a s kterými leptony se při reakci vyskytují. O hmotnosti neutrina e a tau se neví prakticky nic nicméně se naměřilo, že neutrino mu má přibližně o 0.07 eV větší klidovou hmotnost než neutrino e.


Mají v té hmotnosti neutrina pěkný binec:

http://en.wikipedia.org/wiki/Neutrino

Hodila by se jim, protože vykazuje oscilaci, kterou by částice bez hmotnosti vykazovat neměla. Domeřit se ale nemůžou.


HonzaVacek - 13/11/2009 - 22:22

No, binec to je Je ovšem otázka, jestli se vůbec někdy dopárají. Nejsem si úplně jistý, ale mám pocit, že hmotnost neutrina byla zavedena právě jenom z toho důvodu, aby se vysvětlilo, že ze Slunce přichází méně neutrin než by mělo. Jenomže když se kouknu na naše sluníčko jak se teď chová, tak si nejsem tak moc jistý, že je to ta správná cesta a jestli by se deficit neutrin neměl hledat spíše tam, než ho vysvětlovat oscilacemi neutrin.


Adolf - nepřihlášený - 13/11/2009 - 23:25

citace:

Einstein tedy kraluje i nadále a vědci teoretičtí fyzikové mají další důvod k přemýšlení.
[Upraveno 13.11.2009 Alchymista]


Já myslím, že v tom žádné ověřování Einsteina není. I kdyby snad foton klidovou energii měl, o čemž pochybuji, na teorii relativity by to nic nezměnilo, jen by se krajní rychlosti už neříkalo rychlost světla, ale jen krajní rychlost.


HonzaVacek - 13/11/2009 - 23:39

citace:
Já myslím, že v tom žádné ověřování Einsteina není. I kdyby snad foton klidovou energii měl, o čemž pochybuji, na teorii relativity by to nic nezměnilo, jen by se krajní rychlosti už neříkalo rychlost světla, ale jen krajní rychlost.


No já nevím, ale podle mě by se změnilo hodně. STR je zasložena na faktu, že rychlost světla je ve všech inerciálních soustavách stejná a navíc lze touto rychlostí přenášet informaci, není to tedy nic hypotetického. V případě, že by klidová hmotnost byla nenulová, nebyla by rychlost c již konstanta. V různých inerciálních sestavách by byla c různá. STR a OTR by pak asi nefungovaly


Adolf - nepřihlášený - 14/11/2009 - 00:00

citace:

Mozete mi prosim napisat o nejakom jave, pri ktorom nieje foton, alebo ina castica, casticou ale vlnou? Ja totizto uz taky dost dlho nepoznam.



Snad všechny jevy obsahující inteferenci, difrakci, průchod více než jednou štěrbinou, odraz od více než jednoho zrcadla, vázané kvantové stavy. Ale platí to i pro klasické látkové částice s nenulovou klidovou hmotností.

Je sice pravda, že naše modely jsou plné různých berliček pro představivost a všechno je trochu jinak, ale tak, že by částice byly létající kuličky a ty vlny byly jen jakousi mapou jejich chuti k tomu či onomu pohybu. Je to nějak jinak. Tu imaginaci za tou matematickou reprezentací asi nestihl za těch skoro 90 let kvantové teorie nikdo vymyslet.

Kdybych mohl doporučit něco hodně laicky přístupného a zároveň docela solidně ve vědě založeného o interpretaci kvantové teorie a všemožných jevech a experimentech, které musí vysvětlit, doporučil bych Schrödingerova koťata. Autor je i tvůrcem a zastáncem zajímavé interpretace této teorie, kterou na konci knihy alternativně ke Kodaňské interpretaci předkládá. Já osobně jeho interpretaci nazývám intepretací transakce komunikačního protokolu.


Adolf - nepřihlášený - 14/11/2009 - 00:05

citace:
Moze castica letiet rychlostou svetla?


Pokud má klidovou hmotnost, tak nemůže, protože by musela mít nekonečnou energii a tudíž i celkovou hmotnost. Pokud má nulovou klidovou hmotnost, jako třeba foton, graviton aj., pak se musí pohybovat jedině rychlostí světla a žádnou jinou.


Adolf - nepřihlášený - 14/11/2009 - 00:10

citace:
K vlne, ked my vysvetlite ako vznika ciastocny odraz svetla pomocou vlny, tak uznam ze foton je vlna. Zaroven asi poziadam aby Feynmanovy odobrali nobelovu cenu.


Zkuste mi bez vlny vysvětlit odraz, díky němuž se vidím ve výloze, do které koukám.


HonzaVacek - 14/11/2009 - 00:11

Navíc mě teď napadá ještě jeden důvod, proč musí být klidová hmotnost fotonu nulová. Pokud bychom v STR označili rychlost c jako nějakou limitní, tak potom v kvantové elektrodynamice se nám stejně objeví polní částice elektromagnetického pole s nulovou klidovou hmotností pohybující se rychlostí c, které nakonec budeme říkat foton. Nebo ne?

Já tedy nevím, ale kdyby se zjistilo, že hmotnost fotonu je nenulová, tak by najednou bylo všechno jinak.


Adolf - nepřihlášený - 14/11/2009 - 00:20

citace:

No já nevím, ale podle mě by se změnilo hodně. STR je zasložena na faktu, že rychlost světla je ve všech inerciálních soustavách stejná a navíc lze touto rychlostí přenášet informaci, není to tedy nic hypotetického. V případě, že by klidová hmotnost byla nenulová, nebyla by rychlost c již konstanta. V různých inerciálních sestavách by byla c různá. STR a OTR by pak asi nefungovaly


Ta krajní rychlost by měla pořád stejnou povahu, jen by to byla krajní rychlost a ne "rychlost světla". Konec konců, ona je to mj. i rychlost gravitace. Pravda hůř by se nám měřila. Museli bychom zrušit dosavadní stav, kdy se rychlost světla neměří, je dána definicí, a měří se jen, jak je dlouhý metr. - Máme definitoricky určenou rychlost světla, a definitoricky určenou délku sekundy. Čím dál přesnějším měřením určujeme, jak je dlouhý metr. Z tohoto pohodlného stavu bychom se dostali do blázince, kdy bychom pořád upřesňovali hmotnost fotonu a skutečnou krajní rychlos a měli při tom neskutečný zmatek v základních jednotkách.


Adolf - nepřihlášený - 14/11/2009 - 00:25

citace:

Já tedy nevím, ale kdyby se zjistilo, že hmotnost fotonu je nenulová, tak by najednou bylo všechno jinak.


Bylo, ale teorie relativnosti by zůstala stejná, jen by se tam udělala úprava výrazů. Elektrodynamické modely by povinně přibraly kalibraci na mezní rychlost. Strunisté by z toho vyrazili zlatý poklad, že by se tam vecpali se strunami a úplně nám zamotali zbytek ještě nezamotaných mozkových závitů.


Adolf - nepřihlášený - 14/11/2009 - 00:35

citace:
myslím, že druhý zákon termodynamiky by porušený nebol, už preto, že sa zachytáva energia z väčšej plochy a sústreďuje na plochu menšiu, čo vôbec nie je to isté ako ohrievanie teplejšieho telesa chladnejším (ostatne aj to sa dá - na tomto princípe pracujú všetky tepelné pumpy ako napríklad chladničky, klimatizácie...


Ty pumpy pracují vždy s KOMPRESOREM že?

Možná i magnetické chladničky - používané zatím jen jako experimentální zařízení pro chlazení demagnetizací - bychom měli považovat v jistém rozšířeném smyslu za kompresorové. (Až se hne dál vývoj materiálů, asi se dočkáme i magnetických chladniček doma místo chladících kompresorů. - Stejně je to ale kompresor.)


HonzaVacek - 14/11/2009 - 00:48

No, ještě jsi mě nepřesvědčil Hlavním důvodem proč se nakonec dospělo k STR bylo to, že rovnice elmag. pole nebyly invariantní vůči Galileově transformaci. Takže se nakonec dospělo k Lorentzově tr., která vyústila v STR. Pokud by ale neplatilo, že rychlost světla je právě c, tak Maxwellovy rovnice přestávají být vůči LT invariantní, čili elektromagnetické děje dopadnou v různých vztažných soustavách jinak. V principu tedy lze najít privilegovanou vzatžnou soustavu a tím pádem STR nefunguje.


Adolf - nepřihlášený - 14/11/2009 - 00:49

citace:

1. castica - pokryva vsetky zname javy
2. vlna - vysvetluje len malu cast javov




Myslím, že kvantová mechanika je koncipována tak, že vlna vysvětluje skoro vše. (Všepopisující veličinou je vlnová funkce.) Ale je to jedno. Není tu kontraverze vlna NEBO částice. Je to problém hybridizace, jak to má být zároveň vlna i částice. Nikdo to plně nezvládl. Ten fenomén je ale prostě vlnou i částicí.


Adolf - nepřihlášený - 14/11/2009 - 00:52

Chtěl jsem tu odpovědět na námitky proti kompresnímu pojetí některého ohřevu zprostředkovaného fotony. Vzpomínám, že to tu je, ale v tuto hodinu a se všemi sklenicemi to už nejsem schopen najít. Tak mi prommiňte, že odpověď odložím na jindy.


Derelict - 14/11/2009 - 00:57

citace:
citace:

Já tedy nevím, ale kdyby se zjistilo, že hmotnost fotonu je nenulová, tak by najednou bylo všechno jinak.


Bylo, ale teorie relativnosti by zůstala stejná, jen by se tam udělala úprava výrazů. Elektrodynamické modely by povinně přibraly kalibraci na mezní rychlost. Strunisté by z toho vyrazili zlatý poklad, že by se tam vecpali se strunami a úplně nám zamotali zbytek ještě nezamotaných mozkových závitů.


Adolfe, mam tu pro pobaveni par orisku ... ;o)
Pokud plati E=mc^2, foton majici energii by mel mit zaroven hmotnost. Tedy pokud plati vyraz m=E/c^2. Zaroven, cim vyssi frekvence fotonu, tim vyssi energii prenasi, tim vyssi odpovidajici hmotnost by mel mit. Tedy:
1) Ma foton "hmotnost" nebo ne?
2) Pokud hmotna castice nemuze dosahnout rychlosti svetla, protoze by mela nekonecnou hmotnost, tedy svetlo nemuze existovat, protoze foton by mel nekonecnou hmotnost. Nebo ne ?

Radeji s temito osklivymi vtipky skoncim, nebo nikdo neusne ;o)))

P.S.: Prosim ostatni ctenare o shovivavost, tohle je pouze osklivy vtip, ktery se nema vypravet pred spanim ;o)


HonzaVacek - 14/11/2009 - 01:05

citace:
Adolfe, mam tu pro pobaveni par orisku ... ;o)


No hlavně právě díky tomu E=mc^2 by došlo k tomu, že světlo s různými vlnovými délkami by se šířilo i různou rychlostí.


Adolf - nepřihlášený - 14/11/2009 - 01:09

citace:
No, ještě jsi mě nepřesvědčil Hlavním důvodem proč se nakonec dospělo k STR bylo to, že rovnice elmag. pole nebyly invariantní vůči Galileově transformaci. Takže se nakonec dospělo k Lorentzově tr., která vyústila v STR. Pokud by ale neplatilo, že rychlost světla je právě c, tak Maxwellovy rovnice přestávají být vůči LT invariantní, čili elektromagnetické děje dopadnou v různých vztažných soustavách jinak. V principu tedy lze najít privilegovanou vzatžnou soustavu a tím pádem STR nefunguje.


No Lorenzova transformace zůstane s teoretickou krajní rychlostí jako c.

Elektromagnetické děje by dopadly pro nás těžko měřitelně maličko jinak.

Myslím, že privilegovanou vztažnou stoustavu lze najít už i podle obecné teorie relativnosti.

Jinak ale, spousta lidí se těmito problémy uživí. Ti, kdo nebudou tohle potřebovat k obživě, použijí vzorec jen tak přesný, jak se uvádí v teorii informace, kde se tvrdí, že přílišná přesnost je na úkor relevance, jak se uvádí v ekonomické teorii racionální neznalosti, která tohle zpřesňuje do výpočtů rentabilit, a do překladatelského oříšku v jednom klasickém českém filmu, kde ta moudrost zněla: "Kolik třešní tolik višní." - Relevantní rozlišení kyselosti by jistě na potřebné úrovni bez rozlišování třešní a višní bylo vyvinuto.


Derelict - 14/11/2009 - 01:15

citace:
citace:
Adolfe, mam tu pro pobaveni par orisku ... ;o)


No hlavně právě díky tomu E=mc^2 by došlo k tomu, že světlo s různými vlnovými délkami by se šířilo i různou rychlostí.


GRB 090423
Red Shift 8,2

http://www.eso.org/public/outreach/press-rel/pr-2009/pr-17-09.html " target=_blank> http://www.eso.org/public/outreach/press-rel/pr-2009/pr-17-09.html


Derelict - 14/11/2009 - 01:28

citace:
No Lorenzova transformace zůstane s teoretickou krajní rychlostí jako c.

Elektromagnetické děje by dopadly pro nás těžko měřitelně maličko jinak.

Myslím, že privilegovanou vztažnou stoustavu lze najít už i podle obecné teorie relativnosti.




Pokud bude rychlost svetla dohodnuta na meritko 50 snarku za jeden zvahlav, budou stale maxwelovy rovnice platit, lorenzova transformace se nezmeni. Oboje je matematicky aparat, kteremu je jedno zda se dosadi rychlost svetla 300 000 km/s nebo dve kdoule. Tim chci zaroven rict, i kdyz by byla hmotnost fotonu limitne blizka nule, je celkova zmena tak nepatrna, ze ji neni mozne temer dopocitat - jestli to vubec bude potreba. Tady se nevychazi z nejake absolutni rychlosti, ale z rychlosti svetla ve vakuu. Coz je vicemene experimentalni vysledek. Jestli bude teoreticka rychlost castice s nulovou hmotnosti o 5 mikrometru za uhersky rok vetsi nez rychlost svetla ma smysl patrne pouze pro overeni platnosti teorii.


HonzaVacek - 14/11/2009 - 01:33

citace:

GRB 090423
Red Shift 8,2



Tak tady jsem teď nějak ztratil nit Asi jsem na tom už jako Adolf


Derelict - 14/11/2009 - 01:38

citace:
citace:

GRB 090423
Red Shift 8,2



Tak tady jsem teď nějak ztratil nit Asi jsem na tom už jako Adolf


Gama Ray Burst, red shift 8,2 ~ 13,1 miliard let. Nekde jsem zaregistroval informaci o potvrzeni mezi rozdilem "priletu" fotonu s vyssi a nizsi energii, ale vypada to, ze bud neumim hledat nebo to byla kachna ;o)


Alchymista - 14/11/2009 - 01:48

citace:
Honza Vacek
Jenomže když se kouknu na naše sluníčko jak se teď chová, tak si nejsem tak moc jistý, že je to ta správná cesta a jestli by se deficit neutrin neměl hledat spíše tam, než ho vysvětlovat oscilacemi neutrin.
Myslíš, ako že niekedy v čase, keď merali neutrínový tok Slnka už dochádzalo v jadre k nejakým "nestabilitám" a tie teraz vyliezajú na povrch ako podivnosti v slnečnej aktivite? Domnievam sa, že pokiaľ by to bola pravda, tak nás čaká klimatický pruser skutočne kapitálnych rozmerov - asi tak na úrovni vyhynutia dinosaurov.

citace:
Adolf - nepřihlášený
Ty pumpy pracují vždy s KOMPRESOREM že?
Obvykle ano, ale nie nevyhnutne, existuje lumpárna zvaná Peltierov jav.


Hmotnosť fotonu - pokiaľ by to malo mať reálny základ, celkom podstatne by to zamotalo celú kozmológiu, pretože temná hmota by sa zrejme stala tak trochu nadbytočnou. Myslím, že by to zároveň zbúralo celú súčasnú stavbu kvantovej fyziky - a to by malo skutočne ďalekosiahle následky.


HonzaVacek - 14/11/2009 - 01:51

citace:
Gama Ray Burst, red shift 8,2 ~ 13,1 miliard let. Nekde jsem zaregistroval informaci o potvrzeni mezi rozdilem "priletu" fotonu s vyssi a nizsi energii, ale vypada to, ze bud neumim hledat nebo to byla kachna ;o)


Aha, on vlastně Adolf tady něco takového dneska nadhodil a myslím, že nedávno jsem o tom někde četl. Nemusí to být kachna. Jenže ani to ještě nemusí odporovat konstantnosti c ve vakuu. Jednak nevíme jaký je mechanismus výchu vlastního GBR a pak ani asi pořádně neznáme mechanismy na kvantové úrovni, které mohou cestou zaření podle vlnové rozseparovat.

Jinak ještě k té klidové hmotnosti fotonu, i kdyby byla jen nepatrně odlišná od nuly, tak by se to měřitelně projevilo. Stačí si napsat, že i pro energii fotonu platí E = mc^2. Normálně se tak počítá jeho hmotnost (neklidová). Pokud by ale jeho klidová hmotnost byla nenulová, musela by se pravá strana rozepsat s relativistickým výrazem pro hmotnost a pak stačí udělat poměr pro dva fotony s různou energií. Klidové hmotnosti vypadnou, takže na nich nebude záležet a jejich rychlosti se budou lišit dost podstatně.


HonzaVacek - 14/11/2009 - 02:00

citace:
Myslíš, ako že niekedy v čase, keď merali neutrínový tok Slnka už dochádzalo v jadre k nejakým "nestabilitám" a tie teraz vyliezajú na povrch ako podivnosti v slnečnej aktivite? Domnievam sa, že pokiaľ by to bola pravda, tak nás čaká klimatický pruser skutočne kapitálnych rozmerov - asi tak na úrovni vyhynutia dinosaurov.


Až tak jsem to nemyslel, ale jde o to, že tok neutrin ze Slunce se měří poměrně krátkou dobu a vlastně ani nevíme jestli intenzita toho toku nemá také nějaké dlouhodobé cykly jako jsou např. cykly sluneční aktivity a je i možné, že tyto cykly spolu i nějak korespondují a a jsou vzhledem k sobě fázově posunuté. Ale to je jenom spekulace


martinjediny - 14/11/2009 - 09:22

Este sa vratim k zrazke atomu s fotonom.

Ak sa atom rozhodne neabsorbovat foton tak potom:
1/ dojde k odrazu fotonu?
2/ zmenia sa vlastnosti fotonu?
3/ zmeni sa enregia atomu? ohreje sa?


MIZ - 14/11/2009 - 10:11

citace:

Adolfe, mam tu pro pobaveni par orisku ... ;o)
Pokud plati E=mc^2, foton majici energii by mel mit zaroven hmotnost. Tedy pokud plati vyraz m=E/c^2. Zaroven, cim vyssi frekvence fotonu, tim vyssi energii prenasi, tim vyssi odpovidajici hmotnost by mel mit. Tedy:
1) Ma foton "hmotnost" nebo ne?
2) Pokud hmotna castice nemuze dosahnout rychlosti svetla, protoze by mela nekonecnou hmotnost, tedy svetlo nemuze existovat, protoze foton by mel nekonecnou hmotnost. Nebo ne ?

Zdůrazním, co už bylo zodpovězeno: Ano, je to tak, ALE: Kromě této zde popsané hmotnosti mají ještě částice tzv. "klidovou hmotnost", to je potřeba rozlišovat. To je ta hmotnost, která narůstá, pokud částici urychluji vůči mé vztažné soustavě.
E=mc^2 samozřejmě platí, už přes 100 let. Při pohledu na odvození tohoto vztahu však vidíme v předchozí fázi ten pravý důvod převratnosti Einsteinovy teorie relativity: Celková energie tělesa je rovna je ho energii kinetické + KLIDOVÉ energii. Einstein zde světu tvrdí, že energii má nejen letící kámen - tomu snadno uvěřím, když mě trefí - ale také kámen, nehybně ležící přede mnou!
Ano, Einsteinovi Nobelovku za relativitu nedali. Trvalo dlouho, než byl význam jeho teorie doceněn, možná ten proces jejího plného docenění pokračuje ještě dnes. Dnes veškeré naše poznání stojí na dvou pilířích - teorii relativity a kvantové teorii.

Pro začátečníky doporučuji "Pátrání po Schrödingerově kočce" od Johna Gribbina. [Ano, napsal i "Pátrání po Velkém třesku". ] Tato kniha však končí známou Kodaňskou interpretací, dnes už považovanou všeobecně za překonanou. Proto hned pokračujte "Schrödingerovými koťaty".
Pro úplné začátečníky 2 tipy: 1/ Prof. Schrödinger se neproslavil pěstováním koček, jen se po něm pojmenoval takový myšlenkový pokus se zabíjením kočky, kterým vám zatím nebudu motat hlavu. Proslavil se Schrödingerovou rovnicí, ve které vystupuje tzv. vlnová funkce, důležitý to fenomén při pokusech o popis světa. 2/ Až budete číst cokoliv o kvantové teorii nebo vůbec kdykoliv při hloubání v oboru přírodních věd, neptejte se "proč?". Ani "Ku*a, jak je tohle do*le možný?!".


MIZ - 14/11/2009 - 10:18

citace:
Este sa vratim k zrazke atomu s fotonom.

Ak sa atom rozhodne neabsorbovat foton tak potom:
1/ dojde k odrazu fotonu?
2/ zmenia sa vlastnosti fotonu?
3/ zmeni sa enregia atomu? ohreje sa?

1/ Jak jsem už naznačil, já osobně hodlám téma odrazu fotonu bojkotovat.
2/ Ne. Nemají proč.
3/ Ne. Nemá čím.
Je to jednoduché. Je to jako dvojková soustava. Binární situace. Buď ano nebo ne. Pokud foton není atomem pohlcen [ani na chviličku], nic se nestalo, nic se nemění, onen foton s oním atomem pro sebe vzájemně neexistují.


Adolf - nepřihlášený - 14/11/2009 - 22:46

citace:
Keď človek občas číta o pokrokoch vo výskume vlastností rôznych nanomateriálov, napríklad uhlíkových nanotrubíc, tak ten nápad by nemusel byť zasa celkom márny.
Ovšem cena takéhoto materiálu by bola zrejme celkom šialená.


Proto vidím možnou blízkou budoucnost jen u dlouhodobých kosmických letů.


Adolf - nepřihlášený - 14/11/2009 - 23:11

citace:

Nevím, nejsem kvantový fyzik, pouze radiotechnik, ale indukční ohřev bych zrovna kompresí nenazýval. Tam dochází k ohřevu vyvoláním el. proudu, což je v podstatě převod elmag. energie na kinetickou.



Když má masa nositelů elektrických nábojů spády proudit z jednoho místa na druhé, je to také tlakový spád, jako když chce voda téci ve vodovodu. Ten přenost energie se provede na velké množství částic v rámci vekého makroskopickéo celku a vazeb v něm koordinovaně v určitém směru.


Adolf - nepřihlášený - 15/11/2009 - 00:06

citace:

Adolfe, mam tu pro pobaveni par orisku ... ;o)
Pokud plati E=mc^2, foton majici energii by mel mit zaroven hmotnost. Tedy pokud plati vyraz m=E/c^2. Zaroven, cim vyssi frekvence fotonu, tim vyssi energii prenasi, tim vyssi odpovidajici hmotnost by mel mit. Tedy:
1) Ma foton "hmotnost" nebo ne?
2) Pokud hmotna castice nemuze dosahnout rychlosti svetla, protoze by mela nekonecnou hmotnost, tedy svetlo nemuze existovat, protoze foton by mel nekonecnou hmotnost. Nebo ne ?




Foton má vždy hmotnost. I bez komplikací představami o možné klidové hmotnosti. Jeho hmotnost je dána jeho energií. Dokonce díky tomu má i gravitační účinek a při průchodu fotonu gravitačním polem si s ním přerozdělují trochu energie.
Kdyby měl foton nepatrnou klidovou hmotnost musel by se pohybovat těsně pod krajní rychlostí. Podle všech měření zatím rozdíl zjištěn nebyl, ale každé naše měření má jen omezenou přesnost, proto fyzikové určují tzv. maximální možnou hmotnost fotonu - tedy takovou, že její vliv je pod schopnostmi našeho rozlišení měřením. Jestli má foton takovou hmotnost, tak si jejího vlivu při naší přesnosti měření nevšimneme. Zatím každé další měření pořád tuto hmotnost snižuje. Ale v principu, protože přesnost není nekonečná, asi vždy budeme určovat maximální klidovou hmotnost fotonu, pokud fyzika nepřijde s nějakým novým konceptem.

Fyzika jako věda takto musí postupovat. Teoretická teze, že foton má nulovou hmotnost musí být falzifikována a testována na svoji falzifikaci. Musí být určeno, co by se dělo, kdyby ta hmotnost nulová nebyla, a testovat se na jaké míře významnosti to nemusí v nějakém rozsahu platit. Jinak by to nebyla věda ale něco jako vědou kamuflované karbonářské oteplovací náboženství.

Časem se určitě přijde na něco, co nám fyziku pěkně zamíchá a ze současných jejích zákonů udělá jen nějakou podmíněnou aproximaci, jako z Newtonovské fyziky.


Barrymore - 15/11/2009 - 00:24

citace:
citace:

Nevím, nejsem kvantový fyzik, pouze radiotechnik, ale indukční ohřev bych zrovna kompresí nenazýval. Tam dochází k ohřevu vyvoláním el. proudu, což je v podstatě převod elmag. energie na kinetickou.



Když má masa nositelů elektrických nábojů spády proudit z jednoho místa na druhé, je to také tlakový spád, jako když chce voda téci ve vodovodu. Ten přenost energie se provede na velké množství částic v rámci vekého makroskopickéo celku a vazeb v něm koordinovaně v určitém směru.


To je ta relativita - co je to vlastně komprese? Záleží na úhlu pohledu.


Alchymista - 15/11/2009 - 02:25

citace:
Foton má vždy hmotnost. I bez komplikací představami o možné klidové hmotnosti. Jeho hmotnost je dána jeho energií. Dokonce díky tomu má i gravitační účinek a při průchodu fotonu gravitačním polem si s ním přerozdělují trochu energie.

Fotón má energiu - o jeho "hmotnosti" možno uvažovať vždy len v spojení s jeho energiou ale nikdy nie samostatne.
U väčšiny iných elementárnych častíc to možné je, práve preto, že majú kľudovú hmotnosť, môžu existovať v pokoji, ale fotón nie - ten sa musí pohybovať vždy rýchlosťou svetla v danom prostredí a žiadnou inou.

Navyše energia fotonu sa vyjadruje zásadne v tvare E=h*f kde h je planckova konštanta a f je frekvencia.
Pokúšať sa vyjadriť hmotnosť fotónu z rovnice E=mc2 je nezmysel, jednoducho preto, že táto rovnica popisuje niečo celkom iné.

Predstava fotónu ako "objektu" je principiálne mylná.
Fotón je elektromagnetická vlna
a preto je principiálne nemožné sledovať dráhu fotónu medzi miestom, kde vznikol (kde bol vyžiarený) a miestom, kde zanikol (kde bol pohltený). S nenulovou pravdepodobnosťou sa počas cesty medzi zdrojom a detektorom môže nachádzať kdekoľvek v priestore (a to v celom vesmíre!). Z toho istého dôvodu nemá zmysel uvažovať o rýchlosti jednotlivého fotónu pozdĺž jeho dráhy.

Povedané ešte inak - "fotón" je barlička, ktorú si vymysleli fyzici, aby nejak, aspoň trochu normálne, dokázali absorbovať experimentálny fakt, že telesá vyžarujú a prijímajú elektromagnetickú energiu po kvantách úmerných frekvencii. [Upraveno 15.11.2009 Alchymista]


David - 15/11/2009 - 07:07

Kvantová teorie pouze pomáhá pochopit to, že se světlo chová jako hmotné těleso i jako elektromagnetické vlnění.


Adolf - nepřihlášený - 15/11/2009 - 12:29

citace:
Pokúšať sa vyjadriť hmotnosť fotónu z rovnice E=mc2 je nezmysel, jednoducho preto, že táto rovnica popisuje niečo celkom iné.




Ve speciálních úlohách, které se ale snad hodí tak akorát budovatelům modelů vesmíru, se s tím počítat musí. Z energie záření či elektromagnetického pole se takto vypočte hmotnost a z ní plynoucí gravitační účinek. Při některých nepružných absorbcích fotonu taktéž. U fotonu jako "ohnivé koule" v supercolliderech, z něhož pak vznikají spršky částic (jako pár částice antičástice aj.) pochopitelně to rozdělení hmotnosti fotonu do následných částic s klidovou hmotností také má svou cenu. I u superenergetických fotonů, které vždy vzápětí zaniknou za vzniku hmotných částic (tak energetický foton se v našem vesmíru neudrží - možná "ionizace vakua"). Energie fotonů se prostě občas přelije do docela látkových hmot a gravitační působí úplně stejně jako kterákoliv jiná hmota.


Adolf - nepřihlášený - 15/11/2009 - 12:30

citace:
Kvantová teorie pouze pomáhá pochopit to, že se světlo chová jako hmotné těleso i jako elektromagnetické vlnění.


A to samé dělá i látková částice.


martinjediny - 15/11/2009 - 18:01

citace:
...Predstava fotónu ako "objektu" je principiálne mylná.
Fotón je elektromagnetická vlna
a preto je principiálne nemožné sledovať dráhu fotónu medzi miestom, kde vznikol (kde bol vyžiarený) a miestom, kde zanikol (kde bol pohltený). S nenulovou pravdepodobnosťou sa počas cesty medzi zdrojom a detektorom môže nachádzať kdekoľvek v priestore (a to v celom vesmíre!). Z toho istého dôvodu nemá zmysel uvažovať o rýchlosti jednotlivého fotónu pozdĺž jeho dráhy.
...

Takze ak to spojim s doterajsim vysvetlenim. (dufam ze spravne)

Ak atom neabsorboval foton, nema zmysel hovorit o dopade, ci odraze, pretoze foton mohol byt v lubovolnom mieste a mohol atom obist podobne ako pri dvojstrbinovom teste prejde oboma strbinami sucasne.

proste pre seba vzajomne "neexistuju"?


Adolf - nepřihlášený - 15/11/2009 - 21:14

citace:

Ak atom neabsorboval foton, nema zmysel hovorit o dopade, ci odraze, pretoze foton mohol byt v lubovolnom mieste a mohol atom obist podobne ako pri dvojstrbinovom teste prejde oboma strbinami sucasne.

proste pre seba vzajomne "neexistuju"?


Fotonu - tedy vlnové funkci popisující jeho pozorovatelné - se připlete do cesty atom. Existuje pravděpodobnost, že foton bude pohlcen, že bude odražen a že se rozptýlí průchodem přes atom. Tedy vlnový balík se zformuje podle toho. Kus se ho začne šíři zpět, kus dál rozptýleně, a vlnová funkce atomu se také podle toho roztrojí (může to být i složitější). Amplitudy těch vlnových balíků, které budou interferovat samy se sebou (třeba vlna odrahu s vlnou rozptylu aj.). Amplitudy budou určovat pravděpodobnosti pozorovatelných - třebas výskytu, nebo hybnosti. Situace tedy bude existovat v superpozici, odrahu, absorbce a rozptylu dozadu v případně dalších variacích.

Pokud ale dojde k pozorování, za atomem a před atomem bude fotografická deska a najednou se na jedné z desek objeví bod, kde foton absorbovala deska, na druhé desce se takový bod neobjeví, ani atom nebude ve stavu po absorbci, bude ale třeba ve stavu po rozptýlení fotonem, co přes něj prošel, odpovídajícím zčernání na té či oné desce. Ta pozorovaná zčernání a ty stavy atomu se vyskytnou s pravděpodobností danou čtvercem té amplitudy funkce. Když dojde k detekci tohoho výskytu, ta vlnová funkce se zhroutí a přestane existovat.

Tvar toho vlnového balíku je vytvořen podle všech kanálů, kterými částice potenciálně může projít. Ty vlny prošlé různými kanály spolu vzájemně interferují a superponují. Takže balík i jediné částice, která projde sítem, má tvar který ví o všech děrách v sítu. Někde se vlny spolu skládají a jinde spolu interferují. Jsou to vlny pravděpodobnosti. Člověk by si tedy mohl myslet, že ty pravděpodobnosti se budou sčítat. Ale ne sčítají se jejich amplitudy a pravděpodobnost je jejich čtvercem. Dojde-li tedy např. k superpozici dvou vln se stejnou amlitudou, stoupne tam pravděpodobnost pozorovatelné čtyřikrát.



wintermute. - 15/11/2009 - 21:32

Takze este raz foton nieje vlna, foton je castica (a dufam, ze to tu zu budem vysvetlovat naposledy). Bohuzial sposob akym by som vam to bol schopny vysvetli ja, by tu zabral niekolko stran vypoctov a niekolko mesiacov vysvetlovania. Preto si vezmem na pomoc priamo autora kvantovej elektrodynamiky. Na velmi jemny uvod mozem doporucit Feynmanovu prednasku: "Neobycajna teoria svetla a latky". Vysla aj v cestine aj v slovencine. Jedna sa o velmi povrchny uvod do Kvantovej elektordynamiky.

Pred tym nez tu teda vysvetlim v com je problem s ciastocnym odrazom svetla (ktoru matematicky model vlny neodkaze vysvetlit), ktory vysvetluje prave kvantova elektrodynamika, si dovoli citat prave z tejto knihy:


"Chcel bych zduraznit, ze svetlo ma casticovou povahu. Je velmi dulezite vedet, ze svetlo se chova jako proud castic; zvlaste pro ty z vas, kteri si, ze skoly pamatuji, jak vas ucili, ze se svetlo chova ako vlnevi. Ja vam ted rikam jak se chova doopravdy - jako castice."


Podotykam, ze sa jedna o Richarda P. Feynmana drzitela nobelovej ceny, prave za teoriu Kvantovej elektordynamiky. Pomocou tejto teorie bolo mozne aj prvy krat v historii spocitat magneticky moment elektronu. Podotykam, ze vypocitana hodnota bola v neuveritelnej zhode s nameranou hodnotou a dodnes prave hodnota magnetickeho mementu ukazuje na presnost tejto teorie.

Takze dufam, ze sa uz zhodneme na tom, ze foton je castica a nie vlna. Ako som uz napisal predtym pravdaze pomocou modelu vlny dokazete spocitat niektore javy omnoho jednoduchsie ako pomocou QED. Ale je to len velmi zjednoduseny model spravania, ktory napriklad nedokaze vysvetlit ciastocny odraz svetla, uz sa to tu niekto pytal, ze ako ze ko chcem pomocou castice vysvetlit to ze sa vidime v odraze vo vyklade. Som rad ze sa to spytal lebo prave tento problem nedokazeme pomocou vlnovej teorie vysvetlit. Jedina teoria, pomocou ktorej presne pocitate, kolko fotonov sa pri ciastocnom odraze odrazi, je prave kvantova elektordynamika, ktora popisuje spravanie fotonu ako castice. Dovolim si dalsi citat od Feynmana:


"Dnes nemame zadny dobry model mechanizmu castecneho odrazu na dvou rozhranich; pouze pocitame pravdepodobnost toho, ze do daneho fotonasobice dopadne foton odrazeny skelenou deskou. Zvolil sem tento vypocet jako prvni priklad metody, kterou nam poskytuje teorie kvantove elektrodynamiky."


Pre tych co sa pytali, co je problemom ciastocneho odrazu. Problem sa da demonstrovat na nasledojucom pokuse. Pod 90stupnovycm uhlom budeme vystrelovat fotony (svetlo jednej farby) na sklenenu dosku. A budeme ich zachytavat pomocou dvoch fotonasobicov. Jedneho umiestneneho nad sklom v uhle odrazu a druheho "v skle" - to je len pre zjednodusenie, do skla sa prirodzene umiestnit neda (ono to funguje aj pod sklom len to uz je od dvoch povrchov, ale to funguje tiez len by sa o nieco lisilo mnozstvo odrazenych a zachytenych fotonov). Vystrelime postupne jeden po druhom 100 fotonov. 96 z nich zachyti fotonasobic v skle a 4 zachyti fotonasobic nad sklom. Toto je len teoreticky experiment. Prakticky bol prevedeny s dvomi rozhraniami. Ale prikladom jedneho rozhrania je vodna hladina, kde dojde k odrazu na jednom rozhrani.

Ako pomocou modelu vlny vysvetlite, ze sa 4 fotony odrazia do fotonasobica nad sklom?

S ciastocnym odrazom mal problem uz Newton, ktory tiez predpokladal, a spravne, ze svetlo su castice aj ked on k tomu mal zly dovod.

Inac ked niekto stale trva na tom, ze foton je vlna, tak nech navrhne odobratie nobelovej ceny Feynmenovi. Kvantova elektrodynamika (QED) dokaze vysvetlit doposial vsetky zname opticke javy, co sa neda povedat o ziadnej teorii.

Inac ked som hladal vyroky od Feynmana tak som narazil aj na tento:

"Musime odlozit nas zdravy selsky rozum, abychom si udelali predstavu o tom, co se deje v atomech".

Podotykam este, ze dnes uz vysvetluje kvantova elektordynamika vsetky javy okrem gravitacie. Problem je v tom, ze niektore javy nieje mozne touto teoriou spocitat presne, nie ze by to neumoznovala, proste nemame na to dostatocny pocitacovy vykon, preto sa pouzivaju len priblizne vypocty.


Adolf - nepřihlášený - 15/11/2009 - 22:01

"Chcel bych zduraznit, ze svetlo ma casticovou povahu. Je velmi dulezite vedet, ze svetlo se chova jako proud castic; zvlaste pro ty z vas, kteri si, ze skoly pamatuji, jak vas ucili, ze se svetlo chova ako vlnevi. Ja vam ted rikam jak se chova doopravdy - jako castice."

V tom Feynman - největší fyzik 2. poloviny 20. století určitě nekecal. Ovšem z toho vyplývá, že když je foton částice, je i vlnou, jako všechny částice.

Na vysvětlení odrazu ve výloze stačí v prvním přiblížení klasická kvantová úloha průchodu částice potenciálovou bariérou a rozštěpení vlnového balíku na odraženou a procházející složku. Kvantová elektodynamika to úžasně upresňuje, ale v ničem nepopírá. Ty Feynmanovyy částice se neřídí ničím jiným než vlnovou rovnicí.


wintermute. - 15/11/2009 - 22:19

citace:

V tom Feynman - největší fyzik 2. poloviny 20. století určitě nekecal. Ovšem z toho vyplývá, že když je foton částice, je i vlnou, jako všechny částice.



Asi to vzdavam toto nema vyznam.

citace:

Na vysvětlení odrazu ve výloze stačí v prvním přiblížení klasická kvantová úloha průchodu částice potenciálovou bariérou a rozštěpení vlnového balíku na odraženou a procházející složku.



Este raz, tie castice mozete vystrelovat po jednej. A 4 znich sa odrazia. Co to ma s kvantovym tunnelovanim? Ako to vysvetli prave presne 4 zo 100 cez kvantove tunnelovanie?

citace:

Kvantová elektodynamika to úžasně upresňuje, ale v ničem nepopírá. Ty Feynmanovyy částice se neřídí ničím jiným než vlnovou rovnicí.



Ako toto napisat, je ako napisat, ze teoria relativity je fajn, ale iba v podstate uzasne spresnuje Newtonovu mechaniku ale v nicom ju nepopiera.


Adolf - nepřihlášený - 15/11/2009 - 22:40

QED:

Ty částice lze házek jednu po druhé i v klasické QT proti bariéře a 4 se odrazí a 96 projde. Jen QED tu bariéru trochu upřesňuje.

Vůbec bych vztah QED nesrovnáva se vztahem klasické a relativistické mechaniky. Relativistická QT už byla dávno na světě, když ji Feynman použil na elektrodynamiku. Jen pomocí aparátu QT známého už od Diraca definoval částice elektromagnetismu. QED je konstrukt v rámci QT mimo ni či nad ni. Je to něco jako když když pomocí Newtonovské mechaniky vytvořili mechaniku takových objektů, jako jsou tekutiny - speciální forma newtonovských objektů. Je to prostě důsledně kvantová definice elektromagnetismu s následným rozpracováním.


wintermute. - 15/11/2009 - 22:44

citace:
QED:

Ty částice lze házek jednu po druhé i v klasické QT proti bariéře a 4 se odrazí a 96 projde. Jen QED tu bariéru trochu upřesňuje.



Ok, a ked k tomu pridame hrubku skla? Tam mi uz ta bariera nesedi. Aj ked sa priznam, ze som si nie isty ci by vysli tie 4 aj bez toho. Ale to by som si musel spocitat.


wintermute. - 15/11/2009 - 23:11

citace:
citace:
QED:

Ty částice lze házek jednu po druhé i v klasické QT proti bariéře a 4 se odrazí a 96 projde. Jen QED tu bariéru trochu upřesňuje.



Ok, a ked k tomu pridame hrubku skla? Tam mi uz ta bariera nesedi. Aj ked sa priznam, ze som si nie isty ci by vysli tie 4 aj bez toho. Ale to by som si musel spocitat.



Alebo este lepsi priklad. Ako vysvetlite pomocou kvantoveho tulelovania to, ked je druhy povrch vzdialeny niekolko metrov, ze za od prveho odrazi menej fotonov ? A ked bude druhy povrch vzdialeny o meter aj nieco tak uz to nebudu 4 ale 10? alebo 16 a ked zase ho o nieco zvacite tak zase 0 ?


martinjediny - 16/11/2009 - 00:08

Nechcem zhadzovat hlbku vasho sporu,
ale ako strojar pricuchnuvsi k fyzike som si chcel overit moznost
1/ prenosu tepla s chladnejsieho na teplejsie "koncentraciou" ziarenia
co bolo zodpovedane kladne,

2/ zaujimal ma problem uhla dopadu fotonu na plochu. (nemam na mysli sin uhla sklonenej plochy a teda rozlozenie intenzity na vacsiu plochu)
Zaujimal ma prave jeden foton, ktory sa zrazi uz s konkretnym atomom.

Takze ak ide o plochu takmer cierneho telesa, (>0.93) ,tak je uz len otazka, ktoremu atomu v poradi odovzda svoju energiu a to v celku?
A tym teleso ohreje.

Dozvedel som sa spustu veci a na mnohe som ani nevededl, ze sa mam na ne pytat. takze dik.


wintermute. - 16/11/2009 - 19:21

citace:

2/ zaujimal ma problem uhla dopadu fotonu na plochu. (nemam na mysli sin uhla sklonenej plochy a teda rozlozenie intenzity na vacsiu plochu)
Zaujimal ma prave jeden foton, ktory sa zrazi uz s konkretnym atomom.

Takze ak ide o plochu takmer cierneho telesa, (>0.93) ,tak je uz len otazka, ktoremu atomu v poradi odovzda svoju energiu a to v celku?
A tym teleso ohreje.



No tak foton je zvycajne pohlteny elektronom v urcitom pripade aj jadrom. Foton nemoze odovdat cast energie, vzdy odovzda vsetku energiu, takze v celku.

A ktoremu v poradi, no pre konkretny pokus sa da spocitat aj konkretna pravdepodobnost. Existuje aj pravdepodobnost, ze preleti, zalezi na konkretnom materiale a jeho hrubke. Tak isto moze byt pohtlteny a okamzite na to vyziareny. Aj ked fotony a elektorny je asi najlahsie zachytit. Tak isto by som sa asi nebavil o odraze, skor o pohlteni a opatovnom vyziareni.

Ako len zo zvedavosti, potrebujete to kvoli niecomu konkretnemu alebo len tak zo zvedavosti?

Inac pri modernej fyzike je niekedy dolezitejsia otazka ako odpoved.


martinjediny - 16/11/2009 - 21:03

:)

pred pol rokom som si nechal laserom vyrezat ram matrice ir reflektora. na dokoncenie este kus laminovania, povrchovky,... skratka este kus prace, ale nemal som sa k tomu aj pre inu robotu aj som si nebol vysledkom. Mal som predbezne vypocty, ale niektore casti som urobil dost intuitivnea nahrubo, radsej nebudem moje "sedliacke" uvahy prezentovat, i ked tentokrat ste mi potvrdili v com som si nebol isty a postup je v podstate spravny. Moje uvahy boli v podstate ako keby sa tato tema mala vysvetlit 5 rocnemu decku. Kedze tato debata je povzbudiva, tak to skusim dokoncit. Ale najprv vymalujeme detom izbu

Obcas pri citani fyziky si ten ktory experiment aj postavim. Som strojar, ale sirsie obzory nezaskodia.

teraz neviem este 100% ci to bude fungovat a ani ci to bude mat dalsie pouzitie.
Raz sa mi pri gravitacii podarila celkom slusna blamaz. Bola to sice smola nahody, ale radsej som s prezentovanim pokusov opatrnejsi.


Alchymista - 16/11/2009 - 21:24

na niektoré otázky a odpovede z kvantovej a relativistickej fyziky by bolo asi vhodnejšie navštíviť fórum na http://aldebaran.cz/ , kde sa dá nájsť dosť odpovedí a osadenstvo je vo fyzike dobre podkuté, takže dokážu aj celkom rozumne a zrozumiteľne odpovedať.


martinjediny - 16/11/2009 - 22:01

na aldebarane prave mali problem so ziarenim Slnka a dosiahnutim vyssej teploty.
A tam som nevedel rozlisit kto keca a kto tomu rozumie.
Nasiel som aj ine fora ale zdali sa mi byt malo zive.

Tu som si tu otazku dovolil preniest v suvislosti s ISRU.


x - 19/11/2009 - 18:15

Pokladal jsem to v ramci jine odpovedi v jinem vlakne - ale tam se ten dotaz moc nehodil a tak ho pisu sem nebot me to zajima:

Tedy odpoved si tusim jen nevim zdali je tak spravna jak si myslim - tedy zdali se to da brat tak jednoduse jak se domnivam.

Jakou ma vlastne energie teleso pohybujici se po obezne draze - mylim tu dodanou po startu ze Zeme - kdyz teleso - druzice lezi na Zemi - tak bude brat tu kterou ma za vychozi stav - me zajima ten rozdil co ziskal navic.
To je to minimum co proste se bude muset vzdy dodat kazdemu telesu leticimu na obeznou drahu Zeme.
Nemyslim jak efektivne ji doda reaktivni pohon - snad jedine odpor vzduchu behem startu je treba vzit v uvahu.
Ci jste jak je presne s reaktivnim pohonem - energie ziskana spalenim paliva - tedy ta ktera se zuzitkuje spalnnim paliva proti tomu co ziska raketa - tedy i system "prerozadelovani" enrgie mezi spalinami a raketou.
A to ze se zpocatku nese temer jako naprosta vetsina hmotnosti rakety predevsim palivo asi nebude ted brat v uvahu - je to zrejme jen dalsi neefektivita reaktivniho pohonu zalozeneho na spalovani.


ales - 19/11/2009 - 19:01

Pro "x": Poměrně obsáhlé odpovědi jsme Ti už dnes napsali do zdejšího tématu Fyzika kam jsem Tvou otázku (ohledně energií) dopoledne přesunul.


x - 19/11/2009 - 19:04

citace:
Pro "x": Poměrně obsáhlé odpovědi jsme Ti už dnes napsali do zdejšího tématu Fyzika kam jsem Tvou otázku (ohledně energií) dopoledne přesunul.


Prominte, nejak jsem si to nevsiml - trochu jsem spechal a tak jsem si prolitl ted jen jedno tema kam jsem to psal. Dekuji za upozorneni a klidne tam presunte i tento dotaz.


Véna - 20/11/2009 - 13:19

Mám jednu laickou otázku. Máte někdo udělaný přehled, kdy by byla cena za vypuštění rakety / cena za palivo / startovni hmotnost / prazdna hmotnost / hmotnost nákladu ???

Proč se ptám, neb by mě zajímalo, jak vleké procento z ceny rakety je vlastní palivo a jak velke procento je vlastni nosic. Jinymi slovy, zda ma vicepouzitelnost stupnu svuj smysl ... Urcite ma smysl vicestupnovost ...

Diky za info,

Vašek


alamo - 20/11/2009 - 13:38

"Máte někdo udělaný přehled, kdy by byla cena za vypuštění rakety / cena za palivo / startovni hmotnost / prazdna hmotnost / hmotnost nákladu ???"

v snahe nájsť odpoveď na túto otázku, tu bola pred časom založená téma
http://www.kosmo.cz/modules.php?op=modload&name=XForum&file=viewthread&tid=1386
"ekonomika prevádzky nosičov"
vyšumelo to nejak do stratena


ales - 20/11/2009 - 14:03

Solidní přehled výkonů a celkových cen řady raketových nosičů má Vláďa Ajgl na http://www.ajgl.cz/space/Launchers.htm .

Extra cena za palivo tam samozřejmě není, protože výrobci prostě takovéto informace nezveřejňují a "vycucat si něco z prstu" nemá smysl. Obávám se, že zjistit čistou cenu za palivo, může jen někdo, kdo má velmi blízko k nějakému účetnímu z firmy, provozující nosnou raketu. Takových lidí bude velmi málo. Nelze prostě očekávat, že lze snadno zjistit jakoukoliv informaci o čemkoliv. Tohle je prostě příliš těžký a speciální úkol.


-=RYS=- - 20/11/2009 - 14:30

Asi nejlevnejsi 1kg na LEO je za pomoci Lamdy.
http://www.kosmo.cz/modules.php?op=modload&name=kosmo&file=index&fil=/m/nosice/ostatni/lambda/index.htm

Pokud by se Lamda zmodifikovala na dnesni technologie (kompozity, CPU atd.), tak by cena byla jeste mensi.


Miloš Hůla - 20/11/2009 - 14:48

Domnívám se, že o KPL by to nemuselo platit, jedná se o výrobky, které je možné koupit i u nás. Samozřejmě zjistit množstevní slevu při objemech plnění rakety bude větší či menší spekulace, ale např. Linde technoplyny vozí kapalný kyslík do každé nemocnice, letecký petrolej se musí koupit také, a min stlačené helium je také běžně ke koupi. Bude to trochu práce, ale udělat si představu o ceně paliva by bylo zajímavé. Pokud by palivo tvořilo velký podíl, nemohla si NASA původně od STS slibovat řádový pokles přepravních nákladů


x - 20/11/2009 - 18:55

citace:
Solidní přehled výkonů a celkových cen řady raketových nosičů má Vláďa Ajgl na http://www.ajgl.cz/space/Launchers.htm .

Extra cena za palivo tam samozřejmě není, protože výrobci prostě takovéto informace nezveřejňují a "vycucat si něco z prstu" nemá smysl. Obávám se, že zjistit čistou cenu za palivo, může jen někdo, kdo má velmi blízko k nějakému účetnímu z firmy, provozující nosnou raketu. Takových lidí bude velmi málo. Nelze prostě očekávat, že lze snadno zjistit jakoukoliv informaci o čemkoliv. Tohle je prostě příliš těžký a speciální úkol.


Myslim ze cena paliva by se dala zjistit podle cen na trhu -
i kapalny vodik a kyslik se na trhu prodava a podle mne pro raketovy motor nebude zase nejak treba extra ciste - tedy bezna cistota pro technicke ucely - ale nejsem odbornik na raketove motory.
A kolik paliva je v rakete se zhruba snad bezne uvadi.


Vlado 1 - 21/11/2009 - 17:19


Narazil jsem na problém , určit libraci Měsíce z pohledu ze slunce . Já si myslím ,že může byt jen na šířku a to + - 5 st. Milým se ?


JuDu - 28/11/2009 - 15:30

Ahojte,

Mam otazku pre matematikov: ak zoberieme rovnikovy polomer Zeme R(E)=6378,1 km, hmotnost Zeme ako M(E)=5,9736E24 kg, univerzalnu gravitacnu konstantu ako G=6,673E-11 Nm2kg-2, tak vyska geostacionarnej drahy mi vychadza 35866,328 km. Avsak ak si kontrolujem drahy satelitov, ktore kruzia okolo Zeme (napr. Orbitron -om), tak tam su cisla od 35770 km az 35780 km.
Kde som spravil chybu ? Treba zapocitavat aj ine vplyvy ?

Dakujem pekne

Juraj


DH - 28/11/2009 - 16:11

citace:
Ahojte,
Mam otazku pre matematikov: ak zoberieme rovnikovy polomer Zeme R(E)=6378,1 km, hmotnost Zeme ako M(E)=5,9736E24 kg, univerzalnu gravitacnu konstantu ako G=6,673E-11 Nm2kg-2, tak vyska geostacionarnej drahy mi vychadza 35866,328 km. Avsak ak si kontrolujem drahy satelitov, ktore kruzia okolo Zeme (napr. Orbitron -om), tak tam su cisla od 35770 km az 35780 km.
Kde som spravil chybu ? Treba zapocitavat aj ine vplyvy ?
Dakujem pekne
Juraj


Napada me zapocitat zplosteni Zeme (podle meho testovani ukazuje Orbitron vysku satelitu nad povrchem modelu geoidu WGS84, zatimco vy pocitate idealni kouli). Na rovniku je vyboulena, takze Vam vychazi vetsi vyska nez skutecna.

Dalsi vec bude asi neprimy vliv te deformace - gravitacni pole bude jine nez symetricke pole idealni koule. Nad rovnikem bude zrejme silnejsi (je treba uvazoat vyssi efektivni hmotnost Zeme), a proto pro stejnou dobu obehu musi byt satelit dal a mit vyssi rychlost. To uz ale odhaduji. Proste je to realny rozdil mezi idealizovanym modelem a skutecnosti.

Zkuste se pro zajimavost venovat perturbacim drahy. Zejmena u nizsich satelitu (na nizsich drahach) budete prekvapen, jak moc je jejich pohyb nekeplerovsky.


ArX - 28/11/2009 - 17:17

citace:
Ahojte,

Mam otazku pre matematikov: ak zoberieme rovnikovy polomer Zeme R(E)=6378,1 km, hmotnost Zeme ako M(E)=5,9736E24 kg, univerzalnu gravitacnu konstantu ako G=6,673E-11 Nm2kg-2, tak vyska geostacionarnej drahy mi vychadza 35866,328 km. Avsak ak si kontrolujem drahy satelitov, ktore kruzia okolo Zeme (napr. Orbitron -om), tak tam su cisla od 35770 km az 35780 km.
Kde som spravil chybu ? Treba zapocitavat aj ine vplyvy ?

Dakujem pekne

Juraj


Uvazujete jako periodu geostacionarniho satelitu dobu trvani synodickeho dne, coz je cca 23h a 56 min, nebo rovnych 24 h? Velikost chyby by tomu nasvedcovala...


ales - 28/11/2009 - 17:19

citace:
Mam otazku pre matematikov: ak zoberieme rovnikovy polomer Zeme R(E)=6378,1 km, hmotnost Zeme ako M(E)=5,9736E24 kg, univerzalnu gravitacnu konstantu ako G=6,673E-11 Nm2kg-2, tak vyska geostacionarnej drahy mi vychadza 35866,328 km. Avsak ak si kontrolujem drahy satelitov, ktore kruzia okolo Zeme (napr. Orbitron -om), tak tam su cisla od 35770 km az 35780 km.
Kde som spravil chybu ? Treba zapocitavat aj ine vplyvy ?
Domnívám se, že podstatné je počítat s tím, že rotační perioda Země vůči hvězdám není 24 hodin, ale 23,934 hodin, tedy 86164 sekund. Pak už to i s výše uvedenými hodnotami vyjde mnohem lépe (mě to vychází na cca 35786 km).


JuDu - 29/11/2009 - 10:04

Dobry,

Dakujem za odpovede, chyba bola v tom, ze som pocital idealny cas, t.j.: T=24*60*60 sekund za den, pricom spravna hodnota bola T=23.934*60*60. Potom vyska drahy je 35788.845 km. Este raz vdaka.
Rozdiel v casoch bol len 237,6 sekund a vo vyske 77,48 km.
Existuje nejaka literatura, kde bez velkeho obrazkoveho balastu su odvodene vzorce, ktore platia pre pohyb okolo Zeme ? Mam par knih, su zaujimave ale su pre obycajneho citatela, ktory potrebuje hlavne pribeh a nehlada matematicke suvislosti.
Nejaka univerzitna publikacia alebo aj skripta by neboli na zahodenie ..

Esta raz vdaka

Juraj


Barrymore - 15/12/2009 - 08:38

Dobrý den,
měl bych jeden skutečně laický dotaz ohledně přetížení a zrychlení. Pokud se budu pohybovat volným pádem směrem k Zemi (zanedbejme odpor vzduchu), pohybuji se se zrychlením 9,81m/s2, ale nepůsobí na mě přetížení a po dopadu (auvajs) se už nepohybuji se zrychlením, ale působí na mě přetížení 1g. To mi je jasné, ale jak to fyzikálně zdůvodnit?


Tomáš Habala - 15/12/2009 - 09:03

Sila, ktoru vnimas, je vzdy vyslednicou vsetkych sil, ktore na teba v donom okamihu posobia. Ked stojis na Zemi, tak tato vyslednica sa sklada len z pritazlivej sile Zeme (ine sily mozeme v tejto uvahe zanedbat). Ked padas, je vyslednica suctom dvoch sil - pritazlivej, ktora posobi v smere zrychleneho pohybu a zotrvacnej, ktora posobi proti smeru zrychleneho pohybu. Vysledkom je 0.


bejcek - 15/12/2009 - 09:07

Vychází to z prvního Newtonova zákona - zákona setrvačnosti.
Těleso uvedené do pohybu - pádu se samo od sebe nedostane do klidu. Padající těleso nárazem na zem je v okamžiku uvedeno do klidu, to je to auvajs. Při pádu se stále zrychlujeme, působí na nás gravitace - přitažlivá síla naší planety a brzdí nás odpor vzduchu. Pokud máme malé brždění je auvajs. V kosmonautice je velká snaha při návratu - přistání těles se k auvajs nedostat.

Zákon setrvačnosti: Každé těleso setrvává v relativním klidu nebo v rovnoměrném přímočarém pohybu, dokud není přinuceno silovýn působením jiných těles tento stav změnit.


Derelict - 15/12/2009 - 09:49

Trochu jsem se zamotal v terminech. V jakych jednotkach SI je mozne uvest auvajs ? Uaaaa je v m/s, auvajs by melo byt v joulech ?


Barrymore - 15/12/2009 - 09:58

citace:
Trochu jsem se zamotal v terminech. V jakych jednotkach SI je mozne uvest auvajs ? Uaaaa je v m/s, auvajs by melo byt v joulech ?




Auvajs je označení přechodu člověka ze stavu trojrozměrného do stavu dvourozměrného

Jde mi o ty dva ustálené stavy - před auvajs a po auvajs. Při auvajs se kinetická energie spotřebuje na likvidaci jednoho rozměru tělesa (většinou výšky).


Wintermute. - 15/12/2009 - 10:42

citace:
Dobrý den,
měl bych jeden skutečně laický dotaz ohledně přetížení a zrychlení. Pokud se budu pohybovat volným pádem směrem k Zemi (zanedbejme odpor vzduchu), pohybuji se se zrychlením 9,81m/s2, ale nepůsobí na mě přetížení a po dopadu (auvajs) se už nepohybuji se zrychlením, ale působí na mě přetížení 1g. To mi je jasné, ale jak to fyzikálně zdůvodnit?


Okrem toho co tu bolo uvedene ohladom Newtona je mozno pouzit aj vseobecnu teoriu relativity, poste sa len pohybujete rovno v priestore pokym sa nestretnete s nejakym "auvajs" ktory vam v tomto prirodzenom pohybe zacne branit. V okamihu stretu s "auvajs" na vas bude posobit ine pretazenie ako 1g (podstatne vyssie). A to pretazenie sposobi, ze na vas uz naveky bude posobit len 1g.

Ale mozete to zobrat aj z ineho pohladu, proste vy si len tak padate vesmirom a v tom do vas narazi zem, to je to "auvajs". To 1g bude nasledovat potom preto, ze sa nachazate v priestore kde je pre vas prirodzene pokracovat do stredu zeme. Taku deformaciu priestoru sposobila zem, Avsak zem vam v tomto "rovnom" pohybe v prietore brani a to vy pocitujete ako gravitacne zrychlenie. Kym sa nevyskytne ziadne "auvajs" tak si len proste letite "rovno" v priestore a nemate dovod na to pocitovat akekolvek silove ucinky. Ale to co som vam napisal plati len ak ste hmotny bod ak ste teleso tak to plati len do urciteho zakryvenia priestoru. Akonahle by ste sa dostali do blizkosti velmi hmotneho telesa nie ako hmotny bod ale ako clovek tak to "auvajs" by ste pocitili skor ako by ste na to teleso dopadli.

Zhladiska klasickej Newtonky by vas zabilo to, ze na rozne miesta na vasom tele by posobila rozna sila. Napr. keby ste leteli nohami napred (vhodna poloha na to co sa s vami stane) tak na nohy by posobila vacia sila ako na hlavu co by malo za nasledok destrukcny auvajs.

Z hladiska Vseobecnej teorie relativity by vas zabilo pre vas devastujuce zakryvenie priestoru ale s uplne rovnakym priebehom aj vysledkom ako pri pohlade klasickom Newtonovskom.


Wintermute. - 15/12/2009 - 10:55

citace:

Auvajs je označení přechodu člověka ze stavu trojrozměrného do stavu dvourozměrného

Jde mi o ty dva ustálené stavy - před auvajs a po auvajs. Při auvajs se kinetická energie spotřebuje na likvidaci jednoho rozměru tělesa (většinou výšky).



Pri vyssich energiach dokonca do prechodu do stavuje jednorozmerneho (to treba chapat len ako medzistav na prechod do nularozmerneho:). Kedze taka cierna diera dokaze zdeformovat priestor tak, ze v urcitej vzdialenosti od nej by sa zlikvidovali dokonca dva vase rozmery, dalo by sa hovorit len tom aky ste dlhy zvysne dva rozmery by boli nepodstatne. Tomu by sa uz ale mohlo hovorit superauvajs. Dalsou vlastnostou superauvajs je, ze po nom nasleduje automaticky likvidacia aj toho jedneho rozmeru, takzvany prechod do nularozmerneho stavu nazvaneho singularita. Takze zatial co po auvajs pocitujete 1g, po superauvajs uz nepocitujete ziadne g.


ales - 15/12/2009 - 11:05

citace:
... jak to fyzikálně zdůvodnit?
Jde o to, jak velká síla na tebe působí "proti gravitaci". Pokud stojiš na Zemi, tak proti gravitaci působí přesně stejná síla a to je ten "pocit přetížení" (což by mělo být stejné jako "pocit zrychlení"). Pokud volně padáš (ve vakuu), tak proti gravitaci nepůsobí žádná síla a výsledkem je "pocit stavu beztíže" ("pocit zrychlení" v tu chvíli nemusíš mít žádný).


Adolf - 16/12/2009 - 00:16

O co vlastně v této diskus jde? O dvě vzájemně se o tělo přetahujcí síly, jež vyvolávají v těle tlaky, napět atp.?


Alchymista - 16/12/2009 - 20:24

Zobrané do dôsledku - to, čo zabíja po skoku z okna, je mechanické namáhanie dôsledok zrýchlenia (resp. zbrzdenia) pri dopade.
Dopad z ľubovoľnej rýchlosti totiž prebehne v konečnom a nenulovom čase na konečnej dráhe danej rozmermi ľudského tela (krátery v asfalte a betóne sa nevyskytujú a v zemine sú celkom vzácne ).


Barrymore - 18/12/2009 - 08:56

Díky za odpovědi. Už v tom mám jasno.


JuDu - 9/1/2010 - 12:10

Ahojte Vsetci,

Na internete su k dispozicii aktualne satelitove snimky Zeme (napr. http://oiswww.eumetsat.org/IPPS/html/latestImages.html).
Zaujimalo by ma, "kde" sa beru geodeticke body, podla ktoreho urcia presnu orientaciu kamere/satelitu.
Je to asi krizovanie zemepisnych suradnic (sirka a vyska) ale ako sa urci nejaky pevny bod, resp. nula ? Snima sa cely globus a potom geometrickou metodou sa urci stred kruhu ? Je nejaky 'radiomajak' podla ktoreho sa urci nula ?

Dakujem pekne

Juraj


DH - 9/1/2010 - 13:44

citace:
...ale ako sa urci nejaky pevny bod, resp. nula ? Snima sa cely globus a potom geometrickou metodou sa urci stred kruhu ? Je nejaky 'radiomajak' podla ktoreho sa urci nula ?


Nejsem si jisty jestli presne rozumim, ale myslim, ze toto je ukol systemu stabilizace a orientace satelitu. Zakladem je pozice (a natoceni) v "inercialnim prostoru". K tomu se pouziva u druzic predevsim senzor polohy hvezd, ktery fotografuje hvezdnou oblohu a z polohy vyznamnych stalic vzhledem k telesu druzice vypocita presne natoceni telesa druzice vzhledem ke hvezdam. Stred takoveho "inercialniho" systemu je umisten do stredu Zeme.

Zemekoule se v tomto systemu otaci konstantni rychlosti, takze v danem case existuje velmi presna transformace mezi zemepisnou souradnou soustavou a druzicovou soustavou. Jinymi slovy, protoze druzice vi, jak ma na sobe senzory namontovane, vi take presne, kam koukaji (v inercialni i zemepisne soustave). A poridi-li snimek v danem case, neni problem urcit polohu jakehokoliv zemskeho bodu na snimku, i kdyz bude cely povrch zatazen mraky :-)

Krome toho, tyto prepocty pravdepodobne se pravidelne kalibruji (jediny zdroj nepresnosti by mohla byt snad mirne zmenena orientace senzoru vzhledem k senzoru polohy druzice, ale kdovi).


VYFUN - 14/1/2010 - 14:03

Jak lezou na kosmodromu kosmonauti do Sojuzu? Přes orbitální modul?

Pokud ano. Je to pro pomocný personál také dost nepohodlné.


Ervé - 14/1/2010 - 14:50

Z obslužné plošiny vlezou bočními dveřmi do orbitálního úseku a z něj sestoupí do návratové kabiny. Je to trochu nepohodlné, ale ušetřilo to jedny dveře do Sojuzu, tedy významnou váhu. Posádce uvnitř orbitálního modulu pokud vím jeden technik pomáhá - po nástupu a upoutání posádky uzavře poklop mezi kabinou a orbitálním úsekem a pak vyleze a zavře orbitální úsek a aerodynamický kryt.


pospa - 14/1/2010 - 16:16

citace:
Posádce uvnitř orbitálního modulu pokud vím jeden technik pomáhá - po nástupu a upoutání posádky ...
A kde ten technik stojí/visí/leží, když pomáhá poslednímu - třetímu členu posádky s upoutáním do prostřední sedačky? Moc místa už mu v té chvíli nezbývá - existují nějaké fotky z takové procedury?


Ervé - 15/1/2010 - 08:19

To by mně taky zajímalo, žádnou fotku jsem neviděl. Není kdo by to vyfotil a není místo, podle mně leží na podlaze orbitálního modulu a horní polovinu těla má prostrčenou dolů. Pokud si dobře pamatuju, pomocník v BO je uveden v knize Sojuz.


ales - 15/1/2010 - 08:27

Domnívám se, že do Sojuzu se leze velmi podobně jako do čínské lodi Shenzhou (která ze Sojuzu principiálně vychází). Velmi pěkné video z nástupu kosmonautů do lodi Shenzhou 7 je na(je tam vidět i pomocník v orbitálním modulu i sestup kosmonautů do přistávacího modulu).


pospa - 15/1/2010 - 09:35

citace:
Velmi pěkné video z nástupu kosmonautů do lodi Shenzhou 7 -je tam vidět i pomocník v orbitálním modulu i sestup kosmonautů do přistávacího modulu).
Pěkné, ale ...
to "hlavní" tam vidět není - jak ten pomocník pomáhá při upoutávání do sedaček. Myslím, že s tím už si každý z posádky musí poradit sám - délku popruhů mají seřízenou z nácviku předstartovních procedur předem, lehké skafandry Sokol nejsou natlakované, tak snad je to zvládnutelné svépomocí.


Alchymista - 15/1/2010 - 12:57

Tiež si myslím, že s usadením do kresiel si musia kozmonauti poradiť sami - technik v OB im podľa videa pomáha pri vstupe do SA, ale pri usádzaní ich zrejme len kontroluje. Ostatne, s usadením si musia poradiť sami aj pred návratom na Zem.


JuDu - 22/1/2010 - 15:16

Dobry den,

Vedeli by ste mi povedat, co sa stane s velitelskou kabinou lodi Sojuz po pristani ? Je to mozne, ze pristroje sa vymontuju a po kontrole sa posielaju naspat do kozmu ? Kedze kabina je z masivneho materialu (ocel), po pristani sa to rozbije a posle sa naspat do vysokej pece ?

Dakujem pekne

Juraj


martinjediny - 22/2/2010 - 22:43

Ak mion ma zivotnost cca 2,2mikrosekundy, kolko ma zivotnost mionicky atom?


Alchymista - 23/2/2010 - 00:09

Juraj Duducz
Kabína sa neopravuje - bolo by to jednak veľmi pracné a po tepelnom namáhaní, ktorým kabína prejde pri prielete atmosférou, na ňu už nikdy nikto nevystaví "certifikát kvality".
Ale zrejme sa ani priamo nelikviduje - na to zasa nie je rozumný ekonomický dôvod, je to maximálne pár sto kilogramov kovu.
Zopár kabín je v rôznych múzeách na území Ruska a bývalých sovietskych republík a niektoré sú aj v zahraničí, napríklad Sojuz 28 je v Leteckom múzeu Kbely.
Location of Russian Manned Spacecraft http://www.astronautix.com/articles/loccraft.htm

Časť ostatných sa zrejme použije pri rôznych skúškach a testoch, časť môže poslúžiť pri poľnom výcviku a zvyšok? Zrejme existuje niekde nejaký dosť veľký sklad...



Adolf - 23/2/2010 - 00:12

citace:
Ak mion ma zivotnost cca 2,2mikrosekundy, kolko ma zivotnost mionicky atom?


Asi taky zhruba tolik:

http://wikipedia.infostar.cz/m/mu/muonium.html


Vyfun - 23/2/2010 - 09:52

citace:
Dobry den,

Vedeli by ste mi povedat, co sa stane s velitelskou kabinou lodi Sojuz po pristani ? Je to mozne, ze pristroje sa vymontuju a po kontrole sa posielaju naspat do kozmu ? Kedze kabina je z masivneho materialu (ocel), po pristani sa to rozbije a posle sa naspat do vysokej pece ?

Dakujem pekne

Juraj
Já bych tipl, že kabina bude spíš z hliníkových slitin. Dural atd… A někde jsem také četl, že se určité přístroje ze Sojuzů používají znova a znova (navigační systémy).


M: - 23/2/2010 - 10:34

citace:
citace:
Ak mion ma zivotnost cca 2,2mikrosekundy, kolko ma zivotnost mionicky atom?


Asi taky zhruba tolik:

http://wikipedia.infostar.cz/m/mu/muonium.html

Je to mozne, ale viem, ze : "Mimo atomové jádro je neutron nestabilní se střední dobou života 885,7 ± 0,8 sekund[1], přičemž se rozpadá na... " vid. wiki

netusim nakolko sa to da zovseobecnovat a naviac mion nie je v mionickom atome v jadre.
Len moja zvedavost mi nedala nespytat sa ci to ma tiez nejaky vpliv.


Adolf - 23/2/2010 - 10:55

citace:
Je to mozne, ale viem, ze : "Mimo atomové jádro je neutron nestabilní se střední dobou života 885,7 ± 0,8 sekund[1], přičemž se rozpadá na... " vid. wiki

netusim nakolko sa to da zovseobecnovat a naviac mion nie je v mionickom atome v jadre.
Len moja zvedavost mi nedala nespytat sa ci to ma tiez nejaky vpliv.


Neutron je v jádře vždy vázán silnými jadernými interakcemi, a to jej nějak stabilizuje. Mion, ať už je na orbitě či je jádrem, interaguje jen elektromagneticky, což se nijak moc zřetelně na jeho životnosti neprojevuje.


HonzaVacek - 23/2/2010 - 15:09

citace:
netusim nakolko sa to da zovseobecnovat...


Ono se to zevšeobecňovat nedá. Například u zmíněného neutronu to není až tak jednoduché. Není ani pravda, že vázaný neutron v jádře se nerozpadá. To by se dalo říci o stabilních jádrech, ale existují izotopy, kdy dochází k rozpadu i některého z vázaných neutronů. Příkladem může být např. tritium. Na druhou stranu je zde proton. Ten pokud je volný, tak je pravděpodobně stabilní, ale pokud je vázaný v jádře tak může u některých izotopů docházet k tzv. beta+ rozpadu, kdy se některý z protonů jádra přemění na neutron, pozitron a elektronové neutrino.

Jinak srovnávat vázaný neutron v jádře a vázaný mion v elektronovém obalu nelze. Je to tak jak píše Adolf. V atomovém jádře kromě elektro-slabé interakce, která může za beta rozpad, vstupuje do hry ještě silná interakce, která je dominantní, kdežto u mionia hrají úlohu jenom elektromagnetická a slabá interakce a z hlediska samotného rozpadu ten mion asi ani nepozná, že je nějak vázaný. On ten mion má totiž klidovou energii 207x větší než elektron 106 MeV (eletron má 510 keV), a tahle přebytečná energie se při rozpadu odnáší, kdežto vazbové energie elektronů v elektronových obalech jsou o několik řádů nížší.


JuDu - 8/7/2010 - 14:52

Dobry den,

Porovnaval som vysky orbitalnych drah systemu Galileo a GPS. Pise sa,ze satelity sytemu GPS lietaju vo vyske 20200 km nad povrchom Zeme (26600 km od stedu Zeme) a satelity systemu Galileo planuju dat do vysky 23222 km (satellite altitude). Ak vyska Galileo je pocitana od povrchu, mohla by nastat situacia, ze GPS satelity by mohli prijimat signal z Galilea ?
Samozrejme len teoreticky, lebo nemaju senzor na to, ale mohli by ?

Dakujem pekne

Juraj


Jan Bastecky - 8/7/2010 - 16:33

citace:
Dobry den,

Porovnaval som vysky orbitalnych drah systemu Galileo a GPS. Pise sa,ze satelity sytemu GPS lietaju vo vyske 20200 km nad povrchom Zeme (26600 km od stedu Zeme) a satelity systemu Galileo planuju dat do vysky 23222 km (satellite altitude). Ak vyska Galileo je pocitana od povrchu, mohla by nastat situacia, ze GPS satelity by mohli prijimat signal z Galilea ?
Samozrejme len teoreticky, lebo nemaju senzor na to, ale mohli by ?

Dakujem pekne

Juraj


podle mého názoru se mohou přijímat oba systémy navzájem. Anténní systémy obou typů navigačních družic nejsou nijak přesně směrové (není třeba vyzařovací úhel větší než cca 30stupnu).
Pouze musíte při výpočtu polohy vně dráhy druhého systému řešit jiné znaménko u řady korekcí výpočtu.


ales - 26/10/2010 - 22:26

(Posílám dotaz za nick "ekel".)

Dobrý den,

můj možná laický dotaz směřuje k myšlence, zda je možné např. vůz sledovat v reálném čase přes družici/satelit s rozlišením a zoomem umožňující číst např. SPZ ?

Je to vůbec možné, jakým způsobem to lze realizovat a jaká je odhadovaná cena ?

Děkuji za Váš čas i odpověď


Conquistador - 26/10/2010 - 22:48

Možné to určitě je pod správným uhlem družice tak aby nebyla přímo nad vozem (protože by mohla sledovat jen půdorys) ale z uhlu na vůz aby šly vidět boky a také SPZ, pak už jen závisí na výkonu a rozlišení kamerové nebo fotografické aparatury.

Nevím jestli je přímo možné v reálném čase možnost číst SPZ (jestli je tak kvalitní kamera) a nebo až zpětně se záznamu a pozdějším úpravách snímku v speciálních programech na zvýšení kvality a detailů.

Jinak sledovaní automobilu v reálném čase určitě možné je, je k tomu ale zapotřebí vice družic jak jedna, pokud by se jednalo o nějaké dlouhodobé sledování. Takovéhle funkce a operace ale spíše patří pod bezpečnostní a armádní složky, které mají v rozpočtu na takové operace vyčleněné finance (bavíme se o státech jako Rusko, USA, VB, atd.).

Dnes se spíše používá na vyhledávání a sledovaní automobilů a lidi v určitých oblastech bezpilotních prostředku, protože jsou levnější, flexibilnější co se nasazení týče, přecijenom manévrovat a měnit dráhu družice nebo spíše většího počtu špionážních družic je složitější a dražší než měnit kurz bezpilotního prostředku který je k tomu primárně navržen a muže cíl i velké výšky sledovat dlouhodobě.


Alchymista - 27/10/2010 - 04:48

Myslím, že to z družice ešte stále nejde a ani tak rýchlo nepôjde.
Exituje taká fyzikálna nepríjemnosť, zvaná Rayleighovo kritérium pre uhlové rozlíšenie. Vzorček má tvar
sin theta = 1,220 . lambda / D
kde theta je uhol v radiánoch, lambda vlnová dĺžka žiarenia a D priemer objektívu

Výsledok je potom taký, že objektív s priemerom 1 meter dokáže zo vzdialenosti 300km v zelenom svetle 550nm rozlíšiť dva body vzdialené najmenej 0,2m (za ideálnych podmienok) a to na prečítanie značky rozhodne nestačí. Na prečítanie značky by bolo treba rozlíšenie aspoň okolo 0,02m a to znamená objektív o priemere cca 15 metrov (alebo najmenej tri objektívy prepojené ako interferometer so základňami dlhými 15 metrov a následne náročným spracovaním obrazu).

Obraz sa dá trochu vylepšiť počítačovým spracovaním, napríklad dekonvolúciou pomocou rôznych algoritmov a spracovaním série snímok, ale zázraky sa nedejú - v lepšom prípade sa dekonvolúciou dá dosiahnuť rozlíšenie niečo vyše 0,1m (teda nie viac ako 50% zlepšenie), čo je stále na danú úlohu príliš málo, v horšom vzniknú obrazové artefakty a celá práca je zbytočná.


Ervé - 27/10/2010 - 08:29

Myslím že nové KH mají rozlišení kolem 5 cm, ale až po počítačovém zpracování, stejně tak nejsou schopné sledovat jedoucí auto - družice vyfotí sledovanou oblast, data přenese a pak se zpracují, možná rychle, ale kdo ví? Ve filmech to jde, v reálu tomu nevěřím, USA má jen 3 funkční KeyHole, Rusko, Čína ani NATO nemají žádné družice s podobným rozlišením (nejlepší mají tak 0,2 m).


Conquistador - 27/10/2010 - 09:44

špionážní družice muže sledovat cil, nemusi ho jen jednou vyfotit ale i chvili sledovat, zavisi na výšce a sklonu dráhy, ono sice druzice která je výše zemi obíhá pomaleji ale zas potřebujes vykonejší aparaturu a jak poznamenal kolega alchymista, oběktiv by musel byt opravdu obrovský, zatimco družice na velmi nízké orbitě (pravě ty špionážní) mají rozlišení lepší ale cil sledují po kratší ddobu a chtělo by to soustavu vice družic které by jsi sledování předávali.

Takže sledovat vozidlo by šlo, je to přeci jenom celkem velky objekt, ale takovou praci opravdu dnes lepe zastanou bezpilotni prostredky protože je to přeci jenom omnoho levnejší a nasazení je flexibilnější.

Jinak máte pravdu z části je to pouze velká fantazie filmařů.


Alchymista - 27/10/2010 - 18:08

Ervé - možné by to aj bolo, dokonca aj "priamo".

"Bus-1", ktorý je základom pre Key Hole "Misty" alebo "Advanced Crystal", má priemer okolo 4 metrov, aerodynamický kryt na Titan IV používaný pre vynášanie KH má vonkajší priemer okolo 5 metrov. Družica tak môže mať priemer 3,5-4,5 metra, dĺžku 10-15 metrov a váhu do 20ton. Jej objektív alebo hlavné zrkadlo potom môže mať priemer od 2,5 do 4 metrov, podľa toho, ako je to riešené - možností je viac (jj Huble je v princípe prerobená KH, pozerajúca špatným smerom).
S dvojmetrovým objektívom by zo vzdialenosti 300km dalo dosiahnuť rozlíšenie okolo 10 cm, s trojmetrovým okolo 7cm a so štvormetrovým aj uvedených 5cm - počítačovým spracovaním obrazu by sa to dalo ešte ďalej zlepšiť až na 5-6cm u dvojmetrového objektívu a možno 2,5-3cm u štvormetrového.
Je ale treba tiež povedať, že posledné družice série KH majú buď kruhové dráhy vo výške cca 700km, alebo eliptické cca 260x1020km, takže skutočne dosahované optické rozlíšenia budú o čosi horšie.


alamo - 27/10/2010 - 20:27

"..možností je viac (jj Huble je v princípe prerobená KH, pozerajúca špatným smerom). "

a už sa cez Huble, niekto díval na zem?


Ervé - 28/10/2010 - 08:29

Ne, příliš citlivá optika na poškození světlem - na skoro černý Měsíc se HST díval jen po mnoha analýzách a výpočtech, aby nedošlo k poškození odraženým světlem.


-=RYS=- - 28/10/2010 - 09:34

citace:
citace:
Dobry den,

Porovnaval som vysky orbitalnych drah systemu Galileo a GPS. Pise sa,ze satelity sytemu GPS lietaju vo vyske 20200 km nad povrchom Zeme (26600 km od stedu Zeme) a satelity systemu Galileo planuju dat do vysky 23222 km (satellite altitude). Ak vyska Galileo je pocitana od povrchu, mohla by nastat situacia, ze GPS satelity by mohli prijimat signal z Galilea ?
Samozrejme len teoreticky, lebo nemaju senzor na to, ale mohli by ?

Dakujem pekne

Juraj


podle mého názoru se mohou přijímat oba systémy navzájem. Anténní systémy obou typů navigačních družic nejsou nijak přesně směrové (není třeba vyzařovací úhel větší než cca 30stupnu).
Pouze musíte při výpočtu polohy vně dráhy druhého systému řešit jiné znaménko u řady korekcí výpočtu.





Galileo ma celkem 6 dw kanalu v L bandu, pricemz kanal L1 je kompatibilni s GPS. Dohoda konzorcia Galileo a Pentagonu je, ze kanal L1 bude interoperatibilni v ramci obou systemu alespon na urovni, aby se oba systemy vzajemne nerusily. Ale posledni zasedani rady rozhodlo, ze v ramci kompatibility obou systemu se pujde hloubeji a to az tak, ze GPS prijimac by mel zachytit i Galileo na L1 a tim vlastne vice zpresnil pozici, protoze standardni GPS prijimac bude chytat treba 12 druzic GPS a soucasne treba 8 druzic Galileo a obracene, ze prijimac Galileo muze pouzit soucasne jak svuj i L1 kanal, tak i GPS signaly pro vetsi presnost.

Galileo bude pouzivat 6 kanalu v L pasmu a 2 kanaly v C pasmu a NEVADI jina vyska druzic Galilea a GPS a Glonassu (mel jsem v ruce prijimac v kterem je kombinace...dualni prijimac..GPS+Glonass). Reseni teto problematiky tkvi v zakodovani signalu v kterych neni jen "pilot", ale i uzitecna informace podobne jako u casoveho systemu DCF-77 na 77.5kHz, kde v burstu je i 14 bajtu pro PAY komercni pouziti (je to docela levne, kdyz jsem se dival nedavno na cenik). Konkretne kanal L1 u Galilea ma sice stejne kodovane modulovani, ale v binarnim obsahu je trochu jina informace pro vzajemne rozliseni, takze to skutecne nevadi.
Tez GLONASS pouziva L1, ale pouzivaji jinou modulaci...viz druhe doplnkove info v odkazech.

Vice infa zde:
http://ec.europa.eu/dgs/energy_transport/galileo/doc/galileo_stf_ion2002.pdf
http://www.ifen.unibw-muenchen.de/research/signal.htm
http://www.gpsworld.com/gnss-system/galileo/congo-first-gpsgiove-tracking-network-science-research-8835?page_id=1

Doplnujici aplikace:
http://www.gpsworld.com/government/natural-resources/innovation-friendly-reflections-10470?page_id=1
http://www.gpsworld.com/gnss-system/the-system-glonass-forecast-bright-and-plentiful-10580




Bohuzel se system Galileo, respektive jeho kmitoctove rozdeleni kanalu nelibi radioamaterum z celeho sveta. Jde o to, ze kanaly E6 a E6 budou rusit radioamaterske zarizeni vseho druhu i radioamaterske satelity.
Takze v soucasne dobe se v konsorciu resi jeste 6 problemu a jeden z nich je zaruseni radioamaterskeho pasma, ktere je zde na primarni bazi. Zatim se nic s ITU/IARU nedohodlo, ale predpoklad je, ze v extra citlivem useku HAM pasma (SAT, EME, DX provoz) bude mensi vyzareny vykon nebo se tyto vyseky pouzivat nebudou.

Info o problematice zde:
http://www.southgatearc.org/articles/galileo.htm

CTU:
http://www.ctu.cz/cs/download/kmitoctova_tabulka/vyhlaska_105-2010_sb038-10.pdf
http://www.ctu.cz/cs/download/plan-vyuziti-radioveho-spektra/rok_2008/pv-p_20-09_2008-08.pdf


PS:
Rikam si, ze uz se poslalo k Mesici a Marsu dost sond a druzic a je nacase, aby se tam poslala sada kombinovanych navigacne-spojovacich druzic (takove male GPS nebo Galileo), aby byl pokryt celej povrch. Pak by jizda MERu, Curiosity, Lunochodu atd. byla vzdy bezpecnejsi, RYCHLEJSI a tez sekundarne by se dali generovat i "zvlastni" vedecka data na zaklade rozboru navigacnich signalu...viz treba http://radac.nl/UserFiles/File/Brochures/brochure-on%20board%202.pdf)


PS2:
V tom Neubibergu se mluvi taky cesky:
http://ifen.bauv.unibw-muenchen.de/staff/Bartunkova.htm





[Upraveno 28.10.2010 -=RYS=-]


JuDu - 28/10/2010 - 11:14

Dobry den,

Vdaka kazdemu za odpoved.
Ja som predstavil ale taku situaciu, ze nie pozemne prijimace ale samotne satelity aby prijimali signal od "konkurencie". Nemohlo by to sluzit na este presnejsie doladenie drahy alebo zistenie vlastnej polohy ?

Dakujem pekne

Juraj


-=RYS=- - 28/10/2010 - 11:53

citace:
Dobry den,

Vdaka kazdemu za odpoved.
Ja som predstavil ale taku situaciu, ze nie pozemne prijimace ale samotne satelity aby prijimali signal od "konkurencie". Nemohlo by to sluzit na este presnejsie doladenie drahy alebo zistenie vlastnej polohy ?

Dakujem pekne

Juraj


Teoreticky by to technicky slo, ale bohuzel nejde.
Duvod: http://blog.czanso.com/2010/02/prijem-pozice-gps-moduly-a-jejich-omezeni/

PS: Pokud budu delat balonovou sondu, tak radeji pouziji GPS prijimac od Jupitera (http://www.jrmiller.demon.co.uk/projects/ministd/frqstd.htm).
Pokud je vhodny AD ic, tak omezeni vysky/rychlosti neni. Mam zkusenost s AD9850 a zpracovanim celych 100MHz (i kdyz prakticky "hrany" fungovali tak do 60MHz, ale nasledny DSP MC56002 to "pobral"). Pravda, dnes by se to resilo Xilinxem (FPGA), aby upocital signal, jenze to by bylo drahy, takze lepsi je najit vhodny prijimac bez omezeni vysky (Galileo by onen omezujici krok melo mit od 65km).





[Upraveno 28.10.2010 -=RYS=-]


-=RYS=- - 28/10/2010 - 13:18

citace:
Možné to určitě je pod správným uhlem družice tak aby nebyla přímo nad vozem (protože by mohla sledovat jen půdorys) ale z uhlu na vůz aby šly vidět boky a také SPZ, pak už jen závisí na výkonu a rozlišení kamerové nebo fotografické aparatury.

Nevím jestli je přímo možné v reálném čase možnost číst SPZ (jestli je tak kvalitní kamera) a nebo až zpětně se záznamu a pozdějším úpravách snímku v speciálních programech na zvýšení kvality a detailů.

Jinak sledovaní automobilu v reálném čase určitě možné je, je k tomu ale zapotřebí vice družic jak jedna, pokud by se jednalo o nějaké dlouhodobé sledování. Takovéhle funkce a operace ale spíše patří pod bezpečnostní a armádní složky, které mají v rozpočtu na takové operace vyčleněné finance (bavíme se o státech jako Rusko, USA, VB, atd.).

Dnes se spíše používá na vyhledávání a sledovaní automobilů a lidi v určitých oblastech bezpilotních prostředku, protože jsou levnější, flexibilnější co se nasazení týče, přecijenom manévrovat a měnit dráhu družice nebo spíše většího počtu špionážních družic je složitější a dražší než měnit kurz bezpilotního prostředku který je k tomu primárně navržen a muže cíl i velké výšky sledovat dlouhodobě.



Co se tyce toho bezpilotniho prostredku, tak pred 3 rokama testoval kamard z Ladvi (letecky modelar) s klukem co umi navrhnout schema a umi programovat GPS rizeni pres mobil s integrovanym GPS. Model letadla, na zada dal nejaky Siemens PDA s GPSkou a ze systemoveho konektoru vyvedl data do Atmela (AT90xx), ktery daval impulsy do serva prijimace (37MHz Challenger). Spojeni bylo GPRS. Pres vysilacku se letadlo vzneslo do 50m a pak pres notebook (+ PCMCIA Ericsson GPRS) se pripojil k "palubnimu pocitaci" a ten mladik naprogramoval rizeni letadla pres podkladovou mapu (nekde stahl z DVD danou oblast) a textove "rizeni" jen trasu a vysku. Ten Atmel se choval tak, ze na notebooku se zadali souradnice cile + letova vyska a odeslalo se to, pak to Atmel prevzal a doridil letadlo k cili kde pak krouzil (cekal na prevzeti rizeni vysilackou). Ten kluk to mel jako nejakou praci do skoly, koukal jsem na to, zajimave. Kdyz by se slo dal, tak by pres UMTS+EDGE nebo CDMA slo prenaset i zive CGA video (320x270) s nahravanim na cilovem e-booku (nebo "chytrem" telefonu). S tim typkem jsem to pozdeji probiral, ze by se tam mohl dat male modul od Siemense nebo Motoroly (2000Kc) a primo na ten modul by se napojili draty od serv, protoze jadrem modulu byl ARM7 (vyhoda Linuxu). Modul od Motoroly ma 8MB flashe (29f640) +64kb EEPROM (24lc512)+16MB RAMky a komunikacni moznosti quad GSM/GPRS/EDGE + UMTS + 802.11bg a 42 I/O portu (draty od serva, mereni, I2C od live kamery typu pro mobily).
Myslim, ze by si to clovek mohl udelat doma sam, hotove modely letadel se prodavaj (nebo vrtulniku), klicove je naprogramovani Linuxu v letadle a Java aplikace do mobilu, aby clovek netahal e-book/web. Dokonce si myslim, ze kdyz by letadlo bylo dostatecne velke pro fotovoltaiku a elektromoturky, tak by mohl vydrzet ve vzduchu "vecne".
Mozna najdu nekde na disku stare fotky a uverejnim to.


martinjediny - 28/10/2010 - 15:36

citace:
...Dokonce si myslim, ze kdyz by letadlo bylo dostatecne velke pro fotovoltaiku a elektromoturky, tak by mohl vydrzet ve vzduchu "vecne".
Mozna najdu nekde na disku stare fotky a uverejnim to.



Ja som tak prisiel aktivnejsie ku kozmonautike.
Pred par rokmi nieco take robila NASA za tazke miliony. Ja som presvedceny, ze to je bud podvod, alebo pri tom utapaju iny projekt.

Kupil som niektore klucove komponenty, urobil na nich merania a jednozanacne sa da takyto projekt postavit z komercne dostupnych dielov za desatinove ceny. Aj stym, ze vyrobu a projekt si zaplatim.


Dugi - 24/12/2010 - 00:18

Dobrý den,
měl bych dotaz ohledně motorů SRB používaných u raketoplánů. Konkrétně mi jde o technologii jejich zážehu. Předpokládám, že se zřejmě jedná o nějakou hypergolickou reakci, která zažehne palivo, ale zajímalo by mne, jaké chemické látky jsou pro tyto účely použité.
Děkuji Vám předem za odpovědi


Jan Baštecký - 24/12/2010 - 01:05

citace:
Dobrý den,
měl bych dotaz ohledně motorů SRB používaných u raketoplánů. Konkrétně mi jde o technologii jejich zážehu. Předpokládám, že se zřejmě jedná o nějakou hypergolickou reakci, která zažehne palivo, ale zajímalo by mne, jaké chemické látky jsou pro tyto účely použité.
Děkuji Vám předem za odpovědi


SRB se zažehují poměrně komplikovaně. Vlastní zážeh je pomocí plamene od "zažehovacího motoru na TPH" umístěného nad spalovací komorou a nasměrovaného dovnitř odlitého kanálu ve jádře SRB. Cílem je stejnoměrné a současné zažehnutí po celé délce SRB.

Tenhle "zažehovací motor" je teprve před startem odjištěn a natočen do pracovní polohy na středem SRB. Zážeh "zažehovacího motoru" se provádí pyrotechnickou složí, na kterou jsou nasměrovány kanálky od elektrických roznětek.

Co se týče použitých látek, myslím že jde o variace na téma hliník + Ak prášrek OKC \\\\\\\\\\\\\


Dugi - 24/12/2010 - 23:45

Děkuji mnohokrát za odpověď. Ten vlastní "zapalovač" jsem už viděl např. na tomto videu:



Ale nikde jsem o něm nenašel žádné podrobnější informace. Ještě jednou děkuji za pomoc.


eudoxus - 31/12/2010 - 10:51

Mam otazku:
aka misia je povazovana za "deep space"?


JiříHošek - 2/1/2011 - 05:26

http://cs.wikipedia.org/wiki/Deep_Space_1


eudoxus - 2/1/2011 - 08:58

citace:
http://cs.wikipedia.org/wiki/Deep_Space_1

Nemal som na mysli nazov nejakej konkretnej misie, ale obecny terminus technicus. Su to vsetky misie za GEO? Tj. napr. misie k Mesiacu su deep space mission? Alebo deep space zacina v (/za) pasme asteroidov?
Alebo je to cele definovane inak? Trochu som googlil nez som sa spytal, ale nic relevantne som nenasiel. Ten termin sam pouzivam, tak by som v tom chcel mat jasno.


vasek83 - 2/1/2011 - 09:40

citace:
Mam otazku:
aka misia je povazovana za "deep space"?


Apollo 8 byla první deep space mission, tak za LEO a dál, třeba "jen" k Měsíci


raul - 2/1/2011 - 15:18

Co třeba prostor vně sféry gravitačního vlivu země (SOI), což je definováno radiusem 925 tis.km (tedy daleko za orbitem Měsíce ).
http://en.wikipedia.org/wiki/Sphere_of_influence_%28astrodynamics%29

Ale je to asi jen věcí názoru a relativní pojem. Co je 'deep space' pro Zemi, nemusí být zase deep vzhledem ke sluneční soustavě (Heliosféře). Z hlediska současných možností bych to asi vnímal jako prostor vně SOI Země.


admin - 2/1/2011 - 15:38

Nevím přesně, jak by to mělo být, nebo jak se to používá interně v NASA, ESA, ...
Já bych to chápal ve smyslu dělení sluneční soustavy na "inner space"(kamenné planety), "outer space"(od Marsu dál) a "deep space"(za hranicí gravitačního vlivu Slunce).
Tak by mi to vyhovovalo.


Adolf - 2/1/2011 - 17:12

Já osobně pochybuji, že Deep Space je nějaký definovaný pojem. Asi je to nějaké politické zaklínadlo k vytičení zářných zítřků NASA.

Ale existuje jeden pradávný hermetický pojem používaný přinejmenším už Helénistického období starého Řecka. Tehdy ty potvory ve své démonologii a panteonu dělili na bytosti nadměsíční a podměsíční. Tak by se to mohlo z čarování recyklovat do kosmonautiky.


alamo ex - 2/1/2011 - 19:05

"Deep Space"..

mne sa pri tom pojme vybavujú stredoveké mapy, a na nich biele miesta s nápisom "tu sú levi"
"Deep Space" je podľa mňa všetko, čo je mimo trvalú ľudskú prítomnosť
teda aj mesiac.. bez ohľadu na to že "už sme tam boli".. teraz tam nie sme..


Alchymista - 2/1/2011 - 20:25

V pilotovanej kozmonautike je to čokoľvek mimo LEO alebo mimo ochranu zemskej magnetosféry, v nepilotovanej vzdialenosť, ktorá už neumožňuje "priame riadenie" - keď komunikačné oneskorenie presiahne odhadom jednu minútu.


JiříHošek - 2/1/2011 - 21:43

http://iaaweb.org/iaa/Studies/nextsteps.pdf

Tabulka na str. 3 dole.


JuDu - 7/1/2011 - 09:52

Dobry den,

Ak si vylistujete obrazky na i-dnes ( http://technet.idnes.cz/foto.asp?c=A110106_012645_tec_vesmir_vse&r=tec_vesmir&inframe=&strana=1&foto=KUZ324eff_IMG_8286.JPG ) tak mozete najst tuto zaujinavu fotku. Ide o terenne auto (modernejsia verzia ruskeho UAZ-u ?) s protilietadlovym kanonom. Neviem aka je 'SPZ' pre Francusku Guayanu, ale podla obrazka ide o ruske auto ?

Teraz co mam mysliet:
1. pri stavbe kozmodromu bolo potrebne chranit mechanikov
2. Zaujimava verzia cudzio-destrukcneho systemu raketoplanu
3. Omyl, tato fotka by mala patrit niekde inde

Dakujem

Juraj


Ervé - 7/1/2011 - 09:58

Ten kanón je evropské výroby (Oerlikon nebo GIAT), rusové takové nemají, takže bych řekl, že i to auto je francouzské. Buď omyl, nebo pozice francouzské PVO kosmodromu.


Alchymista - 7/1/2011 - 16:24

Ochranu kozmodromu v Kourou má na starosti hlavne francúzska cudzinecká légia (je tam umiestnený 3e REI (pluk), 2e REP/CEA (prieskumná rota?)a 2e REI/4°compagnie). Takže vozidlo, kanón i vojak patria najskôr k nej.

Je to síce malokalibrový protilietadlový kanón (pravdepodobne GIAT, auto je ACMAT VLRA), ale bude súčasťou pozemnej obrany perimetra kozmodromu - je to dnes najčastejšie použitie takýchto zbraní bez automatizovaných zameriavacích systémov (mimoriadne efektívne, mimochodom).
Podobné palebné postavenia (či už pozemné, alebo na vozidle) sa dajú nájsť okolo podobných objektov často, aj keď nebývajú príliš na očiach - obvykle sú umiestnené aj niekoľko kilometrov od objektu v jeho predpolí ("bezpečnostnej zóne"). [Upraveno 07.1.2011 Alchymista]


eudoxus - 7/1/2011 - 18:03

citace:
http://iaaweb.org/iaa/Studies/nextsteps.pdf
Tabulka na str. 3 dole.

Teraz som nasiel informaciu aj v dokumente:
http://www.ok1mjo.com/all/ostatni/space_aircraft/odyssey_telecom.pdf
(odkaz daval pred casom =RYS= 24.12.2010 - 03:49 )
v kapitole 4.1 sa pise:
Deep space generally refers to missions that are farther from the Earth than the Moon (384,000 km).


JuDu - 17/1/2011 - 12:01

Dobry den,

Mam trosku nezvycajnu otazku. Podla Vas kedy je najlepsi okamih na vyfotografovanie geostacionarnych satelitov ? Kedze satelity maju anteny stale natocene k Slnku, asi najlepsie by bolo to odfotit tesne pred prechodom satelitov do tiene Zeme, nie ? Ak je teraz viditelne Slnko na oblohe od 0730 do 1630 (+-) tak by vychadzal cas okolo 1130 - 1145 ?
Vdaka za kazdu (aj kriticku) odpoved.

Juraj


Tomas Habala - 17/1/2011 - 12:25

Geostacionarne druzice obiehaju vo vyske cca 36 000 km nad rovnikom - su nizko nad obzorom. S cim to chcete fotit?


Jan Bastecky - 17/1/2011 - 13:25

citace:
Dobry den,

Mam trosku nezvycajnu otazku. Podla Vas kedy je najlepsi okamih na vyfotografovanie geostacionarnych satelitov ? Kedze satelity maju anteny stale natocene k Slnku, asi najlepsie by bolo to odfotit tesne pred prechodom satelitov do tiene Zeme, nie ? Ak je teraz viditelne Slnko na oblohe od 0730 do 1630 (+-) tak by vychadzal cas okolo 1130 - 1145 ?
Vdaka za kazdu (aj kriticku) odpoved.

Juraj


... ve dne? Na přezářené obloze?

Myslím, že fotovoltaické panely mají stále natočené ke slunci, tedy tady žádné odlesky a výrazné zjasnění nehrozí. Pokud jde o antény, ty jsou pořád natočené k zemi, ale jsou "nerovinné" takže také nebudou poskytivat nějaké výrazné zjasnění.
Tedy půjde o objekty hodně slabé ...

Nicméně je můžeš fotit kdykoliv v noci. Tedy kromě okolí půlnoci poblíž zatmění slunce nebo měsíce (to je vyšší šance, že budou v zemském stínu). V zimě navíc bývá čistější atmosféra.

ještě tip: na webu jsou občas fotky geostacionárních družit, resp. jejich sestav na jednotlivých pozicích (např. na pozici družice Astra je myslím celkem 8 kousků). Lze najít i animaci jejich vzájemného pohybu.

uvedu alespoň jeden odkaz (tečné zatmění skupiny geostacionárních družic):
"http://www.astro.cz/galerie/v/uzivatele/Anto__/20060612_001/2007_04_11+Astra+zatm__n__.avi.html"


ales - 17/1/2011 - 14:02

Geostacionární družice jsou nad obzorem dostatečně vysoko (v ČR přes 30°) a do stínu Země tyto družice skoro nikdy nezapadají (kromě pár hodin kolem půlnoci v době rovnodennosti [kolem 21.3. a 23.9.]). Takže bez obav lze fotit kdykoliv v noci a nejlepší by to mělo být vždy kolem půlnoci ("zimního času") [je jedno, kdy vychází Slunce].


milantos - 17/1/2011 - 17:43

Geostacionární družice je možno fotit prakticky každou noc, a to kdykoliv. Je potřeba si jen uvědomit dosah přístrojů, kterými fotíme, protože jasnost družic ( např. Astra, Hot Bird..) dosahuje v maximu 11-13 mag. ( vyjímečně 8 mag - při reflexu od článků), což představuje 400 -2500x slabší objekty, než hnejslabší hvězdy , pozorovatelné volně očima na tmavé obloze. Ke sledování v dalekohlededu je potřeba dalekohled s průměrem větším než 200 mm.
Pokud jde o zatmění těchto družic, dochází k němu v okolí rovnodenností a to v odbobí +/- 20 dnů. Maximální doba zatmšní je přes 1 hodinu ( v období těsně kolem rovnodennosti). K zatměním nedochází obecně kolem půlnoci , ale v okamžiku, kdy je Slunce na obloze v azimutu o 180° posunuto( = v protipoloze) vůči pozici družice.


-=RYS=- - 19/1/2011 - 04:52

Odfocene a nafilmovane GEO satelity.
19.2stE ASTRA v prubehu 2h.


To same, jen foto.


To same v prubehu 110minut.


A Astra jeste jednou ve fotce.



Nezapomenout, ze druzice Astra na 19.2stE cestuji v prostoru "krychle" o strane 70km.



[Upraveno 19.1.2011 -=RYS=-]


Zbycho - 19/1/2011 - 10:19

citace:
Odfocene a nafilmovane GEO satelity.
19.2stE ASTRA v prubehu 2h.

Nezapomenout, ze druzice Astra na 19.2stE cestuji v prostoru "krychle" o strane 70km.


Prosímtě, mohl bys tento příspěvek (případně podobné další) dát do vlákna "Zajímavé úkazy, viditelnost objektů": http://www.kosmo.cz/modules.php?op=modload&name=XForum&file=viewthread&tid=1304

Možná se k tomu v budoucnu vrátím a určitě bych to nehledal ve vláknu Laické otázky. Děkuji.


JuDu - 19/1/2011 - 10:35

Dobry,

Vdaka kazdemu za odpoved:

- fotit o 1145 - je to moj preklep, myslel som 2345
- nie presne o polnoci - stred medzi vychodom/zapadom Slnka je par minut pred polnocou, nie presne o 23:59:59
- slabe objekty - samozrejme. Mam stojan a predpokladam min. 10 - 15 expozicii, kazdy minutovy. Fotak nemam uplne bez sumu, takze radsej viac kratkych intervalov.

U mna presne na juh je satelit, ktory je cca. 17 stupnov vychodne od Greenwichu. Je to vo vyske asi 34 stupnov na obzorom. Teraz ziari Mesia, takze zatial je len 'teoreticka priprava'.

Este raz vdaka za info od kazdeho.


Jan Bastecky - 19/1/2011 - 12:01

citace:
...
- nie presne o polnoci - stred medzi vychodom/zapadom Slnka je par minut pred polnocou, nie presne o 23:59:59
...


... to neni presne.

Když už to řešíme, tak přesný střed cestuje v důsledku nekruhovitosti oběžné dráhy Zemně kolem slunce. Maximální chyba je cca +-8minut.
Pokud by jste si stavěli sluneční hodiny, pak se křivka průmětu polední (a podobně i půlnoční) polohy slunce v průběhu roku nazývá analema.


milantos - 19/1/2011 - 17:18

Ale pánové, co má půlnoc společného s nejlepší viditelností satelitů, nebo dobou zatmění ? To by platilo pouze pro satelit,který se nám promítá směrem na jih ( to je pro satelit, zavěšený nad 13° východně.) Bude-li družice zavěšena na 40°východně, bude se promítat zhruba 30°od jihu směrem k východu a Slunce bude v protipoloze ( v opozici) již 2 hodiny před půlnocí.
Na videu je pohyb druživ Astra v průběhu 7 3/4 hodiny i s jejich zatměním
http://www.astro.cz/galerie/d/7733-2/13_09_2006+Pohyb+dru__ic+ASTRA.avi
...........
ASTRA 19,2 + průlet Ariane 44 R/B ve vzdálenosti 25.000km


ales - 19/1/2011 - 17:43

Souhlasím s Milanem, že není důležité, kdy má lokální půlnoc pozorovatel, ale jde o to, kdy se GEO družice blíží ke stínu Země. Jediným důvodem pro sledování GEO družice poblíž stínu Země je naděje, že solární panely družic (směřující ke Slunci), budou v té době směřovat i poměrně blízko k pozorovateli a možná by se to mohlo projevit zvýšením jasnosti družice. Moc velkou šanci tomu ale nedávám a je jasné, že to může nastávat jen v období kolem rovnodennosti (jindy družice míjí stín Země ve větší úhlové vzdálenosti). Bez snahy o využití případné zvýšené jasnosti GEO družice je určitě možno je sledovat kdykoliv v noci.


Lukavský - 5/3/2011 - 19:39

Nevíte někdo k čemu je ta složitě tvarovaná vestavba uvnitř aerodynamického krytu? Je vidět ve vlákně Raketoplán X-37 na obrázcích z 4.3. Domníval jsem se, že kryt slouží jen k ochraně nákladu a snížení odporu sestavy při průletu hustými vrstvami atmosféry a k tomu by mohly stačit jen žebrové výztuže pro udržení tvaru. Napadá mne jen protihluková izolace, ale nejsem si jistý.


raul - 6/3/2011 - 00:07

Atlas V používá dva různé aerodynamický kryty ve dvou konfiguracích. 4 m je hliníkový kryt s výztužemi. 5m je kompozitový. Grafit-epoxid na hliníkové kostře. Akustická ochrana na 5m verzi jsou tlumící panely pěny s vloženými tlumícími prvky pro útlum nízkých frekvencí. 5m PLF je v podstatě stejný jako kryt na Ariane V.


Adolf - 16/3/2011 - 20:46

Nevíte někdo, jestli by nešel odněkud z webu stáhnout nějaký Kalmanův filtr?


Jan Baštecký - 17/3/2011 - 10:07

citace:
Nevíte někdo, jestli by nešel odněkud z webu stáhnout nějaký Kalmanův filtr?


Obecný Kalmanův filtr není jednoduchá věc - je totiž vždy vázaný na nějaký systém a jeho vývoj.

V podstatě jde o to, že máte model nějakého systému a postupně naměřená data srovnáváte s vývojem modelu a příslušně opravujete chyby měření. Kalmanův filtr si proto musíte "sestavit" na míru měřenému systému.

pro prvotní inspiraci doporučuji: http://en.wikipedia.org/wiki/Kalman_filter


Jan Baštecký - 17/3/2011 - 10:16

citace:
citace:
Nevíte někdo, jestli by nešel odněkud z webu stáhnout nějaký Kalmanův filtr?


Obecný Kalmanův filtr není jednoduchá věc - je totiž vždy vázaný na nějaký systém a jeho vývoj.

V podstatě jde o to, že máte model nějakého systému a postupně naměřená data srovnáváte s vývojem modelu a příslušně opravujete chyby měření. Kalmanův filtr si proto musíte "sestavit" na míru měřenému systému.

pro prvotní inspiraci doporučuji: http://en.wikipedia.org/wiki/Kalman_filter



... ještě poznamenám, že jednoduché případy odpovídají PID regulátorům a PLL závěsům používaným na strojích. A k tomu se už dá sehnat dostatek informací. Pokud budete bloumat, pak se na mě obraťte a já se pokusím Vám pomoci.


Radek Valter - 28/3/2011 - 15:19

Přírodopis ZŠ

vzpomínáte si někdo na učebnici Přírodopisu pro 4.ročník ZŠ jak tam byl obrázek s váhami kde na jedné misce byl vyfouknutý gumový balonek a na druhé nafouklý? a ten nafouknutý balonek byl na tom obrázku těžší než ten vyfouknutý. tím se tam dokazovalo, že vzduch něco váží. už tehdy jsem si myslel(na rozdíl od učitelky), že je to blbost a myslím si to doteď. vaše názory, popřípadě osobní vzpomínky?



Tomas Habala - 28/3/2011 - 15:24

Dokonca aj keby na jednej miske bola pruzina a na druhej miske taka ista pruzina ale stlacena, tak ta stlacena pruzina by bola tazsia. Stlacena pruzina obsahuje istu potencialnu energiu a podla teorie relativity kazda energia nieco vazi....


derelict - 28/3/2011 - 15:32

citace:
Přírodopis ZŠ

vzpomínáte si někdo na učebnici Přírodopisu pro 4.ročník ZŠ jak tam byl obrázek s váhami kde na jedné misce byl vyfouknutý gumový balonek a na druhé nafouklý? a ten nafouknutý balonek byl na tom obrázku těžší než ten vyfouknutý. tím se tam dokazovalo, že vzduch něco váží. už tehdy jsem si myslel(na rozdíl od učitelky), že je to blbost a myslím si to doteď. vaše názory, popřípadě osobní vzpomínky?



Zde se jedna o chybu ve Vasem uvazovani.Pokud dva balonky vazi stejne, nafouknuty balonek musite povazovat za rozhranim. To ohranicuje oblast ve ktere je vzduch stlacen silou, jenz odpovida napnuti membrany balonku. Pokud budete pocitat, ze vzduch na urovni morske hladiny pri teplote 15C a tlaku 1020hPa bude mit hmotnost 1,1kg/m^3 (snad si to pamatuji dobre), vzduch v balonku musi mit vyssi tlak, tedy i hustotu. Diky tomu je vaha nafouknuteho balonku o neco vetsi.
Jiste nepresnosti se v teto uloze dopustili v jedne veci, neni urceno jak bude balonek nafouknut. Pri vydechu je vzduch opoustejici lidske telo vlhci nez vzduch v okoli. Vlhkost okolo 40% je pri teplote 20C v objemu 1m^3 dalsich cca 9g navic.
Samozrejme, s dostatecne presnymi vahami by jste mohl zmerit i zvyseni vahy dane energii vzduchu a balonku, tepla a dalsic drobnosti, ale to neni obsahem teto otazky.


Vlado1 - 28/3/2011 - 22:03

citace:
Pokud budete pocitat, ze vzduch na urovni morske hladiny pri teplote 15C a tlaku 1020hPa bude mit hmotnost 1,1kg/m^3

Hmotnost 1,1 bude mít při 50 st. C.
Pří teplotě 15 st. je hmotnost 1,3 kg/m3


derelict - 28/3/2011 - 22:06

citace:
citace:
Pokud budete pocitat, ze vzduch na urovni morske hladiny pri teplote 15C a tlaku 1020hPa bude mit hmotnost 1,1kg/m^3

Hmotnost 1,1 bude mít při 50 st. C.
Pří teplotě 15 st. je hmotnost 1,3 kg/m3


Pravda ... priste si to radeji vyhledam.


Radek Valter - 29/3/2011 - 08:14

hmmmm, zajímavé.... pokud tedy půjde o balonek s natlakovanym vzduchem, tak je ho tam víc a musí vážit víc. tak to je asi skutečně chyba v mém uvažování. děkuji za osvětlení temných stránek z dětství :-)
ovšem to s tou pružinou ?? kolik bude ten přírůstek hmotnosti? bude to reálně měřitelné?


Tomas Habala - 29/3/2011 - 08:35

citace:
ovšem to s tou pružinou ?? kolik bude ten přírůstek hmotnosti? bude to reálně měřitelné?

Podla znameho vzorca: E=mc2. Cize m = E/c2.
c2 je dost velka hodnota


Radek Valter - 29/3/2011 - 13:40

takže měřitelné to nebude. pouze teoretiky spočitatelné.


V.J. - 5/4/2011 - 15:28

Měl bych dotaz pro zasvěcené.

ISS klesá cca 100m denně (jsem se někde dočetl..)
Občas se postrčí sama občas ji drcne raketoplán nebo Progres, ATV.
Proč není na ISS nejaký malý iontový motor který by udržolal její dráhu stabilní??? Poměr palivo a impulz by byl výhodnější než jednou za čas motory na plno. Nepočítám že raketoplán nebo Progres tím pálí palivo co mají pro případ problémů navíc.

Díky


Jan Baštecký - 5/4/2011 - 16:10

citace:
Měl bych dotaz pro zasvěcené.

ISS klesá cca 100m denně (jsem se někde dočetl..)
Občas se postrčí sama občas ji drcne raketoplán nebo Progres, ATV.
Proč není na ISS nejaký malý iontový motor který by udržolal její dráhu stabilní??? Poměr palivo a impulz by byl výhodnější než jednou za čas motory na plno. Nepočítám že raketoplán nebo Progres tím pálí palivo co mají pro případ problémů navíc.

Díky


... no jo, jenže když běží motor, tak není stav beztíže a nedá se dělat spousta experimentů ...


Tomas Habala - 5/4/2011 - 16:14

Ja by som skôr povedal, že brzdenie o atmosferu kazí ideálny bezváhový stav. K ideálu by bolo lepšie, keby bol tento vplyv presne vyvažovaný motorom.


Alchymista - 5/4/2011 - 18:39

citace:
Proč není na ISS nejaký malý iontový motor který by udržolal její dráhu stabilní?

1) Energia - Pravdepodobne by ani celý nainštalovaný výkon solárnych panelov nestačil na to, aby uživili iontový motor s potrebným výkonom
2) ISS počas obletu mení priestorovú svoju orientáciu - iontový motor by teda musel byť umiestnený niekde blízko ťažiska - a pohyblivý. Zvyšovanie dráhy Progressom trvá relatívne krátko, takže nutnosť nastaviť ISS do vhodnej polohy je tiež krátkodobá záležitosť.
3) riziko hromadenia elektrického náboja

tuším, že niekde okolo bol príspevok, kde jeho autor počítal potrebný ťah iontového motora a potrebný elektrický príkon a spotrebu pracovného plynu - ale teraz som to nenašiel...


x - 5/4/2011 - 18:49

Je to snad zde:
Ales Holub z 5.2.2010 - 8.39 v Provoz ISS


http://www.kosmo.cz/modules.php?op=modload&name=XForum&file=viewthread&tid=488&pid=54850#pid54850


raul - 5/4/2011 - 22:38

citace:
Měl bych dotaz pro zasvěcené.

ISS klesá cca 100m denně (jsem se někde dočetl..)
Občas se postrčí sama občas ji drcne raketoplán nebo Progres, ATV.
Proč není na ISS nejaký malý iontový motor který by udržolal její dráhu stabilní??? Poměr palivo a impulz by byl výhodnější než jednou za čas motory na plno. Nepočítám že raketoplán nebo Progres tím pálí palivo co mají pro případ problémů navíc.

Díky

Letový model prvního Vasimru MF-200 od AdAstra by měl mít 2 100kW motory a battery pack schopný akumulovat energii z panelů ISS pro 15 minut chodu na plný výkon. Mag. dipóly obou motorů budou navzájem opačně orientovány, aby se rušily a neinteragovaly také s mag. polem země. Součástí bloku bude také radiátor nutný pro chlazení motorů.
O vynesení se uchází Orbital s Taurusem 2 v r.2014 nebo později a upevněn by měl být asi pohyblivě na modulu Unity, orientován zhruba ve směru od Kupole.
Pokud vše vyjde, měly by náklady představovat asi dvacetinu současných 210ml USD ročních nákladů na reboosting ISS.
http://www.adastrarocket.com/aarc/VF200


Alchymista - 5/4/2011 - 23:05

1) Každé "krídlo" solárnych panelov ISS produkuje až 32,8 kW (nové - časom ale degradujú) - osem "krídiel" teda poskytuje teoreticky až 262 kW. Reálne je maximálny výkon solárnych panelov 240-250kW.
2) z 90 minútovej orbity strávi ISS zhruba 35 minút v zemskom tieni - asi 39% času a je napájaná z palubných batérií.

Je teda významné (a nenašiel som):
A) aký je skutočne dostupný priemerný výkon
B) aká časť dostupného výkonu je využívaná na nabíjanie akumulátorov

Alešov výpočet vychádza z predpokladu, že iontové motory by mohli pracovať nepretržite - tak to ale zrejme nebude, naopak je vhodné (výhodné), aby pracovali len na nejakej časti celého obletu (tak funguje i zvyšovanie dráhy pomocou lodí Progress a ATV). To má za následok vyšší potrebný ťah iontových motorov a teda aj ich násobne vyšší príkon.

citace:
Nepočítám že raketoplán nebo Progres tím pálí palivo co mají pro případ problémů navíc
Nie, palivo určené pre zvyšovanie dráhy (a ďalšie manévre - napríklad desaturáciu silových gyroskopov) predstavuje "užitočný náklad" dopravovaný k ISS - "záložné" palivo samotnej nákladnej lode sa počíta osobitne. Hoci ak sa niečo ušetrí pri bezproblémovom stykovom manévri, môže to byť využité aj na ďalšie zvýšenie dráhy stanice, ale nie je to nejak veľa - Progress váži nejakých sedem a pol tony, stanica 55 krát viac.
[Upraveno 05.4.2011 Alchymista]


huno - 12/5/2011 - 15:57

Uz sa niekedy, niektoremu kozmonautovi stalo, ze vo vesmire ochorel?


Jiří Hošek - 12/5/2011 - 21:43

"Další práce byly narušeny onemocněním velitele V. Vasjutina, které kosmonaut 28.10. oznámil řídícímu středisku. Přes veškerou snahu lékařů se na dálku nepodařilo zjistit příčinu Vasjutinových potíží. Navíc se jeho stav výrazně zhoršil, cítil se vyčerpaný a dostavily se značné bolesti břicha. Proto bylo 17.11. rozhodnuto let přerušit. Velitelem posádky byl jmenován V. Savinych a kosmonauti postupně vypnuli většinu aparatury stanice a připravili loď SOJUZ T-14 k návratu."
http://www.kosmo.cz/modules.php?op=modload&name=kosmo&file=index&fil=/m/pil_lety/rusko/sojuz_t/so-t14/index.htm


ales - 12/5/2011 - 22:14

Vasjutin je zatím asi jediným příkladem, kdy musel být kvůli nemoci přerušen (zkrácen) kosmický let. Podle všeho měl Vasjutin nějakou infekci/zánět prostaty nebo močových cest ( http://en.wikipedia.org/wiki/Vladimir_Vasyutin ).

Menší problémy měli i kosmonauti při letu Apollo 7, kteří se v kabině nachladili a dostali rýmu ( http://mek.kosmo.cz/pil_lety/usa/apollo/ap-7/index.htm ). Dalším případem, o kterém vím, je střevní chřipka Bormana a Anderse v Apollu 8 ( http://mek.kosmo.cz/pil_lety/usa/apollo/ap-8/lk.htm ).


Alchymista neprihlasený - 12/5/2011 - 23:44

Ďalším známym prípadom zdravotných problémov je Vasilij Ciblijev, ktorý po zrážke Progress-34 so stanicou Mir začal trpieť kardiovaskulárnymi problémami a srdečnou arytmiou.


Andy - 12/5/2011 - 23:49

V Apollu 13, především Fred Haise, měli také zdravotní problémy. Horečky atd., není se čemu divit, z té vymrzlé lodi...


Jiri87 - 17/5/2011 - 02:31

Mohu se zeptat zdejších odborníků na jejich soukromý odhad, v kterém roce a na jakém stroji poletí poprvé Američané k ISS po odstavení raketoplánů? A dále by mě zajímal váš tip, kdy poletí americká posádka poprvé mimo LEO, kam to bude (Měsíc/asteroid/Mars) a na jaké raketě (NASA/soukr.). Díky


Ervé - 17/5/2011 - 07:24

K ISS podle mně poletí pilotovaný Dragon v roce 2014. V roce 2016 pak Falcon IX Heavy pošle Dragon kolem Měsíce. Ostatní je příliš vzdálené a nejisté. Pokud nástupce Obamy vyhlásí projekt návratu na Měsíc nebo let k asteroidu, mohli by letět v roce 2020. Jenomže při ekonomických potížích USA bych řekl, že projekt letu k asteroidu usne až do roku 2017, takže se poletí 2025 a na Měsíc nebudou peníze až do roku 2030.


admin - 17/5/2011 - 07:34

Já bych dodal, že POPRVÉ amíci letěli mimo LEO 21.12. 1968, viz Apollo 8.


Agamemnon - 17/5/2011 - 10:02

este doplnenie...
ak ta otazka na americanov a iss smerovala iba k ludom a nie vlastnemu letu, tak pojdu na soyuzoch este v tomto roku...
a ak ide o nepilotovany let, tak dragon a mozno dokonca este v tomto roku...


avitek - 17/5/2011 - 14:26

Američané před asi dvěma měsící podepsali s Rusy prodloužení kontraktu na dopravu amerických astronautů na ISS, platné na období 2014-2015 s tím, že to pokrývá dopravu nahoru v roce 2014 a zpět na Zemi od poloviny roku 2014 do poloviny roku 2015. Nedá se však z toho vyvozovat, že v roce 2015 už budou mít USA svůj vlastní pilotovaný kosmický dopravní prostředek.

Termín zprovoznění Dragonu v pilotované verzi v roce 2014 je Muskova idea; zda se to splní je ještě brzy odhadovat. I nákladní dragon má proti plánům z minulého podzimku nejméně 4měsíční zpoždění ...

Jinak původní kontrakt USA-RF za placené služby s dopravou lidí na ISS platí už od začátku loňského roku (2010).


MichalR - 17/5/2011 - 14:36

citace:
Američané před asi dvěma měsící podepsali s Rusy prodloužení kontraktu na dopravu amerických astronautů na ISS, platné na období 2014-2015 s tím, že to pokrývá dopravu nahoru v roce 2014 a zpět na Zemi od poloviny roku 2014 do poloviny roku 2015. Nedá se však z toho vyvozovat, že v roce 2015 už budou mít USA svůj vlastní pilotovaný kosmický dopravní prostředek.

Termín zprovoznění Dragonu v pilotované verzi v roce 2014 je Muskova idea; zda se to splní je ještě brzy odhadovat. I nákladní dragon má proti plánům z minulého podzimku nejméně 4měsíční zpoždění ...

Jinak původní kontrakt USA-RF za placené služby s dopravou lidí na ISS platí už od začátku loňského roku (2010).


Už jsem to psal v sekci Dragon/Falcon:
Elon Musk loni draftoval dva experty; zejémna pak Kena Bowersoxe, bývalého astronauta NASA, který 5x letěl s shuttlem jako pilot či specialista mise, ztrávil půl roku jako velitel na ISS a po usazení na zemi byl ředitelem pro pilotované lety v Johnsově vesmírném centru. Těžko hledat lepšího člověka na pozici člověka odpovědného za plánování a bezpečnost pilotovaných misí.

SpaceX (a tuším že hlavně Elon Musk) má dobrý čich na ty správné lidi. Ono se není co divit, v NASA, zejména pak spousty zkušených expertů v Marshallově a Goddardově centru vesmírných letů) je hodně lidí, kteří "nemají projekt" - není se co divit, NASA nemá dlouhodobou vizi, která by udržela zdravě ambiciozní špičkové lidi.

Tedy, myslím, že přítomnost top expertů v týmu Spacex zajistí, že pilotovaný Dragon nebude průšvih. Možná bych byl opatrnější s odhadem, kdy se jim to celé povede, neboť na vydání licence je zapotřebí spousty a spousty testování a to stojí čas, a nelze očekávat že vše půjde úplně hladce. Ale vůbec nepochybuji o tom, že se jim to povede.


yamato - 17/5/2011 - 15:27

pride mi dost smiesne poukazovat na sklzy u firmy, ktora za 8 rokov postavila 2 nosice, 2 odpalovacie zariadenia a znovupouzitelnu nakladnu lod. Iste, mala sklzy voci svojmu povodnemu planu, ktory vsak bol extremne ambiciozny. Pozrime sa na realitu v aerokozmickom priemysle - za 8 rokov sa dnes ani len nedostane novy motor na testovaci stend, ovela castejsie pocujeme o zruseni projektu. Mimochodom kde je europska Vega? Co je s Angarou? Kde zostal Klipper?
Netvrdim ze v r.2014 poleti pilotovany Dragon, vyvoj presne podla casoveho harmonogramu by bol skor z krajiny zazrakov. Ale je vysoko pravdepodobne ze poleti. A to je dost nezvyk, v porovnani so zvyskom priemyslu


MichalR - 17/5/2011 - 15:59

to yamato: Přesně.

Navíc část vývojářů ve SpaceXu byla zaúkolována na projektu Merlin II, neboť to je vstupenka k zakázkám pro americké vládní agentury s podstatně naditějším rozpočtem, než má jejich chudá příbuzná - NASA.


-=RYS=- - 18/5/2011 - 14:49

citace:
Američané před asi dvěma měsící podepsali s Rusy prodloužení kontraktu na dopravu amerických astronautů na ISS, platné na období 2014-2015 s tím, že to pokrývá dopravu nahoru v roce 2014 a zpět na Zemi od poloviny roku 2014 do poloviny roku 2015. Nedá se však z toho vyvozovat, že v roce 2015 už budou mít USA svůj vlastní pilotovaný kosmický dopravní prostředek.

Termín zprovoznění Dragonu v pilotované verzi v roce 2014 je Muskova idea; zda se to splní je ještě brzy odhadovat. I nákladní dragon má proti plánům z minulého podzimku nejméně 4měsíční zpoždění ...

Jinak původní kontrakt USA-RF za placené služby s dopravou lidí na ISS platí už od začátku loňského roku (2010).


Tady bych souhlasil s panem Vitkem.
Pilotovanej Dragon k ISS nekdy v roce 2014-2015.
Pilotovanej Dragon okolo Mesice nekdy v roce 2015-17.

Ale otazkou je, jak daleko jsou ostatni soukromnici.
Zatim se zpravy zamlcuji. Kazdopadne se Boening na konci dubna poptaval u NASA jak se mohou napojit na TDRS/S...prisla od nich zadost o vygenerovani ID stanice/kryptovacich CCK3 sifer pro sifrovanou TDRS/S komunikaci.
SpaceX zatim o toto nezadadala, zatim komunikaci resi pomoci telemetrie Iridia.
Kazdopadne v povrchove casti Dragona-D (Dragon-C je nakladni verze Dragona) jsou "dipoly"...planarni ploche anteny pro VHF/UHF komunikaci pred startem/po pristani pro komunikaci kosmonautu na VHF-AM kmitoctu s vyhledavacicm vrtulnikem.
Anteny jsou stejne jako u STS soucasti povrchove konstrukce (90% stribro). 6 planarek pro L a S pasmo je soucasti 4 boku Dragona, horni casti (antenni dipol ve viku) a 1 dipol ve spod mezi dlazdicema, cili neco jako uzkej plech, chraneny keramickou dlazdici.
Nekde mam fotku, jak tepelna dlazdice je vyrabena ve fabrice, je to v podstate "tekuta sadra", ktera se vyleje do formy, ale soucasti formy jsou stribrne dipoly. Radiovej signal pronika skrz keramiku, ale ne skrz plazmu pri pristavani.
Soucasti nekolika dipolu je i antena pro vyskovej radar.
Soustava L dipolu je pro spojeni v systemu Iridium.
Jak se zda, je to pro Muska jednodussi/levnejsi reseni nez se slozite napojovat na TDRS/S (slozite se domlouvat s NASA uredniky).
Pochopitelne soucasti Dragonu-D i C bude TDRS/S, ktery se vypne po zadokovani k ISS jako u STS.
Avsak co se tyce Dragona pro spojeni dal nez 40000km od povrchu Zeme...to je otazka.
Zatim NASA nema systemove reseni v podobe spojovacich sond/druzic Mesice/L bodu/HEO druzic.
Jedinne co ma, je DSN sit.
Coz nejen pro Dragon znamena jedinne, aby lod mela vyklopne rameno s antenama jako Apollo.
Pricemz 360st rotator by musel neustale dosmerovavat soustavu anten ke spojovacicm sondam/druzicim.

Z toho plyne, ze stale neexistuje spojovaci si druzic okolo Mesice (neco jako Iridium, ale v S pasmu v podobe S-UMTS, hlavne pro spojeni s povrchovymi aparaty nebo aparaty na nizke-stredni obezne draze Mesice.... aparaty se spoji s "S-UMTS" telco druzicema na vysoke orbite Mesice (odhaduji to na 5000-6000km) podobne jako v pripade Iridia nebo TDRS/S.
Pricemz nekolik spojovacich druzic bude mit ziskovej spojovaci system s pozemnima (Zeme) gatewayema a tez i povrchovejma (Mesic) gatewayema.
V podstate to same silně schazi na Marsu.
Neni tam soustava telco druzic s pozicnim systemem pro povrchove landery/vozitka.

Je otazkou, jestli to Musk pro NASA-DSN neudela-nevynese pomoci FH (byl by zaruceny komercni naklad pro FH, byl by konecne vyuzit).
Dle nekterych msg to tak i vypada, ze Musk pro NASA vynese pomoci FH 18-22 druzic na vysokou telco orbitu Mesice.
Teprve pak by bylo mozno uvazovat o spolehlivem spojovacim a tim i bezpecnym systemu pro obnovene lety na Mesic.

V okamziku, kdy SpaceX nebo nekdo "dalsi" zazada o prideleni ID v TDRS/S systemu budeme vedet, ze se zacina finisovat.


Co se tyce letu k Marsu, tak je to stejny problem jako s Mesicem... schazi tam nekolik S-UMTS multifunkcnich telco druzic, ktere stejne jako s Mesicem budou umet nejen spojit hovory/data, ale i vysilat pozicni data (neco jako GPS/Galileo pro Mesic/Mars) pro pozicni systemy nejen povrchoveho aparatu.

Nejdriv okolo Marsu musime vytvorit sit telco druzic pro funkcni telco komunikaci vcetne GPS pro Mars.

Co se tyce asteroidu/preletu, tak tam pohopitelne telo druzice nebudeme meit, ale pro stale cile jich treba je.

Osobne predpokladam, ze technologie postoupi tak daleko, ze i pro asteroid vytvorime soustavu CubeSatu telco druzic/pozicnich systemu.

Kdo vi, treba Musk vytvori sam soustavu Cubesatu, ktere vyhodi na vysoke obezne drahy Mesice behem prvni "nakladni" cesty na Mesic.

Co se tyce anten, tak treba uz u Iridia-NEXT (druha generace Iridia) se bude vyuzivat malejch (20x20x2cm) plochejch planarek pro spojeni mezi Iridii. Pricemz kazda z 8-mi planarek bude mit v rohu druzice sve rameno z ktereho se anteny budou natacet behem pohybu okolo Zeme, aby se planarky nonstop zamerovali porad na dalsi Iridim druzice.


[Upraveno 18.5.2011 -=RYS=-]


yamato - 18/5/2011 - 15:35

pekny vecny pristup
nadviazem na myslienku vybudovania komunikacnej siete este pred letom (na mesiac, mars, hocikam) - pripada mi logicke postupovat obdobne aj ohladom dopravnej infrastruktury. Ci nie je lepsie zamerat sa na budovanie znovupouzitelnych dopravnych kapacit (mesacny ISRU alulox lander, martansky dvojstupnovy ISRU metalox lander), ktore by boli sice narocne na vyvoj (a mozno ani nie), ale cenu jednotlivej vypravy by zasadne zredukovali. Doterajsi pristup jednorazovej pouzitelnosti a "take all you need with you" je ekonomicky neudrzatelny, aj ked z hladiska pripravy je rychlejsi


Alchymista - 18/5/2011 - 18:51

Problémom telekomunikačných družíc na dráhe okolo Mesiacu je, že tieto dráhy vo všeobecnosti nie sú príliš stabilné. To znamená pomerne časté opravy dráhy, vysokú spotrebu pohonných hmôt a preto relatívne malú životnosť družíc.
Niekde okolo je však pár príspevkov od Alamo, kde sú navrhované osobitné dráhy pre spojovacie družice okolo Mesiacu, ktoré sa do istej miery, niektorými svojimi vlastnosťami, podobajú geostacionárnej dráhe okolo Zeme alebo skôr dráham typu Molnyja.


-=RYS=- - 18/5/2011 - 19:03

citace:
Problémom telekomunikačných družíc na dráhe okolo Mesiacu je, že tieto dráhy vo všeobecnosti nie sú príliš stabilné. To znamená pomerne časté opravy dráhy, vysokú spotrebu pohonných hmôt a preto relatívne malú životnosť družíc.
Niekde okolo je však pár príspevkov od Alamo, kde sú navrhované osobitné dráhy pre spojovacie družice okolo Mesiacu, ktoré sa do istej miery, niektorými svojimi vlastnosťami, podobajú geostacionárnej dráhe okolo Zeme alebo skôr dráham typu Molnyja.


Prece neni problem nabrat Xenon do iontovych motoru, to potom vydrzi hodne dlouho takova druzice Mesice.


Jiří Hošek - 18/5/2011 - 22:32

citace:
Pilotovanej Dragon k ISS nekdy v roce 2014-2015.
Pilotovanej Dragon okolo Mesice nekdy v roce 2015-17.

Ale otazkou je, jak daleko jsou ostatni soukromnici.

Aktuální harmonogramy:

Boeing (CST-100):
2013: Pad abort test
1.pol.2014: Un-crewed orbital flight test
2.pol.2014: Ascent abort test
1.pol.2015: Two-crew orbital flight test
2.pol.2015: Crew transport capable (ISS)

Lockheed Martin (Orion):
2013: Orion flight test 1 (nepilotovaný let do výšky 12000 km, nosič Delta IV-H)
2014: High altitude abort test
2015: Full up unmanned test flight
2016: Crew deep space mission or a lunar flyby (datum závisí na dodání těžkého nosiče)


JakubH - 26/5/2011 - 22:20

Může mi někdo odpovědět, proč trvá raketoplánu sestup od ISS zpátky na Zem asi dva dny, ale Sojuzu jen pár hodin? Díky.


Ervé - 27/5/2011 - 07:20

Protože Sojuz se po oddělení jen vzdálí, zorientuje a zapálí přistávací motor. Na raketoplánu probíhá úklid, balí se skafandry, odpad, kontroluje tepelná ochrana pomocí OBSS a pak se čeká na pozemní vyhodnocení, probíhají experimenty se zážehy motorů pozorovanými ze Země, piloti trénují přistávací manévr, čeká se na dobré počasí na Floridě, aby raketoplán nemusel přistávat na Edwardsu.
Sojuz nemá takovou životnost v samostatném letu, jako raketoplán (zásoby paliva, kyslíku, jídla). Pokud by byly problémy s návratem do atmosféry, musí mít rezervu pro návrat zpět k ISS stejně jako raketoplán. Proto se Sojuz odděluje až v okamžiku, kdy se blíží k návratovému bodu (místo, kdy zapálí brzdící motor a zahájí sestup do atmosféry). Pokud by to bylo třeba, přistál by raketoplán stejně rychle jako Sojuz, pokud by se oddělil zase blízko svému návratovému bodu (pro přistání na KSC nebo Edwardsu). Není to třeba, tak je dobré přípravu návratu řešit až po skončení důležitých prací na ISS a oddělení od ní.


x - 7/6/2011 - 18:11

Je toto pravda - z diskuze na Technetu - IDNES.

"v podstate ano kdybyste mel motor, ktery dokaze letat ve vesmiru (praskovaci letadlo umi jen ve vzduchu) a kdybyste mel palivo, ktere nevazi nepomerne vice nez naklad, ktery vezete (napr. kdybyste takovy stroj napajel ze Zeme, nebo pouzival jaderny reaktor/antihmotovy reaktor), tak muzete stoupat klidne 100km/h porad nahoru a samozrejme ze Zeme odletite, nebo co by vas jako drzelo, kdyz gravitace je porad mensi a mensi a tudiz stoupat je lehci a lehci?

Unikovou rychlost pouzivaji i rakety, ale neni to zadna "konstanta, kterou je nutne prekonat, aby se cokoliv dostalo od Zeme", ale je to protoze nejkritictejsi misto letu do vesmiru (to ceho doopravdy dosahli) je to, ze palivo nutne k vyneseni paliva a nakladu je prilis tezke. Problem byl vyresen pomoci vicestupnovych raket, kdy se hmotnost rakety snizuje tak, ze uz palivo staci, ale ne na trvaly let. Je jednodussi dosahnout unikove rychlosti a vypnout motor nez udrzet po delsi dobu motory a odletet pomalu, ale odletela by i pomalu."

Vim ze v urcite vzadlaenosti je gravitace Zeme zanedbatelna a tak mne zajima co udela to v tomto pripade - stale jsem letel rychlosti 100 km/h od povrchu Zeme - bereme konstatni pro nazornejsi - a v miste s gravitaci Zeme zanedbatelnou ho proste vypnu - tedy snizim na 0 N - poletim tedy k Marsu rychlosti jen 100 km/h ???

Tedy lze letet k Marsu mensi rychlosti nez 2.kosmicka - treba prave z duvodu zachyceni na obezne draze bez brzdeni ???
A k Zemi podobnym stylem - opet z duvodu problemu z brzdenim a tepelnym stitem ???

A nebo radeji jen Mesici - tam je ta orbitalni rychlost kolem Mesice jeste Mensi.

Asi chapu ze je to zrejme mnohem vic energeticky narocnejsi dnes nez 2.kosmicka, ale i tak me to prekvapilo ???


x - 7/6/2011 - 18:14

K tomu doplnim - nejednalo by se v praktickem pouziti o start ze Zeme - ales spise o let z obezne drahy - 1 kosmicka rychlost - xenonovy motor nejspis - ktery by proste pri letu k Mesici i pres trvaly konstatni vykon vubec 2. kosmicke nedosahl ???

Jde to tedy tak - ptam se ???


yamato - 7/6/2011 - 19:16

odporucam trochu sa zahrat s orbiterom, clovek si tu orbitalnu mechaniku lahko predstavi.
Samozrejme ze vo vesmire mozno stupat aj mensou rychlostou ako prva alebo druha kozmicka. Predstavte si raketu, ktora stupa neustale dohora a vobec nenabera horizontalnu rychlost. Pokym bude mat raketa palivo, bude stupat. Ked jej palivo dojde, zacne padat na zem. To plati po celu dobu, kym sa nachadza v gravitacnom poli zeme. Moze teda teoreticky takto vystupat az k mesiacu. Tam zacne posobit gravitacne pole mesiaca a raketa zacne padat na mesiac. V skutocnosti by to nebol kolmy pad, kedze mesiac sa hybe, vysledna trajektoria by teda bola akasi uzka elipsa s periseleniom vo vnutri mesiaca.
K marsu je to zlozitejsie - akonahle by raketa takto pomaly vystupala az von z gravitacneho pola zeme, stava sa obeznicou slnka, s drahou velmi podobnou drahe zeme, od ktorej si nesie hybnost okolo slnka. Takze k marsu opat treba naviest na preletovu trajektoriu. Lenze to skoncime zase s tou druhou kozmickou. Ak by sme chceli k marsu letiet pomalsie, museli by sme zbrzdit hybnost udelenu zemou, a potom motorom posobit proti gravitacnej sile slnka.


derelict - 8/6/2011 - 07:47

Ahoj,
muze mi nekdo pomoci s vysvetlenim?

Vcera mi prisel e-mail nasledujiciho zneni
Space Weather News for June 7, 2011
http://spaceweather.com

MAGNIFICENT ERUPTION: This morning around 0641 UT, magnetic fields above sunspot complex 1226-1227 became unstable and erupted. The blast produced an M2-class solar flare, an S1-class radiation storm, and a massive CME. A recording of the blast from NASA's Solar Dynamics Observatory ranks as one of the most beautiful and dramatic movies of the SDO era. Must-see movies at http://spaceweather.com

AURORA WATCH: High-latitude sky watchers in both hemispheres should be alert for auroras during the late hours of June 8th or 9th when a CME from today's eruption could deliver a glancing blow to Earth's magnetic field. Aurora alerts are available from http://spaceweatherphone.com (voice) or http://spaceweathertext.com (text).

You are subscribed to the Space Weather mailing list, a free service of Spaceweather.com.

New subscribers may sign up for free space weather alerts at http://spaceweather.com/services/ .

Kdyz se divam na klasifikace erupci, jsou tu nasledujici prehledy:
http://www.swpc.noaa.gov/NOAAscales/index.html#SolarRadiationStorms
http://spaceweather.com/glossary/flareclasses.html

Otazky:
1) Ërupce teto sily by nemela zpusobit polarni zari viditelnou v CR, je to pravda?
2) Jaky vliv maji erupce na planovane aktivity EVA a cinnost kosmonautu?
3) Je uz k dispozici modul, kde se mohou kosmonaute "schovat", neco jako radiacni kryt?


Ervé - 8/6/2011 - 08:55

odpověď na dotaz x: gravitační ztráty u Země by byly obrovské - pole sahá hodně daleko. Pro zrychlení z 1. kosmické rychlosti nebo vyšší se to používá u sond poháněných iontovým motorem. Pro přelet k Marsu kromě rychlosti potřebujete uletět i těch nejméně 70 miliónů km mezi Zemí a Marsem, přitom nejefektivnější cestou je Hohmanova elipsa - tedy zase potřeba dosáhnout rychlosti nejméně 11,4 km/s (3,6 km/s pro odlet z LEO). Unášivá rychlost Země kolem Slunce je přes 30 km/s.
Solární bouře: 1. nevím, patrně ne.
2. žádný výstup není plánován - takže patrně budou jen sledovat hodnoty radiace na ISS a v letícím Sojuzu TMA-02M.
3. protiradiační kryt je v podstatě modul Zvezda - ten má zesílené stěny ve 2 spacích kójích. I v některém z ostatních modulů jsou dlouhodobým měřením určená místa, která jsou lépe krytá před radiací a kde by v případě potřeby byli ostatní členové posádky.
Ví někdo podrobnější informace?


derelict - 8/6/2011 - 09:42

citace:
Solární bouře: 1. nevím, patrně ne.

take se obavam, navic bude asi zatazeno. V CR jsem videl polarni zari zatim jenom dvakrat.
citace:

2. žádný výstup není plánován - takže patrně budou jen sledovat hodnoty radiace na ISS a v letícím Sojuzu TMA-02M.
3. protiradiační kryt je v podstatě modul Zvezda - ten má zesílené stěny ve 2 spacích kójích. I v některém z ostatních modulů jsou dlouhodobým měřením určená místa, která jsou lépe krytá před radiací a kde by v případě potřeby byli ostatní členové posádky.
Ví někdo podrobnější informace?


Take by mne zajimaly, spolu s limitem, ktery je schopna ISS "ustat". Mam pocit, ze viditelnost polarni zare v nasich podminkach by mela byt u erupce tridy X. To by asi nejlepe zodpovedel nekdo, kdo se venuje slunci ;o)


bejcek - 8/6/2011 - 11:51

Vysvětlení sluneční činnosti můžete najít na: http://hvezdy.astro.cz/slunce/727-slunce

Sluneční erupce jsou obrovské exploze ve fotosféře. Třídy erupce se značí se písmeny: A,B,C,M,X podle velikosti rtg záření. Nejsilnější jsou M a X. To je hrubá stupnice, třídy se dělí jemněji čísly od 1 do 9. (např M2).
Erupce třídy M a X mohou vyvolat úkazy v okolí Země (polární záře). Ty ovšem budou nyní těžko u nás viditelné, polární záře není ohňostroj a v současné době máme stále světlo v horních částech atmosféry. Máme dlouhé dny, není astronomická noc. (Vidíme pak dobře na družice )
Posádce ISS moc velké nebezpečí z ozáření nehrozí, létá pod ochrannými pásy magnetického pole Země. Letět tak na Měsíc či k Marsu, to by bylo něco jiného.


Ervé - 8/6/2011 - 14:13

Takže podle astro to byla ještě relativně slabá erupce - M2 je 8x slabší než erupce třídy X1, X1 je desetrát slabší než X10, X10 je 10x slabší než X20 naměřená v roce 1989 a 2001 (M2 je tedy 28x slabší než byly tyto), a X20 jsou 10 až 20x slabší než X? zjištěná 2003.


pospa - 8/6/2011 - 15:37

Srovnání velikosti oné erupce se Zemí.
(autor machi)



http://www.unmannedspaceflight.com/index.php?act=attach&type=post&id=24501


Adolf - 10/6/2011 - 08:00

citace:

1) Ërupce teto sily by nemela zpusobit polarni zari viditelnou v CR, je to pravda?



Jde o strašně divnou erupci, o které si badatelé troufají tvrdit, že takovou ještě nikdo nikdy neviděl, což může být způsobeno i tím, že při předchozích událostech nebyly k dispozici takové přístroje, ale možná i tím, že slunce se v tomto cyklu chová divně. Vědátoři jsou z ní paf a budou si možná na této jediné sluneční bouři dělat houfně kariéru. Vzhledem ke krátkému času jsou o rozboru události pouze velice předběžná hodnocení. Populární info o tom jsem ze space.com hodil na Slunce a klima.

Předpověď severní polární záře, jak jsem ji našel u Wattse, zatím nevidím, že by nám tu podívanou dopřála, ale tyhle věci odhadovat dopředu je pořád ještě tak trochu „hádání z koule“.

http://wattsupwiththat.com/reference-pages/geomagnetism/


Ervé - 10/6/2011 - 10:38

Pořád žádné výpadky ani ztráty spojení družic, trefil nás vůbec ten výtrysk?


Adolf - 10/6/2011 - 11:24

citace:
Pořád žádné výpadky ani ztráty spojení družic, trefil nás vůbec ten výtrysk?


Geomagnetická bouře tím vyvolaná je už za námi a nebyla nijak dramatická. Dokonce z těch, co takové předpovědi OPRAVDU dělají, ji nikdo dramatickou nepředvídal.
http://www.gi.alaska.edu/AuroraForecast

Jednou z extrémních zvláštností této sluneční bouře bylo také to, že koronální výron hmoty CME byl sice extrémně veliký, ale s ním spojená erupce nebyla nic moc, v důsledku čehož došlo k velice neobvyklé situaci - nas sluncem se udělal hřibovitý mrak nevídaně gigantických rozměrů, ale místo aby uletěl do vesmíru, jak je při CME standard, skoro všechno zase "vypršelo" na slunce zpátky. Uletěla toho jen trošička, a i když to letělo na Zem, tak to nijak extrémní nebylo, a ta bouře nic moc nebyla.
Viz tyhle křivky solárně-geomagnetické aktivity:

Na sílu té bouře lze usuzovat z té modřé křivky dole.


Ervé - 13/6/2011 - 07:48

Takže 28.5. došlo k mnohem silnější erupci?


Adolf - 13/6/2011 - 09:31

citace:
Takže 28.5. došlo k mnohem silnější erupci?


Zacvičilo to se zemským magnetickým polem víc.

To ovšem neznamená, že samotná erupce byly o moc silnější. Ono to závisí na orientaci a dost i na tom jestli se povede koronální výron hmoty, který nás zasáhne. V takovém případě je to hodně silné. K tomu výronu ale vůbec dojít nemusí anebo ta plazma letí úplně jinak a tady to moc vliv nemá.

Tentokrát došlo ke zvláštnímu jevu, došlo k naprosto extrémnímu koronálnímu výronu hmoty, při čemž tu byla i erupce, která však nebyla nijak mimořádná, a tak neměla sílu odhodit tu strašnou masu plazmy od Slunce. Takže ten hřibovitý mrak nad sluncem zase "vypršel" zpátky. A i když to směřovalo k Zemi, tady se skoro nic nestalo. Ten výron nás asi spíš před tou erupcí ochránil, než aby zesílil její účinek. Byl tu tedy mimořádně velký výron, jen středně silná erupce a jen dost mdlá geomagnetická bouře.

Celé to bylo tentokrát extrémně divné a nikdo tomu moc nerozumí. Možná ty zítřejší objevy o slunci, co chtějí v Novém Mexiku ohlásit, budou o tom.


M: - 16/6/2011 - 15:21

citace:
Myslím, že ... na hranici sféry vlivu Země, ...

preniesol som to sem z inej temy,...

Zijem v predstave, ze ucinok gravitacie Zeme je vypocitatelny kdekolvek vo vesmire. Ale som zvedavy, co si pod "hranici sféry vlivu Země " mam predstavit.


MIZ - 16/6/2011 - 16:22

IMHO se implicitně myslí sféra dominantního vlivu... Tj. až potamtaď, vocamcaď už dominuje Měsíc nebo Slunce nebo jiné těleso.


M: - 16/6/2011 - 16:36

citace:
IMHO se implicitně myslí sféra dominantního vlivu... Tj. až potamtaď, vocamcaď už dominuje Měsíc nebo Slunce nebo jiné těleso.

Asi som sa spytal blbo. tak skusim inak.
mysli sa dominancia:
1:1?
1:10?
1:100?
1:1000000?

Technicky totiz pre let k Mesiacu by mohla byt podstatna dominancia pomaly o dva rady inde jak pre let k Marsu.

Alebo inak. dominancia Zeme konci tam, kde na korekciu drahy mi staci hadi prd?


Alchymista - 16/6/2011 - 18:02

Povedal by som, že v systéme dvoch telies, napríklad Zem-Slnko, je to priestor zhruba medzi bodmi L1 a L2. Tam gravitačne dominuje menšie z oboch telies.
Dá sa to vyjadriť aj tak, že v priestore, kde gravitačne dominuje nejaké teleso, môžeme počítať dostatočne stabilné obežné dráhy okolo tohoto telesa a nemusíme príliš brať ohľad na vplyv iných telies. Napríklad Mesiac - je vzdialený od Zeme 390 000 km a jeho dráha je stabilná v období rádu miliardy rokov. Dá sa teda predpokladať, že jeho dráha leží hlboko v sfére gravitačnej dominancie Zeme a tvrdiť, že táto sfér siaha niekoľkonásobne ďalej, než k dráhe Mesiacu - vzdialenosť k L1 a L2 v systéme Slnko-Zem je 1 500 000 km od Zeme.
[Upraveno 16.6.2011 Alchymista]


ales - 16/6/2011 - 19:47

Sféra vlivu planety se obvykle definuje jako oblast, ve které má planeta dominantní gravitační vliv na pohyb menších kosmických těles v jejím okolí (což už tu správně napsali MIZ a Alchymista). V prvním přiblížení je to tedy ten Martinův poměr 1:1 (např. mezi gravitační silou Země a Slunce). Většinou se uvádí hodnota o něco nižší, než je přesné vyrovnání gravitačních sil s "nadřazeným" tělesem (u Země je tímto tělesem Slunce). Takže pro Zemi se uvádí poloměr sféry vlivu cca 900 000 km, pro Měsíc cca 60 000 km a např. pro Mars cca 600 000 km. Přesněji je to napsáno např. na stránce http://en.wikipedia.org/wiki/Sphere_of_influence_(astrodynamics) . Uvnitř těchto oblastí je možno problém gravitačního působení "n" těles zjednodušit na "problém dvou těles" a tak snadněji počítat oběžné dráhy.

"Hranice sféry vlivu Země" je tedy také (jinak řečeno) vzdálenost od Země, ve které např. sonda přechází z geocentrické dráhy na heliocentrickou dráhu (pak už dráhu sledujeme vzhledem ke Slunci).


Vlado1 - 3/7/2011 - 20:08

Když vypustím ze Země raketu kolmo vzhůru rychlostí 10 km/s , dopadne na stejné místo?
Nepočítám ze sluneční ani s pozemským větrem a ani z odporem vzduchu.


Derelict - 3/7/2011 - 20:15

citace:
Když vypustím ze Země raketu kolmo vzhůru rychlostí 10 km/s , dopadne na stejné místo?
Nepočítám ze sluneční ani s pozemským větrem a ani z odporem vzduchu.


Na to urcite selsky rozum staci. Jen mi prosim odpovez nasledujici otazky:
a) Stala rychlost s jednim impulsem na zacatku?
b) Stala rychlost zajistena pohonem?
c) Zrychleni?


Vlado1 - 3/7/2011 - 20:27

citace:
Na to urcite selsky rozum staci. Jen mi prosim odpovez nasledujici otazky:

SR je SR ale jistota je jistota
a) Stala rychlost s jednim impulsem na zacatku.


martinjediny - 3/7/2011 - 21:29

citace:
citace:
Na to urcite selsky rozum staci. Jen mi prosim odpovez nasledujici otazky:

SR je SR ale jistota je jistota
a) Stala rychlost s jednim impulsem na zacatku.


kedze si blizko unikovej rychlosti, poleti velmi daleko, nez zacne padat spat. urcite uz nebude zanedbatelny vplyv gravitacie inych telies na smer letu.

Alebo otazka smerovala k rotacii Zeme?


martalien2 - 3/7/2011 - 22:23

Nedopadne, protoze se Zeme mezitim pootoci...


Vlado1 - 3/7/2011 - 22:44

citace:
Nedopadne, protoze se Zeme mezitim pootoci...

Ale při startu , je raketě udělená i uhlová rychlost z otáčení Země.


raul - 3/7/2011 - 23:26

citace:
Ale při startu , je raketě udělená i uhlová rychlost z otáčení Země.

Nedostane se ani na úroveň GEO. Spadne tak za 6 hodin o nějakých 10tis km západně, o kterých se Země mezitím pootočí - v případě rovníku. V případě pólu spadnout na stejné místo může.
Úhlová rychlost se rozhodně nezachová. Doporučuji třeba http://cs.wikipedia.org/wiki/Koriolisova_s%C3%ADla [Upraveno 04.7.2011 raul]


Vlado1 - 5/7/2011 - 12:52

Doporučuji třeba http://cs.wikipedia.org/wiki/Koriolisova_s%C3%ADla

To mi moc nepomohlo. Tady jsem se dozvěděl , že vír na hladině , nebo ulita šneků , se má na severu stáčet doleva a na jihu doprava. A ono to tak není. Čmelák, neslyší a neumí číst a tak létá.

To jen tak mimo. Myslím si že pří stoupání nahoru a padání by měl být započten vektor rychlosti otáčení Země ( 465 m/s ) . To je jako výstřel z jedoucího tanku.


martinjediny - 5/7/2011 - 15:51

citace:
Doporučuji třeba http://cs.wikipedia.org/wiki/Koriolisova_s%C3%ADla

To mi moc nepomohlo. Tady jsem se dozvěděl , že vír na hladině , nebo ulita šneků , se má na severu stáčet doleva a na jihu doprava. A ono to tak není. Čmelák, neslyší a neumí číst a tak létá.

To jen tak mimo. Myslím si že pří stoupání nahoru a padání by měl být započten vektor rychlosti otáčení Země ( 465 m/s ) . To je jako výstřel z jedoucího tanku.


Vlado, zem sa otaca okolo zemskej osi uhlovou rychlostou.
Prepocitane podla zemepisnej sirky ziskas rychlost v m/s. (takze na poloch nula)
Ak atmosferu zanedbas, tak smer vystrelenia projektilu a vektor rychlosti z otacania Zeme sa scitaju a mas svoj smer letu.
potom to je uz len balistika, kam dopadne.
akurat, ze Zem sa bude tocit dalej konstatnou uhlovou rychlostou, takze raketa by musela vzhladom k zvacsujucej sa vzdialenosti od osi otacania postupne tangencialne zrychlovata potom brzdit, aby letela stale nad tym istym miestom.

Standardne to ale nepozorujeme, nakolko raketa je pozorovatelna v malych vyskach a tam su uhlove rychlosti este takmer rovnake a navyse rakety obvykle rotaciu predbiehaju, lebo v smere rotacie maju drahu najvyhodnejsiu.

Raul spravne spocital, ze nedoletis ani an GEO a padnes cca za 6 hodin. Ja som len odhadoval, ze to bude v desiatkach hodin a za GEO. hold pocit je pocit. Takze vplyv Mesiaca na drahu nebude nijak podstatny. Zalezi, aky si detailista.


Vlado1 - 5/7/2011 - 18:41

Zalezi, aky si detailista.
Nejsem detailista ale jde mi o fakta a o pravdu nic než pravdu.
Mě vychází že když raketa poletí 6 hod(natočení 90 st.) , tak vektor rychlosti otáčení vynese raketu mimo Zem.
6 x 3600 x 0, 465 = 10044 km R Země je 6378km . Ono seto trochu přikloní k Zemi a zrychlí a tak si myslím že raketa zůstane na oběžné, protáhlé , eliptické dráze. Nebo se mýlím?


martinjediny - 5/7/2011 - 19:13

citace:
Zalezi, aky si detailista.
Nejsem detailista ale jde mi o fakta a o pravdu nic než pravdu.
Mě vychází že když raketa poletí 6 hod(natočení 90 st.) , tak vektor rychlosti otáčení vynese raketu mimo Zem.
6 x 3600 x 0, 465 = 10044 km R Země je 6378km . Ono seto trochu přikloní k Zemi a zrychlí a tak si myslím že raketa zůstane na oběžné, protáhlé , eliptické dráze. Nebo se mýlím?



1/ V tomto pripade nejde o vrh kolmy hore, ale o sikmy vrh vid vektor 10+0,46

2/ Zo ziadneho povrchu telesa sa neda dostat na obeznu drahu bez korekcie drahy tam hore.
Nakresli si zem a elipsu drahy. Tazisko Zeme je ohniskom elipsy, bod navratu hlavnym vrcholom elipsy a elipsa prechadza bodom startu.
Kazda takato elipsa pretne Zem.


Vlado1 - 5/7/2011 - 21:22


1/ V tomto pripade nejde o vrh kolmy hore, ale o sikmy vrh vid vektor 10+0,46
-------------------------------------------
V tomto případě jde o vrh kolmý z pohledu ze Země a dráhu po parabole z pohledu od Slunce.


martinjediny - 6/7/2011 - 00:38

citace:

1/ V tomto pripade nejde o vrh kolmy hore, ale o sikmy vrh vid vektor 10+0,46
-------------------------------------------
V tomto případě jde o vrh kolmý z pohledu ze Země a dráhu po parabole z pohledu od Slunce.


"tedy otáčející se Země) není soustavou inerciální. "

vid
http://cs.wikipedia.org/wiki/Foucaultovo_kyvadlo


Vlado1 - 6/7/2011 - 09:00

"tedy otáčející se Země) není soustavou inerciální. "

vid
----------------------
Inerce –setrvačnost, platí i na Zemi a v celém vesmíru.

Pokud bychom Foucaultovo kyvadlo zavěsili na rovníku, rovina kyvu zůstane vzhledem k ose otáčení Země nezměněna.
Náklon se bude měnít dle ročního období +- 23,5 st..


ales - 6/7/2011 - 10:50

citace:
Nejsem detailista ale jde mi o fakta a o pravdu nic než pravdu.
Mě vychází že když raketa poletí 6 hod(natočení 90 st.) , tak vektor rychlosti otáčení vynese raketu mimo Zem.
6 x 3600 x 0, 465 = 10044 km R Země je 6378km . Ono seto trochu přikloní k Zemi a zrychlí a tak si myslím že raketa zůstane na oběžné, protáhlé , eliptické dráze. Nebo se mýlím?


Kdepak, na tohle se takhle přímočarý výpočet ("selský rozum") moc nehodí. Gravitace silně ovlivní (zmenší) to "vynášení rakety mimo Zem", takže ve skutečnosti raketa samozřejmě dopadne zpět na Zemi. Ano, výsledná dráha bude "protáhlá" a "eliptická", ale současně také bude "suborbitální" (perigeum bude hluboko pod povrchem Země). Výpočet výsledné dráhy je bohužel značně složitý a je při něm třeba převést "stavový vektor" (směr a rychlost v určitém bodu dráhy) na "klasické parametry dráhy" (velkou poloosu, excentricitu, ...). S využitím online převodníku na http://members.verizon.net/~bvsheela/Geocities/vec2ele.html mi (stejně jako Raulovi) vychází, že velká poloosa dráhy bude cca 17700 km a excentricita bude cca 0,9993545. Z toho pak plyne, že oběžná doba této "suborbitální dráhy" bude cca 6,5 hodiny, apogeum bude cca 30000 km nad povrchem a maximální šířka elipsy nepřekročí ani 1500 km (u povrchu Země bude rozdíl průsečíků dráhy ještě mnohem menší). Po dopadu rakety zpět na povrch Země (cca 6 hodin po "startu" z rovníku) tedy už bude Země pootočena mnohem více, než je šířka eliptické dráhy, takže raketa rozhodně nedopadne na místo startu, ani "někam poblíž" (pokud se samozřejmě nestartovalo z pólu).


martinjediny - 6/7/2011 - 13:50

citace:
citace:
Nejsem detailista ale jde mi o fakta a o pravdu nic než pravdu.
Mě vychází že když raketa poletí 6 hod(natočení 90 st.) , tak vektor rychlosti otáčení vynese raketu mimo Zem.
6 x 3600 x 0, 465 = 10044 km R Země je 6378km . Ono seto trochu přikloní k Zemi a zrychlí a tak si myslím že raketa zůstane na oběžné, protáhlé , eliptické dráze. Nebo se mýlím?


Kdepak, na tohle se takhle přímočarý výpočet ("selský rozum") moc nehodí. Gravitace silně ovlivní (zmenší) to "vynášení rakety mimo Zem", takže ve skutečnosti raketa samozřejmě dopadne zpět na Zemi. Ano, výsledná dráha bude "protáhlá" a "eliptická", ale současně také bude "suborbitální" (perigeum bude hluboko pod povrchem Země). Výpočet výsledné dráhy je bohužel značně složitý a je při něm třeba převést "stavový vektor" (směr a rychlost v určitém bodu dráhy) na "klasické parametry dráhy" (velkou poloosu, excentricitu, ...). S využitím online převodníku na http://members.verizon.net/~bvsheela/Geocities/vec2ele.html mi (stejně jako Raulovi) vychází, že velká poloosa dráhy bude cca 17700 km a excentricita bude cca 0,9993545. Z toho pak plyne, že oběžná doba této "suborbitální dráhy" bude cca 6,5 hodiny, apogeum bude cca 30000 km nad povrchem a maximální šířka elipsy nepřekročí ani 1500 km (u povrchu Země bude rozdíl průsečíků dráhy ještě mnohem menší). Po dopadu rakety zpět na povrch Země (cca 6 hodin po "startu" z rovníku) tedy už bude Země pootočena mnohem více, než je šířka eliptické dráhy, takže raketa rozhodně nedopadne na místo startu, ani "někam poblíž" (pokud se samozřejmě nestartovalo z pólu).


V okamihu, ked problem spravne nakreslite, mate 90% riesenia zasebou.

http://www.ulozisko.sk/419487/SR-VLADO.jpg

[URL=http://www.ulozisko.sk/419487/SR-VLADO.jpg][/URL]

K vrcholu drahy sa SR dopracoval spravne.
Ale SR bez taziska Zem zjavne minul. Takto sa na Mesiac skutocne lietat neda...

Sorry, bod G mi usiel. Takze uprava obrazku. [Editoval 06.7.2011 martinjediny]


martinjediny - 7/7/2011 - 09:45

Aby som ta ale neuviedol do omylu, ze SR na vsetko staci,

tak k vrcholu paraboly si sa dopracoval len vdaka zanedbatelne malej tangencialnej zlozke, ktora prakticky kopirovala potencialy gravitacneho pola, a ze dosiahnutu vysku vypocital niekto iny.

http://www.ulozisko.sk/419674/SR-potencial.jpg

[URL=http://www.ulozisko.sk/419674/SR-potencial.jpg][/URL]

Az bude tvorit tangencialna zlozka podstatnu cast rychlosti, budes musiet ratat aj s touto zlozkou v pohybe cez nehomogenne gravitacne pole. A ak nemas rad integraly, tak jedine pocitac a metoda konecnych prvkov.


pav_david - 21/7/2011 - 11:20

dobrý den, mám otázku.

V jiných vláknech se zde několikrát omílalo automatické přistání raketoplánu buran a fakt, že STS to neumí. když jsme nad tím přemýšlel tak jsme došel k názoru , že buran nepřístaval automaticky, ale že byl řízen na dálku ( v poslední fázy přistání), máte k tomu někdo nějaké informace, či nazory?

zajímám se hodně o letectví, ale přiznám se, že nemám přehled jak je to u moderních letadel, často čtu že pilot nemusí šáhnout na řízení po celou dobu letu, ale zaroveň mám pořád pocit že právě start a přistání je stále ponecháváno na pilotovy, že automatika je zajišťuje přiblížení ke konečném přistání, nebo jak se to říká.


Alchymista - 21/7/2011 - 12:55

STS to nedokáže/nedokázal... pretože to američania nechceli. Pritom majú systémy automatického pristátia napríklad na lietadlových lodiach, takže namontovať takýto systém na STS by nebol technicky problém, keby bola vôľa. Neviem presne, či tento systém mali už v čase letu Buranu, každopádne dnes to majú zvládnuté na plne funkčnej úrovni.
Automatické pristátie je obvykle záležitosť spolupráce palubného a pozemného vybavenia - sú to dve spolupracujúce časti toho istého systému ILS - instrumental landing system (viď http://en.wikipedia.org/wiki/Instrument_landing_system).
Plne automatické pristátie so systémom ISL spadá do kategórie ILS IIIB alebo IIIC.

Z technického hľadiska je systém automatického pristátia systémom automatického navedenia na pristátie pri nulovej viditeľnosti, teda schopným viest lietadlo na pristátie až po dotyk s dráhou, s tým, že výstupy nie sú vyvedené na prístroje, z ktorých by ich odčítavali vystresovaný piloti, ale do vstupov chladnokrvného stroja - autopilota. A ručim za to, že mu živí piloti pristátie v minimách vždy milerádi prenechajú... Ak môžu...

Treba povedať, že vybavenie kategórie IIIC nie je v bežnej prevádzke nasadené nikde na svete - podmienky pre kategóriu IIIC totiž vyžadujú nielen automatizované navedenie a pristátie pri nulovej viditeľnosti, ale i automatizované vedenie lietadla po zemi (po pojíždecích dráhach). V prípade Buranu i STS táto časť vybavenia nie je potrebná - stroj zostáva po pristátí na pristávacej dráhe, ktorú má vyhradenú len pre seba. To v bežnej prevádzke civilného letiska neprichádza do úvahy - zablokovanie pristávacej dráhy na normálnom civilnom letisku je "mimoriadna udalosť", bez ohľadu na (ne)letové podmienky.

Existujú ale aj lietadlá, ktoré sú schopné pristátia za nulovej viditeľnosti len pomocou vlastného palubného vybavenia. Sú to predovšetkým niektoré vojenské dopravné lietadlá pre špeciálne misie, ale údajne aj americký prezidentský B747 a ďalšie podobné lietadlá.

ešte k Buranu - je možné (neviem o tom), že diaľkové riadenie bolo v Burane nainštalované, funkčné a v aktívnom stave (a obsluha, napríklad v sprievodnej Mig-25, pripravená prevziať riadenie), ale použité určite nebolo.
Svedčia pre to dve udalosti:
Buran "opakoval" okruh na pristátie (to sa rusom prevalilo až pomerne nedávno) - pristál až z druhého priblíženia, pretože pri prvom priblížení autopilot vyhodnotil, že bočný vietor je príliš silný - pozemná kontrola pritom takmer vydala pokyn na autodeštrukciu, pretože stroj sa pritom otočil smerom na mesto - a pristál na druhý pokus. Taký manéver by obsluha zrejme neriskovala.
Druhá vec je vysoká presnosť pristátia - bod dotyku s dráhou bol v tolerancii 1 meter od vypočítaného bodu. To svedčí skôr pre automatiku než pre živú obsluhu - oneskorenie riadiaceho okruhu cez rádiovú linku je pomerne dlhé, sú to síce len stotiny až desatina sekundy, ale pri rýchlosti okolo 350-400km/h je to dosť, automatika je obvykle rýchlejšia a presnejšia.
[Upraveno 21.7.2011 Alchymista]


pav_david - 21/7/2011 - 13:15

děkuji za vyčerpávající odpověď, to že jsou dnes tyto systémy na tak vysoké úrovni jak popisujete, jsem ani netušil.

vrátím se k tomu raketoplánu buran, je tedy pravděpodobné, že se jednalo skutečně o autonomní přistání? bez operátora na nedaleké základně ? přeci jen to bylo v roce 1988, měli tedy již tenkrát zvládnuté technologe které se dnes jak píšete rutinně používají?


Alchymista - 21/7/2011 - 13:25

Treba brať do úvahy, že technika Buranu bola (a dodnes je) absolútna technologická špička nielen v sovietskych merítkach.
Dá sa povedať, že Buran sa po technickej a technologickej stránke dostal tam, kam by sa mohol dostať STS, keby jeho vývoj pokračoval ďalej - keby sa stavali modernejšie / modernizované raketoplány na základe skúseností s ich prevádzkou.
[Upraveno 21.7.2011 Alchymista]


Ervé - 21/7/2011 - 13:39

Buran neměl dva proudové motory, nezdá se mi ani tvrzení o druhém pokusu, máte někde důvěryhodné zdroje?


Alchymista - 21/7/2011 - 13:44

Príliš dôveryhodné nie - bolo to uvedené v ruskom programe v súvislosti s koncom programu STS a z jedného vojenského fóra, kde autor písal o akútnej hrozbe povelu k autodeštrukcii Buranu po prechode na opakované priblíženie - v prvých momentoch si mysleli, že autopilot zblbol a stroj sa vymkol kontrole.


pospa - 21/7/2011 - 13:47

citace:
vrátím se k tomu raketoplánu buran, je tedy pravděpodobné, že se jednalo skutečně o autonomní přistání? bez operátora na nedaleké základně ?
Ano, přistání bylo automatické.
Určitě tu bude řada lidí, kteří mají v malíku více podrobností o historii Buranu než-li já, ale pamatuju si aspoň na jeden "detail" z toho unikátního přistání.
Buran se k ranveji přibližoval "po větru", tedy v opačném směru než potom přistával (jako ostatně všechny letadla přistával proti větru). Na finále dráhy se pak mohl z protisměru dostat buď levotočivou nabo pravotočivou zatáčkou. Jelikož se po větru přibližoval k dráze ne úplně paralelně, ale pod určitým úhlem z boku, operátoři v řídicím středisku seldující v reálném čase automatické přistání očekávali, že si palubní počítač vybere tu zatáčku na finále, která byla blíž/kratší. K jejich velkému překvapení Buran udělal přesný opak - na finále dráhy se dotočil mnohem delší, vzdálenější zatáčkou a následně perfektně dosednul nedaleko od prahu dráhy.
Při poletovém rozboru pak museli všichni dát palubnímu počítači zapravdu, jelikož vyhodnotil momentální meteo-situaci velmi správně a kvůli bočnímu větru vybral právě tu delší zatáčku aby těsně nad zemí a při dotyku kol nemusel vyrovnávat snos větru větším náklonem celého stroje.

Vot éto těchnika...


Alchymista - 21/7/2011 - 13:56

Ervé - myslím, že je to uvedené aj v tomto video -cca po 30 minute

edit: Mirek - diky. Niekde som to čítal, ale presný zdroj teraz nedokážem nájsť. [Upraveno 21.7.2011 Alchymista]


Dugi - 31/7/2011 - 11:54

Dobrý den všem,
měl bych dotaz ohledně čerpadel, která pumpují palivo z nádrží do spalovací komory u raketových motorů. Předpokládám, že systémy se budou lišit, podle použitého paliva, tedy, že čerpadlo na kapalný kyslík/vodík bude mít asi jiný pohon, než čerpadlo, které se stará o transport RP-1.
Jsou tato palivová čerpadla poháněna nějakým vnitřním zdrojem elektrické energie, nebo je pohání nějaká chemická reakce (např. hydrazin + oxidy dusíku)?
Děkuji předem za odpověď.


Alchymista - 31/7/2011 - 20:29

Čerpadlá sú obvykle poháňané turbínou, a sú to odstredivé čerpadlá.
V závislosti na type rakety a motoru môže byť každá zložka dopravovaná vlastným turbočerpadlom alebo je turbína spoločná pre obe zložky (častejší prípad). Tlakový plyn pre pohon turbíny sa získava spalovaním oboch zložiek paliva, alebo napríklad z rozkladu peroxidu vodíka (ten sa niekedy používa i na "štart" turbíny a na spalovanie zložiek paliva sa prechádza až následne).

S ohľadom na prečerpávané objemy iný spôsob pohonu čerpadiel prakticky neprichádza do úvahy - potrebné príkony idú bežne do megawattov.
Napríklad raketa Saturn V mala päť motorov F-1, ktoré za dve a pol minúty (cca 160 sekúnd) spálili vyše 2100 ton paliva (LOX + RP-1) - teda každý motor okolo 425 ton, čiže viac než dva a pol tony tekutého kyslíku a petroleja každú sekundu pri tlaku viac ako 70 atmosfér (70 barov bol tak v spalovacej komore motoru, takže čerpadlá museli dávať do rozprašovačov palivo a okysličovadlo pri tlaku ešte o dosť vyššom, aby sa dobre rozprášilo a vytvorilo správnu zmes).

Motor Rocketdyne F-1 (Saturn V) (obrázok z Wiki)
Turbočepadlová jednotka je na pravej strane, spalovacia komora pre pohon turbíny je označená ako "Gas Generator"
[Upraveno 31.7.2011 Alchymista]


Dugi - 31/7/2011 - 22:59

citace:
V závislosti na type rakety a motoru môže byť každá zložka dopravovaná vlastným turbočerpadlom alebo je turbína spoločná pre obe zložky (častejší prípad). Tlakový plyn pre pohon turbíny sa získava spalovaním oboch zložiek paliva, alebo napríklad z rozkladu peroxidu vodíka (ten sa niekedy používa i na "štart" turbíny a na spalovanie zložiek paliva sa prechádza až následne).[Upraveno 31.7.2011 Alchymista]


Díky za osvětlení celé problematiky. Ještě se chci zeptat - tím vlastním palivem pro turbíny pohánějící čerpadla bývá většinou jaká chemikálie? Jedná se jen o zmíněný peroxid vodíku, nebo se používají i jiné látky?


Alchymista - 1/8/2011 - 00:27

Bežne sa používa palivo a/lebo okysličovadlo, ktoré sa spaluje aj v danom raketovom motore - je to tak najjednoduchšie, netreba osobitné nádrže paliva a okysličovadla pre turbočerpadlá. Takže sa spaluje LOX+RP-1, LOX+LH, UDMH+N2O4...

Presnejšie:
Existuje niekoľko spôsobov, ako získať tlakový plyn pre pohon turbíny

1) otvorený cyklus ("Gas-generator cycle" "ЖРД открытого цикла")

Palivo a okysličovadlo sa spáli v generátore tlakového plynu v optimálnom pomere, preženie turbínou a vypustí do okolia - dosť neefektívne, pokiaľ si predstavíme, koľko paliva a okysličovadla sa takto "vypustí" mimo trysku motoru (nepodieľa sa na vytváraní ťahu, ale je započítané do špecifického impulzu, ktorý je tak nižší).
Takýto systém sa dá použiť i pri jednozložkovej pohonnej hmote na pohon turbíny, napríklad pre kataliticky rozkladaný peroxid vodíku alebo hydrazín.

2) uzavretý cyklus ("Staged combustion cycle" "ЖРД закрытого цикла")

Palivo a okysličovadlo sa spáli v generátore tlakového plynu v NEoptimálnom pomere (obvykle s prebytkom paliva), preženie turbínou a vedie ďalej do spaľovacej komory raketového motoru, kde redukčný tlakový plyn dohorí s okysličovadlom.

2A) Vylepšenu variantou uzatvoreného cyklu je uzatvorený cyklus s úplnym odparením zložiek ("Full flow staged combustion cycle" "Замкнутая схема с полной газификацией компонентов")

Palivo a okysličovadlo majú každé vlastné oddelené turbočerpadlá a plynové generátory (teda turbočerpadlá sú dve). Plynové generátory produkujú redukčný (pre palivo) alebo oxidačný (pre okysličovadlo) tlakový plyn, ten sa mieša s palivom alebo s okysličovadlom - a odparuje ho, odparené zložky prechádzajú turbínami a sú vedené d spalovacej komory motoru. Je zrejmé, že takýto systém sa hodí predovšetkým pre kryogénne pohonné hmoty - tekutý kyslík a tekutý vodík. Má tiež vyššiu tepelnú účinnosť motoru, pretože zložky vstupujú do spaľovacej komory už odparené, v plynnom stave

3)cyklus s odparením jednej zložky ("Expander cycle" "ЖРД с циклом фазового перехода")

Turbína je poháňaná tlakovým plynom, ktorý vzniká odparením jednej zo zložiek pohonnej zmesy (obvykle paliva) pri chladení trysky motoru. Takéto riešenie je vhodné predovšetkým pre motory s kryogénnymi palivami (tekutý vodík), ale dá sa použiť i pre RP-1 (RP-1 sa líši od bežných leteckých petrolejov predovšetkým práve tým, že má len veľmi malý "destilačný zvyšok", ktorý by mohol inak upchať trubky chladenia spalovacej komory a trysky, a obsahuje len zložky, ktoré pri svojom prípadnom tepelnom rozklade alebo polymerizácii nevytvárajú usadeniny - "asfalty" a "asfaltové" alebo uhlíkové/grafitové "príškvarky").


Za povšimnutie stojí, že na schémach je palivo pred vstupom do spaľovacej komory využité na chladenie trysky a spalovacej komory motoru. Tým sa využije časť tepelnej energie a zlepší sa účinnosť motoru, pretože palivo potrebuje menej tepla na svoje odparenie v spalovacej komore, alebo je do spalovacej komory dodávané už odparené, v plynnom stave.

Ďalšia vec je, že schémy zobrazujú "prevádzkový stav" motorov a nezaoberajú sa problémom, ako motor vôbec naštartovať a roztočiť turbočerpadlá pri spustení, keď je motor ešte "studený". Otvorený a uzatvorený cyklus s tým príliš problém nemá, ale u cyklu s odparením jednej zložky to už problém je...

Štartovacia sekvencia a stabilizácia chodu motoru po spustení je obvykle najzložitejšia časť vývoja kvapalinového raketového motoru s uzatvoreným cyklom (ale i raketového motoru na TPH).
Už bolo zopár prípadov, kedy vývoj motoru s uzatvoreným cyklom skončil neúspechom práve pre neschopnosť dosiahnuť stabilný rozbeh agregátov motoru - spalovacia komora fungovala dobre pri dodávke paliva externými čerpadlami (to nie je príliš veľké umenie), ale nepodarilo sa vyriešiť turboagregát alebo častejšie, jeho spoluprácu so spalovacou komorou v uzatvorenom cykle (aj preto má Merlin-I pracovný cyklus otvorený - je to tak oveľa jednoduchšie). [Upraveno 01.8.2011 Alchymista]


Dugi - 1/8/2011 - 00:31

Mnohokrát děkuju za vyčerpávající odpověď.


nadas - 1/8/2011 - 22:55

Dobrý den !
Mám dotaz ohledně další výstavby ISS na ruském segmentu .
Neví někdo z jakého důvodu je nutno před připojením modulu Nauka odstranit modul Pirs ? Proč nejde Nauku připojit na něj ? Byla by to přece založní výstupní komora , kdyby se něco pokazilo na Poisku ...
Nějaký důvod k tomu určitě mají , jenom se mi nikde nepodařilo vypátrat jaký . Dík za odpověď . Nadas


Conquistador - 2/8/2011 - 02:08

citace:
Dobrý den !
Mám dotaz ohledně další výstavby ISS na ruském segmentu .
Neví někdo z jakého důvodu je nutno před připojením modulu Nauka odstranit modul Pirs ? Proč nejde Nauku připojit na něj ? Byla by to přece založní výstupní komora , kdyby se něco pokazilo na Poisku ...
Nějaký důvod k tomu určitě mají , jenom se mi nikde nepodařilo vypátrat jaký . Dík za odpověď . Nadas


Pirs je na stanici skoro od počátku, a je možné že mu končí zaručená technická záruka a jestli není modul za ty léta opotřeben, dále Nauka je velmi masivní modul a možná kvůli tomu musí byt připojen k centrálnímu uzlu ruského segmentu.

Pokud ovšem Pirs dále dobře funguje (pokud je pořád jako nový) tak spíše by mne zajímalo proč se rusové nepokusí o jeho záchranu a opětovném připojením na dolní uzel Nauky.. (mohl být připojený na Progresu, který by po odpojení od stanice vedle ní posečkal do příletu Nauky a pak modul na spodní uzel Nauky napojil, i když s podobným manévrem re-dockingu mají rusové špatnou zkušenost z MIRu).


Alchymista - 2/8/2011 - 16:11

Na modul PIRS sa pripája Sojuz s hmotnosťou 7500-7800kg.
Modul Nauka má mať hmotnosť 20300kg, teda najmenej 2,5x vyššiu.
Teda PIRS je jednak "starý" - je vo vesmíre už 10 rokov, a jednak asi nemá dostatočnú štrukturálnu pevnosť, aby mohol niesť záťaž na úrovni okolo 30 ton (Nauka + Sojuz + Airlock + ďalšie vybavenie).
Navyše, Nauka (FGB-2, FGB-1 je Zarya) má niesť aj záložný "attitude control system" pre ISS, čo znamená, že silové namáhanie medziľahlých modulov na ohyb, ťah a tlak môže byť i značne vysoké (tlačí pred sebou vlastne celú hmotu stanice - cca 400 ton). A na to zrejme nie je Pirs stavaný vôbec.
To je teda dôvod, prečo nemôže zostať na pôvodnom mieste.
Prečo ho nepresunú na iné miesto, napríklad na koniec Nauky, je otázka - možno preto, že by bol až príliš ďaleko od nejakej významnej osy a tam opäť namáhaný viac, ako sa pôvodne počítalo. Možno mu tiež chýbajú nejaké dôležité časti vybavenia, napríklad rozvod pre prečerpávanie paliva do modulu Nauka a podobne. Alebo je, ako napísal Conquistador, tak starý a jeho presun tak riskantný, že sa to jednoducho neoplatí riskovať.
[Upraveno 02.8.2011 Alchymista]


M: - 2/8/2011 - 18:36

citace:
Na modul PIRS sa pripája Sojuz s hmotnosťou 7500-7800kg.
Modul Nauka má mať hmotnosť 20300kg, teda najmenej 2,5x vyššiu....
Este je dobre si uvedomit, kde je tazisko sojuzu, a kde nauky...


fritz.lochmann - 2/8/2011 - 19:31

citace:
Prečo ho nepresunú na iné miesto, napríklad na koniec Nauky ... Alebo je, ako napísal Conquistador, tak starý a jeho presun tak riskantný, že sa to jednoducho neoplatí riskovať.

Na voľnom konci Nauky má byť predsa v roku 2013 pripojený spojovací modul http://www.russianspaceweb.com/iss_node.html tak načo zachovať Pirs?
Inak tu je to pomerne dobre vidieť:

[Upraveno 02.8.2011 fritz.lochmann]


Leoš - 11/9/2011 - 16:07

Zdravím kosmofórum,

kolem komety elenin toho je na internetu tuny blábolů o nejpodivnějších konspiracích a nějakých májských cyklech. tyhle informace mě nezajímají, bohužel není jednoduché mezi nimi najít něco čemu se dá věřit.

Na několika místech (třeba tady http://www.livingfuture.cz/clanek.php?articleID=10409 - ty zdroje tam působí celkem věrohodně) jsem se ale dočetl, že se kometa začala rozpadat, nebo minimálně, že klesá její zářivost. Může to znamenat, že může změnit svou dráhu a neproletí kolem Země v původně odhadované vzdálenosti?


Alchymista - 11/9/2011 - 21:24

Pokiaľ by sa kométa nejak výraznejšie rozpadala, jej jasnosť by pravdepodobne skôr narastala, než klesala.

Pokiaľ bola zasiahnutá výtryskom koronárnej hmoty zo slnka, pre pokles jasnosti existuje jedno dobré vysvetlenie:
to, čo pozorujeme ako "kométu" je hlava kométy, ktorú tvorí prach a plyn uvoľnený z jadra - telesa podobného malému asteroidu o priemere rádovo kilometre. Prach a plyn v hlave kométy sa pomaly rozptyluje do priestoru a dosahuje do vzdialenosti až niekoľko tisíc kilometrov od jadra kométy. Preto je obvykle kométa dobre viditeľná i voľným okom, ale samotné jadro by bolo veľmi slabý objekt, pozorovateľný len výkonnejšími amatérskymi ďalekohľadmi - ako iné menšie asteroidy.
Plyn a prach v hlave je na jadro gravitačne viazaný len veľmi slabo, takže keď hlavu kométy zasiahne výron slnečnej koronárnej hmoty, značnú časť prachu a plynu doslova "odfúkne", čo sa prejaví dočasným poklesom jasnosti kométy - nejaký čas potom trvá, kým sa z jadra uvoľní ďalší plyn a prach, ktorý doplní hlavu a nahradí "odfúknutý" materiál.
Takéto udalosti nie sú nijak výnimočné, už boli pozorované aj prípady, kedy podobná udalosť (zrážka kométy s CME) spôsobila, že kométe bol "odtrhnutý" napríklad chvost a nejaký (krátky) čas existoval "samostatne".

Pokiaľ by došlo k rozpadu kométy na niekoľko častí, tieto by sa zrejme začali pohybovať po samostatných dráhach v závislosti na rýchlosti rotácie pôvodného jadra.
Pravdepodobne by sa výraznejšie prejavili i negravitačné poruchy dráhy jednotlivých úlomkov, pretože by pri rozpade boli odhalené hlbšie časti jadra, obsahujúce viac prchavých zložiek, ktoré by boli náhle vystavené slnečnému žiareniu a intenzívnejšie sa odparovali, čo by mohlo spôsobiť reaktívny efekt.
V tom čase by zrejme výrazne vzrástla i jasnosť kométy, pretože plyn a prach by bol uvoľňovaný z väčšej plochy a intenzívnejšie ako z povrchu pôvodného telesa - hlava kométy by sa zväčšila. To by trvalo do doby, kým by sa úlomky vzdialili od seba natoľko, že by sa dali rozlíšiť ako samostatné objekty, každý s vlastnou hlavou.

Prípadný rozpad kométy by samozrejme mohol spôsobiť, že sa niektorý úlomok priblíži k Zemi viac, ako uvádzajú výpočty pre pôvodný objekt, ale dodatková rýchlosť, ktorú by mohol pri rozpade získať, je najviac niekoľko desiatok metrov za sekundu (pri vyššej rýchlosti rotácie by sa kométa pravdepodobne už dávno rozpadla), takže nejaká zmena dráhy možná je, ale tá je zaujímavá skôr pre výpočty jej budúcej dráhy pri ďalších návratoch, ale nie pri aktuálnom prielete - čas najväčšieho priblíženia k Zemi by sa po rozpade zmenil možno o niekoľko hodín a najmenšia vzdialenosť k Zemi môžno o pár percent. A potom je celkom jedno, či bude najmenšia vzdialenosť 0,24AU (cca 36 000 000 km) , alebo 0,25AU alebo 0,22 AU (cca 33 000 000 km) - je to 90 alebo 82 násobok vzdialenosti Mesiaca.

[Upraveno 11.9.2011 Alchymista]


Ervé - 12/9/2011 - 11:38

Pokud si dobře pamatuju, Pirs je k Zvezde připojený velkým stykovým uzlem o vnitřním průměru kolem 1000 mm - ten usnadňuje přenos nákladů i průlez. Progressy, Sojuzy a ATV mají ale malý stykový uzel o průměru 800 mm - takže Nauka má horní uzel taky o průměru 1 m a dolní o 80 cm. Připojovat 80 cm průlez k 1 m jaksi nejde...


Na modul PIRS sa pripája Sojuz s hmotnosťou 7500-7800kg.
Modul Nauka má mať hmotnosť 20300kg, teda najmenej 2,5x vyššiu....
Este je dobre si uvedomit, kde je tazisko sojuzu, a kde nauky...


-=RYS=- - 12/9/2011 - 16:51

citace:
Pokiaľ by sa kométa nejak výraznejšie rozpadala, jej jasnosť by pravdepodobne skôr narastala, než klesala.

Pokiaľ bola zasiahnutá výtryskom koronárnej hmoty zo slnka, pre pokles jasnosti existuje jedno dobré vysvetlenie:
to, čo pozorujeme ako "kométu" je hlava kométy, ktorú tvorí prach a plyn uvoľnený z jadra - telesa podobného malému asteroidu o priemere rádovo kilometre. Prach a plyn v hlave kométy sa pomaly rozptyluje do priestoru a dosahuje do vzdialenosti až niekoľko tisíc kilometrov od jadra kométy. Preto je obvykle kométa dobre viditeľná i voľným okom, ale samotné jadro by bolo veľmi slabý objekt, pozorovateľný len výkonnejšími amatérskymi ďalekohľadmi - ako iné menšie asteroidy.
Plyn a prach v hlave je na jadro gravitačne viazaný len veľmi slabo, takže keď hlavu kométy zasiahne výron slnečnej koronárnej hmoty, značnú časť prachu a plynu doslova "odfúkne", čo sa prejaví dočasným poklesom jasnosti kométy - nejaký čas potom trvá, kým sa z jadra uvoľní ďalší plyn a prach, ktorý doplní hlavu a nahradí "odfúknutý" materiál.
Takéto udalosti nie sú nijak výnimočné, už boli pozorované aj prípady, kedy podobná udalosť (zrážka kométy s CME) spôsobila, že kométe bol "odtrhnutý" napríklad chvost a nejaký (krátky) čas existoval "samostatne".

Pokiaľ by došlo k rozpadu kométy na niekoľko častí, tieto by sa zrejme začali pohybovať po samostatných dráhach v závislosti na rýchlosti rotácie pôvodného jadra.
Pravdepodobne by sa výraznejšie prejavili i negravitačné poruchy dráhy jednotlivých úlomkov, pretože by pri rozpade boli odhalené hlbšie časti jadra, obsahujúce viac prchavých zložiek, ktoré by boli náhle vystavené slnečnému žiareniu a intenzívnejšie sa odparovali, čo by mohlo spôsobiť reaktívny efekt.
V tom čase by zrejme výrazne vzrástla i jasnosť kométy, pretože plyn a prach by bol uvoľňovaný z väčšej plochy a intenzívnejšie ako z povrchu pôvodného telesa - hlava kométy by sa zväčšila. To by trvalo do doby, kým by sa úlomky vzdialili od seba natoľko, že by sa dali rozlíšiť ako samostatné objekty, každý s vlastnou hlavou.

Prípadný rozpad kométy by samozrejme mohol spôsobiť, že sa niektorý úlomok priblíži k Zemi viac, ako uvádzajú výpočty pre pôvodný objekt, ale dodatková rýchlosť, ktorú by mohol pri rozpade získať, je najviac niekoľko desiatok metrov za sekundu (pri vyššej rýchlosti rotácie by sa kométa pravdepodobne už dávno rozpadla), takže nejaká zmena dráhy možná je, ale tá je zaujímavá skôr pre výpočty jej budúcej dráhy pri ďalších návratoch, ale nie pri aktuálnom prielete - čas najväčšieho priblíženia k Zemi by sa po rozpade zmenil možno o niekoľko hodín a najmenšia vzdialenosť k Zemi môžno o pár percent. A potom je celkom jedno, či bude najmenšia vzdialenosť 0,24AU (cca 36 000 000 km) , alebo 0,25AU alebo 0,22 AU (cca 33 000 000 km) - je to 90 alebo 82 násobok vzdialenosti Mesiaca.

[Upraveno 11.9.2011 Alchymista]



Rekl bych, ze uz ted je mozne "ohmatat" si pevne jadro ci nekolik jader pomoci radioteleskopu pro vyssi pasma (10/15/22GHz) s talirem alespon 50m. Mozna sousedni nemecky 102m radioteleskop nebo soustava teleskopu pro vyssi pasma v Chile.
Z radioteleskopu se toho vycte vice.
Pri "ohmatavani" asteroidu bylo treba, aby se priblizil na 20megakm.
Pri 33-35megakm se da zjistit, jestli to je jedno teleso nebo nekolik jader. Take se da zjistit prumery.
Kdyz by to bylo 15-20megakm, tak se da udelat i povrchova 3D mapa.
Ale na tohle je zapotrebi vzdy radioteleskopu pro vyssi pasma.
Na inetu jsou snimky treba i z Areciba na 2.5GHz, ale jen vyssi pasma dokazou presneji ohmatat teleso.


Ervé - 13/9/2011 - 07:49

Pokles jasnosti bych dal do jasné závislosti na zásahu CME - ten odfoukl část ohonu komety. Kdyby se kometa rozpadla, jasnost by stoupla.


martinjediny - 16/9/2011 - 00:33

Klasické elektrárne používajú jednotný systém značenia KKS, kedy kazda trasa a kazdy prvok ma unikatne cislo jednotné pre vsetky profesie, takze aj strojari aj elektrikari aj programatori hovoria stale o tom istom, napr. cerpadle.

Ake cislovanie pouzivaju rakety / lietadla? Je jednotne, alebo kazda firma zvlast?


martas - 23/9/2011 - 08:22

Zajímalo by mě, odkud čerpá data např server www.n2yo.com o aktuální poloze družic. Předpokládám, že se jedná o výpočty na základě aktuálních parametrů dráhy tělesa z nějaké databáze. Pokud ano, tuší někdo, jak často se ty paramery aktualizují?


-=RYS=- - 24/9/2011 - 16:41

citace:
Zajímalo by mě, odkud čerpá data např server www.n2yo.com o aktuální poloze družic. Předpokládám, že se jedná o výpočty na základě aktuálních parametrů dráhy tělesa z nějaké databáze. Pokud ano, tuší někdo, jak často se ty paramery aktualizují?


U kazde druzice asi v jiny cas.
U ISS jsem to vysledoval jednou za 6 hodin.

Zkus jim (radioklub) napsat po paketu odkud berou aktualni data. Je mozne, ze maj VPN linku do nekolika sledovacich mist.


Lukavský - 24/9/2011 - 20:02

citace:
Zajímalo by mě, odkud čerpá data např server www.n2yo.com o aktuální poloze družic. Předpokládám, že se jedná o výpočty na základě aktuálních parametrů dráhy tělesa z nějaké databáze. Pokud ano, tuší někdo, jak často se ty paramery aktualizují?


Aktualizaci dat lze vypozorovat z dvouřádkových TLE parametrů dráhy, kde je uveden čas, pro který uvedená poloha platí. Lze to snadno vyčíst například v program Orbitron (http://www.stoff.pl/ ), kde v záložce informací o satelitu je tato doba (období) uvedena. Já to nesleduji, ale vím že u běžných dobře viditelných satelitů je po aktualizaci dat čas poslední polohy u veřejně přístupných dat z databáze NORAD kratší než jeden den.


HonzaVacek - 24/9/2011 - 20:34

TLE lez získat např. tady : http://celestrak.com/


-=RYS=- - 26/9/2011 - 04:15

Martas se ptal:
1) Odkud cerpa server N2YO data o aktualni poloze druzic.
2) Jestli se jedna o vypocty na zaklade nejake databaze.
3) Jak casto se databaze aktualizuji.

Uz jsem odpovedel a predpokladam, ze odpovedi pana Lukavskeho a Honzi Vacka nejsou asi presne ty odpovedi na ktere se Martas ptal.
Takze se pokusim presneji odpovedet.

1)
Z ruznych databazi, nejcasteji z Celestraku a https://www.spacedatasource.org/perl/login.pl.
Avsak neni to jen odtud, protoze minimalne do TRACKCOMU a RAFAELU maj primou VPN.


2)
Ano, jedna se o vypocty ze souhrnu vsech tehto databazi.


3)
Aktualizuji se z web databazi kazdych 15minut a z ostatnich "databazi" do 6h.



Ted k tem databazim.
Pro HAM svet (radioamatersky svet) existuji 2 formaty databazovych dat.
Nejcastejsi (i ten pouziva N2YO) je Amsat format, vypada takto:
http://www.amsat.org/amsat/ftp/keps/current/amsat.all

Ale v profi praxi se spise pouziva NASA 2radkovy format a vypada takto:
http://www.amsat.org/amsat/ftp/keps/current/nasa.all

Pro zpracovani tehle dat existuje hromada radioamaterskych programu. Ja sam pouzivam Orbitron (od Sebastian Stoff)
a InstantTrack (od Franklin Antonio N6NKF), ale jsou i dalsi jako Sat Explorer, SatTrack, SatWin.
Jelikoz vetsina tehle softu vychazi od radioamateru pro radioamatery, tak umoznuji vice veci nez jen sledovat kde je satelit, ale i jakou oblast satelit ozari na radioamaterskych kmitoctovych pasmech nebo maji tyto softy moznost ovladat radioamaterske rotatory ve vertikalnim i horizontalnim smeru tak, aby vetsinou Yagi anteny sledovali objekt na obloze.

Tyto HAM soft umi sledovat i Slunce a Mesic. Mesic kvuli spojeni typu EME (spojeni odrazem od povrchu Mesice... Earth-Moon-Earth) a Slunce kvuli zmereni kvality prijimacu a predzesilovacu.
Nektere soft umi navic zamerit body ve vesmiru, napriklad kdyz chces sledovat vesmirne sondy DSN nebo si doladit presnost rotatoru treba podle pulsaru ve vesmiru o kterem je znama presna poloha vzdy.

Tyto "keplery" je treba aktualizovat pro Zemskou orbitu maximalne do 2 tydnu, jinak predpovet preletu "ujede" o par minut....za mesic i o hodinu.

Pochopitelne pro LEO objekty neni treba hned instalovat Orbitron a zaktualizovavat keplery (TLE), staci se prihlasit na tuto stranku:
http://www.heavens-above.com/
Kde si zadas svou polohu:
http://www.heavens-above.com/LocationFromGoogleMaps.aspx?lat=0&lng=0&loc=Unspecified&alt=0&tz=CET
A pak kliknes na objekt, ktery chces sledovat...see:
http://www.heavens-above.com/amateursats.asp?lat=50.33494&lng=13.52966&loc=Unspecified&alt=230&tz=UCTm2


Orbitron vypada takto:
http://www.ok1mjo.com/all/ostatni/space_aircraft/Orbitron_v3-71_screencopy.jpg

Co se tyce radioklubu N2YO, tak tady mas kontakt z ARRL:
Sufitchi, Ciprian, N2YO
4519 Cub Run Rd
Chantilly, VA 20151
Previous call sign: AG4TX
Licensee ID: L00507498
License Class: Extra
FRN: 0006971642
Radio Service: HV
Issue Date: 07/16/2002
Expire Date: 07/16/2012
Date of Last Change: 01/04/2007 (Administrative Update Applied)
http://www.qsl.net/n2yo/


[Upraveno 26.9.2011 -=RYS=-]


martas - 26/9/2011 - 12:26

Děkuji všem za vyčerpávající odpovědi.


Vlado1 - 4/10/2011 - 12:12

Rychlost 2 M ve výšce 1 km je 2450km/h.
Kolik km/h bude rychlost 2M ve výšce 20 km nebo 100 km ? (rychlost zvuku)


Ctenar - 4/10/2011 - 12:32

Slušný a srozumitelný popis na http://tscm.com/mach-as.pdf.


Vlado1 - 4/10/2011 - 14:46

citace:
Slušný a srozumitelný popis na http://tscm.com/mach-as.pdf.
Děkuji za odkaz. Tá potvora překladač, mi nechce překresli nomogramy.


risa.ber - 11/11/2011 - 23:20

Chtěl jsem se zeptat, proč je startovní rampa pro nosiče Sojuz otočná? Určitě to má něco společného s azimutem, kterým má letět. Jen mi to připadá divné, když steně v počátku letí vlastně kolmo.


Ervé - 14/11/2011 - 06:39

Řídící systém rakety musí být nastavený, aby držel směr letu, raketa se do něj stáčí několik desítek sekund po startu. Řídící systémy raket musí být co nejjednodušší (aby byly spolehlivé a levné) takže je jednodušší otočit s raketou kolem osy než komplikovat řídící systém.


Ervé - 3/1/2012 - 06:31

Když startovala v prvních zkušebních letech Vostoku figurína kosmonauta, kde byl umístněný a jak přistával pes (např. Zvezdočka) který letěl taky?


yamato - 10/1/2012 - 16:18

technicka otazka od nefyzika pre fyzikov: predstavme si reaktivny motor, ktory by najprv v turbine spaloval H2 a O2. Vyrobena elektrina by sa pouzivala v elektrotermalnom motore, ktory by pouzival odpadnu vodnu paru ako pohonne medium. Je mozne z takehoto motoru vyzdimat vyssie Isp ako z klasickeho hydrolox raketoveho motora? (trebars za cenu nizsieho tahu)


M: - 10/1/2012 - 16:29

citace:
technicka otazka od nefyzika pre fyzikov: predstavme si reaktivny motor, ktory by najprv v turbine spaloval H2 a O2. Vyrobena elektrina by sa pouzivala v elektrotermalnom motore, ktory by pouzival odpadnu vodnu paru ako pohonne medium. Je mozne z takehoto motoru vyzdimat vyssie Isp ako z klasickeho hydrolox raketoveho motora? (trebars za cenu nizsieho tahu)

Nie som fyzik, ale nedalo.
Vzdy rataj mnozstvo prikon x ucinnost.
Ak mas dobry motor s kvalitnou spal. komorou a tryskou tak zaratas akurat ich ucinnost.

Pri tvojom systeme mas rovnako ucinnu spal komoru a trysku, pricom ucinnost dalej znizujes prudenim cez turbinu,turbinou, generatorom, vedenim odpadnej pary a tepelnymi stratami, ucinnostou elektrotermalneho pohonu...


yamato - 10/1/2012 - 17:41

jop, myslel som si


Machi - 10/1/2012 - 18:18

citace:
predstavme si reaktivny motor, ktory by najprv v turbine spaloval H2 a O2. Vyrobena elektrina by sa pouzivala v elektrotermalnom motore, ktory by pouzival odpadnu vodnu paru ako pohonne medium. Je mozne z takehoto motoru vyzdimat vyssie Isp ako z klasickeho hydrolox raketoveho motora? (trebars za cenu nizsieho tahu)


Zřejmě ne. S vodou se počítalo v elektrotermálních motorech s odporovým ohřevem, ale tam se Isp nevyšplhá přes 300-350s (LOx asi 400-450s).


MASH - 30/7/2012 - 23:35

Zdravím, mám jednu laickou otázku:
Je možné prohlásit, že geostacionární družice OBÍHÁ Zemi? Zajímá mě sousloví "obíhá Zemi". Podle mě družice na GEO neurazí vůči povrchu Země žádnou dráhu a planetu tedy nikdy neoběhne. Díky za fundované názory


martinjediny - 31/7/2012 - 00:06

Fundovany nie, ale nazor mam
1/ je to otazka spravnej volby vztaznej sustavy.
2/ smernica orbitalneho letu je rovnobezna s vyslednicou zrychleni.
bez obehu a teda bez odstredivej sily budes mat trochu problem vysvetlit realitu.

aj osoby v centrifuge v sedacke oproti, voci tebe su v pokoji, napriek tomu ich nejaka zahadna sila macka do sedadla v smere od teba prec...


arccos - 31/7/2012 - 01:10

Samozřejmě, že obíhá. Tento pohyb se nevztahuje k povrchu Země, ale k jejímu středu (nebo těžišti). Ostatně, jak píše Martin, jinak by to spadlo.


Alchymista neprihlásený - 31/7/2012 - 04:50

Otáča sa Mesiac, keď je stále otočený k Zemi tou istou stranou?
To je v princípe rovnaká otázka.
Správna voľba súradnicového systému je pre správny pohľad na problém celkom podstatná-


MASH - 31/7/2012 - 07:18

Velmi děkuji za první reakce! Dovolím si ještě doplnit: Aby nebyla mýlka ve významu mé otázky, já tak nějak tuším princip družice na GEO. Chápu, že se pohybuje (jinak by spadla, případně odletěla), ale šlo mi právě o to sousloví "obíhá planetu" a nikoliv o to, jestli se "pohybuje". Jestli vůči Měsíci, Slunci, hvězdám a nebo jestli obíhá nějakou černou díru ve středu galaxie
Přiznám se navíc, že s tím těžištěm/středem Země jsem to moc nepochopil (inu jsem laik :-P ). To jako, že střed/těžiště tělesa se pohybuje vůči povrchu toho stejného tělesa jinak? A teď nemyslím zrovna tekuté jádro naší planety .
Na závěr si, s dovolením, položím otázku hodnou dítěte - tedy s naprosto absurdním přirovnáním: Takže, pokud by GEO družice skutečně "obíhala planetu", tak mohl bych tvrdit, že pokud stojím na zemském povrchu, že moje hlava také obíhá planetu (né, jestli pluje vesmírem, ale skutečně, jestli "obíhá planetu")? Resp. kdybych měřil 36 tisíc kilometrů a byl ze super pevného a nedeformovatelného materiálu, stál bych na zemi a hlavu bych měl 5 cm od nějaké geostacionární družice, obíhala by ta moje hlava planetu?
Omlouvám se za to naprosto hloupé přirovnání, jde mi o ten princip (z laického pohledu). Ještě jednou dík a doufám, že vás tu moc nerozčiluju


Agamemnon - 31/7/2012 - 07:31

- tazisko je bezrozmerny, teoreticky bod - tj. nema smer... takze voci nemu sa neda povedat, ze je satelit stale nad tym istym bodom planety...
ked sa pozries na sustavu planeta-satelit "zhora", tak pri satelite budes vidiet, ze opisuje kruznicu okolo planety - obieha ju
- tvoja hlava... hmm... pohybuje sa voci tazisku... takisto cely ty, povrch zeme a pod... ale nema dostatocnu rychlost (orbitalnu) na to, aby doslo k obehu a ty/tvoja hlava/povrch zeme spadli vdaka gravitacie na zem (tj. dotykaju sa povrchu zeme)... keby si bezal dostatocne rychlo (8.2 km/s na povrchu), tak by si obiehal okolo zeme...
- ak by si mal vysku 36tis. km, tak tvoja hlava by mala rovnaku rychlost ako maju geo druzice (tj. dostatocnu na to, aby sa udrzala na orbite)... tam su samozrejme ine problemy potom

na to aby si obiehal planetu, je podstatna "dopredna"/orbitalna rychlost (rychlost, ktora je dotycnicou k drahe orbity)... geo satelity tuto rychlost maju... tvoja hlava na zemi nie... tvoja hlava vo vyske geo drahy by ju mala... tato musi byt dostatocna, aby ta nestiahla gravitacia k povrchu
[Edited on 31.7.2012 Agamemnon]


DH - 31/7/2012 - 07:51

Geostacionární družice obíhá Zemi, ale neobíhá její povrch.

Každá družice vždy těleso obíhá (pasažér ne družici zaměřený na jednu hvězdu vidí, jak Země obíhá po velkém kruhu kolem něj).

To je vlastnost všech družic. Podle toho, jak je jejich dráha eliptická nebo kruhová, jak je skloněná vůči rovině rovníku a zda směr oběhu "souhlasí" nebo "nesouhlasí" se směrem otáčení Země, máme milióny a milióny různých drah.

Co je zajímavé, je to, jakou křivku opisuje "pata kolmice" od družice po mapě povrchu (po globu, nebo po Mercatorově projekci). Ty křivky mohou být takové ty dobře známé "sinusovky" z řídících středisek (dráha blízká kruhové, skloněná), mohou vypadat dost exoticky (Molnija, velmi protáhlá eliptická dráha), mohou být přímka po rovníku (např. rovníková LEO družice).

Čím bližší bude dráha družice ideální GEO dráze, tím pomaleji se bude pata kolmice pohybovat po povrchu a tím bude blíže k rovníku. Pro ideální GEO dráhu se pata kolmice na mapě "zastaví". V realitě žádný ideál neexistuje, takže skutečná pata se bude "lehce pohybovat" a řízení ze Země ji bude muset čas od času motorem postrčit zpátky.

Čili sousloví "družice obíhá planetu" je správné a logické. Spíš by se dalo říci, že planeta se pod GEO družicí "podtáčí" právě tak, aby ji sledovala. Jako kdyby auto po dálnici udržovalo stejnou rychlost jako ultralight nad ním. Dalo by se potom říct, že ultralight přelétá nad autem?


Alchymista - 31/7/2012 - 09:53

"Geostacionárna" družica, teda družica, ktorá zostáva pre pozorovateľ na povrchu zeme v stálej polohe, musí splniť niekoľko podmienok:
(A) musí mať obežnú dobu zhodnú s dobou rotácie Zeme,
(B) musí obiehať po približne kruhovej dráhe,
(C) musí obiehať v smere rotácie Zeme (teda v prográdnom smere),
(D) rovina jej dráhy musí byť presne kolmá na rotačnú osu Zeme a
(E) dráha musí svojím apogeom, perigeom a ďalšími parametrami čo najlepšie sledovať rez gravitačnej ekvipotenciálovej plochy v rovine zemského rovníku.

Ak nie je splnená podmienka (E) - a ona sa vlastne prakticky splniť nedá, pretože geoid nie je guľa - nič podstatné sa nedeje, len sa družica bude mierne pohybovať voči povrchu Zeme a na udržanie sa v nejakej definovanej chybovej ploche (presnejšie priestore) bude musieť svoju dráhu neustále alebo periodicky korigovať (ideálna dráha nie je presne kruhová, ale trochu eliptická) - ale to musí robiť tak či tak, pretože dráha družice je rušená aj gravitačným vplyvom Mesiacu a Slnka, pri skutočne dôslednom vyšetrovaní ešte aj vplyvom gravitácie veľkých planét. Na korekcie slúžia napríklad aj iontové motorčeky, ktoré niektoré geostacionárne družice používajú - majú síce mrňavý ťah, ale na potrebné drobné korekcie to stačí. (Ale podmienkou (E) som si nie celkom istý...)

Ak nie je splnená podmienka (D), hovoríme obecne o geosynchrónnej družici - takáto družica bude pre pozorovateľa v pravidelných intervaloch stúpať nad obzorom k zenitu a potom zasa klesať k obzoru, prípadne aj vychádzať a zapadať. Priemet jej dráhy na povrch Zeme vytvára "osmičku".

Ak by nebol splnená podmienka (C), družica bude na kruhovej 24 hodinovej dráhe, ale v retrográdnom smere - družica sa bude pohybovať proti smeru rotácie Zeme
Ak by nebola splnená podmienka (B), hovoríme o dráhach typu "Molnija" - sú to eliptické dráhy, družica na takejto dráhe je z jedného miesta na zemskom povrchu dlhodobo pozorovateľná, ale priemet jej pohybu na zemský povrch je pomerne zložitý.
Ak by nebola splnená podmienka (A), dráha obvykle nie je nejak synchronizovaná s rotáciou Zeme - ale môže byť, existujú napríklad aj zaujímavé 12 hodinové, alebo 8 a 6 hodinové dráhy.

Problém obiehania - tu skutočne záleží na uhle pohľadu.
Vrchol 36000 kilometrov vysokej veže (alebo hory, či hlava obra) postavenej na rovníku splňuje prakticky všetky podmienky pre geostacionárnu družicu - ale tu by sme nemali hovoriť o "obehu", pretože vrchol veže je "pevne prepojený" so základmi a povrchom Zeme. Pojem "obeh" je spájaný skôr so vzájomným pohybom dvoch samostatných objektov, pre MASHom uvedený príklad je asi vhodnejšie uvažovať ako o "rotácii".
[Upraveno 31.7.2012 Alchymista]


M: - 31/8/2012 - 13:06

Do akej zmeny rychlosti mozme pri prenose dat zanedbat dopplerov efefekt?
Najma pri starte rakety, kedy je zmena rychlosti, z pohladu prenosu dat uz podstatna?

Laicky (strojarsky) mi tam vychadza do 1km/s smiesne mala odchylka frekvencie.
Co je akceptovatelna odchylka frekvencie? V com je pes zakopany?


x - 31/8/2012 - 13:57

"V com je pes zakopany? "

Odpovím jen obecně jak to jako elektronik vím, ale né z problematiky spojení ve vesmíru.
Každý prijímač - tedy pokud je alespoň dělanej klasicky - moderní DSP a jejich možnosti už tolik neznám - má na vstupu úzký a pevný filtr a nebo jen přeladitelný v úzském rozsahu z technických důvodu tomu tak je - ale ten nejvyšší hig-end a jeho možnosti neznám - tedy říkám to obecně.

Čím je ten filtr užší, tak přijímač pochytá jen málo nežádoucích signálů z jiných frekvencí - které mohou být mnohem silnější než ty přijímáme - ty by totiž další obvody toho přijímače zcela zahltili a tak znemožnili jeho funkci.

A tak proto je ten filtr úzký a tudíž může jen omezně reagovat na změny frekvence signálu.

Tolik moje vysvětlení - obecně platné - pro běžné konstrukce přijímačů.


-=RYS=- - 31/8/2012 - 14:24

citace:
Do akej zmeny rychlosti mozme pri prenose dat zanedbat dopplerov efefekt?
Najma pri starte rakety, kedy je zmena rychlosti, z pohladu prenosu dat uz podstatna?

Laicky (strojarsky) mi tam vychadza do 1km/s smiesne mala odchylka frekvencie.
Co je akceptovatelna odchylka frekvencie? V com je pes zakopany?


http://www.ok1mjo.com/all/ostatni/space_aircraft/Doppleruv_jev_by_OK1DDD.png
http://www.svengrahn.pp.se/trackind/UpOnSband/UpOnSband.htm

Pro rakety/druzice co startuji a letej k GTO plati, ze pouzivaj pokud mozno jednoduchej sirokopasmovej druh fazove modulace jako je PSK ci MSK/MFSK (DPSK/BPSK/PI4-DQPSK/PI8-D8PSK/MFSK).
http://www.uhf-satcom.com/molniya/

Kdysi to byl problem, v soucasnosti to problem neni.
Dnes je vse na bazi SDR, kde prijimac zachytava/demoduluje a vyhodnocuje cele potencionalni pasmo.
Napriklad, pokud je dw radioband sirokej 20MHz (pri PI4-DQPSK to je cca 16Mbps) na stredovym kmitoctu treba 2200MHz, tak se pouzije SDR, ktery on-line prijima potrebnou sirku pasma (2190-2210MHz).
Ale aby se vykryl Doppler, tak se SDR nastavi tak, aby prijimal vetsi sirku pasma a to o tolik, kolik je teoretickej predpoklad "ujeti" kmitoctu nahore a dole + 20% rezerva, takze ve skutecnosti bude SDR prijimat zhruba (od oka) 2160-2240MHz, tedy 80MHz.
Takze kdyz se bude kmitocet "courat", tak je to jedno, protoze SDR neprijima tech 20kHz a nepreladuje se, ale prijima celejch 80MHz a "digitalne" se preladuje. Respektive nepreladuje, ale snazi se "obrysove" vytahnout data z celeho 80MHz useku.
Je to podobne jako kdyz mas rucni radiostanice. Prenaseji hovorove pasmo 300-2500Hz.
A tebe zajima jen pasmo siroke treba 200Hz, tak ho naladis nekam na 1200Hz jako stred a jak se toto pasmo a datove-zvuky v nem zacnou hybat 500Hz nahoru nebo 500Hz dolu, tak jsi porad v pohode. Protoze neprijimas rozsah 1100-1300Hz, ale rozsah sirokej 2200Hz. Takze i kdyz ty zvuky budou uhybat do vysek nebo basu, tak porad chytas "cele" pasmo pres DSP (usi).
A takto podobne pracuji moderni SDR prijimace.
Napriklad na Marsu maji druzice SDR prijimac siroky tusim 35MHz a v tomto pasmu jsou ty 4 radiokanaly co se pouzivaji v ramci Proximity-1. Kazdej kanal je sirokej cca 650kHz (pri max rychlosti 256kbps).
Ale to nevadi, kdyz prijimac na MRO je sirokej SDR 35MHz.
Coz znamena, ze kdyz v jednom okamziku uslysi soucasne MSL i MER, tak se nic nedeje. MSL jede na kanalu treba 4 a MER na k1 a oba kanaly jsou v ramci prijmu 35MHz. Takze SDR prijimac MRO zachyti soucasne obe vysilani a zpracujeje.
Kdyz MSL pristaval, tak vysilal na 2 UHF radiokanalech.
Na kanale 6 vysilal data 8kbps up (sirka radiobandu 20kHz) a zaroven vysilal na kanale 1 az 250 tonu soucasne (MFSK modulace jednoho tonu je siroka asi 10Hz...peak, takze sirka radiokanalu pro vsechny tony soucasne je asi 5kHz a presto to SDR na Odyssey/MRO zvlada diky SDR o celkove sirce 35MHz)...tedy vcetne ujizdeni kmitoctu behem pristavani + priblizovani druzice na orbite k nejblizsimu mistu k landeru a pak ujizdeni kmitoctu (tonu) pri vzdalovani.

Akceptovatelne odchylka frekvence je v tomto pripade ujeti az po krajni zpracovatelny kmitocet te sirky 35MHz v pripade Marsu.
Cili pokud by kanal1, ktery vcetne Dopplera se snizuje LSB spodni cast kmitoctu od stredu smerem ke kraji-hranici spodniho kmitoctu, ktery jeste SDR druzice dokaze zpracovat, tak je to na hranici zpracovatelnosti.
Priklad:
Celkovej kmitocet UHF se 6 kanaly je dejme tomu (ted z hlavy nevim) 430-465MHz, kde 1 kanal je 436MHz (tedy pri sirce 650kHz... 435.675Mhz a nejvyssi cast kmitoctu je 436.325MHz), tak v pripade, ze pri odletu od nadhlavniku MSL co prave pristala zacne kmitocet/tony klesat smerem dolu k hranicnimu kmitoctu 430MHz, tak jeste u kmitoctu (kde se dotyka spodni cast pracovniho kmitoctu LSB) 433MHz to je v ramci tolerance a moznosti SDR prijimace. Ale jak to zacne klesat jeste mene az k 431MHz a mene, tak to uz je kriticka odchylka a je to na hranici SDR prijimace. Takze se pred odletem vytvori "tabulka", kde to jeste je akceptovatelna odchylka, coz dle techniku muze byt nejmene 434MHz, aby tam byla jeste rezerva.

Derivaty PSK modulace se pouzivaji, protoze jim tolik nevadi mzikove zkresleni narozdil od QAM modulace. Proto v kosmickych spojich i satelitni televize DVB-S nepouzivaji pro prunik atmosferou QAM, ale jen xPSK, protoze neni rozhodujici zmrsena modulace, ale spise rozdily fazi, ktere davaj logickou 0 a 1. Vzdy roztrhaj normalni modulace ruzne vrstvy atmosfery, takze QAM neni pouzitelna, pro tento typ "skrz atmosferu" se prave pouzivaj ty fazove modulace.
Neco jineho je komunikace jen v kosmickem prostoru, treba mezi dvema sondama/druzicema, kde radiosignal leti jen ve volnem vakuu. V takovych aplikacich se QAM pouziva a ke zkresleni nedochazi (az na par vyjimek z kteryma se da pocitat...vliv Slunce atd..).

Pri pruchodu atmosferou totiz dochazi k odrazum od molekul vzduchu a kapicek vody a tim dochazi k vicecestnemu sireni. QAM to roztrha, protoze dochaze k mzikovym mikroposuvum Dopplera, ale i roztrhana fazova modulace doleti a pomeru fazi, ktere atmosfera neposkodi si prijimac SDR vyhodnoti kde je 1 a kde 0.

Zkus si na Dopplera poslechnout treba zde:
http://www.youtube.com/watch?v=MvIH7UP6XYg
Kde je slyset, jak ton telegrafu ujizdi do basu, pricemz prijimac se nepreladuje.
V HAM praxi se to kompenzuje manualnim/automatickym doladenim TX i RX kmitoctu soucasne. PC soft vypocita ujeti kmitoctu a pres seriovku da TRXu info kam a o kolik si ma ujet s kmitoctem TX a RX.

Toto se u sirokopasmoveho SDR nedeje, protoze bere celej rozsah ujeti a tak tu zmenu frekvence vlastne dojede softwerove.

Klicove je u SDR jak vykonej AD prevodnik je na vstupu, jak rychle vzorkuje a kolika bitama. A tez je dulezite jakej DSP CPU zpracovava ty data z AD prevodniku. Jestli je DSP dostatecne vykonej pro zpracovani signalu a prevodu do uzitecnych dat.



Doufam, ze jsem to vysvetlil srozumitelne.



DH - 31/8/2012 - 14:36

Změna frekvence Dopplerovým jevem není absolutní, ale relativní. Přitom ovšem platí, že rádiová komunikace znamená nějak modulovanou nosnou vlnu. Podle modulační rychlosti vzniká potřeba přenášet nějak široký rádiový kanál kolem nosné frekvence.

Například běžné radioamatérské fonické FM spojení používá kanál o šířce řekněme 12.5 kHz. Je přitom jedno, jestli je to kanál v pásmu VHF (145 MHz) nebo třeba v pásmu L (1296 MHz). Pořád je to těch 12,5 kHz, uprostřed kanálu je nosná.

Ovšem dopplerovský posun nosné (a tedy s ní celého signálu) je v pásmu 145 MHz asi plus mínus 2 kHz během přeletu (maximum při relativní rychlosti přes 7 km/s), v pásmu 435 MHz to dělá plus mínus 6 kHz a v pásmu 2400 MHz to dělá plus mínus 30 kHz.

To znamená, že v pásmu VHF vám signál z LEO družice ujede o kus, v pásmu UHF o půl kanálu vedle a v pásmu S vám zmizí úplně do jiného kanálu.

Anténa, která signál přijímá, není nikdy tak úzkopásmová, aby to vadilo. Vstupní filtr normálně také ne. Problém je ale vnitřek přijímače a demodulátor, který signál přijímá. Demodulátor při špatném naladění začne dávat poškozený datový tok s větší chybovostí (jako když jemně rozladíte analogovou televizi), a vadí to tím víc, čím:

1. přenosová rychlost vyšší, tedy potřebné pásmo širší (čím rychlejší modulace, tím laicky řečeno rychlejší změny a častější hrany je třeba přenést a tedy vyšší odchylky frekvence kolem nosné)

2. frekvence vyšší, tedy větší vliv relativní změny

3. rychlost objektu vyšší, tedy větší relativní změna

Čili jedné komunikaci to vadí více a jiné méně.

Nicméně platí, že doppler již dnes není žádný problém. Pokud je signál dost silný, může se přijímač automaticky dolaďovat na nejsilnější signál. Pokud je dráha predikovatelná, dá se potřebný posun spočítat předem a ladit podle programu (a víte, že je to dobře). Kromě toho, dnešní SDR rádia přijímají celý vstupní frekvenční rozsah najednou a je čistě na softwarovém nastavení, kterou část přijímaného pásma budou demodulovat a jak. Přelaďování potom může být i softwarové.

Důležité je také zmínit, že jestliže přijímač "ujíždí" na jednu stranu a chceme obráceným směrem i vysílat, musíme vysílač rozladit "obráceně". Protože přijímač vidí pozemní stanici v pohybu a tedy také s dopplerem. A musí řešit ten samý úkol, nebo to uděláme za něj už při vysílání.

Rychlost 1 km/s by nevadila pípacímu majáku, ale byla by např. kritická pro analogový televizní přenos.


-=RYS=- - 31/8/2012 - 14:48

Jen pro doplneni.
http://www.uhf-satcom.com/sband/smart1150dpi.pdf
Vsimnete si uplinku/dwlinku, up 2kbps pri PSK je sirka pasma asi 6kHz. Pri 33kbps PSK dw to je asi 80kHz.
Oboji v pasmu S.



Co se tyce FMky na HAM satu LEO, tak jsem to naskladal do pameti po 5kHz kroku a cvakal pameti od vyssiho kmitoctu po nejnizsi kmitocet.
Pouzil jsem 5 pameti.
Kdyz se Amsat objevil , tak byl kmitocet FM nejvyssi, za 5 minut byl kmitocet presne na stredu kde palubni FM vysilac vysila a jak sat odleta, tak kmitocet klesa.


x - 31/8/2012 - 17:29

"Nicméně platí, že doppler již dnes není žádný problém. Pokud je signál dost silný, může se přijímač automaticky dolaďovat na nejsilnější signál. Pokud je dráha predikovatelná, dá se potřebný posun spočítat předem a ladit podle programu (a víte, že je to dobře)."

Toto vím i ze své osobní zkušenosti a tak jsem to nepsal - prostě obvody kmitočtové syntézy nejsou problém dnes ani pro běžně vybavené radioamatéry.

Takže jedině by mohl být příliš úskzý vstupní filtr co jsem tak veděl - nemůžete prostě najednou zpracovat jak silni signál rozhlasového vysílače a zároveň slabý signál z druhého kontinetu - proto musí být vstupním filtrem pásmo rozděleno.

Děkuji za upřesnění současné situace v této problematice.


DH - 31/8/2012 - 17:42

citace:
Takže jedině by mohl být příliš úskzý vstupní filtr co jsem tak veděl - nemůžete prostě najednou zpracovat jak silni signál rozhlasového vysílače a zároveň slabý signál z druhého kontinetu


Nemůžete vyrobit hardwarový filtr, který by byl zároveň hodně ostrý (měl úzkou hranici mezi pásmem propustnosti a pásmem zádrže) a zároveň měl malý průchozí útlum v propustném pásmu. Nikdo to neumí a kdoví jestli to je vůbec fyzikálně možné.

Ale na vstupu potřebujete mít signál co nejlepší, takže použijete jen nějaký preselektor, málo selektivní filtr. Čistě pro hrubé omezení abyste nezahltil čipy nějakým relativně blízkým vysílačem, jak uvádíte.

Jsou situace, které nemají řešení. Například být fyzicky v blízkosti silného zdroje a snažit se těsně frekvenčně vedle něj něco chytat. V takovém případě jakmile se to dostane do antény, už nic neuděláte. Proto potom jedině směrový příjem, pokud jsou to dostatečně vysoké frekvence (dnes naštěstí skoro vše) - dobrý a velký parabolický reflektor potlačí vše kromě hlavního laloku o 30 dB či ještě víc.


x - 31/8/2012 - 17:53

"takže použijete jen nějaký preselektor, málo selektivní filtr."
Zas tak uplně malo selektivní zřejmě není na vstupu zařízení pro pásmo 3,5Mhz - 3,8Mhz je opravdu jen docela úzký filtr - i když se šířkou asi řádově Mhz. Přesně to nemám k dizpozici - není to konstrukce mnou navržena.
Na vyšších frekvencích by tedy tato dosažitelná šířka pásma mohla již i vadit.


martinjediny - 1/9/2012 - 00:33

Zaujimave citanie.
Dakujem za komentare.
M:


Van Guld - 1/9/2012 - 22:59

Zdravím, asi se to už řešilo, ale já to nenašel.
Chci se zeptat, vesmírem sondy a lodě plují nebo letí? Jaká je správna terminilogie?


mrf - 1/9/2012 - 23:14

citace:
Zdravím, asi se to už řešilo, ale já to nenašel.
Chci se zeptat, vesmírem sondy a lodě plují nebo letí? Jaká je správna terminilogie?


Ani neletí ani neplují, pouze setrvávají ve
své inerciální soustavě nebo jsou ve fázi
přechodu na jinou (svou)inerciální soustavu..


alamo - 1/9/2012 - 23:28

dalo by sa povedať že "padajú"?


pospa - 1/9/2012 - 23:34

citace:
Chci se zeptat, vesmírem sondy a lodě plují nebo letí?
Kosmická loď (plavidlo) je logickým překladem anglického výrazu spaceship, spacecraft a nebo ruského kasmičeskyj karabľ.
Jelikož kosmické sondy a lodě při svém pohybu vesmírem po většinu času nejsou ve styku s žádným kapalným nebo plynným prostředím, je logičtější tento pohyb nazývat letem, raději než plavbou, i když název loď, nebo plavidlo by k tomu mohlo svádět. Také člen posádky, řídící pohyb lodi je nazýván pilot (většinou jejich původní profese) a ne kormidelník.
Myslím, že v češtině je běžně ustálená fráze "let kosmické lodi" (spaceflight, kasmičeskyj paljot).
Kosmická plavba by znělo spíš jako archaismus z dob před skutečným počátkem kosmického věku.


Alchymista - 2/9/2012 - 17:14

dovedené ad absurdum:
pohyb telesa v kozmickom priestore nemôže byť ani let, ani plavba, pretože oboje je odvodené od vytvárania vztlakovej sily, či už v režime statickom, podľa archimedovho zákona, alebo v režime dynamickom, odvodenom od bernouliho zákona o kontinuite prúdu, prípadne sa jedná o pohyb reaktívny odvodený od zákona o zachovaní hybnosti (ale to spravidla platí len po veľmi obmedzený čas).

Takže pohyb družice na obežnej dráhe alebo sondy v medziplnetárnom priestore je vrh vo vákuu, teda pohyb zotrvačný v neodporujúcom prostredí, ktorý je ovplyvnený len gravitačnými poliami.
tak.
[Upraveno 02.9.2012 Alchymista]


martinjediny - 2/9/2012 - 21:20

citace:
...Takže pohyb družice na obežnej dráhe alebo sondy v medziplnetárnom priestore je vrh vo vákuu, teda pohyb zotrvačný v neodporujúcom prostredí, ktorý je ovplyvnený len gravitačnými poliami.
tak.
[Upraveno 02.9.2012 Alchymista]

Potom ale najgenialnejsi termin navrhol alamo

I ked to je absurdita typu, co zacne robit dieta, ked sa narodi? ...starnut!

Tak kozmicka lod po vypnuti motorov zacne padat... (a to plati i pri medziplanetarnom lete, len gravitacnych poli nam akosi pribudne)

Ale zas ak zoberiem analogiu s vetronom, pokial je "draha padu" riaditelna, nazyva sa letom...


Beda - 8/3/2013 - 13:01

Dobrý den

Chci se zeptat, proč na SPACE40 má Sputnik 1 v kolonce SSC (předpokládám, že se jedná o katalogové číslo http://www.lib.cas.cz/space.40/HSSC.HTM) uvedenou dvojku?
Co je v tom případě jednička? Nějaká část nosné rakety?
Děkuji.
Beda


Agamemnon - 8/3/2013 - 13:21

jj, nosic je 1


friendly_allien - 8/3/2013 - 13:53

citace:
Dobrý den

Chci se zeptat, proč na SPACE40 má Sputnik 1 v kolonce SSC (předpokládám, že se jedná o katalogové číslo http://www.lib.cas.cz/space.40/HSSC.HTM) uvedenou dvojku?
Co je v tom případě jednička? Nějaká část nosné rakety?
Děkuji.
Beda


... na oběžné dráze lítal poslední stupeň rakety (č.1) a samotná družice Sputnik (č.2).

BTW: Protože ten stupeň byl mnohem větší, tak všichni co si mysleli, že vidí Sputnik, tak ve skutečnosti pozorovali ten stupeň. ;-)


Beda - 8/3/2013 - 14:38

citace:


... na oběžné dráze lítal poslední stupeň rakety (č.1) a samotná družice Sputnik (č.2).

BTW: Protože ten stupeň byl mnohem větší, tak všichni co si mysleli, že vidí Sputnik, tak ve skutečnosti pozorovali ten stupeň. ;-)


Díky. A když Sputnik 2 má trojku, tak v jeho případě se nosič tak vysoko nedostal?


fritz.lochmann - 8/3/2013 - 16:06

citace:
... ak zoberiem analogiu s vetronom, pokial je "draha padu" riaditelna, nazyva sa letom...
Vetroň k letu využíva aj termické prúdy, teda letí hore - stúpa, prekonáva gravitáciu. Čo bude analógiou vetroňa v tomto prípade? Že by slnečná plachetnica?

[Upraveno 08.3.2013 fritz.lochmann]


Patek_Luboš - 8/3/2013 - 17:10

citace:

A když Sputnik 2 má trojku, tak v jeho případě se nosič tak vysoko nedostal?


Sputnik 2 (PS-2) zůstal spojený (podle plánu) s posledním stupněm rakety.


David - 8/3/2013 - 17:51

V případě Sputniku -1 a 2 létaly po orbitě ještě kuželovité kryty užitečného zatížení, takže číslování jim dalo č. 1, pak se oddělila družice /s-1/, resp u S-2 neoddělila -č.2 a družicí /S-1/opuštěný nosič -č.3.


jamsed - 8/3/2013 - 21:32

citace:
V případě Sputniku -1 a 2 létaly po orbitě ještě kuželovité kryty užitečného zatížení, takže číslování jim dalo č. 1, pak se oddělila družice /s-1/, resp u S-2 neoddělila -č.2 a družicí /S-1/opuštěný nosič -č.3.


Omyl.
Do katalogu, který vede NORAD, se zapisují objekty, u kterých je spolehlivě zjištěna totožnost a základní parametry dráhy. Tudíž č. 1 získal horní stupeň rakety a číslo 2 samotná družice Sputnik 1 - kryt užitečného zatížení nebyl do katalogu vůbec zapsán (v roce 1957 nebyly prostředky pro přesné zaměření, opticky nebyl pozorovatelný).
Pořadové číslo katalogu vůbec nesouvisí s pořadím v jakém se tělesa na oběžné dráze oddělují (a to platí dodnes!). Např. fragment 1988-113E má číslo 19711 a hlavní družice Kosmos 1985 mez. ozn. 1988-113A má číslo 19720.
Na počátku kosmické éry taky nebyla pro záznam do katalogu přesná pravidla a tak třeba sonda Pioneer 1, která startovala v roce 1958, byla do katalogu zapsána až v roce 1961 pod číslem 110.

Vlastně existuje jeden objekt, u kterého není zjištěn původ - je to objekt s číslem 05310, který je COSPARem evidován pod mez. označením 1971-000E. [Upraveno 08.3.2013 jamsed]


Erakis - 8/3/2013 - 21:36

citace:
V případě Sputniku -1 a 2 létaly po orbitě ještě kuželovité kryty užitečného zatížení, takže číslování jim dalo č. 1, pak se oddělila družice /s-1/, resp u S-2 neoddělila -č.2 a družicí /S-1/opuštěný nosič -č.3.

Aerodynamické kryty síce na orbite boli, avšak nikdy neboli oficiálne katalogizované. A už vyššie bolo spomenuté, že číslom 1 bol označený nosič, nie Sputnik, takže si david necucaj údaje z prsta. A označenie 1 pre nosič bolo zvolené kvôli tomu, že vtedy to bolo braté tak, že nosič sa dostal na orbitu ako prvý objekt a až potom sa oddelil druhý objekt - samostatný satelit (nebolo to teda kvôli zámene, ako písal friendly_allien).


Ervé - 12/3/2013 - 10:34

Mám dotaz ohledně radiokomunikace: Dá se spočítat, jak velkou anténu a výkonný vysílač by musela mít automatická sonda u Alfy Centauri (4,37 sv.let), abychom od ní dostávali smysluplná data? Řekněme rychlostí 32 a 320 bitů za sekundu na 70m anténu DSN. Ze slabého vysílače a malé antény Voyagerů dostáváme pořád data dost velkou rychlostí (i když na řádově menší vzdálenosti), kvůli šumu asi jednoduchá extrapolace nefunguje.


člověk - 12/3/2013 - 11:18

Ervé:

zkusil jsem to dát do kalkulátoru spojení. Jsou to všechno orientační a ilustrační hausnumera, tak prosím nechytejte za slovo. Řadu parametrů je těžké odhadnout.

- pásmo Ka (frekvence 26 GHz)
- výkon vysílače sondy 10kW do antény (tj. příkon vysílače cca 20kW)
- anténa na sondě 80 dBi (cca 60m parabolický reflektor namířený k Zemi s přesností cca desetin stupně, perfektně tvarově stálý)
- uvažovaná šířka pásma pro posouzení šumu 150 Hz (cca desítky bit/s)
- neuvažován žádný zisk kódování (samoopravný kód) - umožní snížit potřebný výkon či rozměry antény)
- pozemní anténa 85 dBi (cca DSN)
- vzdálenost 3,4 ly = 3,2e16 metrů
- citlivost přijímače na Zemi -140 dBm (mnohem lepší než běžně dostupné)

v takové konfiguraci vychází, že přenos desítek bitů za sekundu by měl být realistický (cca 10 dB rezerva výkonu zdroje), velmi pravděpodobně by bylo možné snížit řádově potřebný výkon vysílače nebo o něco rozměr antény na sondě

neumím posoudit, jak by pomohly chlazené přijímače, digitální zpracování signálu apod.

hodně by pomohla fázovaná anténa na sondě - jako Aegis

můj orientační závěr - řádově jednotky kW výkonu, řádově desítky metrů anténa na sondě - a desítky až stovky bitů/s k nejbližší hvězdě


wintermute- - 12/3/2013 - 22:41

citace:
Mám dotaz ohledně radiokomunikace: Dá se spočítat, jak velkou anténu a výkonný vysílač by musela mít automatická sonda u Alfy Centauri (4,37 sv.let), abychom od ní dostávali smysluplná data? Řekněme rychlostí 32 a 320 bitů za sekundu na 70m anténu DSN. Ze slabého vysílače a malé antény Voyagerů dostáváme pořád data dost velkou rychlostí (i když na řádově menší vzdálenosti), kvůli šumu asi jednoduchá extrapolace nefunguje.


Ono treba asi povedat, ze to co chytame z Voyagerov nieje uplne normalna komunikacia v zmysle, antena -> zosilovac -> demodulator -> data. Dovodom je, ze uz sa neda hovorit o pomere signal/sum. Sum dosahuje vyssiu uroven ako signal. Prijimany sum, sa nahrava, Sampluje na digital s vysokou presnostou, Vzhladom na to, ze vieme ako by mal signal vyzerat a ako nie, tak pomocou superpocitacov sa "demoduluje". Pomocou FFT (rychlej furierovej transformacie) sa z toho sumu doluje nieco co pripomyna signal. V ziadnom pripade s nejedna o real-time proces.

Nemyslim si, ze by bola pre nas mozna komunikacia na taku vzdialelnost, ak sa bavime o vysielacom vykone radovo v stovkach watov.


Alchymista - 12/3/2013 - 23:59

citace:
hodně by pomohla fázovaná anténa na sondě - jako Aegis

No neviem... Ploché pasívne fázované antény majú hodne zaujímavých a užitočných vlastností, ale na toto sa nehodia, pretože majú aj pomerne veľké šumové čísla. A vo fázovacích prvkoch sú aj dosť veľké straty s prvky samotné sú zdrojom šumu. Hlavná výhoda je vo veľmi rýchlom skenovaní priestoru, to ale na diaľkovej sonde nepotrebuješ, tam ide naopak o veľmi presné zamierenie a veľmi úzku vyžarovaciu charakteristiku, čo sa dá zrejme ľahšie dosiahnuť "klasickou" parabolou.


člověk - 13/3/2013 - 08:19

citace:
čo sa dá zrejme ľahšie dosiahnuť "klasickou" parabolou


Asi by si to zasloužilo podrobnější rozbor, ale v případě minimálně pasivní fázované soustavy máte určitě pravdu.

Parabolický reflektor bude mít určitě menší šumovou teplotu a ve stavu beztíže bez podpůrné konstrukce vyjde mnohem lehčí a jednodušší - pro sondu je tedy výhodnější.


Machi - 13/3/2013 - 11:26

V projektu Longshot počítali tuším s dvou nebo čtyř metrovým dalekohledem a laserovou komunikací.


Alchymista - 13/3/2013 - 11:34

Odborný rozbor by bol určite zaujímavý - ale urobiť to je mimo moje schopnosti a vedomosti.

Udržanie tvaru antény a podporná konštrukcia bude podľa mňa zhruba rovnako náročné pre parabolu i plochú anténu. Bavíme sa totiž o anténach s plochou pár sto metrov štvorcových - a udržať potrebnú "rovinnosť" plochej antény je pri ploche porovnateľnej s plochou futbalového ihriska hodne náročná úloha, na splnenie ktorej by už zrejme boli potrebné "aktívne" mechanické prvky, rovnako ako na udržanie správneho tvaru paraboly.
Ono to u plochých antén systémov Aegis vyzerá "celkom jednoducho", ale u systému Patriot či S-300 je dobre vidieť, že ploché antény sú celkom hrubé - "doska" či rám, v/na ktorom sú antény umiestnené sú hrubé minimálne tak 30 cm, skôr viac, takže rám antény je celkom masívny a u námorného systému AN/SPY-1 Aegis alebo stacionárnych pozemných radarov PRO je skrytý v samotnej konštrukcii lode či železobetónovej stavby.
Navyše, pasívne ploché antény majú nevyhnutne aj zložitý vlnovodový trakt s aktívne riadenými prvkami k jednotlivým žiaričom (a ten musí byť "presný" v zmysle fázových a šumových vlastností) - u paraboly je žiarič obvykle len jeden, takže stačí jeden vlnovod.



člověk - 13/3/2013 - 12:19

Pasívní fázované ploché antény jsou zcela mimo diskusi. Velké, komplikované, těžké, mechanicky náročné - jejich jedinou výhodou je skutečně jen možnost rychlého elektronického zaměření "jinam".

Parabolický reflektor musí udržet přesný tvar bezpodmínečně, jinak v dané konfiguraci přijímače v ohnisku nevytvoří dostatečně úzký svazek (tedy zisk a šumovou teplotu).

Aktivní fázovaná soustava si s geometrickým uspořádáním nemusí lámat příliš hlavu - deformace se dají kompenzovat korekcí fázového řízení jednotlivých prvků, a to lze dělat rychle, softwarově a tedy snadno. Anténa je schopná "autokalibrace". VF energie se nemusí přenášet na dálku z centrálního zdroje, vytváří se lokálně v jednotlivých prvcích a k prvkům se přivádí "jen" napájení a datový tok.

Pokud by se technologicky, miniaturizací a nanotechnologií podařilo vyrobit aktivní mnohatisícprvkovou anténu, že by ji bylo možno na lehké konstrukci napnout jako "tlustší plátno", byla by bezpochyby praktičtější a nejvíce flexibilní. Ve vesmíru překousneme konstrukce mnohem křehčí, než u mobilních raketových komplexů S-XXX apod., které jsou určené pro přesun terénem a musí si své vlastnosti uchovat v boji.

Upozorňuji, že to je jen teoretizování. Zatím něco podobného nikdo vyrobit ani vzdáleně neumí. Posouzení skutečných vlastností při použití DNES dostupných technologií samozřejmě musí dopadnout jinak.

Dále nevím jistě, jak je to se šumovými vlastnostmi aktivní fázované soustavy při příjmu. Důležitý rozdíl mezi pozemním příjmem a kosmickým příjmem je také ten, že na Zemi obklopuje anténu z polosféry stran teplý a šumící povrch, zatímco v kosmu jsou to bodové zdroje (hvězdy), těleso sondy a šumová teplota hvězdného pozadí, která je nízká. Je proto otázka, jakou má faktor šumu při této aplikaci váhu.


člověk - 13/3/2013 - 14:30

nějaké informace k tématu:
http://www.microwavejournal.com/articles/17992-evolution-of-aesa-radar-technology


honza78 - 9/6/2014 - 03:39

Zdravím, měl bych dva laické dotazy:

1. Jaké úpravy by potřeboval americký raketoplán, aby byl schopen doletět k ISS v řádu hodin tak, jako nyní ruské lodě?
2. Když nějaká loď dokuje u ISS či od ní oddokovává, tak toto nelze dělat ve směru či proti směru letu ISS, protože by tím loď změnila svou rychlost na orbitální dráze?

Děkuji.


ales - 9/6/2014 - 08:04

Ahoj.

1. Myslím, že raketoplán by pro rychlejší profil letu k ISS nepotřeboval žádné úpravy. Jde v podstatě jen o změnu "metodiky" (výběr vhodných "startovních oken"). Žádné "palivo navíc" není potřeba. Ani Progressy a Sojuzy nepotřebovaly žádné úpravy.

2. Přílet k ISS je podle mne teoreticky možný z libovolného směru, protože vzájemné rychlosti spojovaných těles jsou tak malé (oběžné dráhy jsou prakticky shodné), že jsou zvládnutelné i slabými manévrovacími motorky s přijatelnou spotřebou paliva. Z praktických důvodů se na ISS používají jen směry ve stejné rovině dráhy (tedy "shora", "zezdola", "zepředu" a "zezadu").


Agamemnon - 9/6/2014 - 10:03

atv prilietava po smere letu, iirc


cernakus - 9/6/2014 - 14:15

honza78
1) Pouze SW profily, jako u Sojuzů.
2) Proč si to myslíš?


honza78 - 9/6/2014 - 17:34

Díky všem!

1) Netušíte prosím, proč to tedy tenkrát nezkusili? Vím, že dvoudenní let k ISS využívali ke kontrole tepelného štítu, ale i tak mi přijde, že úspora času by to byla celkem podstatná.
2) Intuitivně mi přišlo, že změna rychlosti ve směru nebo v protisměru oběhu kolem Země je energeticky náročnější než posun do boku či nahoru / dolů.


cernakus - 9/6/2014 - 17:41

2) Není. Dokování a od-dokování se neprovádí nijak závratnou rychlostí. Obvykle se jedná o cm/s. Ve vesmíru je na všechno čas. Oproti tomu orbitální rychlost všech těles je u ISS 7,7km/s. Neboli 770000cm/s. Tedy odpojení či připojení vyžaduje zanedbatelné dV oproti orbitálním manévrům.

Tady se podívej, jak takové připojení vypadá v reálu


je to pěkná nuda IMHO.

1) Protože kosmický program je zkostnatělý a zároveň je pilotovaná kosmonautika nebezpečná a pod přísným drobnohledem. O zrychleném manévru Sojuzu se hovořilo už minimálně v dobách existence Miru a přesto se k tomu odhodlali až po změně avioniky z digitálně-analogové na plně digitální a jejím několikaletém prověření.

Nejde ani tak o to, že by mohlo dojít k umrtí, ale zrychlený let k ISS má mnohem přísnější parametry a kratší okna. Takže nikdo nechtěl riskovat, že STS tato mine a bude muset předčasně přistát pro nedostatek deltaV (obecně není u kosmických lodí velká rezerva, některé orbitální manévry tak nelze opakovat). To by totiž vyletělo minimálně 500 mega oknem. [Upraveno 09.6.2014 cernakus]


Ervé - 10/6/2014 - 07:50

Pokud vím, jedním z důvodů delší trajektorie je snížení stresu (moc úkolů hned první den), dál je dán čas astonautům/kosmonautům, aby se vyblinkali ještě v raketoplánu/sojuzu a neznečišťovali přístroje stanice. Dalším důvodem byla úspora paliva - stačí mírnější/pomalejší manévry, na druhou stranu větší spotřeba stabilizací. Ke Skylabu létali Apolla krátkou trasou při všech letech.


Alchymista - 11/6/2014 - 17:19

V danej dobe bol "rýchly" prílet Apolla ku Skylabu možný vďaka niekoľkým veciam:
- Apollo muselo byť schopné spojiť sa so svojim lunánym modulom "samostane", bez výraznej podpory zo Zeme, v pomerne veľkom rozsahu vzájomných dráh, takže jeho palubné vybavenie bolo na manévre pre stretnutie dvoch telies na orbite špecificky zamerané
- riadiaci a pohonný systém Apolla bol výkonnejší (a zrejme aj celkovo presnejší - primárne určenie pre lety k Mesiacu si to proste vyžaduje), ako u Sojuzu, takže zblíženie telies prebiehalo (mohlo prebiehať) s vyššími zrýchleniami pri manévroch a vyššími rýchlosťami


martinjediny - 2/9/2014 - 11:43

Existuje system jednotneho znacenia pre lietadla a kozmonautiku?
Ma podobne ako v energetike system KKS aj v letectve oznacenie systemov a komponentov univerzalnu skratku, vdaka ktorej je okamzite jasne z oznacenia ze ide napr. o ventil v privodnom palivovom potrubi k motorom s automatickym pohonom?

napriklad
http://www.kks-kod.cz/ ako uvadza oznacenie =K01HAC10AA001 ze ide o prvu armaturu s automatickym pohonom na ekonomizeri kotla K10.

( resp. pokrocilejsie systemy znacenia v energetike www.rds-pp.cz)


dodge - 2/9/2014 - 12:09

citace:
Existuje system jednotneho znacenia pre lietadla a kozmonautiku?
Ma podobne ako v energetike system KKS aj v letectve oznacenie systemov a komponentov univerzalnu skratku, vdaka ktorej je okamzite jasne z oznacenia ze ide napr. o ventil v privodnom palivovom potrubi k motorom s automatickym pohonom?

napriklad
http://www.kks-kod.cz/ ako uvadza oznacenie =K01HAC10AA001 ze ide o prvu armaturu s automatickym pohonom na ekonomizeri kotla K10.

( resp. pokrocilejsie systemy znacenia v energetike www.rds-pp.cz)


V letectví ani v kosmonautice neexistuje systém jednotného značení komponentů a systémů. Existují pouze individuální systémy označování jednotlivých výrobců letadel nebo komponentů či systémů, ale systém srovnatelný se systémem kks používaným v energetice, v těchto oborech neexistuje. [Upraveno 02.9.2014 dodge]


martinjediny - 4/9/2014 - 23:04

citace:
Existují pouze individuální systémy označování jednotlivých výrobců letadel nebo komponentů či systémů, ale systém srovnatelný se systémem kks používaným v energetice, v těchto oborech neexistuje. [Upraveno 02.9.2014 dodge]

Dik, celkom ma to prekvapilo, aspon od cias ISS by som ocakaval nejaku unifikaciu...


ejencik - 14/9/2014 - 09:15

Zajimalo by mne jak fungovaly systemy orientace v prvnich kosmickych lodich Vostok, Mercury, Gemini. V ruznych zdrojich je uvadena orientace podle hvezd, ale to si na tehdejsi dobu neumim prestavit, jeste bych rozumnel zamereni podle slunce, mesice nebo obzoru zeme. Diky za vysvetleni,odkaz nebo klicove slovo pro vyhledavani.


honza78 - 10/10/2014 - 14:49

Dobrý den, mohu se zeptat na význam křivek na obrazovce v Mission Control Center během startu raketoplánu? Jde o čas 4:59 v tomto videu:t=4m58s

Předpokládám, že na ose x je horizontální vzdálenost a na ose y vertikální vzdálenost od místa startu a oranžová křivka znázorňuje skutečný letový profil, ale co ostatní křivky? Některé z nich mají např. souvislost s RTLS? Díky!


Alchymista - 12/10/2014 - 00:22

Je možné, že sú vopred prepočítané dráhy pre rôzne varianty vyvedenia na orbitu a rôzne varianty prerušenia štartu a RTLS manévru.
Vyzerá to, akoby vyznačovali "krajné medze" podmienok, kedy sa dané manévre dajú uskutočniť a zrejme to bolo určené ako pomôcka pre rýchle rozhodovanie v prípade problémov.


pepek - 27/12/2015 - 18:54

????
Na tomto portálu je určitě par chytrých lidí , kteří mi vysvětlí , proč se vymýšlí monstrózní skafandry, když pro práci ve vakuu a v těžkých podmínkách byl již před 50 lety vyvinuty lehký skafandr na zips. Bez omezení pohybu – poskoky, výskoky, kliky, práce s nástroji, golf, řízení vozidla, atd. Ba dokonce si je mohli samy zavléct v omezeném prostou(LM VM) a ve stižených podmínkách (neklidná hladina moře).


feature=youtu.be


pospa - 27/12/2015 - 20:38

citace:
proč se vymýšlí monstrózní skafandry, když pro práci ve vakuu a v těžkých podmínkách byl již před 50 lety vyvinuty lehký skafandr na zips.

Ze stejného důvodu, jako se vyvíjí nové raketové nosiče, kosmické lodě, letadla, auta, lodě, počítače, atd. atd...
Vývoj zkrátka nelze zastavit. Stále se budeme snažit vyrobit věci s lepšími parametry, vyšší užitnou hodnotou, příjemnější pro uživatele, efektivnější pro provozovatele, levnější pro dodavatele i koncového zákazníka...
Je ale pravdou, že do toho taky nezřídka a nemalou měrou vstupují i další faktory, jako je politika, státní zakázky, udržitelná zaměstnanost, mezinárodní vztahy, atd. atd, které jdou většinou proti těm prvně jmenovaným důvodům. Ale vývoj nových variant a verzí toho už dříve vyvinutého podněcují také.
Ale tak už to na světě chodí a dá se to změnit jen málo ... vlastně dá, ale to jsou spíš jen vzácné výjimky, jako například SpaceX, kde mimo jiné také vyvíjejí zcela nový vlastní skafandr.


PS: vývoj nových skafandrů PXS a Z-2 si zadala NASA, aby vyhovovaly novému standartu pro práci vně vyvíjeného Space Exploration Vehicle. Jakýkoli ze starších typů amerických skafandrů pro tento účel není použitelný.


NovýJiřík - 27/12/2015 - 20:50

proč se vymýšlí monstrózní skafandry, když pro práci ve vakuu a v těžkých podmínkách byl již před 50 lety vyvinuty lehký skafandr na zips.



Ze stejného důvodu, jako se vyvíjí nové raketové nosiče, kosmické lodě, letadla, auta, lodě, počítače, atd. atd...
Vývoj zkrátka nelze zastavit. Stále se budeme snažit vyrobit věci s lepšími parametry, vyšší užitnou hodnotou, příjemnější pro uživatele, efektivnější pro provozovatele, levnější pro dodavatele i koncového zákazníka...

Tak tuhle odpověď jsem moc nepochopil. V čem jsou moderní skafandry typu "sněhulák Michelin", ve kterých se skoro nedá pohnout, příjemnější, levnější, efektivnější, s lepšími parametry a vyšší přidanou hodnotou?


pospa - 27/12/2015 - 21:03

citace:
V čem jsou moderní skafandry typu "sněhulák Michelin", ve kterých se skoro nedá pohnout, příjemnější, levnější, efektivnější, s lepšími parametry a vyšší přidanou hodnotou?

Odpověď jsem původně pojal spíš obecně - proč se vyvíjí něco nového a nepoužije se 50 let stará technologie (i když výjinky existují, že )
Později jsem původní příspěvek doplnil o vysvětlení ke konkrétním novým skafandrům.
Nehodnotím, zda-li jsou/budou PXS a Z-2 lepší či horší, než starší typy skafandrů, to ukáže až jejich využití v praxi, ale rozhodně jsou vyvíjeny a testovány podle jiného zadání NASA, než byl vyvíjen například měsíční A7L.


pepek - 28/12/2015 - 22:59


citace:
Tak tuhle odpověď jsem moc nepochopil. V čem jsou moderní skafandry typu "sněhulák Michelin", ve kterých se skoro nedá pohnout, příjemnější, levnější, efektivnější, s lepšími parametry a vyšší přidanou hodnotou?


Já to taky nepochopil. Plně s vámi souhlasím.

Skafandr A7L Apollo , plně funkční vážil 91 kg .
Novodobý skafandr na ISS , váží 145kg.
Jeho pohyblivost je omezena jen na pohyb rukou a v pásu.
Chtěl bych vidět, jak by se, s taktovým skafandrem, dostal do LM.



tycka - 28/12/2015 - 23:25

Třeba mohou mít lepší protimeteroritickou ochranu tak i ochranu před kosmickým zářením - to jen píšu proč tomu tak může být (tedy nevím to přesně - jen odhaduji z jejich vzhledu). Nové věci, které mohou být i větší a hmotnější se často musí vyvíjet jen proto, že se prostě změnili - tedy zpřísnili příslušné normy, které prostě musí splňovat.
Sice je to tedy dle mne méně pohyblivé, ale zato to více ochrání zdraví kosmonautů.
[Upraveno 28.12.2015 tycka]


Andy - 28/12/2015 - 23:47

citace:

Skafandr A7L Apollo , plně funkční vážil 91 kg .
Novodobý skafandr na ISS , váží 145kg.
Jeho pohyblivost je omezena jen na pohyb rukou a v pásu.
Chtěl bych vidět, jak by se, s taktovým skafandrem, dostal do LM.



Ono se staci zamyslet napr. nad tim, na kolik pouziti byl vyvinut AL7 (max 3 vychazky) a na kolik ten na ISS. Pripadne nad jeho mirou pouziti jinou osobou, atd atd...


Alchymista - 29/12/2015 - 01:48

Ako píše Andy - a ďalej:
- pracovné tlaky, dýchacia zmes, doba prípravy kozmonauta pred výstupom, doba "prebrethingu",
- nutnosť zmien tlaku v stanici pred výstupom?, je potrebný pomocník pri obliekaní? je skafander individuálny (na mieru pre daného kozmonauta), alebo univerzálny?, v akom rozsahu? čo je "individuálne" u univerzálneho skafandru?
- odpor skafandru voči pohybu, rozsah pracovného a manipulačného priestoru (kam kozmonaut v skafandi dosiahne), možné uhly otočenia v zápästí, v lakti, v ramene, v členku, v kolene a v bedre
- manipulačné možnosti rukavíc, životnosť a odolnosť rukavíc
- životnosť skafandru po "rozbalení", počet výstupov/cyklov, životnosť skafandru v otvorenom vesmíre (A7L ~24 hodín, A7LB ~48 hodín)
- výkon klimatizácie (človek v kľude produkuje ~120W, šprintér až ~1,6kW), riadenie vlhkosti prostredia, systém odstraňovania CO2
- elektrická a elektronická výstroj - spojenie, kamery, individuálne osvetlenie, schopnosti a možnosti automatiky skafandru, informačný systém (skafandru samotného, o pracovnej úlohe, o okolitom prostredí...)
- pohodlie pri použití

---------------
Apollo A7L: celková hmotnosť 91 kg, pracovný tlak 25,5 kPa, Life Support 6 hod + 30 min
Apollo A7LB: celková hmotnosť 96,2 kg, pracovný tlak 25,5 kPa, Life Support 7 hod + 30 min
---------------
EMU: celková hmotnosť 115 kg, pracovný tlak 29,6 kPa, Life Support 8 hod + 30 min
Enhanced EMU: celková hmotnosť 124 kg(STS)/145 kg(ISS), pracovný tlak 29,6 kPa, Life Support 8 hod + 30 min
---------------
Orlan-MK: celková hmotnosť 120 kg, pracovný tlak 40 kPa, doba autonomnosti 7 hod + ? (skutočná doba práce až 8+ hod)
Orlan-MKS: celková hmotnosť 110 kg, pracovný tlak 40 kPa, doba autonomnosti 7 hod + ?
+ vhodný pre kozmonautov s telesnou výškou od 165 do 190cm
+ životnosť 5 rokov
+ klimatizácia zvláda stredný telesný výkon 300kcal/h (~350W) a maximálne 600kcal/h (~700W)
---------------
Krečet (sovietsky "mesačný", cca 1969): celková hmotnosť 106 kg, pracovný tlak 40 kPa, doba autonomnosti 10 hodin, pracovná životnosť 48 hodín

[Upraveno 29.12.2015 Alchymista]


NovýJiřík - 29/12/2015 - 13:10

citace:
Třeba mohou mít lepší protimeteroritickou ochranu tak i ochranu před kosmickým zářením - to jen píšu proč tomu tak může být (tedy nevím to přesně - jen odhaduji z jejich vzhledu). Nové věci, které mohou být i větší a hmotnější se často musí vyvíjet jen proto, že se prostě změnili - tedy zpřísnili příslušné normy, které prostě musí splňovat.
Sice je to tedy dle mne méně pohyblivé, ale zato to více ochrání zdraví kosmonautů.

OK. Nevařme z vody a počkejme, co nám, doufejme, že brzy, představí SpaceX. Možná se pak nestačíme divit. Anebo třeba taky ne, uvidíme.


kopapaka - 29/12/2015 - 13:35

NovýJiřík: Jenže SpaceX zatím ukázalo ve videu záchranný skafandr pro Dragon a tam by s trochou nadsázky stačil stratosférický skafandr z MiGu-21...

Jinak nechápu smysl otázky - měsíční skafandr neměl zabudované nouzové záchranné trysky se zásobou paliva a další věci. Skafandr pro EVA na ISS zas nepotřebuje pohyblivé nohy...


NovýJiřík - 29/12/2015 - 14:26

Jenže SpaceX zatím ukázalo ve videu záchranný skafandr pro Dragon a tam by s trochou nadsázky stačil stratosférický skafandr z MiGu-21...



Jenže nevěřím tomu, že by SpaceX nepracovala i na skafandru venkovním, už proto, aby její kosmonauti při cestách na ISS nebyli v případě potřeby výstupu z lodi závislí na místních zásobách. O cestách kamkoliv dál ani nemluvě.


martinjediny - 18/3/2017 - 17:29

citace:
... Nevíte někdo náhodou jak je to s omezením průměru vynášeného nákladu? Zda se může udělat nástavba například s dvoj, nebo trojnásobkem průměru nosné rakety?

neexistuje dovod aby to teoreticky neslo
problem je u existujucich rakiet, ktore su stavane na nejaku velkost. najvacsi vplyv ma zmena priemeru krytu.
s nou suvisi hmotnost AD krytu, tazisko a aerodynamicke namahanie.
A s tym spojeny profil letu a znizenie nosnosti.


Kedze technologie maju nejaku obvyklu mernu hmotnost, tak sa da predpokladat, ze vacsi vynasany satelit bude mat vacsiu hmotnost, zatial co vacsi AD kryt nosnost zmensi.

preto pokial nepride vyslovene atypicky naklad, tak nepredpokladam, ze by malo mat zmysel uvazovat nad vacsimi krytmi, ako s akymi bola raketa naprojektovana.

Ak si pozries Alesove simulacie, tak najvacsi problem nebude vo velkosti straty rychlosti pri velkom priemere, ale zo zmeny AD namahania a struktralnej pevnosti rakety. preto bude musiet byt zmeneny profil letu a ten moze mat zasadny vplyv na nosnost.
rovnako nemusi byt ani zasadny problem s narastom hmotnosti krytu, nakolko hovorime o prvom stupni, kde je tolerancia k C realtivne vysoka.


Libertarián - 18/3/2017 - 17:48

citace:
citace:
... Nevíte někdo náhodou jak je to s omezením průměru vynášeného nákladu? Zda se může udělat nástavba například s dvoj, nebo trojnásobkem průměru nosné rakety?

neexistuje dovod aby to teoreticky neslo
problem je u existujucich rakiet, ktore su stavane na nejaku velkost. najvacsi vplyv ma zmena priemeru krytu.
s nou suvisi hmotnost AD krytu, tazisko a aerodynamicke namahanie.
A s tym spojeny profil letu a znizenie nosnosti.


Kedze technologie maju nejaku obvyklu mernu hmotnost, tak sa da predpokladat, ze vacsi vynasany satelit bude mat vacsiu hmotnost, zatial co vacsi AD kryt nosnost zmensi.

preto pokial nepride vyslovene atypicky naklad, tak nepredpokladam, ze by malo mat zmysel uvazovat nad vacsimi krytmi, ako s akymi bola raketa naprojektovana.

Ak si pozries Alesove simulacie, tak najvacsi problem nebude vo velkosti straty rychlosti pri velkom priemere, ale zo zmeny AD namahania a struktralnej pevnosti rakety. preto bude musiet byt zmeneny profil letu a ten moze mat zasadny vplyv na nosnost.
rovnako nemusi byt ani zasadny problem s narastom hmotnosti krytu, nakolko hovorime o prvom stupni, kde je tolerancia k C realtivne vysoka.




Díky za odpověď.
Napadlo mě to v souvislosti s nafukovacím modulem od Bigelow Aerospace s tím, že tam spíše než o hmotnost půjde o rozměry. Takže vlastně nic nebrání tomu, aby se konstruovaly a vynášely na nízkou oběžnou dráhu velké objekty typu Bigelow, které by mohly být určené pro kosmické turisty.


Alchymista - 18/3/2017 - 19:45

Pri danom priemere základne (vrcholu druhého/tretieho stupňa) sa nedá aerodynamický kryt príliš zväčšovať. So zväčšovaním priemeru sa totiž dosť výrazne mení namáhanie zadnej časti aerodynamického krytu, nielen v absolutnej hodnote, ale aj spôsobom namáhania, a tak rýchlo rastú požiadavky na pevnosť krytu. Plus vznikajú komplikácie okolo oddelovania veľkopriemerového krytu.

Pri veľkých priemeroch krytov existujúcich (a historických) rakiet si možno všimnúť, že existuje istá snaha zväčšovať aj priemer horného stupňa pri jeho súčasnom skrátení, teda smaha potrebný priemer dosiahnuť v dvoch krokoch.


martinjediny - 18/3/2017 - 22:10

citace:
...Pri veľkých priemeroch krytov existujúcich (a historických) rakiet si možno všimnúť, že existuje istá snaha zväčšovať aj priemer horného stupňa pri jeho súčasnom skrátení...


urcite je jednoduchsie naprojektovat raketu s velkym krytom, ako upravovat jestvujucu z maleho krytu na velky priemer...

navyse kryty su vakuovane, takze ide vlastne o tlakovu nadobu a nie len kryciu skrupinku. a pre tlakove nadoby je charakteristicke ze pomer hmotnosti nadoby k objemu je rovnaky bez ohladu na velkost.
objem sice rastie s tretou mocninou a plocha s druhou, ale tiez mi rastie aj hrubka steny, takze dohromady je to pomer tretej mocniny k tretej.


citace:

Napadlo mě to v souvislosti s nafukovacím modulem od Bigelow Aerospace s tím, že tam spíše než o hmotnost půjde o rozměry. Takže vlastně nic nebrání tomu, aby se konstruovaly a vynášely na nízkou oběžnou dráhu velké objekty typu Bigelow, které by mohly být určené pro kosmické turisty.


ak mam podobne kryty a zvacsim kryt dvojnasobne, tak hmotnost mi automaticky stupne 8 nasobne.
sucasne mi 4nasobne stupne AD tlak a navyse pride k problemu, ktory spominal alchymista, ze spodok krytu nie je podoprety po celej ploche a na precnievajucej nemalej plohe vznikaju podtlaky a namahania, ktore som predtym nemusel riesit, co je dalsi narast hmotnosti...

urcite sa to riesit da, aj pokles nosnosti sa da urcit
a so spominanym Bigelovom 330 sa jednoducho mohlo dojst k tomu, ze uz by nemusela napr. stacit znizena nosnost.

B330 ma 20ton.
FH ma 54ton na LEO, co hmotou je dost, ale pre Bigelowa asi nizko, a na GTO ma nosnost 22 ton. to je hmotou na hrane, ale zas asi vyssie, ako potrebuje.
takze nosnost medzi tym, na drahu medzi tym, by vyhovovala zrejme aj s rezervou.

ale ak by zvacsovali kryt tak nosnost by mohla klesnut o tolko, ze na niektore drahy by uz FH vyniest B330 nevladal...

ale cele mi to pride este dost vzdialene a prilis vela neznamych, takze dovodov i buducich rieseni moze byt skutocne vela...


HonzaB - 19/3/2017 - 18:54

citace:
... navyse kryty su vakuovane, takze ide vlastne o tlakovu nadobu a nie len kryciu skrupinku. a pre tlakove nadoby je charakteristicke ze pomer hmotnosti nadoby k objemu je rovnaky bez ohladu na velkost.
objem sice rastie s tretou mocninou a plocha s druhou, ale tiez mi rastie aj hrubka steny, takze dohromady je to pomer tretej mocniny k tretej.


jen poznámka: Aerodynamické kryty nejsou vakuované a tedy tato úvaha neplatí.
Pouze u citlivých nákladů se uvnitř udržuje suchý vzduch a konstantní teplota. V operačních manuálech některých raket je to rozebíráno podrobněji včetně průběhu poklesu tlaku při letu rakety ...


martinjediny - 19/3/2017 - 23:38

citace:
jen poznámka: Aerodynamické kryty nejsou vakuované a tedy tato úvaha neplatí...

jeden prednasajuci, co sa zucastnil vypustania satelitu ako technik, ma presviedcal, ze AD kryt je vakuovany, ale nijak som si to neoveroval. ani ci vobec, alebo ci len v spec. pripadoch.
btw. technicky sa mi vakuovanie zdalo byt dost narocne a predtym som predpokladal, ze to je jak pises. a zda sa mi to tak aj dostupnejsie.
ale neoveroval som to.


HonzaB - 20/3/2017 - 01:31

citace:
citace:
jen poznámka: Aerodynamické kryty nejsou vakuované a tedy tato úvaha neplatí...

jeden prednasajuci, co sa zucastnil vypustania satelitu ako technik, ma presviedcal, ze AD kryt je vakuovany, ale nijak som si to neoveroval. ani ci vobec, alebo ci len v spec. pripadoch.
btw. technicky sa mi vakuovanie zdalo byt dost narocne a predtym som predpokladal, ze to je jak pises. a zda sa mi to tak aj dostupnejsie.
ale neoveroval som to.


... to máš nejspíš chybu v překladu. Přidávám odkazy (soubory jsou dostupné na netu):

- "Soyuz_Users_Manual_Part1.pdf" , kapitola "3.2.6.Static pressure under the fairing" až "3.3 fairing air-conditioning system"

- "falcon_9_users_guide_rev_2.0.pdf", kapitola "4.3.6 Fairing Internal Pressure"

ps: AD kryt je někdy udržován mírně přetlakovaný, čímž se lépe řídí kontrolované vnitřní prostředí (netěsnostmi uniká vnitřní atmosféra ven a nikoliv okolní vzduch a prach dovnitř).


Alchymista - 20/3/2017 - 12:08

vakuované vs. hermetické - zrejme sa to stratilo v prekladoch a "interpretáciách".

Uzavretie aerodynamického krytu musí byť skutočne veľmi tesné až hermetické, pretože i celkom "tenký" prúd vzduchu pri rýchlosti 1-2km/s dokáže "úžasné veci", takže spoje aerodynamického krytu musia perfektne tesniť, alebo byť uspôsobené na zabránenie prenikania prúdenie pod kryt(labyrinty, turbolátory...) - pre predstavu: plyny zo slepého náboja do samopalu 58 majú cca 5cm od ustia hlavne rýchlosť 600-800m/s.


HonzaB - 20/3/2017 - 13:43

citace:
Uzavretie aerodynamického krytu musí byť skutočne veľmi tesné až hermetické, pretože i celkom "tenký" prúd vzduchu pri rýchlosti 1-2km/s dokáže "úžasné veci", takže spoje aerodynamického krytu musia perfektne tesniť, alebo byť uspôsobené na zabránenie prenikania prúdenie pod kryt(labyrinty, turbolátory...)

Souhlas ... ;-)
citace:
- pre predstavu: plyny zo slepého náboja do samopalu 58 majú cca 5cm od ustia hlavne rýchlosť 600-800m/s.

... a s tímto nesouhlasím. Porovnáváte neporovnatelné.
Proud plynu u usti hlavne má kompletně jiné energetické parametry. Vysoky tlak (v hlavni je tlak ve stovkách barů), vysokou rychlost, obrovské množství jak statické i kinetické energie na objemovou jednotku.
Oproti tomu aerodynamický kryt je maximálně namáhán v okamžiku "maxQ", který bývá při rychlostech kolem 300-400m/s a ve výšce 11-14km (což odpovídá tlaku cca 0.3 baru). Tedy objemová energie plynu je o několik řádů nižší ...

ps: ve vedlejším tématu probíraná numerická simulace počítá i parametry v bodě "maxQ".... ;-)


Petr_Šída - 20/3/2017 - 23:24

citace:
vakuované vs. hermetické - zrejme sa to stratilo v prekladoch a "interpretáciách".

Uzavretie aerodynamického krytu musí byť skutočne veľmi tesné až hermetické, pretože i celkom "tenký" prúd vzduchu pri rýchlosti 1-2km/s dokáže "úžasné veci", takže spoje aerodynamického krytu musia perfektne tesniť, alebo byť uspôsobené na zabránenie prenikania prúdenie pod kryt(labyrinty, turbolátory...) - pre predstavu: plyny zo slepého náboja do samopalu 58 majú cca 5cm od ustia hlavne rýchlosť 600-800m/s.




vhodná štěrbina a vývěva je na světě, pro pevnost konstrukce nic dobrého ...

jak je vlastně řešené odtlakování? soustavou přetlakových ventilů?


Alchymista - 21/3/2017 - 08:11

citace:
citace:
- pre predstavu: plyny zo slepého náboja do samopalu 58 majú cca 5cm od ustia hlavne rýchlosť 600-800m/s.
... a s tímto nesouhlasím. Porovnáváte neporovnatelné.
Proud plynu u usti hlavne má kompletně jiné energetické parametry. Vysoky tlak (v hlavni je tlak ve stovkách barů), vysokou rychlost, obrovské množství jak statické i kinetické energie na objemovou jednotku.
päť centimetrov od ústia je už tlak plynov v prúde prakticky vyrovnaný s okolím (neviem z hlavy, koľko presne je priemer otvoru na nástavci, ale 5cm je viac ako desať "kritických priemerov" trysky). Námietku ale beriem, hoci - neslobodno si zamieňať podmienky prúdenia v hlavni a prúdenia mimo hlaveň.

citace:
vhodná štěrbina a vývěva je na světě, pro pevnost konstrukce nic dobrého ...
jak je vlastně řešené odtlakování? soustavou přetlakových ventilů?
Preto tiež vojenský piloti nosia pri letoch nadzvukovou rýchlosťou výškové obleky - pokiaľ by v 11-12km došlo pri nadzvuku k odhermetovaniu kabíny, tak ejekčný efekt prúdenia okolo trupu urobí v kabíne podtlak zodpovedajúci výške viac ako 20-25km (pre Mig-21 pri Mach 2,05 v 11000m to malo byť "v najhoršom prípade" okolo 0,01atm - tj. ekvivalent výšky vyše 30km)

Na "odtlakovanie" priestoru pod krytom stačí aj celkom jednoduchá odpružená klapka ("dvierka"), maximálne ešte chránená z vnútornej strany systémom jednoduchého labyrintu (drážka tvaru U a do nej so štrbinou vložená doska), brániaceho tomu, aby prúdenie v otvore klapky pri odtlakovaní dosiahlo nejak vysoké hodnoty.
Pokiaľ sa nad problémom "trochu pobáda", tak sa dá urobiť aj iný trik - funkčný ekvivalent obojsmerného vyrovnávacieho ventilu vo forme venturiho trubice.


martinjediny - 12/10/2017 - 23:07

hral som sa s uvahami pre deorbit telesa a maximalne zatazenie mi vychadza pre vztlakove teleso pri cca 4,5 km/s...
moze to byt spravne?



Alchymista - 13/10/2017 - 03:05

V akej výške?


ales - 13/10/2017 - 09:43

Máš na mysli zatížení tepelné, nebo mechanické? Každopádně oboje silně závisí na profilu sestupu (úhlu, strmosti) a na parametrech toho vztlakového tělesa (rozměry, hustota, tvar, klouzavost, ...). Obecně bych ale řekl, že každé aerodynamické zatížení prostě stále roste s rychlostí a hustotou atmosféry. 4,5 km/s jako maximum tak může vyjít jen pro určité konkrétní podmínky (zpomalování nebo zrychlování a přitom současně i změna výšky [hustoty atmosféry]). Jaké podmínky jsi předpokládal?


martinjediny - 13/10/2017 - 19:01

urcite uhol nabehu vyrazne ovplyvnuje tepelne namahanie...
nezavislo od uhlu ale maximum bolo (pre rovnake uhly pocas zastupu) dosiahnute pri cca 4,5km/s
v zavislosti od uhlu sa menila len spickova dosihnuta teplota.
preto by malo mat zmysel pri 4,5km/s mat najmensi uhol nabehu.


vychadzal som z predpokladu, ze klzavost telesa je nezavisla od hustoty atmosfery. rozdiel hustoty sposobuje len rozdielnu rychlost potrebnu k vyvodeniu vztlaku.

takze som predpokladal, ze teleso si samo klesne do hustoty atmosfery, v ktorej uz bude nadnasane. Pomer vztlaku a celneho odporu ostava staly, takze potom viem vypocitat pracu z odporu a prejdenej drahy...

vypocet supersonickeho vztlaku bude snad porovnatelny v rozsahu 8000 - 2000 m/s
takze aj ked hodnoty nemusim mat presne, tak princip by mohol byt porovnatelny.

https://drive.google.com/open?id=0BwoCTlyAHBdXakV0ZmNkRkQ4M1k


Archimedes - 16/10/2017 - 00:14

Takhle od oka to může sedět, případně to můžu prohnat vlastní simulací.
Jen u té teploty ze ztrátového výkonu - zdaleka ne všechna jde do tělesa, většina jde v rázové vlně pryč kolem tělesa.
Zhruba
[plošná hustota tepelného toku, W/m2] = 0.0002 * [v, m/s]^3 * ( [rho_atm, kg/m3] / [poloměr křivosti náběžné strany, m] )^0,5


martinjediny - 16/10/2017 - 12:20

citace:
...[plošná hustota tepelného toku, W/m2] = 0.0002 * [v, m/s]^3 * ( [rho_atm, kg/m3] / [poloměr křivosti náběžné strany, m] )^0,5

urcite pri ohreve nabeznej strany ma vplyv aj medzna vrstva, ale mne slo hlavne o porovnanie tepelneho namahania pri vztlakovom telese v zavislosti od rychlosti.
preto som sa uspokojil so zjednoduseniami...

...ten tvoj vzorec je zaujimavy, skusim sa s nim pohrat.
dakujem.


martinjediny - 7/11/2017 - 17:52

cital som si prispevok http://www.kosmo.cz/modules.php?op=modload&name=XForum&file=viewthread&start=18&page=1&tid=1817#pid141334

a tak ma napadlo...

do akych nizkych teplot sa daju pouzit peltierove clanky ako chladice?

da sa podkrocit 1/5TD "Debye temperature" pre cezium? t.j. 7,5 K ?
https://en.wikipedia.org/wiki/Seebeck_coefficient


nadas - 11/10/2018 - 23:06

Dobrý den ,
zajímalo by mne jak vlastně funguje záchranný systém Sojuzu ( SAS ), vím jak pracuje záchranná věžička na rampě a v úvodních fázích startu , ale jak je to po odhození věžičky před odhozením aerodynamického krytu , a jak po jeho odhození ??
Děkuji Nadas


martinjediny - 16/3/2021 - 19:45

len taká úvaha...
ako bude vyzerat ciolkovskeho rovnica, ak vytok paliva je desiatkach a stovkach km/s?
Nebude efektivnejsie modifikovat ju na energie?


Alchymista - 17/3/2021 - 00:25

Podľa mňa sa nezmení - nemá dôvod.
Výtoková rýchlosť bude síce vysoká, ale to je tiež všetko... Ešte obvykle klesne hmotnosť paliva a vzrastie "hmotnosť nákladu", pretože samotný pohonný systém bude relatívne ťažší.

citace:
Nebude efektivnejsie modifikovat ju na energie?

Možno aj bude... Každopádne rovnica sa dá prepísať z pohľadu "rýchlost a hmotnost" do pohľadu "hmotnost a energia". Len pri tom asi príde o svoju eleganciu a jednoduchosť...


martinjediny - 17/3/2021 - 09:35

citace:
...Každopádne rovnica sa dá prepísať z pohľadu "rýchlost a hmotnost" do pohľadu "hmotnost a energia". Len pri tom asi príde o svoju eleganciu a jednoduchosť...

urcite relativisticka fyzika nema tak krasnu a intuitivnu jednoduchost ako newtonovska... preto sa pytam Intuitivne by som mal setrit palivo, lebo pri naraste rychlosti mi narasta hmotnost castice... len vzdy je to nieco za nieco a este som to neanalyzoval...

Ked obcas pisem na wiki, tak rad aj s prikladmi... A vtedy som si vsimol, ze pomer hmotnosti elektronu a protonu nezodpoveda pri rozdielnej rychlosti pomerom newtonovskej fyziky... a to uz pri 1eV, zatial co napr. iontove motory idu nad 10keV https://sk.wikipedia.org/wiki/Ur%C3%BDch%C4%BEova%C4%8D_%C4%8Dast%C3%ADc#R%C3%BDchlos%C5%A5_vs._energia



yamato - 17/3/2021 - 10:57

citace:
Intuitivne by som mal setrit palivo, lebo pri naraste rychlosti mi narasta hmotnost castice...



a vies ze toto ma uz tiez napadlo? ale viac ti k tomu nepoviem, lebo pri relativistickej fyzike som skor fanusik nez znalec


Alchymista - 17/3/2021 - 11:59

Aby malo relativistické zmeny hmotnosti "zmysel uvažovať" , výtoková rýchlosť musí predstavovať významnú časť rýchlosti svetla. Pri 100 000km/s je rozdiel len niečo okolo 5%, pri 200 000km/s okolo 25%. Pri 100 a 1000km/s je "rozdiel" na šiestom a siedmom platnom mieste.


yamato - 17/3/2021 - 12:06

citace:
Aby malo relativistické zmeny hmotnosti "zmysel uvažovať" , výtoková rýchlosť musí predstavovať významnú časť rýchlosti svetla. Pri 100 000km/s je rozdiel len niečo okolo 5%, pri 200 000km/s okolo 25%. Pri 100 a 1000km/s je "rozdiel" na šiestom a siedmom platnom mieste.


pockaj, chces povedat ze by to naozaj fungovalo? ze pri relativistickych rychlostiach mi klesne spotreba, pretoze stupne hmotnost vytokovych castic?


Ervé - 17/3/2021 - 12:12

citace:
Dobrý den ,
zajímalo by mne jak vlastně funguje záchranný systém Sojuzu ( SAS ), vím jak pracuje záchranná věžička na rampě a v úvodních fázích startu , ale jak je to po odhození věžičky před odhozením aerodynamického krytu , a jak po jeho odhození ??
Děkuji Nadas


Po odhození věžičky dojde při poruše nosiče nejdřív k pyrotechnickému oddělení Sojuzu od rakety. Pak se odhodí aerodynamický kryt (pokud tam ještě je) a zažehne raketový motor Sojuzu pro oddělení od nosiče. Po několika sekundách chodu motor vypne a dojde k rozdělení Sojuzu na přístrojový úsek, kabinu a orbitální modul. Pak padají volným pádem dolů, dokud se neotevře padák kabiny Sojuzu.


Alchymista - 17/3/2021 - 12:17

ervé - dátum...


Alchymista - 17/3/2021 - 12:35

citace:
pockaj, chces povedat ze by to naozaj fungovalo? ze pri relativistickych rychlostiach mi klesne spotreba, pretoze stupne hmotnost vytokovych castic?
iste, fungovať by to fungovalo, ale reálna účinnosť procesov urýchlovania rýchlo klesá.
U klasického raketového motoru je to jednoduché - chemickou reakciou zahreješ plyny na nejakých 4200°C a tryskou vypustíš von, čím dosiahneš výtokovú rýchlosť cca 4200m/s. To je tiež zhruba maximum, ktoré takouto jednoduchou technikou dosiahneš...
U čohokoľvek zložitejšieho narazíš na kruciálny problém - odkiaľ na to urýchlovanie vziať energiu???
Kým sa bavíme o výtokových rýchlostiach povedzme do 100 000km/s (iontové motory) a malých celkových výkonoch (ťah v jednotkách newtonov), a o "priestore medzi Merkurom a Jupiterom", dajú sa použiť solárne panely (tj. externé napájanie pohonu, vonkajší zdroj energie). Ale ďalej to začína byť fakt problém...
Energiu na urýchlenie častíc treba odniekiaľ získať a nejak do častice napumpovať.
Navyše - ona nám "oťažie" už v procese urýchlovania, takže do nej treba napumpovať o primerane veľký (rovnaký) kus viac energie
Obed zadarmo nikde nedostaneš, a až vôbec nie od matky prírody.
[upraveno 17.3.2021 12:46]


Derelict - 17/3/2021 - 12:45

citace:
pockaj, chces povedat ze by to naozaj fungovalo? ze pri relativistickych rychlostiach mi klesne spotreba, pretoze stupne hmotnost vytokovych castic?

1. Je potřeba se dostat na nějakých 95-98% rychlosti světla, aby to začalo mít smysl...
2. U 99,9% se začíná projevovat slušný efekt, čím víc devítek, tím lépe.
Bohužel E=mc^2 platí i obráceně. Čím více se snažím částice urychlit, tím vyšší množství energie jim musím dodat. Protože jakýkoliv motor ma jenom určitou účinnost, část výkonu se mění na teplo. Zkuste si to spočítat pro motor s účinností 85% a hmotu 1g, urychlenou na rychlost 99,999% rychlosti světla. Ztrátový výkon, který je potřeba uchladit je prostě příšerný.


yamato - 17/3/2021 - 13:05

iste iste, obed zadarmo nebude (do kelu!) ale aj tak ma to pomerne prekvapilo

pri reaktivnych pohonoch vnimam ako hlavny limitujuci faktor zasobu reaktivnej latky na palube. Cim viac chceme "reagovat", tym viac latky potrebujeme, lenze tym stupa hmotnost

takato relativisticka finta znamena, ze dokazem ten hlavny "problem" presunut z hmoty na energiu. T.j. mnozstvo hmoty na palube nemusim zvysovat, pokial mam tolko energie ze tu hmotu urychlim na relativisticke rychlosti

viem ze to je tiez problem a energia nie je zadarmo, ale tak nejako intuitivne sa mi paci, ze zrazu riesim hafo energie namiesto toho, aby som riesil hafo hmoty


Alchymista - 17/3/2021 - 13:18

Nie celkom presne, ale ano. Ešte lepší pohľad je, že si v rakete vezenieš energiu a reakčnú hmotu.
Klasické rakety to vtipne riešia tak, že palivo predstavuje aj zásobu/zdroj energie aj reakčnú hmotu.


martinjediny - 17/3/2021 - 13:26

Ale v pripade potreby mozes dostat injekciu solarnych zrkadiel, laseroveho, resp. mikrovlnneho dela a pod.,

resp. odlet zo slnecenj sustavy zacne letom do stredu, aby si bol blizko zdroja...

alebo inak, ak to prezijes...


Ervé - 17/3/2021 - 14:00

citace:
ervé - dátum...


Aha, to sem si nevšiml.


Ervé - 17/3/2021 - 14:04

citace:
Ale v pripade potreby mozes dostat injekciu solarnych zrkadiel, laseroveho, resp. mikrovlnneho dela a pod.,

resp. odlet zo slnecenj sustavy zacne letom do stredu, aby si bol blizko zdroja...

alebo inak, ak to prezijes...


Jenže tam je jasný problém v tom, že využiješ Slunce pro zrychlení, ale pokud letíš za Jupiter, tak v cíli už nezpomalíš jinak než vezeným motorem s palivem nebo o atmosféru.


yamato - 17/3/2021 - 14:53

citace:

Jenže tam je jasný problém v tom, že využiješ Slunce pro zrychlení, ale pokud letíš za Jupiter, tak v cíli už nezpomalíš jinak než vezeným motorem s palivem nebo o atmosféru.


a od ciela naspäť už sa nedostaneš nijako

rôzne koncepty beamed-energy sme tu už rozoberali, hlavný problém vidím v tom, že aj ten najkolimovanejší beam po pár storisíc kilometroch už je rozplzlý na obrovskej ploche, takže väčšina energie je čistá strata.

chce to antihmotu...


martinjediny - 17/3/2021 - 16:13

citace:
...chce to antihmotu...

vyrobit ju este ako tak vieme, ale rozumne vyuzit... ... to by vydalo aj na tri nabelove ceny...


Archimedes - 18/3/2021 - 11:19

citace:
Aby malo relativistické zmeny hmotnosti "zmysel uvažovať" , výtoková rýchlosť musí predstavovať významnú časť rýchlosti svetla. Pri 100 000km/s je rozdiel len niečo okolo 5%, pri 200 000km/s okolo 25%. Pri 100 a 1000km/s je "rozdiel" na šiestom a siedmom platnom mieste.

Relativistická (Meščerského) rovnice vypadá zhruba podobně a ten hyperbolický tangens se skoro neprojeví, dokud to není opravdu hodně těsně k rychlosti světla.


yamato - 18/3/2021 - 16:23

len nenápadne rypnem, že LHC urýchľuje častice na 99.9999991% rýchlosti svetla


Derelict - 18/3/2021 - 16:41

citace:
len nenápadne rypnem, že LHC urýchľuje častice na 99.9999991% rýchlosti svetla

len nenápadne rýpnem, že LHC má odber takmer 120MW ;o)


Alchymista - 18/3/2021 - 17:12

citace:
citace:
len nenápadne rypnem, že LHC urýchľuje častice na 99.9999991% rýchlosti svetla
len nenápadne rýpnem, že LHC má odber takmer 120MW ;o)

žabári... rýp fakt nenápadne...

Taký Raptor má tepelný výkon ... ehm ... 6,5-7,3 gigawattu

prietok metanu 133kg/s, energia 50-55MJ/kg
https://hypertextbook.com/facts/2004/BillyWan.shtml [upraveno 18.3.2021 17:18]


yamato - 18/3/2021 - 17:20

citace:
citace:
len nenápadne rypnem, že LHC urýchľuje častice na 99.9999991% rýchlosti svetla

len nenápadne rýpnem, že LHC má odber takmer 120MW ;o)


ale nádej žije


martinjediny - 18/3/2021 - 21:45

Preco sa pri iontovych motoroch pouziva mriezka a nie napriklad viacnasobny tunel, ako pri linearnych urychlovacoch?
https://upload.wikimedia.org/wikipedia/commons/0/08/Linear_accelerator_animation_16frames_1.6sec.gif
[upraveno 18.3.2021 21:47]


Archimedes - 18/3/2021 - 22:35

citace:
Preco sa pri iontovych motoroch pouziva mriezka a nie napriklad viacnasobny tunel, ako pri linearnych urychlovacoch?

Protože iontové motory musejí řešit hlavně problém tahu, ne výtokové rychlosti.
Už i za malých urychlovacích napětí (stovky V až kV, tohle mnohastupňové urychlování za takových napětí není potřeba) mají násobně větší výtokovou rychlost než chemické motory. Co jim ale chybí, je tah. Vzhledem k tomu, že je problém dosáhnout i tahu poměrně malého a s výtokovou rychlostí navíc stoupá nutně příkon na jednotku tahu, je příliš vysoká výtoková rychlost pro současné energetické zdroje na kosmických tělesech nejen nevyužitelná, ale i kontraproduktivní.

Tady o tom mluvím cca od 11:00 a 55:00:


martinjediny - 19/3/2021 - 00:28

citace:
...Tady o tom mluvím cca od 11:00 a 55:00:
www.youtube.com/watch?v=5YyzloNkSbk

Vdaka, trochu slabsi zvuk, ale inak pekne spracovane.

Ale
Napriklad Deep Space-1 s prazdnou hmotnostou 377kg, mala naviac 70kg xenonu.
S tym spravila za 670 dni dv=4200m/s

1/ pri napr. dvojnasobnej vytokovej rychlosti by minuli nezanedbatelnu 1/2, (samozrejme pri 2nasobnej spotrebe energie), takze je otazka, ci by do usetrenych 35kg narvali dalsich 2500W panelov..., alebo by len zrychlovali dvojnasone dlho...
2/ stale sa hovori o zivotnosti mriezky... Linearny urychlovac fokusuje. Predpokladam, ze teda lepsie odolava.
3/ ten efekt vo tvojom videu, ze musi byt vystupna mriezka rovnako nabita ako sonda nie je tak kriticky pri viacnasobnej elektrode, lebo v "najhorsom" stratim len jedno delta U.

[upraveno 19.3.2021 09:07]


Archimedes - 19/3/2021 - 10:13

citace:
1/ pri napr. dvojnasobnej vytokovej rychlosti by minuli nezanedbatelnu 1/2, (samozrejme pri 2nasobnej spotrebe energie), takze je otazka, ci by do usetrenych 35kg narvali dalsich 2500W panelov..., alebo by len zrychlovali dvojnasone dlho...
Už takhle to zrychlování trvalo dlouho... Technologicky, skoro zdvojnásobení výtokové rychlosti se u mřížkových motorů asi dosáhnout dá, s urychlovacími napětí 3-4kV se experimentuje.
Krom toho, lze za cenu snížení tahu použít "lehčí" palivo - SpaceX používá (kvůli ceně i dostupnosti) místo xenonu krypton, to zvedá Isp o 25%, s argonem by to byl proti xenonu skoro dvojnásobek. Ten stejný motor (stavěný na nějaký výkon) obvykle záměnu jednoho inertního plynu za jiný snáší dobře, prvotní testy se stejně typicky dělají s argonem.
citace:
2/ stale sa hovori o zivotnosti mriezky... Linearny urychlovac fokusuje. Predpokladam, ze teda lepsie odolava.
To by platilo jen při malých proudech iontů. Při velkých plošných proudech (aby tah stál za to) by nutně začalo docházet k rozptylu toho svazku a ke stejným potížím, jako u eroze mřížek.
Kde by tenhle princip lineárního urychlovače vcelku smysl dávat mohl, jsou motorky s nějakými výrazně "makroskopičtějšími" částicemi, které jsou v poměru ke svému náboji mnohem těžší než ionty.
Krom koloidních motorků byly před nějakou dobou navrženy miniaturní motorky na nanočástice a tam je "urychlovací tunel" navržený jako sekvence stacionárních napětí. Úprava na ten střídavý urychlovač by se ale asi udělat dala.
https://www.researchgate.net/publication/228634681_Scalable_Flat-Panel_Nano-Particle_MEMSNEMS_Thruster
https://www.researchgate.net/publication/253041403_Nanoparticle_Electric_Propulsion_for_Space_Exploration
citace:
3/ ten efekt vo tvojom videu, ze musi byt vystupna mriezka rovnako nabita ako sonda nie je tak kriticky pri viacnasobnej elektrode, lebo v "najhorsom" stratim len jedno delta U.
No ale pokud _střední_ napětí na koncovém segmentu bude nulové, v podstatě je to stejný případ. A krom toho, celá ta soustava se dá s trochou snahy udělat jako plovoucí (u koncové elektrody si stačí domyslet uzemnění). [upraveno 19.3.2021 12:52] [upraveno 19.3.2021 15:10] [upraveno 22.3.2021 07:35]


martinjediny - 25/3/2021 - 14:20

Presuvam debatu zo slunce a klima, pre vacsi okruh zaujemcov...

1/ Nakolko je slnecna konstanta konstantna, ak sa vplyvom pohybu barycentra zmeni poloha obeznej drahy vzhladom na Slnko?
2/ Bude prijem energie na Zemi vyssi ked je barycentrum v strede Slnka, alebo ked je 800000 km nad jeho povrchom?
3/ Bol zaznamenany rozdiel v solarnej konstante, ked sme obiehali Slnko vzhladom na vyosenie barycentra v najkratsej a najvacsej vdzialenosti? (napr. rok 2020)(skorokonjukcia 4 velkych planet) [upraveno 25.3.2021 14:20]


Toto téma přichází z:
http://www.kosmo.cz

Url tohoto webu:
http://www.kosmo.cz/modules.php?op=modload&name=XForum&file=print&fid=3&tid=1027